OB/GYN PP

Réussis tes devoirs et examens dès maintenant avec Quizwiz!

A 37-year-old G0 complains that she experiences mood swings, irritability, bloating and headaches monthly for two to three days prior to her menstrual cycle. Her medical history is unremarkable and physical examination is normal. The physician advises her to keep a calendar of her symptoms. He also recommends a balanced diet, avoidance of caffeine and alcohol, and daily regular exercise. The patient has never exercised regularly and wonders how this will help her mood swings and bloating. Which of the following would provide the best explanation for the benefits of exercise on her PMS symptoms? A. Endorphins B. Cortisol C. Progesterone D. Estrogen E. Androgen

"A Exercise increases circulating endorphins in the brain which are ""feel good"" hormones and act similar to serotonin. Therefore, in addition to being a benefit to cardiovascular health, regular exercise can significantly decrease symptoms of PMS."

A 20-year-old G2P2 healthy woman presents for her post-partum check six weeks after a full term normal spontaneous delivery. She has a 13 month old in addition to the six-week newborn, and is already feeling overwhelmed. She desires a reliable form of contraception. On exam, her vital signs are normal. BMI is 27. The remainder of the exam is unremarkable. Of the following, what is the most effective form of contraception for this patient? A. Intrauterine device B. Tubal ligation C. Depo-Provera D. Oral contraceptive pills E. Essure

"A Long-acting reversible contraceptives (LARC) methods such as contraceptive implants and intrauterine devices are a good option for this patient. Despite high up-front costs and the need for office visits for insertion and removal, LARC methods provide many distinct advantages over other contraceptive methods as Depo-Provera and oral contraceptives. While Depo-Provera is an effective form of contraception, it may not be the best choice in this woman with a high BMI. For this young mother who desires a reversible, but reliable form of contraception, the high effectiveness, continuation rate and user satisfaction of LARC methods would be of most benefit. Emerging evidence indicates that increasing the use of LARC methods also could reduce repeat pregnancy among adolescent mothers and repeat abortions among women seeking induced abortion. (""Increasing Use of Contraceptive Implants and Intrauterine Devices To Reduce Unintended Pregnancy,"" ACOG Committee Opinion, No. 450, 2009). Tubal ligation and Essure are permanent and are not appropriate for this patient."

A 54-year-old G2P2 presents to your office for a health maintenance examination. Her last menstrual period was eight months ago. She complains of severe vasomotor symptoms, vaginal dryness, and dyspareunia, and she desires treatment for her symptoms. She has otherwise been in good health and has no significant past medical or surgical history. Her family history is significant for a mother who has severe osteoporosis at the age of 75 and a grandmother who died of breast cancer at the age of 79. She does not report any smoking, alcohol or drug use. On physical exam her BP=130/78, Pulse=84, BMI=26. The remainder of her exam is within normal limits except for severe vaginal atrophy noted on the pelvic examination. The best recommendation for this patient would include which of the following? A. Lowest effective dose of combination hormone replacement therapy for the shortest duration possible B. Long term hormone replacement therapy to treat her vasomotor symptoms and prevention of osteoporosis C. Testosterone cream D. Progesterone cream E. Biosphophonates

"A The American College of Ob-Gyn (ACOG) recommendations on hormone replacement therapy considers hormone replacement therapy (HRT) the most effective treatment for severe menopausal symptoms that include hot flashes, night sweats and vaginal dryness. The physician should counsel the woman about the risks and benefits before initiating treatment. ACOG recommends ""the smallest effective dose for the shortest possible time and annual reviews of the decision to take hormones."" HRT should not be used to prevent cardiovascular disease due to the slight increase in risk of breast cancer, myocardial infarction, cerebrovascular accident, and thromoboembolic events. A woman with an intact uterus should not use estrogen-only therapy because of the increased risk of endometrial cancer. In addition to the same risks as FDA approved treatments, bioidentical hormones such as testosterone and progesterone cream may have additional associated risks. Bisphosphonates are used to treat osteoporosis and will not relieve her symptoms."

A 58-year-old G3P3 woman who has been menopausal since age 50 comes to you for a health maintenance examination. She is in good health, eats a balanced diet, exercises regularly, and has an unremarkable physical exam. Her bone mineral density as determined by central dual energy X-ray absorptiometry is -1.7. She wants to discuss treatment for her osteopenia. What is the next step in the management of this patient? A. Evaluate her risk factors for fracture B. Determine her frequency of exercise C. Assess her exogenous dietary intake of estrogen D. Assess her exogenous dietary intake of progesterone E. Repeat DEXA scan in one year

"A The World Health Organization (WHO) defines osteopenia (low bone mass) as -1 to -2.5. The American College of Ob-Gyn (ACOG)) Committee Opinion recommends that physicians interpret T scores between −1.5 and −2.0 in combination with the patient's risk factors for fracture. The authors state: ""Clinicians must be careful because the diagnosis of osteopenia often is interpreted as indicating a pathologic skeletal condition or significant bone loss, neither of which is necessarily true. Until better models of absolute fracture risk exist, postmenopausal women in their 50s with T scores in the osteopenia range and without risk factors may well benefit from counseling on calcium and vitamin D intake and risk factor reduction to delay initiation of pharmacologic intervention."" Some of the risk factors for fracture include prior fracture, family history of osteoporosis, race, dementia, history of falls, poor nutrition, smoking, low body mass index, estrogen deficiency, alcoholism, and insufficient physical activity."

"A 42-year-old patient came to the gynecologist to get anticonception advice. She smokes 5 cigarettes daily and is treated for arterial hypertension. Periodic bleeding are abundant and last up to 7 days. Which of the following methods should be advised? A. progestogen-only ""mini pill"". B. subcutaneous implant releasing etonogestrel. C. intrauterine device with copper, Cu-IUD. D. intrauterine device releasing levonorgestrel. E. sterilization."

"A. progestogen-only ""mini pill"". Possibly D. intrauterine device releasing levonorgestrel. (Answer unconfirmed)"

"A 6-year-old girl in kindergarten has begun to bed-wet after having been toilet trained for three years. She has also been found ""playing with herself"" for the past several weeks. In a private conversation the child states, ""My daddy has been playing with me down there."" Which of the following is the most appropriate course of action at this time? A. Suggest family counseling B. Notify police C. Speak with her kindergarten teacher D. Speak with both parents and child together E. Admit the child to the hospital"

"B Child abuse can happen in any circumstance. It is not always the ""boyfriend"" who commits such a crime. Often the perpetrator is a ""pillar of the community."" The child must be able to talk in a safe environment, certainly not in the presence of the person accused of the deed."

A 22-year-old G1P0 woman who underwent dilation and curettage for a presumed missed abortion has a pathology report revealing a partial molar pregnancy. Compared to a complete mole, which of the following is true about a partial mole? A. Karyotype 69XXY, fetus present, higher risk of developing post-molar GTD B. Karyotype 69XXY, fetus present, lower risk of developing post-molar GTD C. Karyotype 46XX, fetus present, higher risk of developing post-molar GTD D. Karyotype 46XX, fetus present, lower risk of developing post-molar GTD E. Karyotype 46XX, fetus absent, lower risk of developing post-molar GTD

"B Molar pregnancies are classified as either complete or partial, depending on several histologic, pathologic and genetic characteristics. Partial moles may contain fetus/fetal parts, placenta/cord; complete moles do not. Partial moles are triploid karyotype (usually 69XXY, 69XXX, or 69XYY) resulting from fertilization of egg by dispermy; complete moles are diploid resulting from fertilization of ""empty egg"" by single sperm (46XX, 90%) or by two sperm (X & Y = 46XY 6-10%). Partial moles show marked villi swelling; complete moles show trophoblastic proliferation with hydropic degeneration. Clinically, partial moles present with lower Beta-hCG levels, affect older patients, have longer gestations, and are often diagnosed as missed or incomplete abortions. Complete moles usually present with larger uteri, preeclampsia and higher likelihood of developing into post-molar GTD."

An 88-year-old nursing home resident is brought in for evaluation of blood found in her diapers. She has a history of lichen sclerosis of the vulva, which was diagnosed fifteen years ago. She quit smoking in her fifties. Examination of the external genitalia reveals an elevated, firm, erythematous, ulcerated lesion arising from the left labia, measuring 2.5 cm in greatest dimension. What is the most likely diagnosis in this patient? A. Malignant melanoma B. Squamous cell carcinoma C. Lichen sclerosus D. Paget's disease E. Verrucous carcinoma

"B Squamous cell carcinoma accounts for approximately 90% of vulvar cancers. Patients commonly present with a lump and they commonly have a long-standing history of pruritus. The chronic itch-scratch cycle of untreated lichen sclerosus, or any other chronic pruritic vulvar disease, is thought to stimulate the development of squamous carcinoma. The mean age of squamous cell carcinoma is 65 years and smoking is known to increase the risk of development of vulvar cancer, especially in the setting of HPV infection. With lichen sclerosus, the skin appears thin, inelastic and white, with a ""crinkled tissue paper"" appearance. Paget's disease of the vulva is associated with white plaque-like lesions and poorly demarcated erythema, not a discrete mass. Verrucous carcinoma has cauliflower-like lesions. Melanoma typically presents as a pigmented lesion."

A 28-year-old G1P1 presents to your office. She delivered four days ago and tearfully reports that yesterday and the day before she had trouble sleeping, felt anxious and was irritable. She feels somewhat better today, but is still concerned. What is the most likely diagnosis? A. Hypothyroidism B. Blues C. Depression D. Normal postpartum state E. Anxiety

"B The patient is describing symptoms of postpartum blues that affects 50% women within three to six days postpartum. Symptoms include insomnia, easy crying, depression, poor concentration, irritability or labile affect and anxiety. Symptoms often last a few hours per day and are mild and transient. ""Blues"" are probably related to biochemical changes of puerperium. Postpartum depression symptoms, such as mood changes, insomnia, phobias and irritability are more pronounced than with the ""blues."""

An 18-year-old college student is seen in the emergency room, claiming she was raped by a 29-year-old janitor in her dorm four hours ago. He threatened her with a knife and she did not resist. She appears calm and has a flattened affect when the history is taken. Currently, she is sexually active with a fellow student and is taking birth control pills. The alleged attacker used a condom, which she helped him put on. The student is 5 feet 2 inches tall and weighs 110 pounds. She is a cheerleader for the college football team. Examination reveals no bruising and gynecologic examination reveals no apparent injuries. Which of the following statements is the most likely explanation for this patient's presentation? A. The attack was not emotionally traumatic B. The student is in shock C. The attack never happened D. The student cooperated with the attacker E. The student suffers from chronic depression

"B Women who are raped are often in denial or shock. A woman may shower after an attack, destroying evidence in an attempt to deny what happened or to ""clean herself."" She may blame herself for the attack and feel she should have resisted more."

"A 30-year-old G2P2 was raped by a stranger while she was walking through a park alone at 2:00 am. The assailant did not use a condom. An intern evaluated the patient in the emergency room. He took a detailed history, performed a complete physical exam and collected forensic specimens. He obtained cultures for gonorrhea and chlamydia, and obtained an RPR, hepatitis antigens, an HIV test, a urinalysis and culture, and a pregnancy test. He provided the patient with postcoital contraceptive medication. Which of the following additional actions is most appropriate? A. Notify police B. Notify the patient's parents or her closest relative or friend C. Offer the patient antibiotic prophylaxis for sexually transmitted infections D. Counsel the patient regarding the practice of ""safe behavior,"" including not walking in deserted areas alone E. Offer the patient testing for herpes simplex virus (HSV)"

"C Antibiotic prophylaxis should be offered to all adult rape victims. Although patients are often reticent to do so, they should be gently encouraged to work with the police. This has been associated with improved emotional outcomes for victims. Counseling the patient regarding the practice of ""safe behavior"" at this time may make the patient feel blame, when the blame should be placed on the rapist. HSV antibody testing is not indicated."

A 23-year-old G2P0 woman at 33 weeks gestation presents to labor and delivery with acute nausea, vomiting and epigastric pain. Her blood pressure is 145/90; she has 1+ protein on a urinalysis. Her labs are shown below: Hematocrit: 42% Bilirubin: 1.4 mg/dL White blood count: 11,000/mcL Bilirubin: 1.4 mg/dL Platelets: 42,000/mcL Lipase: 11 u/L Aspartate aminotransferase (AST): 391 u/L Creatinine: 0.8 mg/dL Alanine aminotransferase (ALT): 444 u/L Uric acid: 7.7 mg/dL Glucose: 100 mg/dL Fibrinogen: 405 mg/dL Which of the following is the most likely diagnosis in this patient? A. Mild preeclampsia B. Hepatitis C. HELLP syndrome D. Cholecystitis E. Acute fatty liver

"C HELLP syndrome is a disease process in the spectrum of severe preeclampsia. The acronym stands for ""hemolysis, elevated liver enzymes, low platelets"" and can lead to swelling of the liver capsule and possibly, liver rupture. It may or may not be accompanied by right upper quadrant pain. It is possible to only have thrombocytopenia and elevated transaminases, without clear hemolysis (elevated bilirubin and anemia), especially if a diagnosis is made early. This patient does not have seizures and, therefore, does not have eclampsia. The clinical scenario is not consistent with hepatitis or cholecystitis. Acute fatty liver almost always manifests late in pregnancy. Symptoms develop over several days to weeks and include malaise, anorexia, nausea and vomiting, epigastric pain, and progressive jaundice. In many women, persistent vomiting in late pregnancy is the major symptom. About half of all women have hypertension, proteinuria, and edema signs suggestive of preeclampsia. There is usually severe liver dysfunction with hypofibrinogenemia, hypoalbuminemia, hypocholesterolemia, and prolonged clotting times. As acute fatty liver worsens there is marked hypoglycemia."

A 28-year-old G1P1 presents to your office. She delivered four weeks ago and tearfully reports that she is not sleeping, feels anxious and has thoughts of jumping out her 15th floor window. What is the most likely diagnosis? A. Postpartum anxiety B. Postpartum blues C. Postpartum depression D. Postpartum psychosis E. Bipolar disorder

"C The patient is describing symptoms of depression. Symptoms such as mood changes, insomnia, phobias and irritability are more pronounced than with the ""blues."" She has not described any of the even more advanced psychotic symptoms of visual or auditory hallucinations."

A 48-year-old G2P2 complains of progressively heavier and longer menstrual periods over the last year. Prior to this year the patient had normal periods. She denies any symptoms other than fatigue over the last few months. Physical examination is unremarkable except for the pelvic examination. The patient is noted to have an irregularly shaped 16-week size uterus. The patient's hematocrit is 28%. What is this patient's most likely diagnosis? A. Endometrial hyperplasia B. Endometrial carcinoma C. Uterine fibroids D. Uterine leiomyosarcoma E. Adenomyosis

"C The patient's history and physical examination is typical for a perimenopausal woman with probable uterine fibroids. Although it is possible that she could have underlying endometrial hyperplasia, the most likely diagnosis is uterine fibroids. Uterine leiomyosarcoma should be considered in a postmenopausal woman with bleeding, pelvic pain coupled with uterine enlargement, and vaginal discharge, but it is exceedingly rare. Endometrial hyperplasia is more common in perimenopausal women who do not ovulate regularly and postmenopausal women. Endometrial carcinoma is typically a disease of postmenopausal women, although 5-10% of cases occur in women who are menstruating and 10-15% of cases occur in perimenopausal women. Adenomyosis may result in a symmetrically enlarged ""boggy"" uterus, but usually presents with dysmenorrhea in addition to menorrhagia."

A 42-year-old G2P2 presents to the office because she recently noticed a pigmented lesion on her vulva. She does not know how long it has been there and it doesn't bother her except that she is worried that she may have warts. Her yearly Paps have been negative. Her prior exams have been reported as normal. She is a nonsmoker. Examination of her vulva reveals a pigmented, flat lesion, approximately 1.5 cm in largest diameter. It is non-tender. No induration is present. Her groin nodes are not enlarged. Her vagina and cervix are well estrogenized and without obvious lesions. Which of the following is the most likely diagnosis? A. Vulvar condyloma B. Squamous cell carcinoma C. Melanoma in situ D. Paget's disease E. Lichen sclerosus

"C This lesion may be melanoma and a biopsy must be done to exclude this diagnosis. The concerning features are the size and irregularity of the lesion. Melanoma represents 5% of vulvar cancer, which is not insignificant given the lack of sun exposure and the relatively small surface area. There is no variability in the coloration, ulceration or thickening of the lesion to suggest malignancy at this time. Squamous cell carcinoma is typically not pigmented. High-grade vulvar intraepithelial neoplasia may be flat and pigmented. Paget's disease is usually erythematous with a lacy white mottling of the surface. Condyloma lesions have a characteristic verrucous appearance. With lichen sclerosus, the skin appears thin, inelastic and white, with a ""crinkled tissue paper"" appearance."

A 38-year-old G0 woman comes to the office because she noted a persistent yellow, frothy discharge associated with mild external vulvar irritation. She denies any odor. She tried over the counter anti-fungal medication without success. The discharge has been present for over three months, gradually increasing in amount. Douching has resulted in temporary relief, but the symptoms always recur. Pelvic examination reveals mild erythema at the introitus and a copious yellow frothy discharge fills the vagina. The cervix has erythematous patches on the ectocervix. A sample of the discharge is examined under the microscope. What is the most likely finding? A. Strong amine fishy odor when KOH applied to sample B. Marked polymorphonuclear cells with multi-nucleate giant cells C. Motile ovoid protozoa with flagella D. Budding yeast and pseudo-hyphae E. Clue cells

"C This patient most likely has trichomoniasis. The erythematous patches on the cervix are characteristic of ""strawberry cervicitis."" Trichomonads are unicellular protozoans, which are easily seen moving across the slide with flagella. The slide must be examined immediately. The discharge is mixed with saline and placed on the slide with a cover slip. Women with trichomonas vaginal infections may have a frothy, yellow-green vaginal discharge. Clue cells are seen on a saline wet mount in women who have bacterial vaginosis. Clue cells are characterized by adherent coccobacillary bacteria that obscure the edges of the cells. A drop of KOH releases amines from the cells and a fishy odor is noted if bacterial vaginosis is present. Yeast vaginitis is characterized by a thick white clumpy discharge which results in erythema, swelling and intense pruritus. Multinucleate giant cells and inflammation may be herpes."

A 20-year-old nulliparous previously healthy woman presents to the emergency room with painless vaginal bleeding. Her last menstrual period was 16 weeks ago. On physical exam, her vital signs are: temperature 98.6°F (37.0°C); heart rate 120 beats/minute; and blood pressure 140/90. Abdominal and pelvic examination confirms a 20-week sized uterus with a small amount of blood in the vagina. Beta-hCG is 68,000 mIU/mL. Fetal doppler tones are not auscultated. Which of the following findings would you expect to see on a pelvic ultrasound of this patient? A. Fetus with no cardiac activity B. Fetal pole in one of the fallopian tubes C. Multifetal gestation D. Uterus with a snowstorm appearance E. Empty uterus with an enlarged, complex adnexal mass

"D A complete mole has a characteristic ""snowstorm"" appearance on ultrasound. This is due to the presence of multiple hydropic villi. This patient has a classic presentation for a molar pregnancy. Vaginal bleeding is universal in molar pregnancies. Uterine size greater than dates (weeks from LMP) can be seen in 25-50% of moles, although size less than dates can be seen in 14-33% of moles. There is no fetus seen in cases of a complete mole. There can be a fetus, which is usually grossly abnormal, in cases of a partial mole. There is detectable Beta-hCG in molar pregnancies. The Beta-hCG values are generally higher than the values observed in normal pregnancy. Caution should be taken against the use of a single-value of Beta-hCG to rule in or out a molar pregnancy. However, when combined with the findings of an enlarged uterus and vaginal bleeding, a Beta-hCG value >1,000,000 mIU/mL may be diagnostic. Tachycardia from hyperthyroidism (10% serum diagnosis; 1% clinical diagnosis) and hypertension from preeclampsia (12-25%) can occur in molar pregnancy"

A 35-year-old G5P4 is at 39 weeks gestation. She does not want to have any more children and would like to have permanent sterilization. She is concerned about the risks associated with postpartum tubal ligation. Which of the following is the most likely complication she might experience? A. Chronic cyclic pain with menstrual cycles B. Increased risk for ovarian cancer C. Decreased enjoyment with sexual intercourse D. Aspiration with general anesthesia E. Future pregnancy

"E The existence of a ""post-tubal ligation syndrome"" in which disruption of blood flow in the area of the fallopian tubes leads to menstrual dysfunction and dysmenorrhea has not been substantiated. Tubal ligation appears to have a protective effect on ovarian cancer incidence. The failure rate associated with surgical sterilization is approximately one percent. Approximately one-third of pregnancies after tubal ligation are ectopic. There is no proven association between decreased sexual enjoyment and tubal ligation. Tubal ligations may be performed under regional or general anesthesia. Postpartum tubal ligations are generally performed using a spinal or epidural anesthesia."

A 23 year-old G1P1 delivered her first baby two days ago after an uncomplicated labor and vaginal delivery. She wants to breast feed and has been working with the lactation team. Prior to discharge, her temperature was 100.4°F, 38°C (other vitals were normal). She denies urinary frequency or dysuria and her lochia is mild without odor. On examination, her lungs are clear, cardiac exam normal, and abdomen and uterine fundus are nontender. Her breasts are firm and tender throughout, without erythema and nipples are intact. Which of the following is the most likely cause of her fever? A. Endomyometritis B. Septic pelvic thrombophlebitis C. Mastitis D. Breast engorgement E. Vaginitis

"D Breast engorgement is an exaggerated response to the lymphatic and venous congestion associated with lactation. Milk ""let-down"" generally occurs on postpartum day 2 or 3. If the baby is not feeding well, the breast can become engorged, which can cause a low-grade fever. Lactating women are encouraged to feed their baby frequently, and use a breast pump to prevent painful engorgement and mastitis. Postpartum fever differential includes endometritis, cystitis and mastitis. These are easy distinguished, based on clinical findings. Vaginitis is not accompanied by fever. Septic pelvic thrombophlebitis is a rare condition and characterized by high fever not responsive to antibiotics and is a diagnosis of exclusion."

"A 10-year-old girl has been drawing sexually explicit pictures during art class. Her behavior in class has ranged from being shy to being aggressive with her fellow students. She won't go to the bathroom unless someone goes with her. Physical examination shows a normally developed girl. Breast budding is occurring and she has wisps of axillary and pubic hair. She will not let you ""look at her bottom."" Which of the following is the most likely cause of this condition? A. Normal adolescent development B. Asperger's syndrome C. Precocious puberty D. Child abuse E. Childhood depression"

"D Children who are sexually abused will often ""act out"" and behave inappropriately. There is often no evidence of physical injury and a careful history must be taken. Asperger's syndrome is an autism spectrum disorder in which problems with social interaction occur and may include physical clumsiness and atypical use of language. Symptoms of childhood depression may be similar to those seen in adult depression."

A 33-year-old G2P2 woman reports a two-year history of severe dysmenorrhea, menorrhagia and pelvic pain following the delivery of her last child. She describes her pelvic pain as primarily in the right lower quadrant, radiating into the vagina. Her pain worsens throughout the day with standing and is associated with pelvic pressure and fullness. Her pelvic examination reveals a mildly enlarged uterus with marked tenderness to palpation of the right adnexa, and no other significant findings. A vaginal ultrasound with color-flow Doppler reveals multiple dilated vessels traversing the right broad ligament to the lower uterus and cervix. The uterus shows no fibroids or other significant changes. Endometrial thickness appears normal. Which of the following is the most likely diagnosis in this patient? A. Endometriosis B. Endometritis C. Adenomyosis D. Pelvic congestion E. Pelvic floor relaxation

"D Pelvic congestion syndrome is a cause of chronic pelvic pain occurring in the setting of pelvic varicosities. The unique characteristics of the pelvic veins make them vulnerable to chronic dilatation with stasis leading to vascular congestion. These veins are thin walled and unsupported, with relatively weak attachments between the supporting connective tissue. The cause of pelvic vein congestion is unknown. Hormonal factors contribute to vasodilatation when pelvic veins are exposed to high concentration of estradiol, which inhibits reflex vasoconstriction of vessels, induces uterine enlargement with selective dilatation of ovarian and uterine veins. This pain may be of variable intensity and duration, is worse premenstrually and during pregnancy, and is aggravated by standing, fatigue and coitus. The pain is often described as a pelvic ""fullness"" or ""heaviness,"" which may extend to the vulvar area and legs. Associated symptoms include vaginal discharge, backache and urinary frequency. Menstrual cycle defects and dysmenorrhea are common. No signs of pelvic floor relaxation were noted on exam."

A 22-year-old nulliparous woman presents with five months of amenorrhea since discontinuing her oral contraceptive pills. She had been on the pill for the last six years and had normal menses every 28 days while taking them. She is in good health and not taking any medications. She is 5 feet 4 inches tall and weighs 140 pounds. Her examination, including a pelvic examination, is normal. Which of the following historical elements would be most useful in determining the cause of amenorrhea in this patient? A. Age at first intercourse B. History of sexually transmitted infections C. Parity D. History of oligo-ovulatory cycles E. Recent history of weight loss

"D Since most women resume normal menstrual cycles after discontinuing oral contraceptive pills (OCPs), they are not usually considered the cause of the amenorrhea. A history of irregular cycles prior to pill use may increase the risk of amenorrhea upon discontinuation. This is sometimes referred to as ""post pill amenorrhea."" A complete work-up should be performed to properly find the cause. Although the other historical elements are all important components of a complete gynecological history, they are not helpful to find the etiology of amenorrhea in this patient. Significant weight loss might cause amenorrhea; however, this patient still has normal body mass index, which makes it unlikely cause of amenorrhea."

"A 70-year-old G3P3 woman presents with a four-year history of constant leakage of urine. Her history is significant for abdominal hysterectomy and bilateral salpingo-oophorectomy for endometriosis. She had four anterior repairs in the past for recurrent cystocele. The leakage started six months after her last anterior repair. Pelvic exam showed no evidence of pelvic relaxation. The vagina was well-estrogenized. Q-tip test revealed a fixed, immobile urethra. Cystometrogram showed no evidence of detrusor instability. Cystourethroscopy showed no evidence of any fistula and revealed a ""drain pipe"" urethra. Which of the following is the best first treatment for this patient? A. Retropubic urethropexy B. Needle suspension C. Artificial urethral sphincter D. Urethral bulking procedure E. Sling procedure"

"D This is a classic example of intrinsic sphincteric deficiency. Urethral bulking procedures are minimally invasive and have a success rate of 80% in these specific patients. The success rates for retropubic urethropexies, needle suspension and slings are less than 50%. An ""obstructive or tight"" sling can be performed to increase the success rate, but the voiding difficulties are significant, even requiring prolonged or lifelong self-catheterization. Artificial sphincters should be used in patients as a last resort."

A 34-year-old chronic hypertensive G1 comes to see you for a consultation at 34 weeks for size less than dates. Her prenatal course has been uncomplicated and the genetic amniocentesis obtained at 15 weeks revealed a normal male. Biometrics today reveal a biparietal diameter consistent with 33 weeks, abdominal circumference of 28 weeks, EFW 1600 g, less than 10th percentile, and an amniotic fluid index of 6. What is the most likely cause of fetal growth restriction in this patient? A. Chromosomal abnormality B. Fetal infection with Rubella C. Fetal infection with cytomegalovirus (CMV) D. Uteroplacental insufficiency E. Maternal infection with Varicella

"D Uteroplacental insufficiency can lead to asymmetric growth restriction. Asymmetric growth restricted infants typically have a normal length, but their weight is below normal. On ultrasound, there is a head-sparing effect, meaning that the head/brain is spared of the reduced blood flow that is a result of uteroplacental insufficiency. Thus, the fetal abdomen measures below normal and the head remains very close to normal. There is an asymmetrical growth pattern that is usually detected during the third trimester and reflects uteroplacental insufficiency. Symmetric fetal growth restriction indicates that all fetal measurements are below normal. As a general rule, such a finding indicates an intrinsic growth failure or an ""early event"" secondary to one or more organ system anomalies, fetal aneuploidy or chronic intrauterine infection. Infectious diseases are known to cause IUGR, but the number of organisms is poorly defined. There is sufficient evidence to show a causal relationship between rubella and CMV infections and fetal growth restriction. Other viruses to consider are syphilis and varicella. The protozoan toxoplasmosis results in IUGR as well. There are no bacteria known to cause IUGR. Symmetrical growth restriction is usually detected in the mid-trimester of pregnancy."

A 34-year-old woman wants to take her combined oral contraceptive (OC) for 6 months continuously without having a period. You advise her that the most likely result of her using extended-cycle contraception, ie, taking combined OCs continuously for greater than 28 days, is: (A) amenorrhea (B) inadequate suppression of pituitary and ovarian activity (C) decreased menstrual cycle symptoms (D) decreased adherence

(A) amenorrhea

A 24-year-old woman, gravida 4, para 0. at 29 weeks of gestation presents in preterm labor. Her cervix is 3 cm dilated and 80% effaced. Intact membranes are noted on speculum examination. The fetal heart rote tracing is Category I. You observe no evidence of chorioamnionitis. She was admiued 3 weeks ago for preterm labor and received a course of antenatal steroids. When she asks about further use of antenatal steroids. you tell her that the optimal number of additional courses of steroids is: (A) 1 (B) 2 (C) 3 (D) 4

(A) 1

A 25-year-old woman, gravida 3, para 2, comes in for a routine prenatal visit. Seasonal influenza vaccines have just been released. She is concerned about the risks she has heard about in the media, and wants to avoid any adverse consequences for her and the ferus. In order to maximize the benefit and minimize the risk for the mother and baby, the best recommendation for timing of this intramuscular vaccination is: (A) as soon as possible during this pregnancy (B) during the third trimester of pregnancy (C) immediately after delivery (D) after breastfeeding is completed (E) not indicated for low-risk women during pregnancy

(A) as soon as possible during this pregnancy

A 25-year-old woman, gravida 3, para 2, comes in for a routine prenatal visit. Seasonal influenza vaccines have just been released. She is concerned about the risks she has heard about in the media, and wants to avoid any adverse consequences for her and the fetus. In order to maximize the benefit and minimize the risk for the mother and baby, the best recommendation for timing of this intramuscular vaccination is (A) as soon as possible during this pregnancy (B) during the third trimester of pregnancy (C) immediately after delivery (D) after breastfeeding is completed (E) not indicated for low-risk women during pregnancy

(A) as soon as possible during this pregnancy

A 29-year-old nulligravid woman comes to the office for contraceptive counseling. She is having heavy menstrual bleeding. Her menstrual interval varies from 25 days to 50 days with heavy flow. Her past medical history is significant for mild systemic lupus erythematosus. Her antiphospholipid antibody titers are negative and she does not have evidence of vascular disease or renal disease. She used the levonorgestrel intrauterine device (IUD) but discontinued it because of unwanted adverse effects. Pelvic examination reveals a normal-sized uterus without adnexal masses. Endometrial biopsy results are significant for disordered proliferative endometrium. The best contraceptive choice for her is a: (A) combination oral contraceptive (OC) (B) copper-containing IUD (C) subdermal etonogestrel implant (D) progestin-only OC

(A) combination oral contraceptive (OC)

A 2-year-old girl with normal growth curves aid bone age has developed Tanner 2-3 breasts. Her follicle-stimulating hormone level is 4.0 MIU/mL and estradiol level is less than 20 pg/mL. The best criteria to aid in the diagnosis of premature thelarche is: (A) current age (B) follicle-stimulating hormone level (C) bone age (D) luteinizing hormone level (E) urinary estrogen

(A) current age

A 58-year-old womam, gravida 2, para 2, presents to your office for her annual well-woman examination. She became postmenopauseal at age 51 years and reports no bleeding or spotting since that time. She has a history of well-controlled hypetension. She exercises five times per week and is a nonsmoker. Her physical examination is within normal limits. Her body mass index (weight in kilograms divided by height in meters squared [kg/m^2]) (BMI) is 25. Her speculum examination reveals atrophic vaginal epithelium, normal cervix, and a small mobile uterus with nonpalpable adnexa. She informs you that endometrial cancer was recently diagnosed in her cousin; she has no other significant family history of cancer. She inquires about whether she needs screening for endometrial cancer. The best next step is: (A) expectant management (B) pelvic ultrasonography to evaluate endometrial stripe (C) endometrial biopsy (D) hysteroscopy with dilation and curettage

(A) expectant management

A 20-year-old white female student requests hormonal contraception for the first time. She used a low-dose oral contraceptive for 4 months but returned complaining that her left leg was swollen, red, and painful. An evaluation revealed a venous thromboembolism. The most common genetic abnormality associated with venous thromboembolism is: (A) factor V Leiden mutation (B) prothrombin mutation (C) antithrombin deficiency (D) protein S deficiency (E) protein C deficiency

(A) factor V Leiden mutation

A 30-year-old nulligravid woman presents with secondary amenorrhea and a serum follicle-stimulating hormone level of 112 MIU/mL found on two separate occasions that were weeks apart. Her karyotype is normal. She is contemplating using her 25-year-old sister as an egg donor. Her sister has had no children and one miscarriage. While discussing the process of egg donation with her sister, you note that she has a slight stutter when she articulates and a long prominent jaw. No other abnormalities are noted. Before using the sister as a donor it would be prudent to recommend that she undergo: (A) fragile X screening (B) adrenal antibodies quantification (C) ovarian biopsy (D) transvaginal ultrasonography of the ovaries (E) anti-Mullerian hormone measurement

(A) fragile X screening

A 29-year-old woman, gravida 6, para 0050, has had a positive home pregnancy test result. Laboratory evaluation for her pregnancy losses included normal maternal and paternal karyotypes, hysterosalpingography, and endocrine parameters. She was previously found to have anticardiolipin immunoglobulin G antibodies. Repeat testing 12 weeks later showed her anticardiolipin lgG levels to be highly elevated. You recommend that she take low-dose aspirin 81 mg a day. The next best step in management is: (A) heparin (B) progesterone (C) prednisone (D) intravenous immunoglobulin (E) paternal mononuclear cell immunization

(A) heparin

A postmenopausal 60-year-old woman with estrogen receptor-positive breast cancer comes to see you for consultation. She has been taking tamoxifen citrate for 1 year and is anxious about the risk of developing endometrial cancer. Endometrial polyps have never been diagnosed and she has had no vaginaI bleeding since menopause 10 years ago. The most appropriate next step for this patient is: (A) observation (B) progestin withdrawal test (C) transvaginal ultrasonography (D) pelvic magnetic resonance imaging (E) endometrial biopsy

(A) observation

A 28-year-old primigravid woman at 18 weeks of gestation calls the outpatient office during a busy weekday. She reports increased shortness of breath when climbing more than two flights of stairs. She has a 15-year history of mild persistent asthma, usually well-controlled with a once-daily low-dose inhaled corticosteroid. The most appropriate next step in her management is to: (A) review the results of her regular peak expiratory flow meter testing (B) increase the use of ber rescue short-acting B2,-agonist inhaler (C) add a long-acting B2-agonist inhaler (D) increase the use of her inhaled corticosteroid to twice daily (E) ask her to report immediately to the nearest emergency room

(A) review the results of her regular peak expiratory flow meter testing

A 21-year-old primiparous woman at term has had an uncomplicaied labor. She achieved complete dilation 3 hours ago. She has been pushing with contractions since then and the fetal head has descended to 3 cm below the maternal ischial spines with some caput. The presenting position is left occiput anterior. The fetal heart rate tracing is Category I. Estimated fetal weight is 3,800 g. Her epidural analgesia is functioning well. The patient reports that she is completely exhausted and requests an operative vagjnal delivery. In counseling this patient for operative vaginal delivery by vacuum you should inform her that the most serious complication related to use of this procedure is: (A) subgaleal hemorrhage (B) cephalahomatoma (C) facial palsy (D) skull fracture

(A) subgaleal hemorrhage

A 21-year-old primiparous woman at term has had an uncomplicated labor. She achieved complete dilation 3 hours ago. She has been pushing with contractions since then and the fetal head has descended to 3 cm below the maternal ischial spines with some caput. The presenting position is left occiput anterior. The fetal heart rate tracing is Category I. Estimated fetal weight is 3.800 g. Her epidural analgesia is functioning well. The patient reports that she is completely exhausted and requests on operative vaginal' delivery. In counseling this patient for operative vaginal delivery by vacuum, you should inform her that the most serious complication related to use of this procedure is: (A) subgaleal hemorrhage (B) cephalohematoma (C) facial palsy (D) skull fraction

(A) subgaleal hemorrhage

A 27-year-old primigravid woman comes to you for her first prenatal visit at 9 weeks of gestation. Her mother had a deep vein thrombosis at age 55 years, which prompted thrombophilia testing for first-degree relatives. Your patient's test result reveals that she is heterozygous for the prothrombin G20210A polymorphism. She reports a history of superficial thrombophlebitis but states that she does not have a specific personal history of venous thrombosis. She reports that she would like to avoid needles if at all safely possible. The most appropriate management that addresses her potential risk of venous thrombosis is: (A) surveillance without anticoagulation (B) low-dose aspirin (C) subcutaneous prophylactic heparin (D) subcutaneous adjusted dose heparin

(A) surveillance without anticoagulation

A 31-year-old woman, gravida 3. para 0, comes to the clinic for evaluation of recurrent pregnancy loss. She used an intrauterine device (lUD) for contraception for 6 years because of a previous venous thrombosis episode at age 22 years. After IUD removal. she had three consecutive miscarriages between 5 weeks and 6 wee.ks of gestation. Chromosomal evaluation of the last products of conception yjelded a nonnal XY karyotype. Saline infusion sonohysterography revealed a normal uterine cavity. A fasting glucose test was normal. The best next step to diagnose her condition is: (A) parental karyotype (B) antiphospholipid antibody syndrome testing (C) endometrial biopsy in luteal phase (D) cervical cultures for Chlamydia traclwmatis

(B) antiphospholipid antibody syndrome testing

A 31-year-old woman. gravida 3, comes to your office at 19 weeks of gest:ation with pelvic pressure. She has not experienced contractions or vaginal fluid leak. Her first pregnancy resulted in a term delivery by forceps. A large cervical defect was identified during her initial office visit at 16 weeks of gestation in her second pregnancy. She ultimately gave birth at 21 weeks of gestation secondary to painless cervical dilation. On evaluation of the current pregnancy, the uterus is soft and non-tender. Speculum examination reveals a large cervical defect extending from the external os to the cervical vaginal junction. Copious white vaginal discharge with a pH less than 4.5, multiple white blood cells, and rare clue cells are seen on wet prep but yeast is absent. Her cecvix is long, closed, soft, and in mid position. Transvaginal ultrasonography shows cervical funneling with a cecvical length of 2.1 cm. The condition that would most likely explain these findings is: (A) bacterial vaginosis (B) cervical insufficiency (C) preterm labor (D) uterine anomaly (E) intrauterine infection

(B) cervical insufficiency

An 18-year-old woman, gravida I, para 0, at 41 weeks of gestation has had rupn,re of membranes for approximately 17 hours. She is in early labor with contractions every 5 minutes with cervical dilation at 5-6 cm. She received an epidural for pain relief 12 hours ago. Her temperature is 38.5° C (101°F). Her pulse is 124 beats per minute (bpm) and the fetal heart rate is 175 bpm with accelerations. She does not have any shortness of breath, dyspnea, or headaches. Her physical examination is notable for fundal tenderness. OD admission, her white blood cell count was 18,000 cells/mm3 and her urinalysis had negative leukocyteesterase and nitrites. The mosl likely cause of her fever is: (A) pyelonephritis (B) chorioamnionitis (C) pneumonia (D) epidural fever (E) meningitis

(B) chorioamnionitis

A 25-year-old nulligravid woman presents with excessive hair growth on her chin and chest over the past 2 years. She has irregular menstrual cycles. Her body mass index (weight in kilograms divided by height in meters squared [kg/m^2] is 35. She is sexually active but does not desire pregnancy. On physical examination, dark hair growth is noted on the chin, lip, and chest On pelvic examination, the external genitalia appear normal and the uterus is normal in size without adnexal masses. Her total testosterone level is 85 ng/dL, prolactin level is 100 ng/dL, and moring basal 17a-hydroxyprogesterone level is 100 ng/dL. She desires treatment of the excessive hair growth. The next step is: (A) eflornithine hydrochloride (B) combination contraceptives (C) flutamide (D) metformin (E) spironolactone

(B) combination contraceptives

A healthy 20-year-old woman has a 3-year history of dysmenorrhea with pain that is moderately severe during the first 3 days of her 5-day menses. Ibuprofen provides some relief. Her menstrual cycles are regular. Pelvic ultrasonography and bimanual examination results are negative. She would like to attempt pregnancy in 9 months. She requests a treatment to decrease menstrual bleeding, improve pain, and provide reliable contraception. You tell her that the best management option for her is: (A) cyclical oral contraceptives (OC.) (B) continuous or long-cycle OCs (C) depot leuprolide acetate (Lupron) (D) depot medroxyprogesterone acetate

(B) continuous or long-cycle OCs

A 32-year-old woman, gravida 2, para 1, with a last menstrual period 7 weeks ago, presents with abdominal pain and vaginal spotting for 2 days. Pelvic examination demonstrates a small uterus and mild right adnexal tenderness. Her human chorionic gonadotropin (b-hCG) titer is 4,000 mIU/mLl. Transvaginal ultrasonography reveals the image shown in Figure 129-1. The most likely pregnancy diagnosis is: (A) angular (B) interstitial (C) intrauterine (D) ovarian

(B) interstitial

A 28-year-old woman, gravida 2, para I, comes to the office for a routine prenatal visit at 38 weeks of gestation. She started her pregnancy at a normal weight for her height and has gained 8.2 kg (18 lb) to date. Her first child was delivered vaginally at 39 weeks of gestation and weighed 3,750 g. Her I-hour glucose tolerance test is normal. Her blood pressure is I 10no mm Hg with a normal urinalysis. On physical examination, the fundal height is 39 cm and Leopold maneuvers suggest an infant in cephalic presentation that weights appro:ximately 3,860g (8.5 lb). The fetal heart rate is 130 beats per minute. Her cervix is closed, long, and firm. Ultrasonography shows a singleton pregnancy in cephalic presentation with ru1 estimated fetal weight of 4,110 g (9 lb). The best next step in mru1agement is: (A) cesarean delivery at 39 weeks (B) expectant management (C) immediate cesarean delivery (D) immediate induction of labor (E) induction of labor at 39 weeks

(B) expectant management

A 27-year-old primagravid woman comes to your office for her first prenatal visit. She has no significant medical history, such as diabetes mellitus, medication exposure, or fammily history' of congenital anomalies. Her husband has undergone surgical correction of an atrioventricular septal defect with no subsequent problems and no other affected family members are reported. After counseling, the mmost appropriate management related to this history is: (A) amniocentesis (B) fetal echocardiography (C) nuchal translucency measurement (D) paternal microarray testing (E) second-trimester ultrasonography

(B) fetal echocardiography

A 32-year-old woman is preparing for infertility treatment. You counsell her that she can reduce her risk of having a fetus with a neural tube defect (NTI) by taking a supplement that contains the micronutrient: (A) thiamine (B) folic acid (C) pyridoxine (D) beta carotene

(B) folic acid

A 17-year-old nulligravid adolescent female with normal secondary sexual chnrncteristics and rcsular predictable spontaneous menstrual periods has experienccd 6 months of secondary amenorrhea, vaginal dryness, and occasional hot flushes. The best screening test to diagnose her condition is to test for: (A) luetinizing hormone (B) follicle stimulating hormone (C) thyroid stimulating hormone (D) estradiol (E) karyotype

(B) follicle stimulating hormone

A 26-year-old woman, gravida 1, para 0, aborta 1, comes to your office with a 6-month history of amenorrhea and night sweats. She reports menarche at age 12 years and regular menses until approximately 9 months ago. She is sexually active and does not use contraception. She has no significant past medical history and her family history is significant only for her mother, with premature menopause and a cousin with mental retardation. On examination, she has Tanner V breast and pubic hair. A pelvic examination shows no significant abnormalities. Her urine pregnancy test result is negative. Thyroid-stimulating hormone and prolactin Ievels are normal, and follicle-stimulating hormone (FSH) level is 103 mIU/mL. On repeat testing, her FSH level is 111 mIU/mL. You order a karyotype, which is 46,XX. The step that will most likely determine the etiology of this patient's premature ovarian failure is: (A) ovarian autoantibody testing (B) fragile X mental retardation (FMRI) premutation testing (C) adrenal autoantibody testing (D) ovarian biopsy

(B) fragile X mental retardation (FMRI) premutation testing

A patient comes to your office at 28 weeks of gestation for lher I-hour glucose tolerance test. A complete blood count is done at the same time. Her platelet count is 95,000/mm3. A repeat platelet count 6 weeks later is 92,000/mm 3• She currently feels well wuth no recent illnesses. She is healthy and takes only a prenatal vitamin. Her initial prenatal laboratory results show a platelet count of 287,000/mm3. She has normal lblood pressure ,u1d her urine dipstick test result is negative for protein. The most likely diagnosis is: (A) preeclampsia (B) gestational thrombocytopenia (C) idiopathic thrombocytopenia purpura (D) drug-induced thrombocytopenia (E) thrombocytopenia due to a viral illness

(B) gestational thrombocytopenia

A 28-year-old nulligravid woman comes in for an infertility evaluation. She was treated for a brain tumor with cranial radiation and chemotherapy 3 years ago. She is currently receiving thyroid replacement therapy and she has no menstrual bleeding or spontaneous cycle without the use of oral contraceptives or hormone therapy. Physical examination reveals vulvovaginal atrophy but she is otherwise normal. Magnetic resonance imaging (MRI) of the brain is negative. Follicle-stimulting hormone (FSH) level is 1.0 MIU/mL; luetenizing hormone level, 1.5 MIU/mL; estradiol level 18 pg/mL; thyroid-stimulating hormone level, 1.5 microunits/mL; and prolactin level, 15 ng/mL. Results or pelvic ultrasonography, hysterosalpingography, and semen analysis are normal. The most appropriate medical management to induce ovulation in this patient is: (A) clomiphene citrate (Clomid. Serophene) (B) gonadotropins (C) metformin (Glucophage) (D) gonadotropin-releasing hormone (GnRH) agonist (Lupron)

(B) gonadotropins

A 24-year-old woman is to undergo a cesarean delivery. Her obstetrician discusses the purpose of the surgery, risks, benefits, and answers her questions. The hospital consent document is signed and witnessed and the patient is taken to the operating room where a colleague will do the cesarean delivery. (A) adequate (B) inadequate

(B) inadequate

A 34-year-old woman has requested that she be prescribed an intrauterine device (IUD). The gynecologist asks the nurse to have the patient sign a consent form and then re-enters and immediately proceeds to insert the IUD. (A) adequate (B) inadequate

(B) inadequate

A 47-year-old woman is in the preoperative care unit before getting a hysterectomy. The attending gynecologist, who is on his way to the hospital, instructs the resident to obtain the informed consent to proceed with the operation. (A) adequate (B) inadequate

(B) inadequate

A 32-year-old woman gravida 1, para 1, calls your office to report decreased milk supply. She had an uncomplicated spontaneous vaginal delivery 4 months ago. Her goal is to exclusively breastfeed for the first 6 months and to continue to breastfeed for the first year. Her infant feeds every 4-6 hours and has had excellent weight gain. She reports no nipple soreness, irritation, or erythema. She returned to work after 12 weeks and has been pumping every 6-8 hours at work. She is concerned because the volume of milk she is pumping has been steadily decreasing, and she will soon be unable to keep up with the breastfeeding needs of her child. The most appropriate next step in management is: (A) no change to current regimen (B) increase frequency of pumping (C) metoclopramide hydrochloride (D) domperidone (E) fenugreek (Trigonella foenum graecum) seed

(B) increase frequency of pumping

A 29-year-old nurse at 10 weeks of gestation comes to your office for consultation after her annual intermediate strength purified protein derivative (PPD) skin test resultl was reported psoitive. A prior PPD skin test result had been negative 18 months ago. The patient reports no current health problems and does not have any cough or weight loss. She has experienced some nausea and fatigue since becoming pregnant. A chest x-ray performed with shielding was negative. The mostl appropriate medication for this patient is: (A) ethambutol (B) isoniazid (C) pyrazinamide (D) rifampin (E) streptomycin

(B) isoniazid

A 30-year-old nonpregnant woman comes to your office and reports worsening intermittent right lower quadrant pain. She was seen 2 days earlier in the emergency room. At the time, she had stable vital signs, no fever, and normal hemoglobin and white blood cell counts. An ultrasonographic scan demonstrated a 6-cm amorphous appearing right ovary with a 4-cm hemorrhagic appearing cyst and no free fluid in the cul-de-sac. Normal arterial blood flow to the right ovary also was noted. On discharge, she was prescribed an oral narcotic analgesic and an oral contraceptive. Her pain remains intermittent, but is of longer duration and more frequent. It also is associated with abrupt nausea and occasional vomiting. On physical examination, you note an uncomfortable appearing woman who is afrebile with a blood pressure of 120/70 mm Hg and a pulse of 110 beats per minute. Abdominal examination reveals tenderness in the right lower quadrant without rebound. Bimanual examination demonstrates a tender enlarged right adnexa. Repeat ultrasonographic studies provide results unchanged from those done in the emergency room. A pregnancy test result was negative. The most appropriate next step in her management is: (A) magnetic resonance imaging (MRI) (B) laparoscopy (C) computerized tomography (CT) scan (D) ultrasonographic-guided needle aspiration

(B) laparoscopy

A 27-year-old primigravid kindergarten teacher calls the office at 16 weeks af gestation because she is worried about exposure to a child with a rash. She reports that fifth disease was diagnosed in one of her pupils. She is asymptomatic. The most appropriate next step in management would be: (A) no further testing (B) maternal antibodies for parvovirus B19 (C) weekly middle cerebral artery Doppler measumuents (D) amniocentesis for polymerase chain reaction for parvovirus B19 (E) perrutaneous umbilical blood sampling

(B) maternal antibodies for parvovirus B19

A 59-year-old woman presents to your office with vaginal bleeding, She reports three episodes of spotting over the past month: the bleeding has now resolved. The patient's primary care physician obtained routine laboratory studies, which revealed a hematocrit level of 37%, normal serum chemistry values, and a normnl thyroid-stimulating hormone level. Transvaginal ultrasonography demonstrates a mildly enlarged uterus with a 3.7-cm submucosal leiomyoma and a 4.2-cm intramural leiomyoma. The endometrial stripe measures 7 mm. On examination, she has a 12-week gestation-sized mobile uierus with no palpable adnexal masses. The most appropriate next step in the patients management is: (A) observation (B) office endometrial biopsy (C) endometrial ablation (D) hysteroscopy with resection of the submucosal leiomyoma (E) hysterectomy

(B) office endometrial biopsy

An 18-year-old nulligravid woman whose last menstrual period was 3 days ago, presents with a 3-day history of lower abdominal pain. She is sexually active and uses condoms intermittently. She says she has neither nausea nor vomiting. Examination reveals a temperature of 39.4 C (100.3 F), cervical motion tenderness, unilateral adnexal tendemess, and a mucopurulent cervicitis. Her urinary human chorionic gonadotropin test result is negative and white blood cell (WBC) count is 9,500 cells per mm^3. Testing for gonorrhea and chlamydial infection was sent and is pending. The appropriate next step in her management is: (A) diagnostic laparoscopy (B) outpatient antibiotics (C) inpatient parenteral antibiotics (D) nonsteroidal anti-inflammatory drugs

(B) outpatient antibiotics

A 30-year-old woman with normal menstrual cycles tells you that she has experienced left-sided pelvic pain for the past 2 months. The pain is occasionally exacerbated by intercourse, but she mostly feels comfortable and reports sexual satisfaction. She is concerned about malignancy. On examination, you clinically palpate a normal-sized and normal-shaped uterus and a 5-cm left adnexal mass that is cystic, nontender, and mobile. The patient's right adnexa is normal. The best next step in the evaluation of this patient is: (A) computed tomography (CT) scan of the pelvis (B) pelvic ultrasonography (C) CA 125 (D) magnetic resonance imaging (MRI) of the pelvis (E) positron emission tomography scan

(B) pelvic ultrasonography

A 27-year-old multiparous woman visits your office with irregular vaginal bleeding and a positive home pregnancy test result. Her last menstrual period was 5 weeks ago and she states that her menses have been irregular since giving birth 2 years ago. She is sexually active and has been trying to conceive. In recent weeks, she has had occasional mild diffuse pelvic pain. Examination reveals a heart rate of 94 beats per minute and blood pressure of 100/60 mm Hg. Her abdomen is soft and non tender with no guarding or rebound. Her speculum examination reveals scant brown blood in the vaginal vault and a closed cervix. On bimanual examination, the patient has a small, anteverted uterus that is minimally tender on examination. Transvaginal ultrasonography shows a uterus with 1-cm endometrial stripe and no gestational sac, no adenexal masses, and no free fluid in the culde-sac. Laboratory results include a quantitative b-hCG level of 1900 mIU/mL and a hemoglobin (Hgb) level of 11.5 g/dL. The most appropriate next step in her management is to: (A) repea ultrasonogtaphy in 1 week (B) repeat B-hCG in 48 hours (C) prescribe methotrexate (D) perfom a dilation and cutterage (E) evaluate with a diagnostic laparoscopy

(B) repeat B-hCG in 48 hours

A 21-year-old nulligravid woman is referred to your office with high-grade cervical intraepithelial neoplasia (CIN) on cytologic screening. Colposcopy is adequate and directed biopsies reveal moderate dysplasia (CIN 2) with negative endocervical curettage. She is interested in future childbearing. In this patient, the preferred management is: (A) repeat cytologic testing in 3 months (B) repeat cytologic testing and colposcopy in 6 months (C) cryotherapy (D) loop electrosurgical excision procedure (LEEP) (E) cold-knife conization

(B) repeat cytologic testing and colposcopy in 6 months

A 33-year-old woman, gravida 3, para 3, with poorly controlled type I Diabetes Mellitus, reports vulvovaginal itching and white discharge. Two weeks ago, you diagnosed her with vulvovaginal candidiasis and treated her with one dose of oral fluconazole. After an examination today, you diagnose vulvovaginal candicliasis again, and you note new fissures. For treatment of her complicated vulvovaginal candidiasis, you recommend one 7-day course of topical azolee followed by oral fluconazole for: (A) one day (B) once a day for 3 days (C) once a day for a week (D) once weekly for several months

(D) once weekly for several months

A 32-year-old woman, gravida 2, para 2, comes to your clinic 5 days post elective repeat cesarean delivery with bloody discharge from her wound. Her postoperative course in the hospital was unremarkable. On physical examination, her oral temperature was 37*C (98.6 F) and removal of the surgical staples from the incision revealed a hematoma. The rectus fascia was noted to be intact and no skin erythema or purulent material was noted in the wound. The hematoma was drained and the wound was packed with saline-soaked gauze. Wet to dry dressings were continued at home twice a day. The patient returned to the office 3 days later and the wound was noted to have granulation tissue without evidence of infection. The best next step in management is: (A) continuation of current regimen (B) secondary wound closure (C) changing to iodophor gauze dressings (D) negative pressure wound vaccum therapy

(B) secondary wound closure

A 21-year-old nulligravid woman has a history of mood irritability noted in the 5 days preceding menses for the past 6 months. Her moods interfere with relationships at home and work, and she withdraws from contact with others during these times of mood change. She began an exercise program and calcium supplementation but her symptoms persisted and, therefore, she was started on oral contraceptives (OCs). Despite use of OCs for 3 months, her symptoms have not improved. The best next step in treatment should be: (A) alprazolam (B) selective serotonin reuptake inhibitors (SSRis) (C) leuprolide acetate (D) progestins

(B) selective serotonin reuptake inhibitors (SSRis)

A 38-year-old woman, gravida 4, para 2, is seen at 29 weeks of gestation for generalized intense itching for the past week. The itching is worst on her hands and feet. She does not report any infectious contacts or new skin care products. Her temperature is 37 C (98.6°F), blood pressure is 135/85 mm Hg, and pulse is 90 beats per minute. She appears in no acute distress. Her sclerae are nonicteric and her oropharynx is normal. She has no rash. Her cervix is closed and long. The fetal heart rate is 135 beats per minute and reassuring. Urine dipstick test result is negative for protein. Laboratory evaluation reveals her white blood count is 9.0 x 10^9/L, hematocrit is 33%, and platelet count is 240,000/mm^3. Metabolic panel shows the serum alanine transaminase level is 55 units/L and the total bilirubin level is 0.7 mg/dL. The most appropriate diagnostic test is: (A) serum alkaline phosphatase (B) serum ammonia (C) serum bile acid levels (D) serum glucose

(B) serum ammonia

A 25-year-old woman presents with 3 months of daily vaginal bleeding following an uncomplicated term delivery. She is not breastfeeding. She generally feels well, but tires easily. On pelvic examination, the cervix and uterus are normal with no blood in the cervical os. However, she has a violaceous, friable, 2-cm anterior vaginal nodule. The most approprinie next step in evaluation is: (A) biopsy of the suburethral nodule (B) serum b-hCG assessment (C) endometrial biopsy (0) pelvic ultrasonography (E) laparoscopy

(B) serum b-hCG assessment

A 30-year-old nullgravld woman presents with secondary amenorrhea and spontaneous bilateral galactorrhea for the past 3 months. Her pregnancy test result is negative, thyroid-stimulating hormone level is 1.8 micro units/ml, and two prolactin values are 110 and 123 ng/ml, respectively. Pituitary magnetic resonance imaging demonstrates a 13-mm adenoma. The next step in her evaluatlon is: (A) cone-down X-ray of sella turcica (B) serum insulin-like growth factor-1 (IGF-1) (C) serum follicle-stimulating hormone (D) digital mammogram (E) neurosurgeon consultation

(B) serum insulin-like growth factor-1 (IGF-1)

A 29-year-old woman, gravida 2, para 0, presents at 37 weeks of gestation with decreased fetal movement for 8 hours. The pregnancy has been uncomplicated to date. The nonstress test is reactive, but several brief variable decelerations are present. In order to assess her amniotic fluid (AF) volume. the most appropriate test is: (A) AF index (B) single deepest pocket (C) three-dimensional ultrasonography (D) two-diameter poc.ket technique

(B) single deepest pocket

A 33-year-old multiparous woman with no reported medical problems tells you that she had regular heavy monthly menses until 4 months ago. Since then, her menses have become unpredictable and more frequent, and have been accompanied by increased cramping. Her general physical examination is unremarkable. On pelvic examination, you observe no vaginal bleeding and the cervix appears normal. Bimanual palpation reveals a non-tender anteverted 8-week-gestational-sized uterus with no identified adnexal masses or tenderness. A pregnancy test is negative and laboratory studies are unremarkable. The best next step for evaluating this patient is: (A) transvaginal ultrasonography (B) sonohysterography (C) computerized tomography scan (D) magnetic resonance imaging

(B) sonohysterography

An 85-year-old woman, gravida 4, para 4 comes to your office for treatment of a bothersome vaginal bulge and accompanying urinary retention. Her medical history is significant for hypertension, coronary artery disease, and emphysema, for which she requires supplemental oxygen at home. Her prior surgical procedures include vaginal hysterectomy at age 45 years and coronary artery bypass grafting 5 years ago. On pelvic examination, she has significant anterior wall and apical prolapse with minimal posterior wall prolapse. Results of a pelvic organ prolapse quantification examination (Figs. 91-1 and 91-2) are as follows: Aa +I, Ba +l.Ap-2.Bp-2. D-3, total vaginal length 8 cm. genital hiatus 3 cm. perineal body 3 cm. consistent with Stage III pelvic organ prolapse. When the anterior prolapse is reduced in the office, she has no incontinence. She desires to retain coital ability. The most appropriate next step in managing this patient is: (A) observation (B) support pessary (C) suburethral sling procedure (D) anterior repair (E) colposacropexy

(B) support pessary

An 85-year-old woman, gravida 4, para 4, comes to your office for treatment of a bothersome vaginal bulge and accompanying urinary retention. Her medical history is significant for hypertension, coronary artery disease, and emphysema, for which she requires supplemental oxygen at home. Her prior surgical procedures include vaginal hysterectomy at age 45 years and coronary artery bypass grafting 5 years ago. On pelvic examination, she has significant anterior wall and apical prolapse, with minimal posterior wall prolapse. Results of a pelvic organ prolapse quantification examination (Figs. 91-1 and 91-2) are as follows: Aa +I, Ba +1, Ap-2, Bp-2, D-3, total vaginal length 8 cm, genitaI hiatus 3 cm, perineal body 3 cm, consistent with Stage III pelvic organ prolapse. When the anterior prolapse is reduced in the office, she has no incontinence. She desires to retain coital ability. The most appropriate next step in managing this patient is: (A) observation (B) support pessary (C) suburethral sling procedure (D) anterior repair (E) colposacropexy

(B) support pessary

A 35-year-old woman, gravida 2, para 2, with a history of polycyslic ovary syndrome and thyroid disease who is nonadherent with medical therapy comes in for treatment of a Bartholin's abscess. She reports a recent history of fever, palpilalions, and diarrhea. The palient was treated with oral penicillin and acetaminophen (tylenol) with codeine for pain before her visit. After injection of the local anestheslic (lidocaine hydrochloride 10 cc 1%), the patient becomes restless, tachycardic, and develops seizures. Her vital signs show a temperature of 38.1 C (101.3 F). The most likely underlying palhophysiology responsible for this presentation is: (A) lidocaine hydrochloride toxicity (B) thyrotoxic crisis (C) sepsis (D) cerobrovascular accident (E) hyperandrogenemia

(B) thyrotoxic crisis

A 50-year-old nulligravid woman has a maternal grandmother with postmenopausal uterine cancer. She is very concerned about ovarian cancer and wants to discuss implementing ovarian cancer screening. ln her situation, the most appropriate management choice is pelvic: (A) examination and education about ovarian cancer symptoms (B) ultrasooography and CA 125 (C) computed tomography (CT) scan and CA 125 (D) magnetic resonance imaging (MRI) scan and CA 125

(B) ultrasooography and CA 125

A 30-year-old patient with infertility has just completed a cycle of in vitro fertilization (IVF) utilizing leuprolide acetate (Lupron) for prevention or a luteinizing hormone (LH) surge. A total of 20 oocytes, were retrieved and a single blastocyst was transferred. Her husband travels frequently and is currently out or town for 2 weeks. You advise her that her best choice for luteal phase support may be: (A) oral micronized progesterone (B) vaginal progesterone (C) human chorionic gonadotropin (hCG) (D) intramuscular progesterone

(B) vaginal progesterone

A 30-year-old woman, gravida 5, para 4, comes to the hospital at 35 weeks of gestation with decreased fetal movement. Her prior pregnancies were uncomplicated and she has no medical problems. Midtrimester anatomy ultrasonography showed choroid plexus cyst and fetal heart defect. She declined genetic testing and her prenatal laboratory results were otherwise normal. Fetal monitoring and ultrasonograph reveal absent fetal heart beat and a diagnosis of fetal demise is made. Her vital signs are normal including a blood pressure reading of 110/70 mmHg. The test(s) most litely to be informative regarding the cause of the stillbirth is (are): (A) maternal protein S and protein C levels (B) hemoglolin A1c (C) amniocentesis for karyotype (D) complele blood count with platelets and urinalysis for protein and Liver enzymes (E) titers for toxoplasmosis, other viruses, rubella, cytomegalovirus, herpes simplex viruses

(C) amniocentesis for karyotype

A 26-year-old primigravid woman comes to your office for counseling at 25 weeks of gestation. Her hepatitis C virus (HCV) antibody titer is positive and her human immunodeficiency virus (HIV) titer is negative. The pregnancy is uncomplicated. To minimize the risk of transmission of HCV to her fetus you recommend: (A) cesarean delivery (B) abstention from breastfeeding (C) avoidance of fetal scalp electrode (D) induction of labor at 39 weeks

(C) avoidance of fetal scalp electrode

A 19-year-old woman visits your office for contraceptive counseling and health maintenance. She received the quadrivalent human papillomavirus (HPV) vaccine at age 16 years, just before beginning sexual activity at age 17 years. She has had three sexual partners. According to the most recent American College of Obstetricians and Gynecologists guidelines on cervical cytologic screening, the best screening recommendation for this patient is that screening should: (A) begin now (B) begin 3 years after the onset of sexual activity (C) begin at age 21 years (D) use liquid-based cytology

(C) begin at age 21 years

A 60-year-old white woman visits your clinic to discuss her risk of osteoporosis. Her 88-year-old mother recently fell and fractured her hip. The patient has mild hypertension for which she takes lisinopril. She smokes one pack of cigarettes a day and drinks two to three glasses of wine every evening. She tries to stay active but does not exercise regularly. Two Winters ago, she slipped on ice and broke her wrist. She used hormone therapy after menopause at age 51 years but stopped after 6 months because of perceived weight gain. On physical examination, she has a blood pressure of 145/92 mm Hg. She has a height of 1.6 m (63 in), weight of 67.1 kg (148 lb), and a body mass index (weight in kilogram divided by height in meters squared [kg/m^2]) (BMI) of 26.3. A recent dual-energy X-ray absorptiometry scan revealed a T-score of -1.2. According to the FRAX tool (the World Health Organization's online Fracture Risk Assessment Tool), her 10-year probability of major osteoporotic fracture is 24%, and her risk of hip fracture is 1.6%. The most effective next step is to prescribe therapy with: (A) raloxifene (B) calcium and exercise (C) bisphosphates (D) calcitonin (E) vitamin D

(C) bisphosphates

A 34-year-old married woman, gravida 3, para 3, comes to your office for an annual examination. She has no significant past medical history and is a nonsmoker. She had a negative Pap test 2 years ago during routine prenatal care. You order human papillomavirus (HPV) testing along with cervical cytology. The cytology test results are negative for intraepithelial neoplasia, but the HPV test result is positive for high-risk HPV subtypes. The best next step in management is: (A) cervical cytology in 3 years (B) HPV vaccine (C) cervical cytology and HPV testing in 1 year (D) colposcopy

(C) cervical cytology and HPV testing in 1 year

A 22-year-old woman transfers to your practice at 36-weeks of gestation. On initial physical examination, you notice vulvar and perianal condyloma accuminata (Fig. 61-1; see color plate). Multiple large warts in tile posterior fourchette extend to the hymenal ring and fill the vestibule to the level of the urethral meatus. In addition, you observe moderate size lesions just inside the vagina. Multiple confluent warts are noted along the anal verge. The total volume of condyloma is approximately 12.0 cm^2. She states that the warts have grown in size over the past month. The warts are pruritic and bleed easily even with gentle touch and wiping.. The most appropriate management option for her is : (A) imiquimod (B) podophyllin (C) cesarean delivery with treatment postpartum (D) vaginal delivery with treatment postpartum (E) laser therapy

(C) cesarean delivery with treatment postpartum

A 17-year-old nulligravid woman, comes to you for an annual examination and to refill her oral contraceptives. She has recently noted vaginal discharge. She has been sexually active for 3 years. She uses a combination oral contraceptive regularly but is inconsistent with condom use. Her most recent Pap test was at age 15 years. In addition to discussing human papillomavirus (HPV) vaccination with her, the test that her gynecologic evaluation should include is: (A) cervical HPV DNA testing (B) cervical cytology screening (C) chlamydial infection testing (D) colposcopy

(C) chlamydial infection testing

A 28-year-old nulligravid woman comes to your office for evaluation and treatment of infertility. Polycystic ovary syndrome (PCOS) was diagnosed when she was an adolescent and she was started on oral contraceptives (OCs) at age 18 years. Her menstrual cycles were regular while taking OC's and her mild hirsutism resolved. She stopped taking the OCs 18 months ago in order to become pregnant. She experiences infrequent menses (one every 2-3 months) and has not been able to conceive. She has no other significant medical problems, and her body mass index (weight in kilograms divided by height in meters squared [kg/m^2]) (BMI) is 23.4. Her thyroid-stimulating hormone and prolactin levels are normal. Hysterosalpingography reveals bilaterally patent fallopian tubes, and her husband had a normal semen analysis. You advise her that the best next step to achieve a liveborn child would be: (A) cyclic progestin (B) metformin hydrochloride (C) clomiphene citrate (D) in vitro fertilization (IVF)

(C) clomiphene citrate

A 31-year-old infertile woman has menses every 45-120 days and reports excessive facial hair and acne. She has a normal hysterosalpingogram and her male partner has a normal semen analysis. The medication that is most likely to lead to ovulation and pregnancy is: (A) letrozole (Femara) (B) metformin hydrochloride (Glucophage) (C) clomiphene citrate (Clomid, Serophene) (D) dexamethasone (E) bromocriptine mesylate (Pai·lodel)

(C) clomiphene citrate (Clomid, Serophene)

A 25-year-old nulligravid white woman comes to your office for contraceptive counseling. She has been using depot medroxyprogesterone acetate (DMPA) for the past 2 years. She is amenorrheic and would like to continue this method of contraception. Her body mass index (weight in kilograms divided by height in meters squared [kg/m^2]) (BMI) is 24, she smokes 1 pack of cigarettes per day, and does not take calcium supplementation. In addition to smoking cessation counseling and calcium supplementation, you recommend that she: (A) have bone mineral density (BMD) testing (B) add low-dose estrogen (C) continue DMPA (D) have urine testing to assess for bone loss

(C) continue DMPA

A 56-year-old woman visits your office for nipple discharge. She reports spontaneous, serosanguinous discharge from her left nipple for the past 3 weeks. She has no family history of breast cancer, and routine screening mammography was negative 4 months ago. On examination, she has no palpable mass or enlarged lymph nodes. You are able to express a small amount of blood-tinged discharge from a single duct on the left nipple, which you send for cytology. The report from the cytology is negative for malignancy. The next seep in her evaluation is: (A) no further testing (B) prolactin (C) diagnostic mammography (D) breast magnetic resonance imaging (E) ductography

(C) diagnostic mammography

A 60-year-old woman who has had continuous, combination estrogen-progestin treatment for 8 years presents with new-onset vaginal bleeding. Transvaginal ultrasonography reveals no evidence of leiomyoma, large polyps, or adnexal pathology. The endometrial stripe measures 6 mm and is homogenous. The most appropriate next step in evaluation is: (A) observation (B) sonohysterography (C) endometrial biopsy (D) dilation and curretage (E) hysteroscopy

(C) endometrial biopsy

A 34-year-old woman underwent a computed tomography (CT) scan of her abdomen and pelvis for vague abdominal pain. The only abnormality noted was an ovarian cyst. Transvaginal ullrasonography confirms a 4-cm unilocular ovarian cyst. The patient has no gynecologic symptoms and has never received hormone therapy. The next step in the management is assessment of: (A) CA 125 (B) inhibin B (C) human choriooic gonadolropin (hCG) (D) lactate dehydrogenase (E) carcinoembryonic antigen

(C) human choriooic gonadolropin (hCG)

A 26-year-old nulligravid woman comes to your office for evaluation and management of infertility. She reports no significant medical problems and has never had surgery. Her menstrual cycles are every 28 days, with mild flow: and bleeding that generally lasts for 5 days. She has a past history of chlamydial infection, but no history of other sexually transmitted diseases or gynecologic concerns. She has been ovulating according to an over-the-counter ovulation predictor kit as well as a basal body temperature chart. She and her husband have engaged in timed intercourse for the past year without pregnancy. Her husband underwent semen analysis, which was normal. The most appropriate next step in managing this patient is: (A) clomiphene citrate (B) intrauterine insemination (C) hysterosalpingography (D) sonohysterography (E) hysteroscopy

(C) hysterosalpingography

A 31-year-old woman, gravida 2, aborta 2, comes for consultation regarding secondary infertility. She reports a 12-month history of increasing amenorrhea and intermenstrual spotting for the past 6 months with less regular periods. Pelvic examination reveals a nontender uterus the size of an 8-week-gestation. Transvaginal ultrasonography reveals a thickened irregular endometrial stripe and a 2-cm submucosal leiomyoma (Fig. 39.1). Laboratory studies reveal a hemoglobin level of 11 g/dL and normaI prolactin and thyroid-stimulating hormone levels. A sonohysterogram performed on cycle day 8 reveals a 1.8 cm intracavity leiomyoma (Fig. 39-2; see color plate). The best next step in management is: (A) clomiphine citrate (Clomid, Serophene) (B) intrauterine insemination (C) hysteroscopic myomectomy (D) in vitro fertilization and embryo transfer (E) ovulation induction with gonadotropins

(C) hysteroscopic myomectomy

A pregnant patient at 8 weeks of gestation comes to your office during the influenza season andvinquires about influenza vaccination. She would like to receive the vaccine if you feel it is safe for her to receive it. She is afraid of injections and prefers the nasaI spray administration route. As her physician you inform her that she should receive: (A) nasal spray vaccine after 12 weeks of gestation (B) injectable vaccine after 12 weeks of gestation (C) injectable vaccine today (D) nasal spray vaccine today (E) no vaccine

(C) injectable vaccine today

A 40-year-old woman comes to your office for prenatal care at 10 weeks of gestation. She desires information regarding a possible chromosome abnormality in her fetus but wishes to avoid invasive testing if possible. You counsel her that the screening test with the highest detection rate and the lowest false positive rate for Down syndrome is: (A) quadruple screening (B) first-trimester combined screening (C) integrated screening (D) sequential scree11ing (E) targeted ultrasonographic survey

(C) integrated screening

A 33-year-old woman, gravida 3, para 2, aborta 1, requests long-term contraception. She has completed childbearing. She has a past medical history significant for a pulmonary embolus after the birth of her youngest child 5 years ago. Past surgical history is significant for a salpingectomy for an ectopic pregnancy, and a sigmoid colon resection for colon cancer. Her body mass index (weight in kilograms divided by height in meters squared [kg/m^2]) (BMI) is 40 and her pelvic examination is normal. The best contraceptive option for her is: (A) combination oral contraceptive (OC) (B) hysteroscopic tubal occlusion (C) intrauterine device (D) laparoscopic tubal occlusion (E) subdermal etonogestrel implant

(C) intrauterine device

A 32-year-old primigravid woman at 16 weeks of gestation was prescribed nitrofurantoin for asymptomatic bacturiuria. You tell her that in addition to possible progression to cystitis and pyelonephritis, another pregnancy complication associated with untreated asymptomatic bacteremia is: (A) maternal anemia (B) neonatal acute respiratory distress syndrome (C) low birth weight (D) preeclampsia

(C) low birth weight

A 30-year-old nulligravid woman has been trying to conceive for the past year. She is unsure as to when she is ovulating and would like to know how she can best detect ovulation. The most reliable indicator of impending ovulation is an increase in (A) estradiol (B) follicle-stimulating hormone (C) luteinizing hormone (D) progesterone (E) basal body temperature

(C) luteinizing hormone

A 30-year-old nulligravid woman has been trying to conceive for the past year. She is unsure as to when she is ovulating and would like to know how she can best detect ovulation. The most reliable indicator of impending ovulation is an increase in: (A) estradiol (B) follicle-stimulating hormone (C) luteinizing hormone (D) progesterone (E) basal body temperature

(C) luteinizing hormone

A 30-year-old nulligrnvid woman has been trying to conceive for the past year. She is unsure as to when she is ovulating and would like to know how she can best detect ovulation. The most reliable indicator of impending ovulation is an increase in: (A) estradiol (B) folliclc-stimulating hormone (C) luteinizing hormone (D) progesterone (E) basal body temperature

(C) luteinizing hormone

A 40-year-old woman comes to your office to ask about breast cancer screening. She has no family history of breast cancer, has never had a breast biopsy, has a negative past medical history, and is not taking any medications. According to the most recent U.S. Preventive Services Task Force (USPSTF) recommendation, the best breast cancer screening strategy for this patient is: (A) mammography beginning now (B) mammography beginning at age 45 (C) mammography beginning at age 50 (D) timing of mammography determined by Gail model risk assessment

(C) mammography beginning at age 50

A 28-year-old woman comes to you for consultation at 17 weeks of gestation because of positive immunoglobulin G (lgG) and immunoglobulin M (lgM) maternal titers for cytomegalovirus (CMV). She had requested CMV testing after speaking to a colleague whose baby was severely affected by an in utero CMV infection. She has had no symptoms of CMV and no known exposures. Ultrasonographic evaluation shows normal fetal growth and no evidence of anomalies. The most appropriate next step in management is: (A) amniocentesis for CMV (B) fetal magnetic resonance imaging (C) maternal CMV avidity testing (D) repeat maternal CMV titer (E) growth ultrasonography at 28 weeks

(C) maternal CMV avidity testing

A 15-year-old adolescent presents with a history of primary amenorrhea. Her pubertal changes are pertinent for breast development but little pubic hair formation. She has a history of bilateral inguinal hernia repair at age 12 years with ovaries noted in the inguinal regions and they were returned to the abdominal cavity on repair of the hernia. On physical examination, the patient has Tanner stage 3 breast development, Tanner stage 1 pubic hair, and absent axillary hair. A pelvic examination reveals a 4-cm deep vagina, no cervix, and no palpable uterus or adnexa. Ultrasonography confirms the absence of the uterus and adnexa. Follicle-stimulating hormone (FSH) and leutinizing hormone (LH) levels are 13.5 MIU/mL and 40.3 MIU/mL, respectively. The next step in the management of this patient is to: (A) repeat serum FSH and LH levels (B) measure serum estradiol and progesterone levels (C) measure serum testosterone (D) order magnetic resonance imaging (MRI) of her pituitary gland (E) order MRI to identify the location of her ovaries

(C) measure serum testosterone

A 32-year-old nul ligravid woman comes to your office reporting fatigue and weight gain after knee surgery 1 year ago for a tom meniscus. She says she gained 18 kg (40 lb) and presently weighs 95.3 kg (210 lb). She is 1.57 m (62 in.) tall with a body mass index (weight in kilograms divided by height in meters squared [kg/1111]) of 38.4. Her waist measw-es 109 cm (43 in.). Her menstrual cycles occur every 28-30 days with moderate flow. You note excess hair on her forearms, cheeks, and chin, and fullness in the thyroid region. She has increased pigmentation on her upper lip, malar prominences consistent with chloasma, and velvety dark pigmentation at the nape of her neck. Fasting glucose level is 108 mg/dL and total cholesterol level is 270 mg/dL. Her blood pressure is 135/90 mm Hg. The most likely diagnosis is: (A) late onset congenital adrenal hyperplasia (B) polycystic ovarian disease (C) metabolic syndrome (D) Cushing syndrome (E) Hashimoto thyroiditis

(C) metabolic syndrome

A 27-year-old woman, gravida 2, para 1, visits your office at 6 weeks of gestation. She experienced severe postpartum depression in her prior pregnancy. She was on medication for depression for 5 years after the birth of her first child, but discontinued use before the start of the current pregnancy. She is experiencing symptoms of decreased appetite, weight loss, and feelings of excessive guilt, similar to her prior depression symptoms. You consider prescribing sertraline hydrochloride, a selective serotonin reuptake inhibitor (SSRI). The most common adverse effect to the fetus or neonate from use of an SSRI is: (A) increased neonatal cortisol and catecholamine levels (B) fetal cardiac anomalies (C) neonatal withdrawal syndrome (D) neonatal primary pulmonary hypertension

(C) neonatal withdrawal syndrome

A 28-year-old nulligravid woman with no medical problems comes to your office for an annual examination and contraceptive counseling. As part of her visit, you discuss her plans for future pregnancy and steps she should take to optimize obstetric outcomes. The one birth outcome that preconceptlon counseling has reduced is (A) aneuploidy (B) low birth weight (C) neural tube defects (D) prematurity (E) preeclampsia

(C) neural tube defects

A 30-year-old African American woman, gravida 3, para 0, at 12 weeks of gestation comes to the office to discuss her risk of recurrent preterm dellivery. The current pregnancy was conceived with the use of in vitro fertilization (IVF). During her first pregnancy, she gave birth at 28 weeks of gestation and during her second pregnancy she gave birth at 29 weeks of gestation; both following the spontaneous onset of preterm labor. She had a loop electrosurgical excision procedure perfomed before her first pregnancy. She smokes 10-15 cigarettes a day. The risk factor that is most strongly associated with a future preterm delivery is her: (A) non-Hispanic black race (B) current IVF conception (C) obstetric history (D) previous cervical surgery (E) tobacco use

(C) obstetric history

A 26-year-old woman, gravida 2, para 1, presents to your office at 33 weeks of gestation with nausea and vomiting, anorexia, and right-sided abdominal pain for 8 hours duration. Abdominal ultrasonography is inconclusive so a computed tomography (CT) scan is performed, which reveals a right-sided appendicolith and periappendiceal abscess. The most appropriate next step in her management is hospital admission and: (A) antibiotics and induction of labor after corticosteroids for fecal lung maturity (B) laparoscopic appendectomy (C) open appendectomy (D) ceserean delivery combined with adenectomy

(C) open appendectomy

A 32-year-old nulligravid woman with infertility is found to have a 6-cm posterior uterine leiomyoma. A hysterosalpingogram shows uterine cavity distortion from a submucosal location, a finding confirmed by transvaginal ultrasonography. ln discussing complications of abdominal myomectomy, you inform her that the most likely risk is: (A) conversion to hysterectomy (B) uterine rupture with labor (C) pelvic adhesions (D) blood transfusion (E) intrauterine synechiae

(C) pelvic adhesions

A 63-year-old woman undergoes a right salpingo oophorcctomy for a 10-cm mass, which is confirmed on frozen section to be high-grade serous ovarian cancer. In addition to total abdominal hysterectomy and bilateral salpingo-oophorectomy (TAH-BSO), omentectomy, and peritoneal biopsies and washings, the procedure(s) that would most accurately surgically stage apparent early stage ovarian cancer wouId be: (A) no further surgery (B) appendectomy (C) pelvic and aortic lymphadenectomy (D) liver biopsy

(C) pelvic and aortic lymphadenectomy

A 22-year-old woman at 12 weeks of gestation is found to have a positive rapid plasma reagin (RPR) test result on routine prenatal laboratory testing. A confirmatory fluorescent treponemal antibody absorbed test result is also positive. Her current titer is 1:32. She says she does not have any lesions or symptoms at the present time. She was treated for early latent syphilis with doxycycline 13 months ago but failed to follow up after treatment. Her RPR titer at the time of initial treatment was also 1:32. She says she experienced a bronchospasm after taking penicillin as a child. The most appropriate next step in her management is: (A) treatment with erythromycin (B) treatment with ceftriaxone (C) penicillin desensitization (D) skin testing (E) repeat titer at 28 weeks

(C) penicillin desensitization

"A 22-year-old primigravid woman at 31 weeks of gestation was admitted 5 days ago for preterm premature rupture of membranes. She is receiving antibiotics for latency and has completed a course of steroids. Her past medical history is negative. She smokes a half pack of cigarettes daily, but has been switched to a nicotu1e patch during the hospital stay. She now reports vaginal bleeding. When questioned, she reports that the baby is ""balling up."" Her heart rate is 110 beats per minute, blood pressure 130/80 mm Hg, and temperature 37.2°C (99.0°F). Fetal heart rate is 160 beats per minute with minimal variabilir/. On abdominal examination, her uterus is continuously firm to palpation. Given this clinical scenario, the most likely diagnosis is: (A) placenta previa (B) preterm labor (C) placental abruption (D) chlamydia! cervicitis"

(C) placental abruption

A 16--year-old nulligravid woman comes to your office reporting primary amenorrhea. She is 1.37 m (4 ft 6 in.) tall and weighs 50 kg (110 lb). Her vital signs are within normal limits. On physical examination, she has Tanner stage 2 breast developmenl and Tanner stage 3 axillary and pubic hair. Ultrasonography reveals a small uterus and ovaries. Her estradiol level is 12 pg/ml, follicle-stimulating homione (FSH) level is 82 MIU/mL, luteinizing hormone level is 76 MIU/ml, and prolactin level is 5 ng/ml. The most probable diagnosis is: (A) hypothalamic amenorhhea (B) constitutional delay (C) premature ovarian failure (D) anorexia nervosa

(C) premature ovarian failure

Cervical cytology on a 32-year-old woman revealed a high-grade squamous intraepithelial lesion (HSIL). Colposcopy shows a small acetowhite lesion, and biopsy confirms cervical intraepithelial neoplasia 3. The patient has one healthy child and desires more children if possible. You perform a loop electrosurgical excision procedure (LEEP) in the office with a 1-cm wide loop. The pathology resulls indicate negative margins on the cervical intraepithelial neoplasia 3 lesion. However, an incidental discovery is made of adenocaroinoma in situ extending to the deep margin of the specimen. The most appropriate next step in management is: (A) follow up in 4-6 months with cervical cytology (B) colposcopy and endocervical curettage (C) re-excision via cervical conization (D) extrafascial hysterectomy (E) radical hysterectomy

(C) re-excision via cervical conization

A 47-year-old premenopausal woman receives a diagnosis of pneumonia. You see her in consultation because she has an elevated CA 125 level of 90 units/mL. She appears well, reports no symptoms other than a persistent cough, and is taking proper antibiotics for community-acquired pneumonia. Physical examination findings, including pelvic examination, are normal, except for egophony and dullness to percussion in the chest. Her mother had postmenopausal ovarian cancer. After resolution of her pneumonia, you recommend: (A) laparoscopy (B) laparotomy (C) repeat CA 125 assessment (D) pelvic ultrasonography

(C) repeat CA 125 assessment

A 55-year-old postmenopausal woman had a 6-cm simple right ovarian cyst diagnosed by transvaginal ultrasonography. She is asymptomatic and does not have a family history of breast cancer or ovarian cancer. The next step in management is: (A) ultrasonographically-guided cyst aspiration (B) laparoscopy (C) repeat ultrasonography in a year (D) computed tomography (CT) scan of pelvis (E) observation

(C) repeat ultrasonography in a year

A 29-year-old woman, gravida 4, para 2, is seen at 29 weeks of gestation for generalized intense itching for the past week. The itching is worst on her hands and feet. She does not report any infectious contacts or new skin care products. Her temperature is 37.0 C (98.6 F). blood pressure is 135/85 mm Hg and pulse is 90 beats per minute. She appears in no acule distress. Her sclerae are nonicteric and her oropharynx is nonnal. She has no rash. Her cervix is closed and long. The fetal heart rate is 135 beats per minute and reassuring. Urine dipstick test result is negative for protein. Laboratory evaluation reveals her white blood count is 9.0 x 10^9/L. hemocrit Is 33%, and platelet count is 240.000/mm^3. Metabolic panel shows the serum alanine transaminase level is 55 units/L and the total bilirubin level is 0.7 mg/dL The most appropriate diagnostic test is: (A) serum alkaline phosphatase (B) serum ammonia (C) serum bile acid levels (D) serum glucose

(C) serum bile acid levels

A 38-year-old woman presents for her first prenatal visit at 15 weeks of gestation. She states that she has felt well so far during the pregnancy, other than some mild nausea, which resolved 2 weeks prior. Physical examination reveals a soft nontender abdomen, a 15-week-gestation-sized uterus, with a mobile right-sided, lower abdominal mass. Ultrasonographic examination confirms gestational age and finds no fetal abnormalities. The right ovary has a 7-cm complex cystic mass. The left ovary is normal and there is no fluid in the pelvis. Laboratory results show a CA 125 level of 52 units/mL In this patient, the factor that determins the greatest risk for persistence of the mass is: (A) patient age (B) physical examination findings (C) size of the mass (D) CA 125 value

(C) size of the mass

During her annual examination, a long-standing patient informs you that Hodgkin disease has been diagnosed in her 16-year-old son. He is due to begin treatment with chlorambucil (Leukeran) and procarbazine (Matulane) and possible radiation. She has been reassured that cure rates with treatment are high but has heard about the chance or subsequent infertility with this therapy. You tell her that her son's best chance of maintaining future fertility would be with the use of: (A) gonadotropin-releasing hormone agonist therapy (B) testicular shield during radiation (C) sperm cryopreservation (D) testicular tissue cryopreservation (E) expectant management

(C) sperm cryopreservation

A 51-year-old woman, gravida 3, para 2, comes to your office for a menopausal consultation. Her most recent menstrual period was 8 months ago. She has been having hot flushes for more than 15 months with increasing intensity and frequency, and they are interfering with her daily activities. She asks you to explain the risks of hormone therapy (HT). You tell her that for women aged 50-54 years, in addition to an increased risk of deep vein thrombosis, estrogen-only hormone therapy increases the risk of: (A) coronary heart disease (CHD) (B) breast cancer (C) stroke (D) cognitive decline (E) pulmonary embolism

(C) stroke

A 17-year-old female asks for a prescription for oral contraceptives for treatment of dysmenorrhea. Her history and physical examination findings are normal. She tells you that she first had intercourse at age 15 years and that she smokes cigarettes. She also asks for a human papillomavirus (HPV) vaccination. You counsel her about smoking cessation, vaccination against high-risk HPV subtypes, and safe sex. The most appropriate time to screen this patient by cervical cytology is: (A) immediately (B) 3 years after initiation of intercourse (C) 1 year after HPV vaccination (D) at age 21 years

(D) at age 21 years

As part of your quality control committee responsibilities in your group practice, you develop a screening program for gestational diabetes mellitus (GDM). The population is composed primarily of Hispanic women. When screening for GDM at 24-28 weeks of gestation, the greatest effect of lowering the threshold for the 1-hour 50-g glucose test from 140 mg/dL to 135 mg/dL, would be on the frequency of: (A) cesarean delivery (B) hyperbilirubinemia (C) intrauterine fetal demise (D) birth weight greater than 4,000 g (E) preeclampsia

(D) birth weight greater than 4,000 g

A 25-year-old nulligravid woman presents to your office with vaginal bleeding. She is otherwise heallhy and has not received medical care for several years. On pelvic examination, she has a 4.5-cm fungating cervical lesion wilh spread to the left parametrium. She has an otherwise negative staging workup. Biopsy of the mass reveals a squamous cell carcinoma. The most appropriale treatment for this patient is: (A) radical hysterectomy and lymphadenectomy (B) radical tradlelectomy and lymphadenectomy (C) radiation (D) chemoradiation

(D) chemoradiation

A 24-year-old Hispanic woman reports alcohol use during pregnancy. She is a college graduate and works for a nonprofit agency making approximately $45,000 a year. The demographic factor most associated with her use of alcohol in pregnancy is: (A) age (B) income (C) ethnicity (D) education

(D) education

A 43-year-old woman comes to your office with a recently self-detected breast lump. She reports that the lump appeared approximately 2 months ago and has slowly increased in size. She tells you that she has not experienced nipple discharge, pain, or skin changes. She has no family history of breast cancer. She had a normal clinical breast examination followed by negative screening mammography 6 months ago. On examination, you confirm a 1-cm, well-defined, firm, mobile mass in the upper outer quadrant of her right breast, without associated lymphadenopathy. You order diagnostic mammography, which shows no masses and no evidence of malignancy. The best next step in management is: (A) reassurance (B) repeat mammography in 6 months (C) breast magnetic resonance imaging (MRI) (D) fine-needle aspiration biopsy

(D) fine-needle aspiration biopsy

A 27-year--old woman presents \\ti.th a 2-year history of vulvar burning with imercoursie. She recently began dating a new partner and they have been unable to have sexual intercourse because of a sharp, cutting pain that occurs immediately with penetration. She has a history of irritable bowel Syndrome for which she is being treated by her primary care physician. On physical examination, you identify areas of tendemess when she is touched lightly with a cotton swab at the 4 o'clock and 8 o'clock positions of the vestibule. You observe no visible lesions or skin changes, and the neurologic examination is normal. Examination of vaginal secretions reveals a normal pH and no evidence of infeccion or vaginitis. You recommend good vttlvar hygiene and discuss the therapeutic options. As part of your counseling, you explain that vestibulectomy should be recommended: (A) right away (B) if her pain is progressive (C) if her pain spreads to the perineum (D) if medical therapy fails

(D) if medical therapy fails

A 27-year-old woman presents with a 2-year history of vulvar burning with intercourse. She recently began dating a new partner, and they have been unable to have sexual intercourse because of a sharp, cutting pain that occurs immediately with penetration. She has a history of irritable bowel syndrome for which she is being treated by her primary care physician. On physical examination, you identify areas of tenderness when she is touched lightly with a cotton swab at the 4 o'clock and 8 o'clock positions of the vestibule. You observe no visible lesions or skin changes, and the neurologic examination is normal. Examination of vaginal secretions reveals a normal pH and no evidence of infection or vaginitis. You recommend good vulvar hygiene and discuss the therapeutic options. As part of your counseling, you explain that vestibulectomy should be recommended: (A) right away (B) if her pain is progressive (C) if her pain spreads to the perineum (D) if medical therapy fails

(D) if medical therapy fails

Gestational diabetes mellitus was diagnosed in a 32-year-old Latin American woman, gravida 1, at 28 weeks of gestation 2 weeks ago. She initiated a 2,000 kcal American Diabetes Association diet and has been monitoring her blood sugar with a home glucometer. Her fasting blood sugar ranges from 120 mg/dL to 140 mg/dl and her 2-hour postprandial sugar ranges from 170 mg/dL to 240 mg/dl. The best next step in management is: (A) increase the fiber content of her diet (35 g/day) (B) begin a low carbohydrate diet (40 gfday) (C) glyburide therapy (D) insulin therapy (E) initinte daiIy exercise regimen

(D) insulin therapy

A 36-year-old married woman, gravida 3, para 3, comes to your office for annual examination and contraception. She reports that she has heavy menstrual bleeding, and would like to resume taking oral contraceptives (OCs), which she took for several years to improve her menses as a teenager. She is morbidly obese, with a history of a pulmonary embolism after her most recent pregnancy 2 years ago. Although she is certain she does not want to become pregnant in the next few years, she thinks her husband may want another child in the future. You counsel her that the best contraceptive for her at this time would be (A) progestin-only OCs (B) combined OCs (C) contraceptive vaginal ring (D) levonorgestrel intrauterine device (IUD) (E) copper-containing IUD

(D) levonorgestrel intrauterine device (IUD)

A 29-year-old woman, gravida 2, para 2, had her annual preventive health care examination 10 days ago. At the time of her visit, she reported no vaginal symptoms. You are now reviewing her Pap test report that states normal cytology and bacterial vaginosis. Your next step in management is to: (A) perform a wet mount (B) treat with metronidazole (C) treat with clindamycin hydrochloride (D) manage her expectantly

(D) manage her expectantly

A 29-year-old woman, gravida 2, para 2, had her annual prevetentive health care examination 10 days ago. At the time of her visit, she reported no vaginal symptoms. You are now reviewing her Pap test report that states normal cytology and bacterial vaginosis. Your next step in management is to: (A) perform a wet mount (B) treat with metronidazole (C) treat with clindamycin hydrochloride (D) manage her expectantly

(D) manage her expectantly

A 47-year-old woman, gravida 4, para 3, spontaneous abortion 1, reports progessive vasomotor symptoms, joint pain and stiffness, and depressed moods. She has been experiencing progressively less regular menstrual periods, occurring every 3-6 weeks. She works fuII time as a highschool teacher and describes a stable marriage. Her phyisical and pelvic examinations are unremarkable. Her medical history is negative for arthralgia, depression. or other medical problems. She is not on any medication but takes multivitamins, calcium, and vitamin D supplements. Her thyroid-stimulating-hormone level is 3.5 microunits/mL. The most likely diagnosis is: (A) midlife crisis (B) degenerative arthritis (C) subclinical depression (D) menopausal transition (E) hypothyroidism

(D) menopausal transition

A 26-year-old woman, gravida 1, at 18 weeks of gestation consults you because of headache. The pain is unilateral, throbbing in nature, and worsens with movement of her head, light, and sound. She has no nausea, visual changes, nasal congestion, or lacrimation. Her headache is episodic, occurs approximately 1-2 times per month, and is relieved by ibuprofen. The most likely type or headache is: (A) sinus (B) cluster (C) tenslon (D) migraine (E) hemicrania continua

(D) migraine

A 32-year-old woman, gravida 2, para 1, at 7 weeks of gestation reports vaginal bleeding early this morning. She has no concurrent medical conditions and the only medication she takes is prenatal vitamins. You diagnose her with early pregnancy failure based on ultrasonographic findings of a crown-rump length of 8 mm and no detectable cardiac motion. She does not want surgical intervention but she would like resolution within 2 weeks. The best next step is: (A) expectant management (B) methotrexate (C) mifepristone (D) misoprostol

(D) misoprostol

A 28-year-old primigravid woman requests an elective cesarean delivery because she is concerned about pain during labor. She has no medical indications for cesarean delivery. You counsel her about pain management in labor. You explain to her that elective cesarean delivery before labor is associated with a higher rate of certain complications. In this patient, the complication that is most likely to be increased with elective cesarean delivery versus planned vaginal delivery is: (A) postpartum hemorrhage (B) sexual dysfunction (C) urinary incontinence (D) neonatal respiratory morbidity (E) postpartum depression

(D) neonatal respiratory morbidity

"A 45-year-old Asian woman comes to you with concerns about ovarian cancer. She has read about screening for ovarian cancer and is concerned to hear that more than one-half of cases of ovarian cancer are diagnosed at an advanced stage. She recently received an e-mail from a friend encouraging her to ""see your doctor and get tested now."" She is healthy. her menses are still regular, but she does have occasional mild hot flashes. Her maternal grandmother had breast cancer at age 70 years and a paternal cousin has colon cancer. There is no family history of ovarian cancer. Physical examination is normal In terms of screening, you recommend: (A) serial CA 12S testing (B) tumor marker panel testing (C) pelvic Ultrasonography (D) no routine screening"

(D) no routine screening

A 14-year-old nulligravid woman presents with severe midline pelvic pain and cramping during her menstrual cycle for the past 6 months. She has migraines with possible aura. Menarche occurred at age 13 years. She has been absent from school for 2 days each month because of her symptoms. She ia not sexually active. Physical examination is normal. The best initial therapy for her dysmenorrhea is: (A) acupuncture (B) combination oral contraceptives (OCs) (C) herbal medication (D) nonsteroidal anti-inflammatory drugs (NSAIDs) (E) levonorgestrel intrauterine device (IUD)

(D) nonsteroidal anti-inflammatory drugs (NSAIDs)

A 33-year-old nulligravid married woman receives a diagnosis of breast cancer. She is scheduled to have a mastectomy and subsequent chemotherapy for 12 weeks with a regimen that includes cyclophosphamide (Cytoxan). The therapy or intervention that might improve her ability to deliver a child after chemotherapy for breast cancer is: (A) use of oral contraceptives during her chemotherapy (B) use of a gonadotropin-releasing hormone (GnRH) agonist during her chemotherapy (C) obtain ovarian tissue and cryopreserve before chemotherapy (D) perform in vitro fertilization (IVF) and cryopreserve embryos before chemotherapy

(D) perform in vitro fertilization (IVF) and cryopreserve embryos before chemotherapy

A 39-year-old woman, gravida 2, para 0, at 36 weeks of gestation has type I diabetes mellitus and chronic hypertension. She is undergoing antepartum fetal assessment with biophysical profile (BPP) testing. After 75 minutes, she has a BPP of 2/10 and an amniotic fluid index of 2 cm. The best next step in management is to: (A) repeat BPP in 24 hours (B) perforn contraction stress test (C) perform vibroacoustic stimulation (D) proceed to delivery

(D) proceed to delivery

A 32-year-old woman, para 3, presents with postcoital bleeding. The result of her most recent Pap test 8 years ago was reported as abnormal. On examination, she has no palpable adenopathy. Her pelvic examination is normal except for a 1-cm raised lesion on the cervix with no parametrial spread. A biopsy of the lesion confirms a well-differentiated invasive squamous cell cancer. Further evaluation is consistent with a stage 1B1 squamous cell cervical cancer. Although she is not planning any further childbearing, she is very concerned about the effect of her treatment on her sexual function. The best treatment option for this patient is: (A) cervical conization (B) simple hysterectomy (C) radical trachelectomy (D) radical hysterectomy (E) chemoradiation

(D) radical hysterectomy

A 32-year-old woman, para 3, presents with postcoital bleeding. The result of her most recent Pap test 8 years ago was reported as abnormal. On examination, she has no palpable adenopathy. Her pelvic examination is normal except for a 1-cm raised lesion on the cervix with no parametrial spread. A biopsy of the lesion confirms a well-differentiated invasive squamous cell cancer. Further evaluation is consistent with a stage IB1 squamous cell cervical cancer. Although she is not planning any further childbearing, she is very concerned about the effect of her treatment on her sexual function. The best treatment option for this patient is: (A) cervical conization (B) simple hysterectomy (C) radical trachelectomy (D) radical hysterectomy (E) chemoradiation

(D) radical hysterectomy

A 25-year-old woman, gravida 2, para 0, aborta 1, visits your office for follow-up of ectopic pregnancy. She desires to avoid surgery if possible. She initially presented 10 days ago w'ith mild lower abdominal pain and vaginal spotting. At that time, her vitaI signs were stable, she had minimal tenderness on abdominal and pelvic examination, and her transvaginal ultrasonography showed a hypoechoic heterogeneous endometrial lining, no free fluid in the cul-de-sac, and a 2-cm ectopic pregnancy. Her serum b-hCG level at that time was 1,500 mIU/mL. Two days later, her b-hCG level was 1,875 mIU/mL. Manual vacuum aspiration revealed no chorionic villi and methotrexate was given using the single-dose protocol. On days 4 and 7, her b-hCG levels were 2,500 mIU/ml and 2,300 mIU/mL, respectively. On day 7, the patient has stable vital signs and mild intermittent lower abdominal cramping. Based on her clinical scenario, the next best step in management is: (A) repeat b-hCG in 48 hours (B) repeat b-hCG in 1 week (C) laparoscopy (D) re-dose methotrexate (E) repeat manual vacuum aspiration

(D) re-dose methotrexate

A 45-year-Old woman is scheduled to undergo laparoscopically assisted vaginal hysterectomy for menorrhagia and uterine leiomyomas. On laparosccpic inspection of the pelvis, a 7-cm complex left ovarian mass appears suspicious for cancer. You obtain pelvic washings. For frozen tissue diagnosis, the most appropriate next step in management is to perform a left: (A) ovarian biopsy (B) ovarian aspiration (C) ovarian cystectomy (D) salpingo-oophorectomy (E) salpingo-oophorectomy and laparoscopically assisted vaginal hysterectomy

(D) salpingo-oophorectomy

A 32-year-old woman, gravida 2, para 1, aborta 1, status post spontaneous vaginal delivery 4 months ago, has postpartum depression. She has no history of prepregnancy depression or antepartum depression. She has been seeing a counselor for nearly 3 months, but is still depressed and is concerned about bonding with her newborn. She is breastfeeding with difficulty, and has anhedonia. She has experienced feelings of inadequacy with appetite disturbance since the delivery. In regard to her depression, the best next step in the management of her depression is: (A) no treatment (B) citalopram (C) ethinyl estradiol (D) sertraline hydrochloride

(D) sertraline hydrochloride

A 68-year-old postmenopausal woman comes to your office for a health maintenance visit. She is asymptomatic, but asks about hormone therapy (HT) and her risk of ischemic heart disease. She has never had any gynocologic surgery. The Women's Health Initiative (WHI) evaluated the health effects of HT in asymptomatic women. You advise her that the study found that the effect of HT on ischemic heart disease for patients such as her was: (A) highly protective (hazard ratio (HR)=02; 80% reduclion) (B) slightly protective (HR=0.8; 20% reduction) (C) no association (HR=1.0; no increase or decrease in risk) (D) slightly increased risk (HR=1.3; 30% increase) (E) marked increased risk (HR=3.0; threefold increase)

(D) slightly increased risk (HR=1.3; 30% increase)

A 55-year-old woman comes in for her first gynecologic examination since the binh of her youngest child 30 years ago. She reports being monitored by her primary doctor and is taking an oral antihypertensive with no other medical problems. She is experiencing hot flushes and irregular vaginal bleeding. On physical examination. you observe fullness in the left supraclavicular and cervical lymph nodes. Her abdomen is nom,al, but a pelvic examination reveals an 8-cm exophytic rumor replacing the entire cervix. She does not demonstrate any parametrial involvement. In addition to a cervical biopsy, the next diagnostic procedure should be: (A) endometrial biopsy (B) fractional djlation and curettage (D&C) (C) cystoscopy and proctoscopy (D) supraclavicular lymph node cytology or biopsy

(D) supraclavicular lymph node cytology or biopsy

A 34-year-old woman has a known twin pregnancy. Her referring physician noted discordant growth on 20-week ultrasonography. The set of ultrasonographic findings most consistent with twin-twin transfusion syndrome is: (A) twin peak sign, twin B with fetal growth restriction, and renal anomaly (B) twin peak sign, decreased amniotic fluid twin B, and hydramnios twin A (C) thin dividing membrane, twin B with fetal growlh restriction, and renal anomaly (D) thin dividing membrane, decreased amniotic fluid twin B, and hydramnios twin A

(D) thin dividing membrane, decreased amniotic fluid twin B, and hydramnios twin A

A 22-year-old nulliparous woman presents to the office at 26 weeks of gestation with fever and chills. Physical examination reveals right flank and costovertebral angle (CVA) tenderness. Her temperature is 38.5°C (101.3°F) and microscopic evaluation of her urine reveals greater than 20 bacteria per high-powered field. The most appropriate next step in management is to: (A) await urine culture results (B) treat as outpatient with oral antibiotics (C) perform renal ultrasonography (D) treat as inpatient with intravenous antibiotics

(D) treat as inpatient with intravenous antibiotics

A 40-year-old woman comes in for evaluation after recently suffering her second pregnancy loss. Neither her own famlly nor her husbands family has a history of more than a single pregnancy loss. Karyotyping of both partners is performed. The risk of either member of the couple carrying a structural chromosomal abnormality is: (A) 55% (B) 45% (C) 7% (D) 4.8% (E) 0.5%

(E) 0.5%

A 38-year-old woman, gravida 3, para 3, comes to your office with concerns about breast cancer. She has a strong family history of breast cancer, with breast cancer diagnosed in her mother, maternal aunt, and older sister all before menopause. In addition, epithelial ovarian cancer was recently diagnosed in her 52-year-old cousin. You recommend BRCA carrier testing, and she is found to be a carrier of the BRCA1 mutation. She wants to know the best way to reduce her risk of breast cancer. You inform her that the strategy associated with the greatest reduction in breast cancer risk is: (A) breast magnetic resonance imaging (B) tamoxifen citrate (C) bilateral salpingo-oophorectomy (D) bilateral subcutaneous mastectomy (E) bilateral total mastectomy

(E) bilateral total mastectomy

A 30-year-old primigravid woman presents for her first prenatal visit at 10 weeks of gestation. She has been in a monogamous relationship for 5 years and neither she nor her husband has ever had a sexually transmitted disease (STD). Her history is generally unremarkable and her physical examination is consistent with a normal first-trimester pregnancy. She has read about the effects of herpes simplex virus (HSV) on newborns and is concerned about HSV in pregnancy. You explain that the most cost-effective approach to testing for her is: (A) cervical viral culture for HSV (B) HSV polymerase chain reaction (C) HSV-1 and HSV-2 antibody test (D) screen her partner (E) culture if she develops a suspicious lesion

(E) culture if she develops a suspicious lesion

A 24-year-old nulligravid woman is taking carbamazepine for epilepsy. She currently uses condoms, but would prefer a more effective contraceptive. She tells you that she does not plan on becoming pregnant for several years. You counsel her that, given her use of carbamazepine, the most effective contraceptive for her would be (A) depot medroxyprogesterone acetate (B) combined oral contraceptives (OCs) (C) etonogestrel implant (D) progestin-only pill (E) intrauterine contraception

(E) intrauterine contraception

A 37-year-old woman, gravida 2, para 0, comes to you for evaluation for recurrent miscarriage after experiencing two consecutive first-trimester losses. She was managed expectantly with her first loss but required dilation and cutterage with the second loss. Overall she is in good heallh, is of normal weight, and has no medicaI problems. She takes no medications. Her husband has fathered no clildren. Hysterosalpingography reveals normal intrauterine architecture. Thyroid-stimulating hormone (TSH) and fasting glucose are within normal limits and the test result for antiphospholipid antibodies (anticardiolipin antibody and lupus anticoagulant test) is negative. The most appropriate next step in her workup is: (A) reassurance and observation (B) endometrial biopsy (C) ureaplasma culrures (D) evaluation for thrombopbilias (E) karyotype of both partners

(E) karyotype of both partners

A 30-year-old woman, gravida 2, para 2, comes to your office for treatment of heavy menstrual bleeding. She reports 28-day cycles with heavy bleeding that last for 7 days and tells you that the bleeding is affecting her quality of life. She has no significant medical or surgical history and takes no medications. On examination, she has small, mobile, anteverted uterus with no nodularity and no adnexal tenderness or masses. Her hemoglobin level is 11.0 g/dL. Although she does not desire pregnancy in the near future, she wants to retain childbearing potential. You advise her that the best option for reducing menstrual blood loss for her is: (A) nonsteroidal antiinflammatory drug (NSAIDs) (B) cyclic progestin for 10-14 days per month (C) cyclic progestin for 21 days per month (D) combined oral contraceptives (OCs) (E) levonorgestrel intrauterine deviice (IUD)

(E) levonorgestrel intrauterine deviice (IUD)

A 36-year-old woman, gravicla 1, para 1, presents for evaluation of cyclic heavy menstrual bleeding and increasing pelvic pressure for the past year. Her medical history is unremarkable. She had a preterm 34-week vaginal delivery 5 years ago. On examination, her vital signs are stable. A pregnancy test result is negative. Her abdomen is soft, nontender with a mass palpable just inferior to the umbilicas. Pelvic examillation revealed an 18-week-sized mobile uterus. Endometrial biopsy showed secretory endometrium. Pelvic ultrasonography assessed her uterine size to be 18 cm x 10 cm x 7 cm with multiple leiomyomas, the largest of which was 8 x 6 x 4 cm. With saline infusion ultrasonographic scanning, none of the leiomyomas had substantial submucous components. The patient's hemoglobin was 8 g/dL. The patient tells you she desires treatment that will reduce both her bleeding and her pelvic pressure. She wishes to preserve her fertility. The safest and most effective long-term treatment option for this patient is: (A) combination oral contraceptives (B) gonadotropin-releasing hormone (GnRH) agonist injections (C) uterine artery embolization (D) magnetic resonance imaging (MRI)-guided focused ultrasound therapy (E) myomectomy

(E) myomectomy

A 36-year-old woman, gravida 1, para 1, presents for evaluation of cyclic heavy menstrual bleeding and increasing pelvic pressure for the past year. Her medical history is unremarkable. She had a preterm 34-week vaginal delivery 5 years ago. On examination, her vital signs are stable. A pregnancy test result is negative. Her abdomen is soft, nontender with a mass palpable just inferior to the umbilicus. Pelvic examination revealed an 18-week-sized mobile uterus. Endometrial biopsy showed secretory endometrium. Pelvic ultrasonography assessed her uterine size to be 18 cm x 10 cm x 7 cm with multiple leiomyomas, the largest of which was 8 x 6 x 4 cm. With saline infusion ultrasonographic scanning, none of the leiomyomas had substantial submucous components. The patient's hemoglobin was 8 g/dL.. The patient tells you she desires treatment that will reduce both her bleeding and her pelvic pressure. She wishes to preserve her fertility. The safest and most effective long-term management option for this patient is: (A) combination oral contraceptives (B) gonadotropic-releasing hormone (GnRH) agonist injections (C) uterine artery embolization (D) magnetic resonance imaging (MRl)-guided focused ultrasound therapy (E) myomectomy

(E) myomectomy

A 50-year-old woman, gravida 3, para 3, had her last menstrual period 12 months ago and has had no vaginal bleeding since. She is a competitive runner who works full time as a regional sales manager requiring her to do a lot of driving. She reports worsening vasomotor symptoms, which have resulted in insomnia and interuptions at work. She requests treatment to manage her symptom so she can maintain her busy lifestyle. She does not wish to use estrogen. The nonhormonal treatment most likely to relieve her vasomotor symptoms is: (A) black cohosh (B) dong quai root (C) clonidine (D) gabapentin (E) paroxetine hydrochloride

(E) paroxetine hydrochloride

A 38-year-old woman, gravida 3, para 3, presents for preoperative counseling before her planned total abdominal hysrerectomy for heavy menstrual bleeding. She has had three cesarean deliveries without complications and she is otherwise in good health without significant medical history or family history. She is not taking any medicalions. The best postoperative thromboembolism prophylaxis for this patient is: (A) aspirin (B) early ambulation (C) graduated compression stockings (D) therapeutic unfractioned heparin (E) pneumatic compression devices

(E) pneumatic compression devices

A 25-year-old primigravid woman comes to your office as a transfer for prenatal care. She is currently at 36 weeks of gestation. She was diagnosed with human immunodeficiency virus (HIV) during routine screening performed in the first trimester. She has been taking highly active antiretroviral therapy (HAART) since then, and has been tolerating her treatment well. Her pregnancy has otherwise been uncomplicated. The results of laboratory testing done at 34 weeks of gestation showed her viral load to be 2,000 copies/mL and her CD4 count was 375 x 10^6/L. In addition to intrapartum intravenous zidovudine therapy and continuation of the other HAART medications, you counsel her that the most appropriate method and timing of delivery to reduce the risk of transmission of HIV to her child is: (A) vaginal delivery after onset of spontaneous labor (B) vaginal delivery after induction of labor at 39 weeks (C) cesarean delivery at the onset of labor (D) scheduled cesarean delivery at 39 weeks (E) scheduled cesarean delivery at 38 weeks

(E) scheduled cesarean delivery at 38 weeks

A 23-year-old woman has vaginal bkeding e\tty 2-3 mooths accompanied by moderate weight gain. However. she is most bocheml by worsening dark upper lip and chin facial hair. Workup suggests polycystie ovary syndrome and you prescribe an oral contraccpti\•e to treai her hirsutism. At &-month follow-up. the best indicator of success is a decrease in: (A) Ferriman-Gallwey score (B) 17a-hydroxyprogesterone levels (C) free testosterone (D) dihydroepiandrosterone Sulfate (E) shaving and plucking intervals

(E) shaving and plucking intervals

A 33-year-old white woman, gravida 3, para 2, at 37 weeks of gestation comes to your office in early labor. She had a term vaginal delivery 5 years ago and a term low transverse cesarean delivery for breech presentation 3 years ago. She is already scheduled for a repeat cesarean delivery at 39 weeks of gestation. Office examination reveals a maternal weight of 96.2 kg (212 lb), a height of 1.63 m (64 in), and a body mass index (weight in kilograms divided by height in meters squared [kg/m^2]) (BMI) of 36.5. She has blood pressure of 132/78 mm Hg and negative urine dip for protein and glucose. Sterile vaginal examination reveals a cervix that is 5 cm dilated, 80% effaced with cephalic presentation at +1 station. You counsel her that her only factor associated with a decreased probability of success of a trial of labor after previous cesarean delivery is her: (A) prior vaginal birth (B) prior cesarean delivery that was performed for breech presentation (C) interdelivery interval greater than 24 months (D) obesity (E) spontaneous labor before 40 weeks

(E) spontaneous labor before 40 weeks

A 29-year-old womam, gravida 2, para 1, comes to your office at 31 weeks of gestation with decreased fetal movement for 2 days. Intrauterine fetal demise is diagnosed and she is admitted for induction of labor. You order diagnostic testing to identify the cause of the stillbirth. The best tissue to obtain for fetal karyotype analysis is: (A) costochondral (B) fascia lata (C) patella (D) placenta (E) umbilical cord

(E) umbilical cord

External examination of a woman giving birth is performed with the use of: 1) first Leopold's maneuver; 2) pelvimeter; 3) fifth Leopold's maneuver; 4) McRoberts maneuver; 5) Bracht maneuver. The correct answer is: A. 1,2,3. B. 4,5. C. 1,2. D. 1,2,4. E. all the above.

A. 1,2,3.

Azoospermia is defined as: A. absence of spermatozoa in semen. B. absence of semen. C. absence of progressive motility of sperms. D. concentration of sperms less than 20 million in 1 mL of semen. E. less than 30 % of spermatozoa with normal morphology.

A. absence of spermatozoa in semen.

A 28-year-old G1P1 previously healthy woman is brought into the emergency department by her husband following a seizure at home. She had been complaining of a severe headache for 2 days prior to this, and had been feeling more and more fatigued and short of breath since the delivery of their child three months ago. She has been breastfeeding, and began to have vaginal spotting one month ago. Her neurologic and physical examinations are unremarkable and her pelvic exam reveals a normal uterus with no adnexal masses. Her work-up reveals multiple nodules on chest X-ray and within the brain and liver, suspicious for metastasis. Choriocarcinoma is highly suspected. Which of the following tests will confirm the diagnosis in this case? A. Quantitative Beta-hCG B. Serum CA-125 C. Transvaginal ultrasound D. Fine needle aspiration of the liver lesions E. Biopsy of a chest nodule

A A diagnosis of choriocarcinoma is made once the presence of Beta-hCG is confirmed. Certainly, intrauterine pregnancy and ectopic pregnancy must be excluded, but this can easily be done depending on the quantitative level. In the presence of metastatic disease of unclear primary, the diagnosis of GTD (choriocarcinoma) must be considered. Ultrasound is useful in ruling out an intrauterine or ectopic pregnancy, but provides no information if the Beta-hCG is negative or below the discriminatory zone. Serum CA-125 is a tumor marker for most epithelial ovarian cancers but, because it is non-specific, its possible elevation in this case is not diagnostic. Because metastatic choriocarcinoma is quite vascular, suspicious lesions should never be biopsied. Tissue diagnosis is the standard in establishing a diagnosis of almost all malignancies, with the exception of choriocarcinoma. Only a positive Beta-hCG in a reproductive-aged woman who has a history of a recent pregnancy (term, miscarriage, termination, mole) is necessary to establish the diagnosis.

A 72-year-old G3P1 has progressive ovarian cancer. She and her husband have already completed a medical power of attorney form. However, the patient did not complete a living will or any other documents expressing her wishes for the initiation of mechanical ventilation or cardioversion in the event of a respiratory or cardiac arrest. Unfortunately, the patient is brought into the hospital after suffering an incapacitating seizure. She is not arousable when she reaches the oncology unit. Her husband Jim is present and willing to act as Mary's surrogate decision-maker. When he decides on the proper course of care, the husband should make decisions based primarily on which of the following? a. What Mary would have chosen b. Mary's best interest c. Hospital Ethics Committee's recommendation d. The family's wishes e. His own wishes

A A person who has power of attorney should make decisions based on what the patient would have wanted for herself, regardless of what they think her best interests might be.

A 38-year-old G1P1 comes to the office for a health maintenance examination. She has noticed some urinary frequency over the past month. She has no dysuria, hematuria, urgency or incontinence. She has normal cycles, no history of abnormal Pap smears or sexually transmitted infections and is sexually active, with one partner. She smokes a quarter of a pack of cigarettes daily, and drinks one glass of wine per day. Her mother had breast cancer at age 30. Her general examination is normal. On pelvic exam, she has normal external genitalia; vagina and cervix are without lesions. Her uterus is normal size, anteverted and nontender. Her left adnexa is normal, right adnexa has a mobile, slightly tender 4 cm mass. Laboratory results show a normal urinalysis, a negative urine pregnancy test and a normal Pap smear. What is the most appropriate next step in the management of this patient? A. Perform a transvaginal ultrasound B. Perform a diagnostic laparoscopy C. Recommend a CT-guided drainage of the mass D. Order a KUB plain film E. Perform an exploratory laparotomy

A A transvaginal ultrasound would be the best way to begin a workup for an incidental finding of an adnexal mass. Using this modality, one can distinguish an adnexal mass from other structures, as well as note the characteristics of the mass (simple vs. complex, solid vs. cystic, thin or thick walled, size, and ancillary structures involved). Using this information, a management plan may be constructed. In a stable, asymptomatic patient, laparoscopy or laparotomy would not be indicated for diagnosis until further imaging studies had been done. A CT-guided drainage would not be a good choice until one has a better understanding of the characteristics of the mass. If the mass was malignant, draining it in this manner could not only seed the needle tract, but it could also spill intra-abdominally, causing the malignancy to spread. A plain film would not be helpful unless the mass contains calcium and thus would not give additional data.

A 26-year-old woman presents with hirsutism and irregular menses. Her mother, who is diabetic, had similar complaints prior to menopause. On physical exam, this patient is noted to have terminal hair on her chin and a gray-brown velvety discoloration on the back of her neck. This lesion is acanthosis nigricans. Which of the following is the most appropriate first test to order for this patient? A. Fasting insulin B. TSH C. 17-hydroxyprogesterone level D. Cortisol level E. Pelvic ultrasound

A Acanthosis nigricans is associated with elevated androgen levels and hyperinsulinemia. Since this woman has a family history of diabetes and also has acanthosis nigricans, the most appropriate test of those listed would be the fasting insulin. The other tests would also be reasonable, but hyperinsulinemia is most likely in this patient.

A 33-year-old G2P1 woman delivered a male infant after a precipitous second stage. On initial assessment, the infant has no respiratory effort. You decide to proceed with positive pressure ventilation. Which of the following techniques will impede positive pressure ventilation on this newborn? A. Adjusting head position to modified flex position B. Adjusting the head to sniffing position C. Securing mask for a good seal D. Compressing the bag just until chest rise is seen E. Having the oxygen flow at minimum 10 L/minute

A Adjusting the head to a modified flex position is typically used in adult CPR. The sniffing position (tilting the neonate's head back and lifting the chin) is the correct position for application of positive pressure ventilation in a newborn infant. It is important to also secure the mask to the infant's face and to observe an initial chest rise. A recommended rate of oxygen flow is 10 L/minute.

A 37-year-old G2P1 woman with poorly controlled chronic hypertension presents in labor at term. Her prenatal course was uncomplicated. She delivers a 3500 gram infant spontaneously after oxytocin augmentation of labor. Immediately postpartum, she experiences excessive bleeding. Her blood pressure is 130/90; pulse 84; and she is afebrile. On examination, uterine fundus is firm and the placenta is intact. Which of the following is the most appropriate next step in the management of this patient? A. Exploration for lacerations B. Methylergonovine C. B Lynch suture D. IV push of oxytocin E. Uterine artery embolization

A After ensuring appropriate backup, establishing intravenous access and stabilizing a patient as needed, the first steps in the management of postpartum hemorrhage are to make sure the uterus is well-contracted, there is no retained placental tissue and to look for lacerations. This patient has a firm fundus, which indicates a contracted uterus. Her placenta is complete, which typically rules out retained placental tissue, so it is important to rule out lacerations, which can lead to hemorrhage. Methylergonovine, prostaglandins and oxytocin are all uterotonics and used to increase uterine contractions and decrease uterine bleeding. Methylergonovine is an ergot alkaloid, which is a potent smooth muscle constrictor. It is also a vasoconstrictive agent and should be withheld from women with hypertension and/or preeclampsia. B Lynch suture is used at time of laparotomy for uterine atony. Oxytocin should not be given as an IV push. Uterine artery embolization can be considered after other sources of bleeding such as lacerations are ruled out.

A 33-year-old G2P1 woman at 29 weeks gestation presents with confirmed preterm premature rupture of membranes. She denies labor. She takes prenatal vitamins and iron. She denies substance abuse, smoking or alcohol use. Her prior pregnancy was delivered vaginally at 41 weeks after spontaneous rupture of membranes. Her blood pressure is 110/70; pulse 84; temperature 98.6°F (37.0°C). Which of the following is the best medication to delay the onset of labor? A. Antibiotics B. Betamethasone C. Calcium channel blocker D. Beta mimetics E. Magnesium sulfate

A Antibiotic therapy given to patients with preterm premature rupture of the membranes has been found to prolong the latency period by 5-7 days, as well as reduce the incidence of maternal amnionitis and neonatal sepsis. Corticosteroids (betamethasone) and tocolytics may also prolong the pregnancy for various lengths of time, but generally not seven days.

A 19-year-old G1 woman at 36 weeks gestation presents for her first prenatal visit, stating she was recently diagnosed with HIV after her former partner tested positive. The HIV Western Blot is positive. The CD4 count is 612 cells/µl. The viral load is 9,873 viral particles per ml of patient serum. Which of the management options would best decrease the risk for perinatal transmission of HIV? A. Treatment with intravenous zidovudine at the time of delivery B. Treatment of the newborn with oral zidovudine only if HIV-positive C. One week maternal treatment with zidovudine now D. Cesarean section in second stage of labor E. Single drug therapy to minimize drug resistance

A Antiretroviral therapy should be offered to all HIV-infected pregnant women to begin maternal treatment as well as to reduce the risk of perinatal transmission regardless of CD4+ T-cell count or HIV RNA level. The baseline transmission rate of HIV to newborns can be reduced from about 25% to 2% with the HAART (highly active antiretroviral therapy) protocol antepartum and continuing through delivery with intravenous zidovudine in labor and zidovudine treatment for the neonate. Cesarean section prior to labor can reduce this rate to 2% (although the benefit is less clear in women with viral loads <1,000 particles per ml.) Multiple agents should be used in pregnancy to minimize the development of drug resistance.

A 20-year-old G1P0 woman at 28 weeks gestation presents to triage with uterine contractions every four minutes. On exam, her cervix is long, closed and posterior. Her urinalysis is normal. Fetal fibronectin is negative. In addition to hydration, which of the following is the most appropriate next step in the management of this patient? A. Expectant management B. Bedrest C. Tocolysis D. Cerclage E. Administer betamethasone

A Approximately 50% of patients with preterm contractions have spontaneous resolution of abnormal uterine activity. The patient should be observed until a correct diagnosis is made. If there is evidence the patient is dehydrated and she is unable to tolerate PO fluids, then IV hydration would be indicated. Preterm labor, which is defined as the presence of regular uterine contractions leading to cervical change, needs to be promptly treated. Tocolysis is not necessary in this case because a diagnosis of preterm labor has not been made (no cervical change). The patient should not be sent home until diagnosis and treatment plans are determined. Since fetal fibronectin is negative and the patient is not in labor, she can be expectantly managed. Cerclage is not necessary, since she does not have an incompetent cervix. Treatment with betamethasone is not indicated unless there is evidence that the patient is at increased risk of delivering preterm. Bedrest is not indicated and has not been shown to reduce preterm birth.

A 35-year-old woman presents to the emergency department with heavy vaginal bleeding at seven weeks gestation. On examination, she has a dilated cervix with blood and tissue present at the cervical os. Which of the following is the most likely chromosomal abnormality to be found in the karyotypic evaluation of the products of conception? A. Autosomal trisomy B. Triploidy C. Tetraploidy D. Monosomy X (45X,0) E. Fragile X mutation

A Autosomal trisomy is the most common abnormal karyotype encountered in spontaneous abortuses, accounting for approximately 40-50% of cases. Triploidy accounts for approximately 15%, tetraploidy 5% of cases, and Monosomy X (45X, 0) identified in 15-25% of losses. The Fragile X mutation involves an expanded number of trinucleotide repeats in the CGG (cytosine-guanine) sequence.

A 67-year-old G3P3 woman presents with severe pelvic protrusion status post hysterectomy. She denies any incontinence. She failed conservative management with a pessary. She underwent a vaginal surgical repair where the pubocervical fascia was plicated in the midline as well as laterally to the arcus tendineus fascia (white line). What defect was repaired in this patient? A. Cystocele B. Rectocele C. Uterine prolapse D. Enterocele E. Urethral diverticulum

A Central and lateral cystoceles are repaired by fixing defects in the pubocervical fascia or reattaching it to the sidewall, if separated from the white line. Defects in the rectovaginal fascia are repaired in rectoceles. Uterine prolapse is surgically treated by a vaginal hysterectomy, but this patient already had a hysterectomy. Enteroceles are repaired by either vaginal or abdominal enterocele repairs. Vaginal vault prolapse is treated either by supporting the vaginal cuff to the uterosacral ligaments, sacrospinous ligament or sacrocolpopexy. Urethral diverticulum does not present with severe pelvic protrusion.

A 27-year-old G1 at 12 weeks gestation presents for first prenatal care visit. She is previously healthy and takes no medications. An ultrasound is performed and a viable pregnancy is confirmed. At the end of the visit, the patient discusses with you her desire to have a Cesarean section for delivery, as she does not wish to go through the pain of labor. Her husband, an orthopedic surgeon, expresses concerns as they desire to have at least three children and he is worried about potential complications with repeated Cesarean sections. What is the most appropriate next step in the counseling of this patient? a. Agree with her decision after proper counseling and perform a Cesarean section at 39 weeks gestation b. Agree with her decision after proper counseling and perform a Cesarean section at 41 weeks gestation if she has not gone into labor by then c. Advise her that it is not possible to plan a Cesarean section for delivery d. Advise her to listen to her husband and plan for a vaginal birth e. Refer her to psychiatric counseling

A Elective cesarean section on demand has been getting more popular among women for a variety of reasons. Although, it might sound unreasonable to undergo a Cesarean section for being afraid of pain, the patient has the right to request it and the physician's duty is to make sure she understands all the risks and potential complications associated with such a decision. Her husband is appropriately concerned but it is up to her to make the decision regarding an elective procedure.

A 72-year-old G3P3 presents to the emergency room complaining of abnormal vaginal discharge for the past two months. She has had two episodes of vaginal bleeding over the last year. She used combination hormone replacement therapy for 10 years when she went through menopause at age 58, but stopped once the Women's Health Initiative report came out. Her last gynecologic exam and Pap smear were two years ago and were normal. She has tried several over-the-counter antifungal creams for what she presumed was a yeast infection; however, she reports no change in the nature of the discharge. She does note that she has some mild lower abdominal discomfort. The only significant finding on exam is a mucopurulent discharge from a multiparous cervical os. She has a 10-week sized globular uterus. Which of the following findings is most concerning for presence of endometrial cancer? A. Vaginal bleeding B. Late menopause C. Abnormal vaginal discharge D. Enlarged uterus E. Hormone replacement therapy

A Endometrial cancer is a disease that typically presents with symptoms and clinical findings that lead to an early diagnosis. The most common symptom is abnormal postmenopausal bleeding. However, other symptoms or clinical findings that may be seen include abnormal vaginal discharge and lower abdominal discomfort. Endometrial cancer can increase the size of the uterus as it grows, but is usually not the most common finding given the early diagnosis of this cancer. Unopposed estrogen replacement therapy does increase the risk, but not when taken in combination. Early menarche and late menopause are additional risk factors that may be related to endometrial cancer development.

A 22-year-old G1 woman with LMP six weeks ago presents for elective termination of pregnancy. She is healthy with no medical problems. An ultrasound performed in the office shows an 8 mm endometrial stripe with no intrauterine gestational sac and no adnexal masses. Which of the following is the most appropriate next step in the management of this patient? A. Obtain a Beta-hCG level B. Perform dilation and curettage C. Give patient methotrexate D. Perform a diagnostic laparoscopy E. Prescribe mifepristone and misoprostol

A Even though the patient reports being pregnant, she is asymptomatic with no gestational sac in the uterus. First step in her management is to establish pregnancy by obtaining a Beta-hCG level. One should not assume she has an intrauterine pregnancy and perform a dilation and curettage or assume that she has an ectopic pregnancy and treat her with methotrexate or surgery until the pregnancy is confirmed.

A 24-year-old patient is considering pregnancy in the next year. Her medical history and physical examination are normal. She is unaware of any significant family history. She exercises regularly and does not smoke or drink alcohol. She asks if her diet has adequate folate and what you recommend to your patients. Which of the following statements about folate is correct? A. Folate supplementation is recommended for this patient B. Most patients get an adequate intake by diet alone C. She is not currently planning a pregnancy and does not need to worry about folate D. Current grain supplementation adequately prevents neural tube defects E. Folate does not need to be started until after a pregnancy is documented

A Folate lowers homocysteine levels. The Nurses Health Study showed fewer nonfatal MIs and fatal coronary events in women with adequate intake doses of folate and vitamin B6. Folate can also help prevent neural tube defects. Studies have shown that diet alone is not effective in achieving adequate levels and routine folate supplementation is therefore recommended. Women of reproductive age should take a daily 400-microgram supplement. Adequate levels are especially important prior to pregnancy and during the first four weeks of fetal development.

A 48-year-old G3P3 presents to the office for a health maintenance examination. Her past medical history, physical exam and labs are normal. Her body mass index (BMI) is normal. Her family history is significant for hypertension in her father and diabetes mellitus in her mother. Her grandfather passed away of colon cancer at the age of 82 and she is worried about getting colon cancer and desires to undergo screening. What is the most appropriate next step in the management of this patient? A. Recommend a colonoscopy at age 50 and, if normal, repeat every 10 years B. Recommend a colonoscopy at age 50 and then every two years C. Order a colonoscopy now and, if normal, repeat in five years D. Order a sigmoidoscopy now and, if normal, repeat in five years E. Recommend a sigmoidoscopy at age 50 and then every two years

A For patients with average risk for colon cancer, the recommended screening is to begin colonoscopy at age 50 and then every 10 years, if normal. Despite having a grandfather who passed away from colon cancer, this patient is not necessarily at increased risk and does not need to be screened at different intervals than the general population. Although a sigmoidoscopy can be an acceptable screening procedure, it would still begin at age 50 and repeat every five years, if normal.

A 60-year-old G4P4 woman presents with a two-year history of urine leakage with activity such as coughing, sneezing and lifting. Her past medical history is significant for vaginal deliveries of infants over 9 pounds. She had a previous abdominal hysterectomy and bilateral salpingo-oophorectomy for uterine fibroids. She is on vaginal estrogen for atrophic vaginitis. Physical exam shows no anterior, apical or posterior wall vaginal prolapse. Vagina is well-estrogenized. Post-void residual was normal. Q-tip test showed a straining angle of 60 degrees from the horizontal. Cough stress test showed leakage of urine synchronous with the cough. Cystometrogram revealed the absence of detrusor instability. The patient failed pelvic muscle exercises and was not interested in an incontinence pessary. Which of the following is the best surgical option for this patient? A. Retropubic urethropexy B. Needle suspension C. Anterior repair D. Urethral bulking procedure E. Colpocleisis

A Genuine stress incontinence (GSI) is the loss of urine due to increased abdominal pressure in the absence of a detrusor contraction. The majority of GSI is due to urethral hypermobility (straining Q-tip angle >30 degrees from horizon). Some (<10%) of GSI is due to intrinsic sphincteric deficiency (ISD) of the urethra. Patients can have both hypermobility and ISD. Retropubic urethropexy such as tension-free vaginal tape and other sling procedures have the best five-year success rates for patients with GSI due to hypermobility. Needle suspensions and anterior repairs have lower five-year success rates for GSI. Urethral bulking procedures are best for patients with ISD, but with little to no mobility of the urethra. Colpocleisis is one option to treat uterine prolapse, and is not indicated for urinary incontinence.

A 62-year-old G5P5 woman presents with a seven-month history of pelvic pain and pressure, as well as abdominal distention and bloating. She experiences occasional constipation, but no melena or hematochezia. She also has mild to moderate urinary frequency without dysuria, hematuria or flank pain. Her medical history is significant for hypertension and obesity. She went through menopause 12 years ago and has never been on hormone therapy. She reports one episode of light vaginal bleeding several months ago. Her family history is significant for postmenopausal ovarian cancer in her mother and maternal aunt, but is otherwise negative for breast, endometrial or colon cancer. Pelvic examination is remarkable for vaginal atrophy, cervical stenosis and difficult uterine and adnexal assessment due to her body habitus. What is the most appropriate next step in the management of this patient? A. Transvaginal ultrasound B. CT scan of the abdomen and pelvis C. Colonoscopy D. Hysteroscopy E. Diagnostic laparoscopy

A Given the patient's age, nonspecific abdomino-pelvic symptoms, recent postmenopausal bleeding episode and family history of ovarian cancer, a transvaginal ultrasound is the next best step as it is more sensitive than CT for evaluation of the uterus and adnexa. Colonoscopy is useful for colorectal cancer screening, as well as evaluation of the patient's gastrointestinal symptoms, but would not provide information regarding pelvic anatomy. Diagnostic laparoscopy would be a more invasive procedure that could be performed as indicated, after these other diagnostic studies. Hysteroscopy might be useful based on the ultrasound results, since it might be difficult to perform an endometrial biopsy in the office.

A 22-year-old G0 woman presents with worsening pelvic pain. She previously underwent a laparoscopic ablation of endometriosis followed by continuous oral contraceptive pills. She had short-term relief from this approach, but now has failed this treatment and is seeking additional medical management. Which of the following mechanisms best explains how a gonadotropin releasing hormone (GnRH) agonist would help alleviate her symptoms? A. Down-regulation of the hypothalamic-pituitary gland production B. Up-regulation of the hypothalamic-pituitary gland production C. Suppression of both LH and FSH mid-cycle surges D. Induction of a pseudopregnancy state E. Competitive inhibitor for estrogen receptors

A Gonadotropin-releasing hormone (GnRH) agonists are analogues of naturally occurring gonadotropin-releasing hormones that down-regulate hypothalamic-pituitary gland production and the release of luteinizing hormone and follicle-stimulating hormone leading to dramatic reductions in estradiol level. Numerous clinical trials show GnRH agonists are more effective than placebo and as effective as Danazol in relieving endometriosis-associated pelvic pain. Danazol, a 17-alpha-ethinyl testosterone derivative, suppresses the mid-cycle surges of LH and FSH. Combined estrogen and progestin therapy in oral contraceptives produces the pseudopregnancy state.

A 25-year-old G0 woman is scheduled to discuss her recent abnormal Pap smear which showed atypical squamous cells of undetermined significance (ASCUS). She has had one Pap smear at age 22 which was normal. Her only significant gynecologic history is genital warts that have not responded to treatment with local application of trichloroacetic acid. She has had eight sexual partners. She uses condoms and oral contraceptives. She has smoked a pack a day for the past two years. Which of the following is the most appropriate next step in the management of this patient? A. HPV typing B. Repeat Pap smear in three years C. Cone biopsy D. Cryotherapy E. Loop Electrosurgical Excision Procedure (LEEP)

A HPV typing is an option in the initial triage of the finding of atypical squamous cells of undetermined significance (ASCUS) on a Pap smear. If a high-risk HPV type is detected, then the patient needs a colposcopy with biopsies. An alternative approach can be close surveillance with repeat Pap smears in 12 months and if normal, she may return to routine screening. Initiation of treatment by way of cone biopsy, LEEP, or cryotherapy is not indicated at this time without a biopsy-confirmed diagnosis of cervical dysplasia.

A 25-year-old G2P1 woman presents at 26 weeks gestation with preterm labor. She is currently receiving tocolytic therapy with magnesium sulfate. The patient's nurse is concerned the patient may have magnesium sulfate toxicity. The patient is alert and has no complaints. Her contractions have stopped and her vital signs are stable. Which of the following findings associated with magnesium sulfate treatment would this patient experience before she develops respiratory depression? A. Areflexia B. Hyperreflexia C. Tachycardia D. Hypertension E. Oligouria

A High levels of magnesium sulfate may cause respiratory depression (12-15 mg/dl) or cardiac depression (>15 mg/dl). Prior to developing respiratory depression the patient should have diminished or absent deep tendon reflexes (areflexia).

A 35-year-old G2P2 comes to the office due to heavy menstrual periods. The heavy periods started three years ago and have gradually worsened in amount of flow and duration. The periods are now interfering with her daily activities. The patient had two spontaneous vaginal deliveries. She smokes one pack of cigarettes per day. On pelvic examination, the cervix appears normal and the uterus is normal in size, without adnexal masses or tenderness. A urine pregnancy test is negative. TSH and prolactin levels are normal. Hemoglobin is 12.5 mg/dl. On pelvic sonography, a 2 cm submucosal leiomyoma is noted. An endometrial biopsy is consistent with a secretory endometrium; no neoplasia is found. Which of the following would be the best therapeutic option for this patient if she desires to have another child? A. Hysteroscopy with myoma resection B. Laparoscopic myomectomy C. Endometrial ablation D. Oral contraceptives E. Dilation and curettage

A Hysteroscopic myomectomy preserves the uterus, while removing the pathology causing the patient's symptoms. A laparoscopic approach is not indicated as the myoma is submucosal and not accessible using a laparoscopic approach. Endometrial ablation destroys the endometrium and can create Asherman's syndrome, thus it is reserved for patients who have completed childbearing. Dilation and curettage is unlikely to remove the myoma and is a blind procedure (carried out without direct visualization). Oral contraceptives would typically help with heavy menses, but are contraindicated in this patient, who is over 35 and smokes

A 20-year-old nulliparous woman presents requesting birth control pills. She received the HPV vaccine series last year, and had her first sexual encounter last month. Otherwise, she is in good health and is a non-smoker. Her pelvic examination reveals normal external genitalia, and a nulliparous cervix without discharge or mucosal lesions. A urine pregnancy test is negative. Which of the following is the appropriate screening recommendation for this patient? A. Return next year for a Pap smear B. Return in three years for a Pap smear C. Perform a Pap smear now D. Perform HPV testing E. Perform colposcopy

A In 2013, the American Congress of Obstetrics and Gynecology (ACOG) updated the following recommendations for cervical cancer screening: • Cervical cancer screening should start at age 21 years. • Women aged 21 - 29 years should have a Pap test every three years. • Women aged 30 - 65 years should have a Pap test and an HPV test (co-testing) every five years (preferred). It is acceptable to have a Pap test alone every three years. • Women should stop having cervical cancer screening after age 65 years if they do not have a history of moderate or severe dysplasia or cancer and they have had either three negative Pap test results in a row, or two negative co-test results in a row within the past 10 years, with the most recent test performed within the past five years. • Women who have a history of cervical cancer, are infected with HIV, have a weakened immune system, or who were exposed to DES before birth should not follow these routine guidelines. ACOG recommends that women who have been vaccinated against HPV should follow the same screening guidelines as unvaccinated women.

A 79-year-old woman is seen in the emergency room in a comatose state. Her daughter states that her mother is always wandering away from the home. She has tried to tie her to her bed, but has found that if she gives her zolpidem (Ambien) every four hours, it keeps her sedated. The daughter works during the day and would put her mother in a nursing home, but she needs the mother's social security check to maintain the household. Examination reveals an unresponsive, thin, dehydrated woman with acute and chronic bedsores. After hydration and treatment of her skin condition in the hospital, which of the following is the most appropriate next step? A. Notify social services B. Notify hospital security C. Discharge patient to hospice D. Consult psychiatry E. Decrease the zolpidem dose

A In a case of obvious elderly abuse, social services must be notified. Physicians must be aware of the social services available for their patients. The patient should not be taking zolpidem, and this should also be discontinued. The patient may ultimately be discharged home or possibly to a skilled nursing facility, depending on her needs, which should be done in consultation with social services and discharge planning. At this time, hospice or consultation with psychiatry are not indicated.

A 28-year-old G1 at approximately 40 weeks gestation presents to triage with mild contractions. You measure her fundal height at 34 cm. You are concerned about intrauterine growth restriction and you want to confirm her dates. In reviewing her records, she reports first feeling fetal movements at 18 weeks gestation. The crown-rump length measurements determined at eight weeks and femur length at 20 weeks are consistent with 40 weeks gestation. Today's assessment reveals biometrics consistent with 34 weeks, amniotic fluid index of 1, and placental calcifications. Which of the following is considered the most reliable method of determining the gestational age in this patient? A. Crown-rump length measurement B. Second trimester ultrasound C. Quickening date D. Third trimester composite biometry E. Placental calcifications

A In order to accurately confirm gestational age at term, one of the following criteria should be met: Fetal heart tones have been documented for 20 weeks by a non-electronic fetoscope or for 30 weeks by Doppler; it has been 36 weeks since a positive serum or urine HCG pregnancy test was performed by a reliable laboratory; an ultrasound measurement of the crown-rump length, obtained at six to twelve weeks, supports a gestational age of at least 39 weeks; and an ultrasound obtained at 13-20 weeks confirms the gestational age of at least 39 weeks, determined by clinical history and physical examination. The crown-rump length can reliably date a pregnancy within five to seven days.

A 29-year-old G1P0 at 42 weeks gestation presents to labor and delivery because of intermittent contractions. She denies ruptured membranes. Her prenatal course was uncomplicated. Her vital signs are: blood pressure 140/96; pulse 72; afebrile; fundal height 32 cm; and estimated fetal weight of2900 gm. Cervix is closed, 25% effaced, -2 station. The fetal heart rate tracing shows occasional late decelerations. Of the following, what is the nextbest step in management? A. Maternal left lateral position B. Intrauterine resuscitation with terbutaline C. Start an amnioinfusion D. Begin magnesium sulfate E. Augment labor with oxytocin

A Initial measures to evaluate and treat fetal hypoperfusion include a change in maternal position to left lateral position which increases perfusion to the uterus, maternal supplemental oxygenation, treatment of maternal hypotension, discontinue oxytocin, consider intrauterine resuscitation with tocolytics and intravenous fluids, fetal acid-base assessment with fetal scalp capillary blood gas orpH measurement. An amnioinfusion may be used to treat patients with variable decelerations. Measures to improve uteroplacental blood flow should be attempted prior to proceeding with Cesarean delivery. Magnesium sulfate is not yet indicated in this patient with one slightly elevated blood pressure. Augmentation of labor mayaccentuate the late decelerations.

A 16-year-old girl is brought by her mother to her primary care physician requesting that her daughter have a gynecological examination. She is fearful that her daughter is sexually active. A private conversation with the girl reveals that she is sexually active and is on oral contraceptives prescribed by a community clinic. Having seen what her older sister went through when she became sexually active, she prefers not to tell her mother about her sexual activity for fear of severe punishment. The pelvic examination is normal. Which is the most appropriate action at this time? A. Tell the mother that her daughter is healthy B. Insist to the daughter that she disclose her sexual activity to her mother C. Reassure the mother that her daughter's sexual activity is a normal activity for most girls her age D. Speak to the mother openly about her daughter's condition E. Disclosure of any medical information is forbidden and a major HIPPA violation

A It is not appropriate to discuss the daughter's sexuality with her mother, although her mother has the right to know about her daughter's overall health as her legal guardian. The daughter should be encouraged to have an open conversation with her mother, but that this is entirely up to her discretion and that she will be supported in whatever decision she ultimately makes about disclosing her sexual activity.

A 29-year-old G1P0 at 42 weeks gestation presents in labor. She denies ruptured membranes. Her prenatal course was complicated by chronic hypertension. Her vital signs are: blood pressure 130/80; pulse 72; afebrile; fundal height 36 cm; and estimated fetal weight of 2400 gm. Cervix is dilatedto 4 cm, 100% effaced, -1 station, and bulging bag of water. The fetal heart rate tracing reveals five contractions in 10 minutes and repetitive late decelerations. What is the most likely cause of her late decelerations? A. Uteroplacental insufficiency B. Umbilical cord compression C. Uterine hyperstimulation D. Occiput posterior position E. Fetal head compression

A Late decelerations when viewed as repetitive and/or with decreased variability are an ominous sign. The can be associated with uteroplacental insufficiency as a result of decreased uterine perfusion or placental function, thus leading to fetal hypoxia and acidemia. Common causes include chronic hypertension and postdate pregnancies. Variable decelerations are associated with cord compression. Uterine hyperstimulation may cause prolonged bradycardia. Occiput posterior position should not be the cause of late decelerations. Fetal head compression may be associated with early decelerations.

A 28-year-old G2P0 (SAB2) experienced her second miscarriage within 14 months. A recent ultrasound was notable for two uterine fibroids. The patient is worried that the fibroids may have caused her early pregnancy losses. She is otherwise healthy and has no previous surgeries. She presents to you for further consultation. Which type of fibroid is more likely to cause subfertility? A. Submucosal B. Intramural C. Subserosal D. Pedunculated E. Cervical

A Leiomyomas are an infrequent cause of infertility, either by mechanical obstruction or distortion (and interference with implantation). When a mechanical obstruction of fallopian tubes, cervical canal or endometrial cavity is present and no other cause of infertility or recurrent miscarriage can be identified, myomectomy is usually followed by a prompt achievement of pregnancy. Submucosal myomas are most likely to cause infertility. Presumed mechanisms include: 1) focal endometrial vascular disturbance; 2) endometrial inflammation, and; 3) secretion of vasoactive substances. Submucosal fibroids are best treated by hysteroscopic resection.

A 30-year-old G1P1 woman presents with a history of chronic vulvar pruritus. The itching is so severe that she scratches constantly and is unable to sleep at night. She reports no significant vaginal discharge or dyspareunia. She does not take antibiotics. Her medical history is unremarkable. Pelvic examination reveals normal external genitalia with marked lichenification (increased skin markings) and diffuse vulvar edema and erythema as shown in picture below. Saline microscopy is negative. Potassium hydroxide testing is negative. Vaginal pH is 4.0. The vaginal mucosa is normal. Which of the following is the most likely diagnosis in this patient? A. Lichen simplex chronicus B. Lichen sclerosus C. Lichen planus D. Candidiasis E. Vulvar cancer

A Lichen simplex chronicus, a common vulvar non-neoplastic disorder, results from chronic scratching and rubbing, which damages the skin and leads to loss of its protective barrier. Over time, a perpetual itch-scratch-itch cycle develops, and the result is susceptibility to infection, ease of irritation and more itching. Symptoms consist of severe vulvar pruritus, which can be worse at night. Clinical findings include thick, lichenified, enlarged and rugose labia, with or without edema. The skin changes can be localized or generalized. Diagnosis is based on clinical history and findings, as well as vulvar biopsy. Treatment involves a short-course of high-potency topical corticosteroids and antihistamines to control pruritus.

A 27 year-old G1P0 at 22 weeks gestation with systemic lupus erythematosus (SLE) presents complaining of malaise, joint aches, and fever. Physical examination reveals the following: pulse 88, temperature 98.6°F (37.0°C), respiratory rate of 22, and BP 150/110 (baseline is 100/70.) Laboratory analysis reveals 1 + proteinuria, AST 35, and ALT 28. Which of the following is the most appropriate initial therapy for the treatment of this patient? A. steroids B. nonsteroidal anti-inflammatory drugs (NSAIDs) C. azathioprine D. cyclophosphamide E. magnesium sulfate

A Lupus is notoriously variable in its presentation, course, and outcome. Clinical manifestations include malaise, fever, arthritis, rash, pleuropericarditis, photosensitivity, anemia, and cognitive dysfunction. A significant number of patients have renal involvement. There is no cure, and complete remissions are rare. Mild disease may be disabling because of pain and fatigue. Nonsteroidal anti-inflammatory drugs are used to treat arthralgia and serositis. Severe disease is managed with corticosteroids. Hydroxychloroquine is used to help control skin manifestations and may be associated with lupus flares if discontinued.

A 40-year-old G1P0 woman at 34 3/7 weeks gestation was found on the floor at work having a grand mal seizure. Her airway was secured. Blood pressure in the ambulance was 140/90. What is the initial treatment for her condition? A. Magnesium sulfate B. Valium C. Hydantoin D. Phenobarbital E. Naloxone

A Magnesium sulfate is the treatment of choice for eclampsia, which is her most likely diagnosis. Valium, hydantoin, tiagabine, and barbiturates can also be used to treat seizures, but are not first-line therapy for eclampsia. They can be added as second agents, or used if magnesium is contraindicated. Naloxone (Narcan) is a drug used to counter the effects of opioid overdose, for example heroin or morphine overdose, and is specifically used to counteract life-threatening depression of the central nervous system and respiratory system.

A 35-year-old G0 comes to the office because of six months of spotting between her periods and a desire for a pregnancy. She reports using 30 pads/cycle the last two months and has blood clots and cramping pain. Prior menses were light and required 15 pads/cycle. She has been trying to conceive for six months. Her work-up included a transvaginal ultrasound which revealed a 2 cm endometrial polyp. What is the next best step in the management of this patient? A. Hysteroscopic polypectomy B. Observation C. Combination birth control pills D. Endometrial ablation E. Hysterectomy

A Management of an endometrial polyp includes the following: observation, medical management with progestin, curettage, surgical removal (polypectomy) via hysteroscopy, and hysterectomy. Observation is not recommended if the polyp is > 1.5 cm. In women with infertility polypectomy is the treatment of choice. Hysterectomy is reserved for women with polyps and premalignant or malignant changes.

A 24-year-old G1 woman at 32 weeks gestation presents with leaking watery fluid from the vagina. On evaluation, preterm premature rupture of membranes is confirmed. She has occasional Braxton Hicks contractions associated with fetal heart rate accelerations. She does not have vaginal bleeding and vaginal fluid phosphatidylglycerol is absent. Her blood pressure is 110/70; pulse 90; temperature 98.6°F (37.0°C). Fundal height is 30 cm and her fundus is tender. Amniotic fluid index (AFI) is 4. Which of the following findings is an indication for delivery in this patient? A. Tender uterine fundus B. Size less than dates C. Fetal heart rate accelerations D. Amniotic fluid index of less than 5 E. Absence of vaginal fluid phosphatidylglycerol

A Maternal signs of chorioamnionitis or other evidence of intra-amniotic infection are indications for delivery. This patient has ruptured membranes and a tender fundus, which indicate chorioamnionitis. Labor at 32 weeks would be allowed to progress and prolonged non-reassuring fetal testing would prompt delivery. There are no criteria for amniotic fluid index or degree of oligohydramnios as an indication for delivery. Most authors agree that the achievement of fetal lung maturity (i.e. positive phosphatidylglycerol or 34 weeks gestational age) is the threshold at which the risk of morbidity and mortality of maintaining the pregnancy in utero outweighs the benefits of prolonging the pregnancy.

A 44-year-old G1P1 was placed on three months of a GnRH agonist in order to diminish the size of a 5 cm submucosal myoma and allow it to be accessible to a hysteroscopic removal. About two weeks prior to surgery, she was no longer having severe menorrhagia although the drug side effects were becoming incapacitating. During the pre-operative visit, she asks you if she could simply stop the GnRH agonist and wait to see if her symptoms eventually return. What can you inform her about the response of the fibroids after the cessation of GnRH agonist therapy? A. Will not grow B. Will resume their former growth potential C. Become calcified D. Will grow at a more rapid rate E. Will re-grow but to only about half their original size

A Maximal response is usually achieved by three months of GnRH agonist treatment. The reduction in size correlates with the estradiol level and with body weight. Hot flashes are experienced by >75% of patients, usually in three to four weeks after start of treatment, although they should not persist for longer than one to two months from end of treatment. After cessation of treatment, menses return in four to ten weeks, and myoma and uterine size return to pretreatment levels in three to four months. The regrowth is consistent with the fact that reduction in size is not due to a cytotoxic effect. However, it is not true that secondary to the GnRH agonist withdrawal they will grow at a more rapid rate.

A 27-year-old patient with no significant prior medical history reports three months of low energy, lack of enjoyment with her daily activities, early morning awakening, and trouble concentrating. What is the next best step in the assessment of this patient? A. Risk of suicide B. Willingness to accept medical treatment C. Family history of mental illness D. Good support system E. Current medication profile

A Most depressed patients who are suicidal are relieved to be asked about it. Although all the items listed are components of a complete history, the most important topic is assessment of suicide risk.

A 52-year-old G3P3 woman presents to your office with severe hot flashes and vaginal dryness for six months. Her last menstrual period was 15 months ago. After discussing the risks and benefits of hormone therapy with this patient, she decides to begin treatment. This patient is most likely to stop hormone therapy secondary to what side effect? A. Vaginal bleeding B. Development of breast cancer C. Hirsutism D. Nausea E. Relief of menopausal symptoms

A Most irregular bleeding due to initiation of hormone therapy occurs in the first six months. It has been cited as the most common reason as to why women stop hormone therapy. Women who are amenorrheic for some period of time are often disturbed by the resumption of any vaginal bleeding/spotting and find it intolerable.

A 25-year-old G1P1 who is breastfeeding her 2 ½-week-old comes to the office with left breast pain and fever. The symptoms began earlier today and are not relieved by acetaminophen. She has no known drug allergies. Blood pressure 120/60; pulse 64; temperature 99.9° F (37.7° C). On exam, there is erythema on the upper outer quadrant of the left breast, which is tender to touch; there are no palpable masses. What is the most appropriate antibiotic therapy for this patient? A. Dicloxacillin B. Erythromycin C. Doxycycline D. Gentamicin E. Cefotetan

A Most postpartum mastitis is caused by staphylococcus aureus, so a penicillin-type drug is the first line of treatment. Dicloxacillin is used due to the large prevalence of penicillin resistant staphylococci. Erythromycin may be used in penicillin allergic patients.

A 42-year-old woman presents to the office with a breast mass. She noticed a firm, slightly tender mass on her right breast during a self breast exam two months ago. She has not noticed nipple discharge or skin changes. It has not enlarged nor decreased in size. The review of systems is negative. She does not use tobacco, alcohol or drugs, and she drinks five cans of caffeinated soda per day. She has a maternal aunt with breast cancer who was diagnosed at age 50. On examination, the left breast is normal; right breast has a 1 cm x 2 cm rubbery mobile mass in the upper outer quadrant. There was no nipple discharge, no visible skin changes, and no axillary or supraclavicular lymphadenopathy. A mammogram three months ago was normal and showed dense breast tissue bilaterally. What is the best next step in the management of this patient to help determine her diagnosis? A. Fine needle aspiration of the mass B. Observation for six months and reexamine C. Repeat mammogram D. Excisional biopsy of the mass E. Decrease caffeine intake and reexamine

A Needle aspiration of a palpable breast mass or lymph node allows for pathologic diagnosis of the mass with minimal discomfort to the patient. Results correlate well with excisional biopsy results. Observation or waiting for the patient to decrease caffeine intake would not be recommended for a patient with a new finding of a palpable breast mass, especially in a patient with a family history of breast cancer. A mammogram does not need to be repeated since one was performed three months ago. Excisional biopsy is not necessary at this point although she might ultimately require it.

A 26-year-old G2P0 woman presents for counseling following manual vacuum aspiration of an eight-week missed abortion. The patient asks whether an uncomplicated first trimester pregnancy termination three years ago might have predisposed her to the subsequent spontaneous abortion. What are the patient's risks associated with the prior surgical abortion in the first trimester? A. Does not predispose the patient to subsequent spontaneous abortion B. Increases the risk of spontaneous abortion two-fold C. Predisposes the patient to primary infertility D. Increases the likelihood of subsequent pregnancy loss in both the first and second trimesters E. Increases the likelihood of spontaneous abortion and future delivery complications

A Neither controlled trials nor surveillance data support the contention that a single, prior first trimester surgical abortion increases the risk of subsequent first trimester pregnancy loss. Indeed, first trimester surgical abortion confers no subsequent obstetric disadvantage, particularly when compared with an appropriate control population. The clinician should reassure this patient that first trimester spontaneous abortion is a common occurrence and that she has not caused this missed abortion.

At the time of physical examination, detection of a lower-abdominal tumor in a 7-year-old girl is best accomplished by palpation of the abdomen coupled with: A) Rectal examination B) Vaginal examination C) Rectovaginal examination D) Abdominal auscultation and percussion

A) Rectal examination

You are counseling a 22-year-old G1 who is at 38 weeks gestation. She has been pushing for four hours and you recommend a trial of forceps. She asks whether a vacuum-assisted delivery is associated with a lower incidence of maternal and fetal trauma. Which of the following is less likely to occur during a vacuum delivery? A. Maternal lacerations B. Fetal cephalohematoma C. Neonatal lateral rectus paralysis D. Neonatal hyperbilirubinemia E. Neonatal retinal hemorrhage

A Newer forms of vacuum extractors cause less maternal discomfort as they are applied to the vertex of the fetal head and do not take up additional space in the maternal pelvis. If properly applied, this leads to a decreased rate of maternal lacerations. Fetal and neonatal complications related to vacuum use include lacerations at the edges of the vacuum cup, particularly if torsion is applied. Torsion may also lead to separation of the fetal scalp from the underlying structures can cause a cephalohematoma and places the fetus at risk of jaundice. Transient neonatal lateral rectus paralysis has been found to occur more frequently in vacuum-assisted deliveries, but, because the paralysis resolves spontaneously, it is unlikely to be of clinical importance.

A 26-year-old G2P1 woman at 33 weeks gestation presents in preterm labor. She has a history of a prior preterm birth at 32 weeks gestation. She has insulin dependent diabetes and has a history of myasthenia gravis. She has regular contractions every three minutes and fetal heart tones are reassuring. Cervix is 3 cm dilated and 0 station. Her blood pressure is 140/90. Which of the following is the most appropriate tocolytic agent to use in this patient? A. Nifedipine B. Terbutaline C. Magnesium sulfate D. Indomethacin E. Ritodrine

A Nifedipine, a calcium channel blocker is the best option for her as she has contraindications to the other agents listed. Terbutaline and ritodrine are contraindicated in diabetic patients and the FDA made a formal announcement in 2011 warning against using terbutaline to stop preterm labor stating that terbutaline is both ineffective and dangerous if used for longer than 48 hours; magnesium sulfate is contraindicated in myasthenia gravis; and indomethacin is contraindicated at 33 weeks due to risk of premature ductus arteriosus closure.

A 27-year-old G1P0 complains of mood swings and fatigue in the week prior to her menstrual period. These symptoms have worsened over the past six months. Some months the symptoms are so severe she misses several days of work. Her medical history is otherwise unremarkable and a physical examination is normal. Which of the following is the most appropriate next step in this patient's management? A. Obtain a symptom diary for two months B. Recommend dietary changes C. Begin treatment with an anxiolytic agent D. Refer for psychiatric consultation E. Obtain a pelvic ultrasound

A Obtaining further history with a menstrual calendar determines the cyclic nature of the PMS or PMDD symptoms and helps guide appropriate therapy. While dietary changes may help, it is first important to establish the diagnosis. An anxiolytic agent or psychiatric consultation is not indicated.

A 32-year-old G2P1 is at 41 weeks gestation. Her cervix is long and closed. She does not report contractions and states there is good fetal movement. The patient strongly desires not to have an induction of labor. She would like to wait until she goes into labor spontaneously. Which of the following treatment options is optimal at this time? A. Perform a non-stress test (NST) and amniotic fluid index (AFI) twice a week with induction of labor for a nonreactive non-stress test or oligohydramnios B. Patient should perform daily fetal movement counts and proceed with induction for decreased fetal movement C. Perform daily biophysical profiles and deliver if 4 or less D. Immediate induction of labor E. Immediate Cesarean section

A Optimal management for the patient with an unfavorable cervix at 42 weeks gestation is controversial. Induction of labor in a patient with a reactive tracing and an unfavorable cervix will minimize any risk of antepartum fetal demise; however, the risk of Cesarean section is significantly increased compared to a patient who goes into spontaneous labor. It is reasonable to follow a patient who is 41 weeks with antepartum fetal testing, such as twice weekly NSTs with amniotic fluid assessment. The risk of fetal death is 1-2/1,000 high-risk pregnancies with a reassuring non-stress test, contraction stress test or biophysical profile. The addition of amniotic fluid assessment may improve the predictive value of a reactive NST and reduce the risk of antepartum fetal demise to even lower levels.

A 19-year-old G1 woman presents in labor at term. Her prenatal course was uncomplicated. She delivers a 3500 gram infant spontaneously after oxytocin augmentation of labor. Postpartum, she experiences excessive bleeding. Which of the following defines postpartum hemorrhage in this patient? A. Greater than 500 cc B. Greater than 750 cc C. Greater than 1000 cc D. Greater than 1500 cc E. Any amount of bleeding that leads to hypovolemia

A Postpartum hemorrhage is defined as bleeding in excess of 500 cc after a vaginal delivery or in excess of 1000 cc after a Cesarean delivery.

A 29-year-old G1 is at 42 weeks gestation based on her last menstrual period and a first trimester ultrasound. Of the following, what factor is most likely to be associated with postterm pregnancy? A. Placental sulfatase deficiency B. Fetal adrenal hyperplasia C. Fetal alpha-fetoprotein deficiency D. Fetal renal anomalies E. Fetal chromosomal abnormalities

A Postterm pregnancies are associated with placental sulfatase deficiency, fetal adrenal hypoplasia, anencephaly, inaccurate or unknown dates and extrauterine pregnancy.

A 36-year-old health worker presents for a health maintenance examination. She is sexually active and not using contraception. She doesn't think she is pregnant, but would be happy if she were. As part of her general preventive care, you discuss immunizations. Which vaccination is contraindicated if this patient gets pregnant now? A. Measles-Mumps-Rubella (MMR) B. Pneumococcus C. Hepatitis B D. Polio E. Influenza

A Pregnancy or the possibility of pregnancy within four weeks is a contraindication to the MMR and varicella vaccinations. Tetanus, Hepatitis B, Polio and Pneumococcal vaccinations would not be contraindicated.

A 25-year-old woman, gravida 3, para 2, comes in for a routine prenatal visit. Seasonal influenza vaccines have just been released. She is concerned about the risks she has heard about in the media, and wants to avoid any adverse consequences for her and the fetus. In order to maximize the benefit and minimize the risk for the mother and baby, the best recommendation for timing of this intramuscular vaccination is . A) as soon as possible during this pregnancy B) during the third trimester of pregnancy C) immediately after delivery D) after breastfeeding is completed E) not indicated for low-risk women during pregnancy

A) as soon as possible during this pregnancy

A 25-year-old G2P1 woman at 20 weeks gestation is diagnosed with preterm premature rupture of the membranes. She denies labor. She takes prenatal vitamins and iron. She denies substance abuse, smoking or alcohol use. Her prior pregnancy was delivered vaginally at 36 weeks after preterm rupture of membranes. Her blood pressure is 110/70; pulse 84; temperature 98.6°F (37.0°C). The patient's fetus is greatest risk for which of the following? A. Pulmonary hypoplasia B. Cardiac anamolies C. Urinary tract anamolies D. Microcephaly E. Compression fractures

A Preterm premature rupture of the membranes that occurs before viability is associated with significant risk of poor outcome. Neonatal survival when rupture occurs between 20 and 23 weeks is approximately 25%. Complications that may be found in the developing fetus include structural abnormalities that are primarily deformations (abnormalities that occur due to an insult after a structure has already formed) rather than malformations (abnormal development of the structure itself). Pulmonary hypoplasia is seen when rupture of membranes occurs before 25 weeks gestation because the lack of amniotic fluid interferes with the normal intrauterine breathing process. The result is failure of normal development and growth of the respiratory tree.

A 27-year-old G0 presents to the clinic because of concerns that she has not been able to get pregnant for the last three months. She married a year ago and was using condoms for contraception, which she stopped three months ago when she decided to start a family. She is in good health and her only medication is a prenatal vitamin. Her periods are regular every 28 days with normal flow; her last period was two weeks ago. She has no history of sexually transmitted infections and no abnormal Pap smears. Her husband is also healthy with no medical problems. She is 5 feet 4 inches tall and weighs 130 pounds. Her examination, including a pelvic exam, is completely normal. What is the most appropriate next step in the management of this patient? A. Reassurance and observation B. Order a hysterosalpingogram C. Order a transvaginal pelvic ultrasound D. Recommend husband gets a semen analysis E. Order mid-cycle blood LH and FSH levels

A Reassurance and observation is most appropriate as the patient has only been trying to conceive for three months. After one month, 20% of couples will conceive; after three months, 50%; after six months, 75%; and after 12 months, 90% will conceive. Primary infertility is defined as inability to conceive for one year without contraception. The patient is young and healthy with no obvious reasons for infertility, so at this point reassurance and observation is the proper management. There is no reason to order any studies now, especially since she has normal cycles.

A 17-year-old is brought to the physician because she has never had a menstrual cycle. She has normal breast and pubic hair development. Physical examination reveals a small vaginal opening with a blind pouch. Pelvic ultrasound reveals normal ovaries, but absence of uterus and cervix. Which of the following is the most appropriate next study in this patient? A. Renal ultrasound B. FSH and LH determination C. Karyotype D. Cortisol level E. Testosterone level

A Renal anomalies occur in 25-35% of females with Mullerian agenesis. The uterus and cervix are absent, but the ovaries function normally and, therefore, secondary sexual characteristics are present. You would expect the karyotype in this patient to be 46,XX and testosterone levels in the female range.

A 17-year-old patient presents with hirsutism, irregular menses and obesity. Her mother is moderately obese with mild hirsutism. Recently, the patient's hirsutism has worsened and she has been depressed. She has also gained 20 pounds in the past two months and has noticed stretch marks on her abdomen. At the time of your examination, you note that she has terminal hair growth on her chin and hair growth on the back of her hands. Her cheeks appear flushed. Her stretch marks are purplish in color. The rest of her exam is normal. Which of the following is the most appropriate first test to order for this patient? A. Overnight dexamethasone suppression test B. 17-hydroxyprogesterone C. Fasting insulin D. TSH E. Pelvic ultrasound

A Since Cushing's syndrome is suspected, either a dexamethasone suppression test or a 24-hour urinary measurement for cortisol can be performed. Elevated cortisol would be indicative of Cushing's syndrome. The other tests listed would be reasonable, but only after Cushing's syndrome had been excluded.

A 32-year-old G1 presents at 35 weeks gestation with decreased fetal movement. Her prenatal course has been complicated by size less than dates. Serial ultrasounds show a decrease of the estimated fetal weight from 60th to 20th percentile. The non-stress test is reactive and the amniotic fluid index is 10. What is your next step in management? A. Continue with weekly non-stress tests B. Obtain umbilical artery systolic: diastolic ratio C. Admission for daily fetal surveillance D. Induction of labor today E. Cesarean section today

A Since the patient reported decreased fetal movement, a non-stress was performed and was reassuring. The NST is based on the principle that when the fetus moves, its heartbeat normally accelerates. The NST assesses fetal health through monitoring accelerations of the heart rate in response to the baby's own movements. Amniotic fluid volume is important because a decreased amount raises the possibility that the fetus may be under stress. Since the fetus does not show growth restriction and fetal status was reassuring, there are no indications for Doppler studies or delivery. In light of the dramatic decrease in growth, it is reasonable to follow this patient with weekly non-stress tests.

A 21-year-old woman comes to the office because of acne, irregular menses and hirsutism. She initially was evaluated six months ago, at which time she was diagnosed with idiopathic hirsutism. She was started on oral contraceptive pills to improve her symptoms. Menstrual periods now occur every month, but her hirsutism has not significantly improved. In addition to the oral contraceptives, which of the following would be an appropriate treatment for hirsutism? A. Spironolactone B. Lupron C. Danazol D. Depo-Provera E. Steroids

A Spironolactone, an aldosterone antagonist diuretic, can also be used in addition to the oral contraceptives for hirsutism. Danazol is primarily used for the treatment of endometriosis and may actually worsen hirsutism and acne. Lupron and Depo-Provera are also reasonable as second-line treatments of hirsutism, had the patient not already been on oral contraceptives. Steroids will not help.

A 27-year-old multiparous woman comes to your office to discuss tubal ligation. She is currently taking no contraception. She worries that a tubal ligation could affect her menstrual periods. You tell her that the most likely result of tubal ligation on her menstrual periods will be: A) no change B) more painful menses C) heavier menses D) more bleeding days E) cessation of menses

A) no change

A 17-year-old woman comes to your office for her first gynecologic visit. She has been sexually active for the last year and always uses condoms. What is the most appropriate management regarding Pap smear screening for this patient? A. Pap smear at age 21 B. Pap smear at this visit and then annually C. Pap smear now and every 3 years D. Pap smear now and then every other year E. Pap smear at age 18

A The American Congress of Obstetricians and Gynecologists (ACOG) recommendation from 2009 is for patients to have an annual Pap smear starting at 21 years of age regardless of history of sexual activity. Cervical neoplasia develops in susceptible individuals in response to a sexually transmitted infection with a high-risk type of HPV (Human papillomavirus). HPV causes carcinogenesis in the transformation zone of the cervix, where the process of squamous metaplasia replaces columnar with squamous epithelium. Squamous metaplasia is active in the cervix during adolescence and early adulthood. Human papillomavirus infections are commonly acquired by young women shortly after the initiation of vaginal intercourse, but, in most, they are cleared by the immune system within one to two years without producing neoplastic changes. The recommendation to start screening at age 21 years regardless of the age of onset of sexual intercourse is based in part on the very low incidence of cancer in younger women. It is also based on the potential for adverse effects associated with follow-up of young women with abnormal cytology screening results.

A 29-year-old G1P0 at 41 weeks gestation presents in early labor. The prenatal course was uncomplicated. Ultrasound at 21 weeks was consistent with gestational age. Her vitals reveal a blood pressure of 128/76; pulse 74; and she is afebrile. Fundal height is 36 cm with estimated fetal weight of 2700 gm. Cervix is dilated to 1 cm, 50% effaced and the fetal vertex is at -2 station. The nurse calls you to evaluate the fetal tracing. Which statement best describes the tracing seen below? A. Normal fetal heart rate with good variability and regular contractions B. Fetal tachycardia with good variability and regular contractions C. Normal fetal heart rate with poor variability and regular contractions D. Fetal tachycardia with poor variability and irregular contractions E. Normal fetal heart rate with poor variability and irregular contractions https://www.apgo.org/images/26-2b.jpg

A The baseline fetal heart rate is normal with good accelerations and regular contractions. There is no tachycardia. This is a reassuring tracing.

A 24 year-old G1P0 at 12 weeks gestation presents for prenatal care. She is 5 feet 4 inches tall and weighs 220 pounds (BMI: 37.8 kg/m2). She wants to know if there is an increased risk on her pregnancy because of her size. Which of the following is the most common complication in this patient? A. Hypertension B. Preterm labor C. Post-term pregnancy D. Small for gestational age E. Shoulder dystocia

A The body mass index (BMI) is equal to a person's weight in kg divided by their height in meters squared. The National Heart, Lung, and Blood Institute identify a normal BMI as 18.5 to 24.9 kg/m2; overweight as a BMI of 25 to 29.9 kg/m2; and obesity as a BMI of 30 kg/m2 or greater. Obesity is further categorized as class I (BMI: 30 to 34.9 kg/m2), class II (BMI: 35 to 39.9 kg/m2), and class III (BMI: > or=40 kg/m2). Increased maternal morbidity results from obesity and includes chronic hypertension, gestational diabetes, preeclampsia, fetal macrosomia, as well as higher rates of Cesarean delivery and postpartum complications. This patient's BMI is approximately 38 so she is a class II and has over a 7-fold increase risk for preeclampsia and a 3-fold risk for hypertension.

A 39-year-old G0 woman presents to the clinic reporting non-tender spots on her vulva for about a week. No pruritus or pain is present. She also notes a brownish rash on the palms of her hands. She admits to IV drug abuse. She was diagnosed as HIV-positive two years ago, but has not been compliant with suggested treatment. On examination, three elevated plaques with rolled edges are noted on the vulva. They are non-tender. A brown macular rash is noted on the palms of her hands and the soles of her feet. What is the most appropriate next step in the management of this patient? A. Obtain a treponemal-specific test B. Biopsy of the lesion C. Colposcopic evaluation of the vulvar lesions D. Culture the base of the lesion E. Initiate empiric treatment with doxycycline and ceftriaxone

A The diagnosis of syphilis is often established by serologic testing. Non-treponemal tests (VDRL or RPR) are non-specific. In this patient with high suspicion for syphilis, specific testing with treponemal antibody can confirm infection. The classic coiled spirochete is easily seen with dark-field microscopy but availability is limited. A characteristic finding is a macular rash on the palms and soles that are often described as copper penny lesions. Colposcopy would not be diagnostic, but certainly is helpful to evaluate for any vulvar lesions thought to be dysplastic. Biopsies can be stained for spirochetes and may show a necrotizing vasculitis, but certainly would not be the most expedient way to make the diagnosis. Penicillin G is the preferred drug for treating all stages of syphilis.

"A 46-year-old female is scheduled to undergo a total laparoscopic hysterectomy for menorrhagia. She comes into your office to discuss the effects a hysterectomy will have on her sex life. She is concerned about how a hysterectomy will affect her ""womanhood"" and read in a woman's magazine that the cervix, as well as the uterus, is necessary for orgasm. Which of the following organs is most responsible for sexual arousal? A. Vulva B. Cervix C. Uterus D. Ovary E. All of the above"

A The external genitalia, specifically the clitoris, are essential organs involved in the arousal component of the sexual response cycle, which consists of desire, arousal, orgasm, and resolution. Hormones such as androgen and estrogen (produced by the ovaries) are key to desire, while genital mechanisms such as clitoral, labial, and vaginal engorgement are key to arousal. With arousal and adequate sensory stimulation, orgasm ultimately may occur consisting of repeated motor contractions of the pelvic floor including uterine and vaginal smooth muscle contractions.

A postmenopausal 60-year-old woman with estrogen receptor-positive breast cancer comes to see you for consultation. She has been taking tamoxifen citrate for 1 year and is anxious about the risk of developing endometrial cancer. Endometrial polyps have never been diagnosed and she has had no vaginal bleeding since menopause 10 years ago. The most appropriate next step for this patient. A) observation B) progestin withdrawal test C) transvaginal ultrasonography D) pelvic magnetic resonance imaging E) endometrial biopsy

A) observation

A 33-year-old G2P1 presents at 34 weeks gestation for consultation because ultrasound revealed a 3900 gm fetus with biometrics consistent with 39 weeks. Her prior pregnancy was complicated by gestational diabetes and a shoulder dystocia. Which of the following complications is this fetus at greatest risk? A. Birth trauma B. Hyperglycemia C. Hypobilirubinemia D. Hypothyroidism E. Congenital anomalies

A The fetus with enhanced general growth or macrosomia is defined by a birth weight at or above the 90th percentile for gestational age. The condition can usually be ascribed to one of three etiologies: enhanced growth potential (50-60%); abnormal maternal glucose homeostasis (35-40%); or underestimation of fetal age (5%). Macrosomic newborns of diabetic mothers experience excessive rates of neonatal morbidity, including birth trauma such as shoulder dystocia and brachial plexus injury. These infants have significantly higher rates of severe hypoglycemia and neonatal jaundice. Neonatal acidosis occurs with poor glycemic control, thus increasing the incidence of fetal demise. While poorly controlled pre-existing diabetes is associated with an increased risk of congenital anomalies, gestational diabetes is not associated with increased risk of congenital anomalies.

A 38-year-old G1P0 woman presents to the hospital at 39 weeks in early labor. She has had routine prenatal care and no antepartum complications to date. She reports good fetal movement and denies vaginal bleeding and leakage of fluid. What is the next best step in the initial assessment of this patient? A. Physical examination B. Nitrazine test C. Fetal ultrasound D. Biophysical profile E. Contraction stress test

A The initial evaluation of patients presenting to the hospital for labor includes a review of the prenatal records with special focus on the antenatal complications and dating criteria, a focused history and a targeted physical examination to include maternal vital signs and fetal heart rate, and abdominal and pelvic examination. A speculum exam with a nitrazine test to confirm rupture of membranes is indicated if the patient's history suggests this, or if a patient is uncertain as to whether she has experienced leakage of amniotic fluid. Performing a fetal ultrasound is not a routine part of an assessment in a patient who may be in early labor. A prenatal ultrasound may be used in cases to determine fetal presentation, estimated fetal weight, placental location or amniotic fluid volume.

A 24-year-old G1P1 presents with a complaint of decreasing breast size and hirsutism noted the last three months. She also notes her skin feels oily and her husband has mentioned her voice seems to be getting deeper. She has no medical or surgical problems and takes no medications. Physical examination reveals oily skin, upper lip and chin terminal hair, and normal appearing breasts. Pelvic examination reveals her clitoris to be 2 cm in length and 1 cm wide. Which of the following is the most likely cause of her symptom constellation? A. Steroid cell tumor B. Hypothyroidism C. Polycystic ovary syndrome D. Congenital adrenal hyperplasia (early onset) E. Anabolic steroids

A The likely cause of this patient's sudden onset of symptom is an increase in androgens due to a tumor. Hirsutism is often the result of a benign condition, however, may be a sign of significant disease if sudden in onset and coupled with virilization. Virilization in the female may be manifested by frontal hair thinning, oily skin or acne, deepening of the voice, clitoral enlargement, menstrual irregularities, and increased muscle strength. Possible causes of virilization include PCOS, hypothyroidism, androgen producing tumors (ovarian, adrenal, or pituitary), and anabolic steroid use. A rare cause may be late onset congenital adrenal hyperplasia.

A 34-year-old G2P2 presents with concerns of hormonal changes. She is worried about facial hair growth, worsening acne, and deepening of her voice. She also realized that she has missed her period for two months, and has been sexually active and had tubal ligation. On examination, she is moderately obese and noted to have severe acne, upper lip and chin terminal hair. Her abdomen is obese with moderate hair growth. Pelvic examination is most notable for an enlarged clitoris, and pelvic exam reveals an enlarged right-sided adnexal mass. Which of the following is the most likely diagnosis in this patient? A. Sertoli-Leydig cell tumors B. Granulosa cell tumor C. Benign cystic teratoma D. Thecoma E. Cystadenoma

A The most likely diagnosis in this patient is a testosterone-secreting ovarian tumor. Sertoli-Leydig cell tumors are commonly diagnosed in women between the ages of 20-40, and are most often unilateral. Rapid onset of hirsutism and virilizing signs are hallmarks of this disease, and include many of the findings in this patient including acne, hirsutism, amenorrhea, clitoral hypertrophy, and deepening of the voice. Abnormal laboratory findings include suppression of FSH and LH, marked elevation of testosterone, and presence of an ovarian mass. The constellation of findings is most consistent with a testosterone-secreting tumor, and a pelvic ultrasound will confirm the presence of an ovarian mass. The other tumors do not cause virilization. Granulosa cell tumors and thecomas are estrogen secreting tumors.

A 23-year-old G1P0 woman presents with cramping, vaginal bleeding and right lower quadrant pain. Her last normal menstrual period occurred seven weeks ago. On physical exam, vital signs are: blood pressure 110/74; pulse 82; respirations 18; and temperature 98.6°F (37°C). On abdominal exam, she has very mild right lower quadrant tenderness. On pelvic exam, she has scant old blood in the vagina and a normal appearing cervix. Her uterus is normal size and slightly tender. On bimanual exam, there is no cervical motion tenderness, and she has slight tenderness in right lower quadrant. Quantitative Beta-hCG is 2500 mIU/ml; progesterone 6.2 ng/ml; hematocrit 34%. The transvaginal ultrasound shows an empty uterus with endometrial thickening, a mass in the right ovary measuring 3 x 2 cm and a small amount of free fluid in the pelvis. Which of the following is the most appropriate next step in the management of this patient? A. Methotrexate B. Antibiotics C. Observation D. Dilation and curettage E. Culdocentesis

A The next best step in management is methotrexate administration. Certain conditions must be met prior to initiating methotrexate therapy for treatment of an ectopic pregnancy. These include: hemodynamic stability, nonruptured ectopic pregnancy, size of ectopic mass <4 cm without a fetal heart rate or <3.5 cm in the presence of a fetal heart rate, normal liver enzymes and renal function, normal white cell count, and the ability of the patient to follow up rapidly (reliable transportation, etc.), if her condition changes. There is no indication for antibiotics in this scenario. Offering observation delays treatment and pain control would not address the underlying cause of the patient's problem. Culdocentesis is not indicated and does not change the management of this patient.

A 29-year-old G1P0 at 41 weeks gestation presents for a prenatal visit. Her prenatal course is complicated by tobacco abuse and intermittent prenatal care. Her last visit was at 35 weeks. Prenatal labs are unremarkable except cervical DNA probe positive for Chlamydia, which was treated, and a Pap smear with low-grade squamous intraepithelial lesion. Ultrasound at 21 weeks was consistent with gestational age. Her vitals reveal a blood pressure of 128/76; pulse 74; and temperature 98° F (36.7° C). Fundal height is 39 cm with estimated fetal weight of 2700 gm. Cervix is dilated to 1 cm, 50% effaced, -2 station. What is the next best step in the management of this patient? A. Non-stress test B. Vibroacoustic stimulation test C. Oxytocin challenge test D. Return visit in one week E. Cesarean section

A The non-stress test is an assessment of fetal well-being that measures the fetal heart rate response to fetal movement. The normal or reactive non-stress test occurs when there are two fetal heart rate accelerations of 15 beats/minute for 15 seconds within 20 minutes. Vibroacoustic stimulation is not indicated unless the NST is non-reactive. Contraction stress test assesses uteroplacental insufficiency and looks for persistent late decelerations after contractions (3/10 minutes); however, it is not necessary to perform, as the non-stress test will assess fetal well being, as well. Observation only would not be proper care as the patient is post-term. In the presence of abnormal testing, labor would be induced or a Cesarean section performed.

A 22-year-old G3P0 woman at 37 weeks gestation with an uncomplicated pregnancy presents to labor and delivery with decreased fetal movements for one day. She denies contractions, loss of fluid, or bleeding. Vital signs are temperature 98.6°F (37.0°C); blood pressure 100/60; pulse 79; respiratory rate 13; fetal heart rate 140s, reactive, with no decelerations. Tocometer reveals one contraction every eight minutes. Fundal height 36 cm, amniotic fluid index is 9. Cervix is firm, long, closed and posterior. What is the next best step in the management of this patient? A. Discharge home with labor warnings B. 24 hour observation C. Biophysical profile D. Contraction stress test E. Induction of labor

A The patient has reassuring fetal testing and may be discharged home with labor warnings: contractions every five minutes for one hour, rupture of membranes, fetal movement less than 10 per two hours or vaginal bleeding. A reactive non-stress test and normal AFI (modified biophysical profile) are sufficient to assess fetal well being at this time. Additional testing and interventions are not indicated at this time.

A 26-year-old G2P2 woman presents with a new onset of vulvar burning and irritation. She is sexually active with a new male partner. She is using oral contraception for birth control and did not use a condom. She thought she had a cold about 10 days ago. Which of the following is the most likely diagnosis in this patient? A. Herpes simplex virus B. Primary syphilis C. Secondary syphilis D. Human immunodeficiency virus E. Trichomonas

A The patient is most likely infected with herpes. Herpes simplex virus is a highly contagious DNA virus. Initial infection is characterized by viral-like symptoms preceding the appearance of vesicular genital lesions. A prodrome of burning or irritation may occur before the lesions appear. With primary infection, dysuria due to vulvar lesions can cause significant urinary retention requiring catheter drainage. Pain can be a very significant finding as well. Treatment is centered on care of the local lesions and the symptoms. Sitz baths, perineal care and topical Xylocaine jellies or creams may be helpful. Anti-viral medications, such as acyclovir, can decrease viral shedding and shorten the course of the outbreak somewhat. These medications can be administered topically or orally. Syphilis is a chronic infection caused by the Treponema pallidum bacterium. Transmission is usually by direct contact with an infectious lesion. Early syphilis includes the primary, secondary, and early latent stages during the first year after infection, while latent syphilis occurs after that and the patient usually has a normal physical exam with positive serology. In primary syphilis, a painless papule usually appears at the site of inoculation. This then ulcerates and forms the chancre, which is a classic sign of the disease. Left untreated, 25% of patients will develop the systemic symptoms of secondary syphilis, which include low-grade fever, malaise, headache, generalized lymphadenopathy, rash, anorexia, weight loss, and myalgias. This patient's symptoms are less consistent with syphilis, but she should still be tested for it. Human immunodeficiency virus is an RNA retrovirus transmitted via sexual contact or sharing intravenous needles. Vulvar burning, irritation or lesions are not typically noted with this disease, although generalized malaise can be. HIV can present with many different signs and symptoms, therefore risk factors should be considered, and testing offered. Trichomonas is a protozoan and is transmitted via sexual contact. It typically presents with a non-specific vaginal discharge. It does not have a systemic manifestation.

A 36-year-old G2P2 woman presents with irregular vaginal bleeding. Six weeks ago, she had her first Depo-Provera injection and now she has unpredictable bleeding. She is concerned by these symptoms. She has a history of hypertension but is currently on no medications. Vital signs reveal: blood pressure 130/90; weight 188 pounds; height 5 feet 5 inches; BMI 31.4kg/m2. Which of the following is the most appropriate next step in the management of this patient? A. Reassurance B. Begin oral contraceptives C. Begin estrogen D. Insert etonogestrel implant (Implanon) E. Perform an endometrial biopsy

A The patient should be reassured since initially after Depo-Provera injection there may be unpredictable bleeding. This usually resolves in 2-3 months. In general, after oone year of using Depo-Provera, nearly 50% of users have amenorrhea.

A 28-year-old G0 woman presents to your office for preconception counseling. She has a history of type 1 diabetes, diagnosed at age six, and uses an insulin pump for glycemic control. She has a history of proliferative retinopathy treated with laser. Her last ophthalmologic examination was three months ago. Her last hemoglobin A1C (glycosylated hemoglobin level) six months ago was 9.2%. Which of the following complications is of most concern for her planned pregnancy? A. Fetal growth restriction B. Fetal cardiac arrhythmia C. Group B Streptococcal infection D. Oligohydramnios E. Macrosomia

A The patient with type 1 diabetes is at risk for many pregnancy complications. In women with insulin-dependent diabetes, the rates of spontaneous abortion and major congenital malformations are both increased. The risk appears related to the degree of metabolic control in the first trimester. Overt diabetic patients are also at an increased risk for fetal growth restriction, although fetal macrosomia may also occur. The former becomes a greater concern as in this patient, with longer-term diabetes and vascular complications, such as retinopathy. Diabetics also have increased risk for polyhydramnios, congenital malformations (cardiovascular, neural tube defects, and caudal regression syndrome), preterm birth and hypertensive complications.

A 53-year-old G2P2 comes to your office complaining of six months of worsening hot flashes, vaginal dryness, night sweats and sleep disturbances. Her last normal menstrual period was six months ago and she has been experiencing intermittent small amounts of vaginal bleeding. Her medical history is significant for hypertension, which is well-controlled by a calcium-channel blocker, adult onset diabetes, for which she takes Metformin, and hyperthyroidism, for which she takes Propylthiouracil. The patient is 5 feet 7 inches tall and weighs 140 pounds. Blood pressure is 120/70. Physical examination is unremarkable. Which of the following medical conditions in this patient is a contraindication to treatment of menopausal symptoms with hormone therapy? A. Vaginal bleeding B. Hypertension C. Diabetes D. Osteoporosis E. Hyperthyroidism

A The principal symptom of endometrial cancer is abnormal vaginal bleeding. Although the patient's worsening symptoms make treatment an important consideration, the specific organic cause(s) of abnormal bleeding must be ruled out prior to initiating therapy. A tissue diagnosis consistent with normal endometrium or a pelvic ultrasound with an endometrial stripe of <4 mm ought to be documented. In addition, risks and benefits of hormone replacement therapy must be discussed with this patient at length prior to beginning treatment.

A 15-year-old G0 presents with severe menstrual pain for the past 12 months. The pain is severe enough for her to miss school. The pain is not relieved with ibuprofen 600 mg every four hours. She is not sexually active and the workup reveals no pathology. The most appropriate next step in the management of this patient is to begin combination oral contraceptives. How do oral contraceptives relieve primary dysmenorrhea? A. Creating endometrial atrophy B. Decreasing inflammation C. Increasing prolactin levels D. Decreasing inhibin levels E. Thickening cervical mucous

A The progestin in oral contraceptives creates endometrial atrophy. Since prostaglandins are produced in the endometrium, there would be less produced. Dysmenorrhea should be improved.

A 34-year-old G2P1 woman presents at 13 weeks gestation. She did not seek preconception counseling and is worried about delivering a child with Down syndrome, given her maternal age. She has no significant medical, surgical, family or social history. Which of the following tests is most effective in screening for Down syndrome in the second trimester? A. Quadruple screen B. Triple screen C. Amniotic fluid for alpha fetoprotein level D. Maternal serum alpha fetoprotein level E. Nuchal translucency measurement with serum PAPP-A (pregnancy associated plasma protein-A) and free Beta-hCG level

A The quadruple test (maternal serum alpha fetoprotein, unconjugated estriol, human chorionic gonadotropin, and inhibin A) is the most effective screening test for Down syndrome in the second trimester. Down syndrome occurs in about 1 in 800 births in the absence of prenatal intervention. The efficacy of screening for Down syndrome is improved when additional components are added to the maternal serum alpha fetoprotein screening. The addition of unconjugated estriol and human chronic gonadotropin (the Triple Screen) results in a 69% detection rate for Down syndrome. Adding inhibin A to produce a quadruple screen achieves a detection rate of 80-85%. An amniotic fluid alpha fetoprotein level is unnecessary. Nuchal translucency measurement with maternal serum PAPP-A and free Beta-hCG (known as the combined test) is a first trimester screen for Down syndrome. It detects approximately 85% of cases of Down syndrome at a 5% false positive rate.

You are asked to give a lecture on a new chemotherapy drug that has demonstrated a reasonable efficacy in women with advanced cervical cancer. The day before giving the lecture, you realize that you own stock in the company that makes the drug. Which of the following statements about conflict of interest is true? a. Pharmaceutical companies can support the costs of medical conferences in which physicians receive continuing medical education credits b. Physicians should engage in agreements in which companies make a substantial donation to an educational activity, when the donation is contingent on the physician's use or advocacy of a product c. The hospital may not interfere with a physician's decision to use a new surgical device d. An investigator may not own stock in a company if he/she does research for that company e. Physicians are not required to disclose any potential conflict of interest before speaking in a national forum

A The relationship of physicians and hospitals with pharmaceutical companies is a sensitive one, as there is potential for conflict of interest. It is acceptable for pharmaceutical companies to support conferences in which physicians receive CME credit. Physician participation in those activities should not be contingent upon physician use or advocacy of the product. An investigator may own stock in a company if he/she does research for that company, as long as he/she declares the conflict of interest and the conflict of interest is addressed.

A 28-year-old G1P0 internal medicine resident at 34 weeks gestation wants to discuss the values on her pulmonary function tests performed two days ago because she was feeling slightly short of breath. She is a non-smoker, and has no personal or family history of cardiac or respiratory disease. Vital signs are: respiratory rate 16; pulse 90, blood pressure 112/70; oxygen saturation is 99% on room air. On physical examination: lungs are clear; abdomen non-tender; fundal height is 34 cm. The results of the pulmonary function tests are: Inspiratory Capacity (IC) = increased Tidal volume (TV) = increased Minute ventilation = increased Functional reserve capacity (FRC) = decreased Expiratory reserve capacity (ERC) = decreased Residual volume (RV) = decreased What is the next best step in the evaluation of this patient? A. Routine antenatal care B. Chest x-ray C. Arterial blood gas D. Spiral CT of the lungs E. Echocardiogram

A The results of her PFT are consistent with normal physiologic changes in pregnancy. Inspiratory capacity increases by 15% during the third trimester because of increases in tidal volume and inspiratory reserve volume. The respiratory rate does not change during pregnancy, but the TV is increased which increases the minute ventilation, which is responsible for the respiratory alkalosis in pregnancy. Functional residual capacity is reduced to 80% of the non-pregnant volume by term. These combined lead to subjective shortness of breath during pregnancy.

A 19-year-old G2P1 woman at 30 weeks gestation presents with preterm premature rupture of membranes six hours ago. She denies labor. She takes prenatal vitamins and iron. She denies substance abuse, smoking or alcohol use. Her prior pregnancy was delivered vaginally at 41 weeks after spontaneous rupture of membranes. Her blood pressure is 110/70; pulse 84; temperature 98.6°F (37.0°C). Pertinent sonographic findings reveal a cervical length of 34 mm and an amniotic fluid index (AFI) of 3. What is the likelihood she will deliver within a week? A. 80% B. 40% C. 20% D. 10% E. 5%

A The time from premature rupture of membranes to labor is inversely related to gestational age. At term, 90% will spontaneously go into labor within 24 hours of PROM. At 28 weeks to 34 weeks, 50% will go into labor within 24 hours and 80% within 48 hours.

A 16-year-old girl goes to the doctor to discuss why she has not had a menstrual cycle. She is healthy and plays weekend volleyball. She studies hard and gets good grades in school. She has a good relationship with her parents. On examination she is 5 feet 1 inch tall and weighs 84 pounds. Breast and pubic hair growth are at a Tanner stage II. External genital examination is normal. What is the most likely reason this patient has not had any menses? A. Inadequate body weight B. Poor nutrition C. Inadequate sleep D. Excessive exercise E. Familial reasons

A There are three known critical elements for secondary sexual characteristics: adequate body weight, sleep and optic exposure to sunlight. These factors especially can delay the onset of menarche. A body weight of 85 to 106 pounds is needed before menses begins. Psychosocial causes of delayed puberty include eating disorders, excessive exercise, and stress or depression.

A 24-year-old G1P0 woman has just delivered 37 week male twins. On your initial assessment, you notice twin A is large and plethoric, and twin B is small and pale. A complete blood count (CBC) is obtained on both twins. What is the most likely finding in this case? A. Twin A is at high risk for polycythemia B. Twin A is at high risk for thrombocytopenia C. Twin B is at high risk for thrombocytopenia D. Twin B is at high risk for tachycardia E. Twin B is at high risk for hyperbilirubinemia

A This case is suggestive of twin-twin transfusion syndrome (TTTS). Polycythemia is a common complication for the plethoric twin. TTTS is a complication of monochorionic pregnancies. It is characterized by an imbalance in the blood flow through communicating vessels across a shared placenta leading to under perfusion of the donor twin, which becomes anemic and over perfusion of the recipient, which becomes polycythemic. The donor twin often develops IUGR and oligohydramnios, and the recipient experiences volume overload and polyhydramnios that may lead to heart failure and hydrops.

A 34-year-old G4P3 woman at 19 weeks gestation presents to the emergency department with chest pain, palpitations and sweating, which began 2-3 hours ago. On further questioning, she states that she has been very anxious lately and is not sleeping well, which she attributes to the pregnancy. She reports that she has lost 40 pounds in the last year without trying. She denies significant medical problems. On exam, the patient appears diaphoretic and anxious, her eyes are wide open, prominent, with easily visible sclera surrounding the pupil. Vital signs are: temperature 100.2°F (37.9°C); pulse 132; and blood pressure 162/84. Her height is 5 feet 10 inches and weight is 128 pounds. Her thyroid is palpably enlarged, with an audible bruit. Electrocardiogram shows sinus tachycardia. Remaining labs are pending. Which of the following therapies is contraindicated at this time? A. Radioactive iodine (I-131) B. Propylthiouracil (PTU) C. Propranolol D. Inorganic iodide E. Intravenous fluid replacement

A This is a patient in thyroid storm, an acute, life-threatening, hypermetabolic state. Radioactive iodine (I-131) concentrates in the fetal thyroid and may cause congenital hypothyroidism, so it should not be intentionally used in pregnancy. Acute treatment of thyroid storm may include thioamides (i.e. PTU), propranolol, sodium iodide and dexamethasone. Oxygen, digitalis, antipyretics and fluid replacement may also be indicated. Maternal mortality with thyroid storm exceeds 25%.

A 42-year-old G4P4 presents with a history of progressively worsening severe menstrual pain. Menses are regular, but she complains of very heavy flow requiring both a menstrual pad and tampon with frequent bleeds through this protection on heavy days. She takes Oxycodone that her husband used for back pain to relieve her dysmenorrhea. She had a tubal ligation four years ago. Pelvic examination shows an enlarged, soft, boggy uterus. No masses are palpated. Pregnancy test is negative, hemoglobin 9.8 and hematocrit 28.3%. What is the most likely diagnosis? A. Adenomyosis B. Endometrial carcinoma C. Endometriosis D. Primary dysmenorrhea E. Endometrial hyperplasia

A This is a typical presentation of adenomyosis (presence of endometrial glands and supporting tissues in the muscle of the uterus). The gland tissue grows during the menstrual cycle and, at menses, tries to slough, but cannot escape the uterine muscle and flow out of the cervix as part of normal menses. This trapping of the blood and tissue causes uterine pain in the form of monthly menstrual cramps. Endometrial hyperplasia and carcinoma are less likely in a woman with regular menses and no inter-menstrual spotting. Endometriosis would most likely have presented earlier in life and would not explain the enlarged uterus.

A 27 year-old G1P0 at 14 weeks gestation presents with a 2-month history of insomnia, feeling depressed, and unintentional weight loss. Symptoms began after the unexpected death of her father. She is not excited about this pregnancy and reports no suicidal ideation. Physical examination reveals a woman of stated age with a flat affect. Which of the following therapies is contraindicated in this patient? A. Paroxetine (Paxil) B. Sertraline (Zoloft) C. Fluoxetine (Prozac) D. Nortriptyline (Norpress) E. Bupropion (Wellbutrin)

A This patient has classic depression. The most commonly used antidepressants are the selective serotonin reuptake inhibitors (SSRIs.) One SSRI, Paroxetine (Paxil) has recently been changed to a category D drug because of the increased risk of fetal cardiac malformations and persistent pulmonary hypertension. The older SSRI compounds, fluoxetine and sertraline, have not been reported to cause early pregnancy loss or birth defects in animals or in humans. Because these agents have few side effects compared with other antidepressants, they are a good choice for pregnant women. Tricyclic antidepressants have a long record of use in pregnancy and there is no increase in the rate of fetal malformation. Bupropion is not an MAO inhibitor, nor is it an SSRI and a report by the Bupropion Pregnancy Registry reports no unusual effects in 90 exposed pregnancies.

A 27-year-old G1P0 woman at 34 weeks gestation presents with increased swelling in her face and hands over the last two days. Blood pressure is 155/99. A 24-hour urine sample for protein is 440mg/dL. BMI is 27. Repeat blood pressure two days later is 150/92. Which of the following is the most likely diagnosis in this patient? A. Mild preeclampsia B. Severe preeclampsia C. Gestational hypertension D. Prehypertension E. Chronic hypertension

A This patient has met criteria for the diagnosis of mild preeclampsia based on her persistent elevation of blood pressure and 24-hour urine results. The amount of protein excreted in the urine varies throughout the day, therefore a sample is collected over a 24-hour time period. Twenty-four hour urine protein values greater that 300 mg are required for the diagnosis of mild preeclampsia. Values greater than 5000 mg (or 5 g) are required for the diagnosis of severe preeclampsia (assuming no other defining criteria are present such as SBP >160 or DBP >110). Blood pressure between 120/80 and 139/89 is termed prehypertension, which is indicative of developing hypertension in the future.

A 27-year-old G0 woman comes to the office as she has been unable to conceive for the last year. She is in good health and has not used any hormonal contraception in the past. She had normal cycles every 28 days until about six months ago. At that time, she began to have irregular menses every two to three months, with some spotting in between. She is not taking any medications. She has no history of abnormal Pap smears or sexually transmitted infections. Her physical examination is normal. Laboratory tests show: Results Normal Values TSH 10 mIU/ml 0.5-4.0 mIU/ml Free T4 0.2 ng/dl 0.8-1.8 ng/dl Prolactin 40 ng/ml <20 ng/ml FSH 6 mIU/ml 5-25 mIU/ml LH 4 mIU/ml 5-25 mIU/ml What is the most appropriate step in the management of this patient? A. Begin levothyroxine B. Begin bromocriptine C. Order a clomiphene citrate ovulation challenge test D. Obtain a brain MRI E. Order a thyroid gland ultrasound

A This patient is having abnormal cycles due to hypothyroidism, which is also the most likely cause of her hyperprolactinemia. The best treatment at this point is to correct the hypothyroidism. It is not necessary to treat the hyperprolactinemia with bromocriptine or order a brain MRI until the hypothyroidism is first corrected. There is no reason to obtain a thyroid ultrasound, as the diagnosis of hypothyroidism is clear from the laboratory results.

A 27-year-old primigravid woman comes to you for her first prenatal visit at 9 weeks of gestation. Her mother had a deep vein thrombosis at age 55 years, which prompted thrombophilia testing for first-degree relatives. Your patient's test result reveals that she is heterozygous for the prothrombin G20210A polymorphism. She reports a history of superficial thrombophlebitis, but states that she does not have a specific personal history of venous thrombosis. She reports that she would like to avoid needles if at all safely possible. The most appropriate management that addresses her potential risk of venous thrombosis is . A) surveillance without anticoagulation B) low-dose aspirin C) subcutaneous prophylactic heparin D) subcutaneous adjusted dose heparin

A) surveillance without anticoagulation

A 17-year-old nulliparous female is brought in by her mother because she has not yet had any menses. She is otherwise in good health, but recently has been experiencing cyclical lower abdominal cramping. She has never had sexual intercourse. She is 5 feet 6 inches tall and weighs 120 pounds. On examination, her breasts are Tanner Stage IV. She has some suprapubic tenderness on abdominal exam. Her pelvic exam reveals normal external genitalia, but there was difficulty inserting a speculum due to patient's discomfort. Beta-hCG < 5 mIU/mL. What is the most likely diagnosis in this patient? A. Genital tract outflow obstruction B. Müllerian agenesis C. Hypothalamic-pituitary dysfunction D. Psychogenic amenorrhea E. Constitutional delay in menarche

A This patient's primary amenorrhea, with normal secondary sexual characteristics, development and cyclical abdominal pain, points to an anatomical cause of amenorrhea, which is preventing menstrual bleeding. An imperforate hymen commonly causes this and the treatment is surgical. In Mϋllerian agenesis, or Mayer-Rokitansky-Kϋster-Hauser syndrome, there is congential absence of the vagina and usually an absence of the uterus and fallopian tubes. Ovarian function is normal and all the secondary sexual characteristics of puberty occur at the appropriate time.

A 22-year-old G0 college student returns for follow-up of mood swings and difficulty concentrating on her schoolwork the week before her menses for the past 12 months. Her past medical history is unremarkable and physical examination is normal. Which of the following would be an appropriate treatment option for this patient? A. Oral contraceptive pills B. Reassurance and observation C. Methylphenidate (Ritalin) D. Gabapentin E. Ginkgo

A This woman has premenstrual syndrome (PMS) with symptoms that warrant treatment. Patients with PMS and premenstrual dysphoric disorder (PMDD) experience adverse physical, psychological and behavioral symptoms during the luteal phase of the menstrual cycle. PMS is characterized by mild to moderate symptoms, while PMDD is associated with severe symptoms that seriously impair usual daily functioning and personal relationships. Mild symptoms of PMS often improve by suppressing the hypothalamic-pituitary-ovarian axis with oral contraceptive pills. Ritalin and Ginkgo are not effective treatments for PMS. Gabapentin is used for neuropathic pain and will not help alleviate her symptoms.

A 34-year-old G4P2 woman at 18 weeks gestation presents with fatigue and occasional headache. She has a sister with Grave's disease. On physical exam, vital signs are normal. BMI is 27. Thyroid is difficult to palpate due to her body habitus. The remainder of her exam is unremarkable. Thyroid function studies show: Results Reference Range TSH 1.8 mU/L (0.30 -5.5 mU/L) Free T4 1.22 ng/dL (0.76 - 1.70 ng/dL) Total T4 14.2 ng /dL (4.9 - 12.0 ng /dL) Free T3 3.4 ng/dL (2.8 - 4.2 ng/dL) Total T3 200 ng/dL (80 - 175 ng/dL) What is the next best step in the management of this patient? A. Continue routine prenatal care B. Check anti-thyroid antibody levels C. Obtain a thyroid ultrasound D. Initiate propylthiouracil E. Initiate methimazole

A Thyroid binding globulin (TBG) is increased due to increased circulating estrogens with a concomitant increase in the total thyroxine. Free thyroxine (T4) remains relatively constant. Total triiodothyroxine (T3) levels also increase in pregnancy while free T3 levels do not change. In a pregnant patient without iodine deficiency, the thyroid gland may increase in size up to 10%. This patient's thyroid function is normal for pregnancy, and her symptoms of fatigue can be explained by other physiologic changes in pregnancy, including anemia, difficulty with sleep, and increase metabolic demand.

A 26-year-old lesbian has chronic herpetic lesions on her lip. She is concerned about the affect this will have on her partner when she performs cunnilingus on her. Physical examination is normal except for a herpetic lesion on the lip, which has been diagnosed as Herpes type 1 in the past. Acyclovir is prescribed. Which of the following is the most appropriate advice to this patient? A. Use a dental dam when having oral sex B. Provide a prescription for acylovir for her partner to take before and after oral sex C. Apply a thin layer of petrolatum jelly over her lips while having oral sex D. Inform her that the Herpes virus type 1 is not contagious to the genitalia E. Discontinue oral sex

A Use of a dental dam or a latex condom cut down the middle is effective in avoiding infection. Type 1 virus can cause ulcers on the vulva, as it is contagious. Prophylaxis of the partner with acyclovir is not a recommended strategy to prevent transmission, and petrolatum jelly is not an effective barrier.

30-year-old G2P1 has an ultrasound at 42 weeks for size greater than dates. The fetus had an isolated enlarged head measurement with a BPD of 11 cm, but otherwise appeared to have normal femur length and abdominal circumference. Polyhydramnios is noted. The estimated fetal weight is 3900 g. There is a 10 cm lower uterine segment fibroid protruding into the uterine cavity. The fetus is in the vertex presentation and the fetal head is above the level of the uterine fibroid. Which of the following is an indication for primary Cesarean delivery in this patient? A. Uterine fibroid B. Fetal hydrocephalus C. Polyhydramnios D. Macrosomia E. 42 weeks gestation

A Uterine fibroids that are in the lower uterine segment may obstruct labor by preventing the fetal head from entering the pelvis. A fetal head with measurements greater than 12 cm could benefit from delivery by Cesarean. The fetus in the case presented does not necessarily have hydrocephalus. The fetus does not have macrosomia which may be defined as an estimated fetal weight greater than 4000 grams in a diabetic and greater than 4500 grams in a non-diabetic patient. Macrosomia defined as greater than 4000 grams, 42 weeks gestation, and polyhydramnios are not indications for primary Cesarean section.

A 19-year-old G1P0 at 39 weeks gestation presents in labor. She denies ruptured membranes. Her prenatal course was uncomplicated and ultrasoundat 18 weeks revealed no fetal abnormalities. Her vital signs are: blood pressure 120/70; pulse 72; temperature 101.0° F (38.3° C); fundal height 36 cm; and estimated fetal weight of 2900 gm. Cervix is dilated to 4 cm, 100% effaced and at +1 station. She receives 10 mg of morphine intramuscularly for pain and soon after has spontaneous rupture of the membranes. Light meconium-stained fluid was noted and, five minutes later, the fetal heart ratetracing revealed variable decelerations with good beat-to-beat variability. What is the most likely cause for the variable decelerations? A. Umbilical cord compression B. Meconium C. Maternal fever D. Uteroplacental insufficiency E. Morphine administration

A Variable decelerations are reflex mediated usually associated with umbilical cord compression as a result of cord wrapped around fetal parts, fetal anomalies or oligohydramnios. The presence of light meconium-stained fluid is not associated with a specific fetal heart rate tracing. Uteroplacental insufficiency is associated with late decelerations. Maternal drugs may cause loss of variability.

A 19-year-old G1P0 woman at 39 weeks gestation presents in labor. She denies ruptured membranes. Her prenatal course was uncomplicated and ultrasound at 18 weeks revealed no fetal abnormalities. Her vital signs are: blood pressure 120/70; pulse 72; temperature 101.0° F (38.3° C); fundal height 36 cm; and estimated fetal weight of 2900 gm. Cervix is dilated to 4 cm, 100% effaced and at +1 station. She receives 10 mg of morphine intramuscularly for pain and soon after has spontaneous rupture of the membranes. Light meconium-stained fluid was noted and, five minutes later, the fetal heart rate tracing revealed variable decelerations with good variability. What is the most likely cause for the variable decelerations? A. Umbilical cord compression B. Meconium C. Maternal fever D. Uteroplacental insufficiency E. Umbilical cord prolapse

A Variable decelerations are typically caused by cord compression and are the most common decelerations seen in labor. Placental insufficiency is usually associated with late decelerations. Head compression typically causes early decelerations. Oligohydramnios can increase a patient's risk of having umbilical cord compression; however, it does not directly cause variable decelerations. Umbilical cord prolapse occurs in 0.2 to 0.6% of births. Sustained fetal bradycardia is usually observed.

A 36-year-old female G1 presents for her prenatal care visit at 35 weeks gestation. She has good dating criteria that were confirmed by a first trimester ultrasound. Her previous medical history is positive for hypertension and type 2 diabetes. You have been following fetal growth with serial ultrasounds. At this visit, ultrasound reveals limited fetal growth over the past three weeks. Biometry is consistent with 32-5/7, EFW 2175 g, <10th percentile. What is the most appropriate next test indicated in the management of this patient? A. Amniotic fluid volume, umbilical artery Doppler systolic: diastolic ratio, non-stress test B. Daily fetal kick counts with follow up ultrasound to reassess fetal growth in one week C. Amniocentesis for fetal lung maturity D. Twice daily fetal kick counts with delivery at 37 weeks gestation E. None, delivery is indicated

A When a pregnancy is complicated by fetal growth restriction, various fetal physiologic parameters require assessment. In growth-restricted pregnancies, oligohydramnios is frequently found. This finding is presumably due to reduced fetal blood volume, renal blood flow and urinary output. Chronic hypoxia is responsible for diverting blood flow from the kidney to organs that are more critical during fetal life. The significance of the amniotic fluid volume with respect to fetal outcome has been well documented. Ninety percent of patients with oligohydramnios delivered growth restricted infants. These infants experienced a high rate of fetal compromise. The systolic/diastolic (S/D) ratio of the umbilical artery is determined by Doppler ultrasound. An increase in the S/D ratio reflects increased vascular resistance. It is a common finding in IUGR fetuses. A normal S/D ratio indicates fetal well-being. As vascular resistance increases, the S/D ratio increases. With severe resistance, there is absence and ultimately reversal of end-diastolic flow. These findings are associated with an increased rate of perinatal morbidity and mortality, and a higher likelihood of a long-term poor neurologic outcome. Options for antenatal testing include the non-stress test, contraction stress test, and the biophysical profile. Any of these may be used in a growth-restricted fetus as a means of detecting possible or probable fetal asphyxia. While fetal kick counts may be of value, additional fetal testing such as twice weekly NST with AFI and weekly umbilical artery Doppler studies is indicated in monitoring fetuses with IUGR.

A 24-year-old Rh-negative G1P1 woman just delivered a healthy term infant who is Rh-positive. You recommend RhoGAM administration but she declines because she does not desire any blood products. What is her approximate risk of isoimmunization if she does not receive the RhoGAM? A. Less than 20% B. 40% C. 60% D. 80% E. 100%

A While 75% of all gravidas have evidence of transplacental hemorrhage during pregnancy or immediately after delivery, 60% of these patients have <0.1 cc of fetal blood in the maternal circulation, which is enough to sensitize a patient. The incidence and size of transplacental hemorrhage increases as pregnancy advances. During the second month of gestation, 5-15% of women will have evidence of feto-maternal hemorrhage. By the third trimester, this number increases to 45% of patients. The risk of isoimmunization is 2% antepartum, 7% after full term delivery, and 7% with subsequent pregnancy.

A 32-year-old G2P2 woman has just had a spontaneous vaginal delivery. She is concerned that no breast milk is yet being produced when she tries to feed her baby. You reassure her that colostrum is rich in protein and nutrients, and that her breast milk will come in 2-3 days when which of the following hormones have been cleared? A. Estrogen and progesterone B. Prolactin and oxytocin C. Human placental lactogen and prolactin D. Progesterone and prolactin E. Growth hormone and GnRH

A With delivery, there is a rapid and profound decrease in the levels of progesterone and estrogen, which removes the inhibitory influence of progesterone on the production of alpha-lactalbumin by the rough endoplasmic reticulum. The increased alpha-lactalbumin serves to stimulate lactose synthase and ultimately to increase milk lactose. Progesterone withdrawal allows prolactin to act unopposed in its stimulation of alpha-lactalbumin production. This may take up to two days.

A 30-year-old G1P0 woman with type 1 diabetes mellitus presents at 10 weeks gestation for a routine visit. She smokes a half a pack of cigarettes per day. Her hemoglobin A1C level is 9.7. What structural anomaly is the fetus at highest risk of developing? A. Cardiac anomalies B. Caudal regression malformation C. Hydrocephalus D. Microcephaly E. Limb reductions

A Women with poorly controlled diabetes immediately prior to conception and during organogenesis have a four- to eight-fold risk of having a fetus with a structural anomaly. The majority of lesions involve the central nervous system (neural tube defects) and the cardiovascular system. Genitourinary and limb defects have also been reported.

A 25-year-old G6P2 woman in active labor is treated with mepiridine (Demerol). The patient reports the use of marijuana to control nausea during her pregnancy. She quickly progresses from 4 cm to fully dilated in 1 hour and is now pushing. A limp unresponsive infant is delivered. Heart rate is greater than 90 beats/minute. The infant has no respiratory effort. Which of the following is the most appropriate next step in the management of this patient? A. Give positive pressure ventilation and prepare to intubate B. Give positive pressure ventilation and prepare to give naloxone C. Give stimulation only and continue to monitor heart rate D. Suction thoroughly and check heart rate E. Suction thoroughly and give naloxone

A You should give positive pressure ventilation and prepare to intubate the infant, if necessary. Any history of substance abuse may be a relative contraindication to the use of naloxone (Narcan) because the mother may have used narcotics during the pregnancy and administration of naloxone to the infant can cause life-threatening withdrawal. Stimulation may not be sufficient for this infant. Suction will not necessarily stimulate a respiratory effort.

Ectopic pregnancy relates to the following situations: 1) fallopian pregnancy (ampullary region); Vaginal bleeding and hypogastric pain occurred in a 25-year-old patient in 8 week of the pregnancy 2) abdomen pregnancy; 3) uterine horn pregnancy; 4) intrauterine pregnancy; 5) caesarean section scar pregnancy. The correct answer is: A. 1,2,3,5. B. 2,3,4,5. C. 3,4,5. D. 4,5. E. only 5.

A. 1,2,3,5.

Indicate the true sentences regarding screening for the neoplastic disorders of female genital organs: 1) screening for vaginal and vulvar cancer is not performed because of the low incidence of these tumours; 2) screening for endometrial cancer is not performed because its first symptoms, i.e. uterine bleeding, are easily noticed by women and reported to their doctors; 3) screening for non-epithelial tumours of the ovaries, the uterus and the vagina is not performed because of the low incidence of these tumours; 4) screening for endometrial cancer is performed and includes ultrasound imaging, the measurement of the markers: CA 125, HE 4 (human epididymis protein 4) and choriongonadotropin; 5) screening for cervical cancer is performed and includes a cytological test, a test which sensitivity reaches 85-90% in leading centers. The correct answer is: A. 1,2,3. B. 1,5. C. 3,4,5. D. 4,5. E. 1,3,4.

A. 1,2,3.

The clinical features of HELLP syndrome include: 1) elevated liver enzymes; 2) low platelet count; 3) hemolysis; 4) thrombocytosis; 5) decreased concentration of indirect bilirubin. The correct answer is: A. 1,2,3. B. 1,3,4. C. 1,3,5. D. 2,3,5. E. 3,4,5.

A. 1,2,3.

The high risk factors of gestational diabetes include: 1) obesity of the pregnant woman (BMI > 30); 2) diabetes in the first-degree relatives; 3) birth weight of neonates of previous pregnancies > 4000 g; 4) age of the pregnant woman < 25 years; 5) no significant obstetric history. The correct answer is: A. 1,2,3. B. 1,2,5. C. 2,3,4. D. 1,3,4. E. all the above.

A. 1,2,3.

Which of the following indicates a complete mole at ultrasound examination? 1) "snow-storm" image; 2) no fetus; 3) presence of theca lutein cysts; 4) presence of fetal echo with a visible heart action; 5) presence of the gestational sac with amniotic fluid. The correct answer is: A. 1,2,3. B. 1,2,5. C. 1,3,5. D. 2,3,5. E. 3,4,5.

A. 1,2,3.

Routine seminological examination is an essential element in assessing the male factor as the cause of infertility. The values of microscopic evaluation of semen are presented below: 1) volume from 1.5-6 mL; 2) sperm concentration > 15 million/mL; 3) viability > 38% live sperm; 4) mobility A and B > 32%; 5) > 4% sperm with normal structure. Indicate the parameters within normal range (normospermia) according to the WHO 2010 standards: A. 1,2,4,5. B. 2,3,4,5. C. 4,5. D. only 5. E. 1,5.

A. 1,2,4,5

Untreated iron-deficiency anemia in a pregnant woman may lead to: 1) intrauterine growth retardation; 2) premature labour; 3) elongation of the pregnancy; 4) congenital anomalies; 5) prolonged labour. The correct answer is: A. 1,2,4,5. B. 1,3,4,5. C. 1,2,3,4. D. 1,2,3,5. E. 3,4,5.

A. 1,2,4,5.

Which of the following may cause retrograde ejaculation? 1) diabetes mellitus; 2) depression; 3) sclerosis multiplex; 4) spinal cord injury; 5) myocardial infarction; 6) femoral bone fracture. The correct answer is: A. 1,3,4. B. 1,4,5. C. 2,5,6. D. 3,4,5. E. 4,5,6.

A. 1,3,4.

The multiple pregnancy is related to the increased risk of numerous obstetrical complications compared with the singleton pregnancy. Which of the following complications are not observed in multiple pregnancies more often than in singleton pregnancies? 1) shoulder dystocia; 2) gestational hypertension; 3) necessity for the cesarean section; 4) postterm pregnancy; 5) perinatal death; 6) placenta praevia; 7) congenital defects. The correct answer is: A. 1,4. B. 2,6. C. 2,7. D. 3,5. E. 4,6

A. 1,4.

Indicate the medications that can be safely used in pregnant patients: 1) aluminum compounds in the treatment of gastroesophageal reflux; 2) non-steroidal anti-inflammatory drugs from the group of non-selective cyclooxygenase inhibitors; 3) nystatin in the treatment of infectious diseases; 4) nitrofurantoin and furazidin in the treatment of urinary tract infections; 5) angiotensin converting enzyme inhibitors in the treatment of hypertension. The correct answer is: A. 1,4. B. 1,2,4. C. 2,3,5. D. 1,3. E. only 1.

A. 1,4. There is no correct answer here. 1 is 100% correct, 4 is 100% correct, 3 - nystatin is category B.

Anti-RhD immunoglobulin is used in RhD minus female patients in a single i.m. injection after the laparoscopic treatment of an ectopic pregnancy (WGA < 12 Hbd) within 72 h of the surgery at a dose of: A. 50 µg. B. 100 µg. C. 150 µg. D. 200 µg. E. 300 µg.

A. 50 µg.

A woman who is rhesus negative undergoes spontaneous miscarriage at 14 week. What dose of anti D immunoglobulin should she receive? A. 50 μg. B. 100 μg. C. 150 μg. D. 300 μg. E. 1500 μg.

A. 50 μg.

Which of the following cannot be a consequence of intrauterine infection? A. Down syndrome. B. hydrocephalus. C. fetal tachycardia. D. preterm labor. E. prelabor rupture of membranes.

A. Down syndrome

Indicate the maneuver used in vaginal delivery of a baby in a longitudinal lie with breech presentation in order to release the child's shoulders: A. Lővset's maneuver. B. Mauriceau-Smellie-Veit maneuver. C. Wiegand-Martin-von Winckel maneuver. D. Gaskin maneuver. E. Kiwisch-Scanzoni maneuver.

A. Lővset's maneuver.

Which of the following ovarian tumors is a virilizing tumor? A. Sertoli-Leydig cell tumor. B. endometrioid tumor. C. Wolffian duct cell tumor. D. corpus luteum cyst. E. fibroblast tumor.

A. Sertoli-Leydig cell tumor.

Which of the following surgical approaches is characterized by invisible scare, better healing in obese women but the limited field of view? A. abdominal approach - the incision in the midline. B. abdominal approach - the incision in the paramedian line. C. abdominal approach - the Pfannenstiel incision. D. vaginal approach. E. laparoscopic approach.

D. vaginal approach.

An umbilical cord prolapse during the second stage of the labor in a cephalic presentation with a longitudinal lie: A. is not dangerous. B. is an indication for the attempt of a manual reduction of the prolapse. C. is an indication for immediate delivery. D. is an indication for a manual rotation of the fetus. E. is an indication for the immediate artificial rupture of membranes.

C. is an indication for immediate delivery.

Which of the following is a non-invasive prenatal test that shows the highest sensitivity in the diagnostics of chromosomal aberrations in a fetus? A. chorionic villus sampling. B. amniocentesis. C. triple test. D. PAPP-A determination. E. examination of cell-free fetal DNA in maternal blood.

E. examination of cell-free fetal DNA in maternal blood.

Absence of menstruation after fertilization and implantation of the embryo in the endometrium is caused by: A. absence of corpus luteum degeneration and the increased synthesis of progesterone and estrogens because of increasing levels of chorionic gonadotropin. B. absence of corpus luteum degeneration and the increased synthesis of progesterone and estrogens because of decreasing levels of prolactin. C. absence of corpus luteum degeneration because of the placenta taking over the production of progesterone and estrogens. D. degeneration of the corpus luteum because of the placenta taking over the production of progesterone and estrogens. E. degeneration of the corpus luteum because of increasing prolactin levels.

A. absence of corpus luteum degeneration and the increased synthesis of progesterone and estrogens because of increasing levels of chorionic gonadotropin.

In a patient with Turner syndrome (karyotype 45,X), within sexual organs, one can observe: A. aplastic ovaries. B. additional ovaries. C. testes and ovaries. D. testes only. E. normal ovaries.

A. aplastic ovaries.

Which of the following was the most frequently occurring malignant tumor in Polish women for the 5 years (2010-2014)? A. breast cancer. B. endometrial cancer. C. cervical cancer. D. ovarian cancer. E. lung cancer.

A. breast cancer.

A 28-year-old woman who fainted again at home reported to the obstetrics and gynecology ER. On admission she presented pale, sweaty face, rapid heart rate, body temperature 36.6˚C. Despite inserting a contraceptive intrauterine device a year ago, she had her last period 8 weeks ago. Preliminary gynecological examination showed: slight vaginal bleeding, slightly enlarged uterus, painful resistance in the projection of the left uterine appendages and doughy tumor in the pouch of Douglas. These symptoms are indicative of diagnosis: A. ectopic pregnancy - tubular miscarriage. B. cervical pregnancy. C. correctly located pregnancy - incomplete miscarriage. D. acute adnexitis. E. ovarian cancer.

A. ectopic pregnancy - tubular miscarriage.

Which of the following is the most probable diagnosis in the case of impaired fertility, chronic pelvic pain, painful periods, and dyspareunia? A. endometriosis. B. ovarian cancer C. cervical cancer. D. adnexitis. E. premature ovarian failure.

A. endometriosis.

The treatment of pre-invasive cervical cancer consists in: A. excision of the affected part of the cervix. B. strict cytological and colposcopic examination once a month. C. simple hysterectomy. D. radical hysterectomy. E. brachytherapy.

A. excision of the affected part of the cervix.

A patient with twin monochorionic pregnancy has been informed about the risk of twin to-twin transfusion syndrome (TTTS). The syndrome is characterized by an unbalanced blood flow between the twins through active interconnecting blood vessels. One of the complications of TTTS that occurs in the recipient is: A. fetal hypertrophy. B. growth retardation. C. anemia. D. oligohydramnios. E. hypovolemia.

A. fetal hypertrophy.

The relation of the long fetal axis to the long axis of the uterus and reproductive canal is called: A. fetal lie. B. position. C. fetal location. D. engagement. E. presentation.

A. fetal lie.

Which of the following is called menarche? A. first menstruation in life, usually occurring at the age of 12-13 years. B. development of pubic hear, usually occurring at the age of 11 years. C. beginning of the hormonal activity of the ovaries. D. evolving function of the adrenals that comes 2-3 years before gonadal function. E. development of the breasts, usually occurring at the age of 10 years.

A. first menstruation in life, usually occurring at the age of 12-13 years.

An absolute indication for primary cesarean section in breech presentation is not: A. foetus weight above 3500 g. B. primipara. C. umbilical cord prolapse. D. placenta previa. E. placenta abruption.

A. foetus weight above 3500 g.

In a 16-year-old girl with amenorrhea, vaginal hematoma (hematocolpos) was diagnosed during ultrasound examination. The most probable diagnosis is: A. imperforate hymen. B. uterine septum C. uterine aplasia. D. Mayer-Rokitansky-Küster-Hauser syndrome. E. ovarian agenesis.

A. imperforate hymen.

The basic strategy of treatment in the case of Bartholin's gland abscess consists in: A. incision of the abscess or its marsupialization. B. surgical removal of the abscess. C. observation. D. antibiotic therapy only. E. wraps with washing soda.

A. incision of the abscess or its marsupialization.

Which of the statements regarding stress urinary incontinence is true? A. increase in intra-abdominal pressure is accompanied by involuntary loss of urine. B. involuntary leakage of urine is accompanied by a strong but short urinary urgency. C. volume of leaking urine is large. D. frequency of micturition at night is increased. E. in urodynamic testing detrusor hyperreflexia is observed.

A. increase in intra-abdominal pressure is accompanied by involuntary loss of urine.

Indicate the true set of physiological changes in the circulatory system of pregnant women: A. increased heart rate, increased cardiac output, increased volume of circulating blood, decreased systemic and pulmonary vascular resistance, decreased arterial pressure. B. increased heart rate, decreased cardiac output, increased volume of circulating blood, increased systemic and pulmonary vascular resistance, decreased arterial pressure. C. increased heart rate, decreased cardiac output, increased volume of circulating blood, increased systemic and pulmonary vascular resistance, increased arterial pressure. D. increased heart rate, decreased cardiac output, decreased volume of circulating blood, increased systemic and pulmonary vascular resistance, decreased arterial pressure. E. decreased heart rate, decreased cardiac output, increased volume of circulating blood, increased systemic and pulmonary vascular resistance, increased arterial pressure.

A. increased heart rate, increased cardiac output, increased volume of circulating blood, decreased systemic and pulmonary vascular resistance, decreased arterial pressure.

Which sexually transmitted disease does not cause symptoms in the genital area? A. infection caused by HIV. B. infection caused by HPV. C. infection caused by HSV. D. syphilis. E. gonorrhea.

A. infection caused by HIV.

The phase of voiding the urinary bladder, initiated by the brain cortex is characterized by the relaxation of sphincter muscles surrounding the urethra and the increased tone of the detrusor muscle. These phenomena are caused by: A. inhibition of the sympathetic activity and the increased pulsation in the parasympathetic system. B. inhibition of the parasympathetic activity and the increased pulsation in the sympathetic system. C. increased pulsation in the somatic system innervating muscles of the pelvic floor. D. inhibition of the sympathetic activity only. E. inhibition of the parasympathetic activity only.

A. inhibition of the sympathetic activity and the increased pulsation in the parasympathetic system.

In women using biphasic hormonal anticonception a typical menstrual period: A. is short and light. B. does not occur. C. occurs irregularly and last for many days. D. is heavy and long. E. occurs every 21 days and is light.

A. is short and light.

Prohibited vaccinations during pregnancy include: A. measles. B. hepatitis B. C. infuenza. D. tetanus. E. yellow fever.

A. measles.

Which of the following is the primary medication used for the long-term antihypertensive therapy in pregnancy? A. methyldopa. B. perindopril. C. enalapril. D. ramipril. E. lisinopril.

A. methyldopa.

Disulfiram-ethanol reaction may be expected in a patient with an infection of reproductive organs, who drank alcohol during the treatment with: A. metronidazole. B. cefuroxime. C. clindamycin. D. furaginum. E. acyclovir.

A. metronidazole.

Which of the following is characteristic of borderline ovarian tumors? A. no destructive infiltration of the stroma. B. occurrence limited to the ovaries only. C. prognosis similar to that of ovarian cancer. D. no effective treatment options available. E. occurrence in postmenopausal women only.

A. no destructive infiltration of the stroma.

A 32-year-old patient did the sperm count test for the diagnostics of couple infertility. He avoided ejaculation for 3 days before the test. The test results show: sperm number 18mln/mL; sperm motility slow and rapid 33% in total; sperm morphology - 5% normally shaped. What conclusion should be formulated? A. normospermia. B. asthenozoospermia. C. kryptozoospermia. D. oligoteratozoospermia. E. asthenoteratozoospermia.

A. normospermia.

The diagnosis of endometriosis should be considered in all the following conditions, except for: A. oligomenorrhea. B. dyspareunia. C. painful menstruation. D. ovarian cyst with dense content. E. pelvic pain.

A. oligomenorrhea.

In which of the following neoplasms brachytherapy is not used as an auxiliary method? A. ovarian cancer. B. endometrial cancer. C. cervical cancer. D. vulvar cancer. E. in the all of the above cancers brachytherapy is used as an auxiliary method.

A. ovarian cancer.

Teratozoospermia according to WHO criteria of 2010 denotes: A. percentage of sperm with normal morphology below 4%. B. percentage of sperm showing normal mobility below 4%. C. no sperm in the ejaculate. D. over 250 million sperm in 1 mL of the ejaculate. E. percentage of sperm with normal morphology below 40%.

A. percentage of sperm with normal morphology below 4%.

Which of the following situations occurring during pregnancy requires a quick termination of the pregnancy? A. premature placental abruption. B. cervical cancer in pregnancy. C. HIV infection. D. adnexal torsion. E. acute appendicitis.

A. premature placental abruption.

The pregnancy in a patient with chronic hypertension is a high-risk pregnancy and maybe complicated with the following disorders except for: A. preterm birth related to the cervical incompetence. B. intrauterine growth retardation. C. superimposed preeclampsia. D. abruption of normally located placenta. E. oligohydroamnios.

A. preterm birth related to the cervical incompetence.

Which of the following is not a risk factor for endometrial cancer? A. BMI > 32 kg/m B. tamoxifen therapy. C. late menopause. D. nulliparity. E. family history of ovarian cancer.

E. family history of ovarian cancer.

Medical history of cervical incompetence is an indication for: A. prophylactic placement of a cervical cerclage. B. placement of a cervical cerclage when uterine contractions occur before 28 week of gestation. C. placement of a cervical cerclage when there are symptoms of vaginal infection, along with antibiotic therapy. D. frequent monitoring of the cervical length with ultrasonography. E. calcium-rich diet in order to improve the consistency of the cervix.

A. prophylactic placement of a cervical cerclage.

Which of the following are assessed by the Bishop score? A. relation between the cervix and the vaginal axis, dilation, cervical effacement, fetal station. B. gestational age, fetal position, cervical dilation and effacement. C. cervical consistency, fetal lie, dilation, cervical effacement, fetal presentation. D. relations between the uterine axis and vaginal axis, dilation, cervical effacement, fetal position. E. relations between the long fetal axis and the vaginal axis, dilation, cervical effacement, strength of uterine muscle contractions.

A. relation between the cervix and the vaginal axis, dilation, cervical effacement, fetal station.

In obstetrics, the term position (positio) means: A. relationship of the foetus' body parts to the parts of the maternal womb. B. relationship of the longitudinal axis of the foetus to the longitudinal axis of the maternal womb. C. relationship of the foetal head to the foetal spine. D. relationship of the foetus' body parts to one another. | E. relationship of a presenting part of the fetus to the inter-spinal line.

A. relationship of the foetus' body parts to the parts of the maternal womb.

Which of the following is not a risk factor for endometrial cancer? A. smoking tobacco. B. obesity. C. diabetes. D. exogenous estradiol. E. late menopause.

A. smoking tobacco.

Which of the following foetal head diameters is the smallest and most conveniently goes through the spaces of the pelvic planes during physiological childbirth? A. suboccipitobregmatic. B. occipitofrontal. C. occipitomental. D. submentobregmatic. E. verticomental.

A. suboccipitobregmatic.

Precocious puberty in girls is defined when tertiary sexual features appear before: A. the age of 8. B. the age of 8, but when menstruation starts concomitantly with pubic hair. C. the age of 10. D. the age of 10, but when menstruation starts concomitantly with pubic hair. E. the age of 11 regardless of race.

A. the age of 8.

Which of the following is not typical of cervical cancer? A. urinary urgency. B. bloody vaginal discharge with unpleasant smell. C. intermenstrual bleeding. D. abdominal pain. E. contact bleeding.

A. urinary urgency.

Hemorrhage, strong hypogastric pain, no palpable fundus of the uterus above the pubic symphysis and rapid deterioration of the patient's general condition during the third stage of delivery are the symptoms and signs of: A. uterine inversion. B. uterine rupture. C. rupture of the vaginal fornix. D. afterbirth retained in the uterine cavity. E. incarcerated placenta.

A. uterine inversion.

A basic recommendation for a woman with polycystic ovary syndrome, BMI>27, irregular menstrual cycles and infertility is: A. weight loss (diet). B. metformin. C. stimulation of ovulation with clomiphene citrate. D. metformin and the stimulation of ovulation with clomiphene citrate. E. progestogens in the second phase of the cycle.

A. weight loss (diet).

The prevention of Rh disease during pregnancy consists in the administration to each pregnant woman: A. who is Rh-D negative, of 300 µg anti-RhD immunoglobulin at 28 -32 week of gestation. B. irrespectively of her Rh-D status, of 300 µg anti-RhD immunoglobulin at 28 -32 week of gestation. C. who is Rh-D negative, of 300 µg anti-RhD immunoglobulin at 28t -32t week of gestation, but only if Rh-D status of the father is provided. D. who is Rh-D negative, of 150 µg anti-RhD immunoglobulin at 28 -32 week of gestation.

A. who is Rh-D negative, of 300 µg anti-RhD immunoglobulin at 28 -32 week of gestation.

Is it acceptable to postpone the next dose when taking contraceptive tablets containing only progestogen (mini pills)? A. yes, up to 3 hours. B. yes, up to 12 hours. C. yes, up to 24 hours. D. yes, up to 72 hours. E. the next dose should absolutely be taken at the same time.

A. yes, up to 3 hours.

The treatment of arterial hypertension in pregnant women includes the use of: A. angiotensin-converting-enzyme inhibitors, methyldopa, labetalol. B. angiotensin-converting-enzyme inhibitors, methyldopa, angiotensin II receptor antagonists. C. labetalol, nifedipine, methyldopa. D. diuretics, angiotensin II receptor antagonists, labetalol. E. labetalol, methyldopa, amiodarone.

C. labetalol, nifedipine, methyldopa.

A female patient was diagnosed with a lack of ovulation, acne and hirsutism. Those symptoms may suggest polycystic ovary syndrome. However, a presence of striae, a buffalo hump and arterial hypertension may suggest the diagnosis of Cushing's syndrome. Which of the following should be performed in order to differentiate those disorders? A. estradiol level. B. dexamethasone suppression test. C. metoclopramide test. D. lutropin level. E. follitropin level.

B. dexamethasone suppression test.

Which of the following is not characteristic of thrombotic thrombocytopenic purpura in a pregnant woman? A. microangiopathic hemolytic anemia. B. disseminated intravascular coagulation. C. thrombocytopenia. D. kidney failure. E. fever.

B. disseminated intravascular coagulation.

Which of the following medications is certified for the treatment of stress urinary incontinence? A. paroxetine. B. duloxetine. C. venlafaxine. D. dapoxetine. E. fluoxetine.

B. duloxetine.

A 24-year-old G1P1 just delivered a healthy infant at term. She has a history of a psychiatric disorder and was treated for depression while in college. Which of the following is she at most increased risk for in the postpartum period? A. Postpartum blues B. Postpartum depression C. Postpartum psychosis D. Postpartum anxiety E. She is not at increased risk for a psychiatric disorder

B A patient's history of a psychiatric illness is a risk factor for the development of a postpartum depression. Patients with a prior history of depression, either situational or spontaneous, are at very high risk for postpartum depression. In fact, one-third of patients with a postpartum psychiatric problem report a prior history. These patients need careful follow up after delivery, which should include an early appointment for a postpartum visit. Questions at this time should be directed to her moods and thoughts.

A 48-year-old G0 comes to the office for a health maintenance exam. She is healthy and not taking any medications. She has no history of abnormal Pap smears or sexually transmitted infections. She is not currently sexually active. Her menstrual cycles are normal and her last cycle was three weeks ago. She smokes one pack of cigarettes per day. Her mother was diagnosed with endometriosis and had a hysterectomy and removal of the ovaries at age 38. She is 5 feet 4 inches tall and weighs 130 pounds. On pelvic examination, the patient has a palpable left adnexal mass. An ultrasound was obtained, which showed a 4 cm complex left ovarian cyst and a 2 cm simple cyst on the right ovary. What is the most appropriate next step in the management of this patient? A. Oral contraceptives B. Repeat ultrasound in two months C. CT scan of the abdomen and pelvis D. Needle aspiration of the cyst E. Abdominal hysterectomy and bilateral salpingo-oophorectomy (TAH/BSO)

B A repeat ultrasound is the most appropriate next step, as this is most likely a hemorrhagic cyst which will resolve on its own. Oral contraceptives are contraindicated in this patient, as she is older than 35 and smokes. A CT scan of the pelvis will not add any more information. Needle aspiration is not the standard of care in this asymptomatic premenopausal patient. There is no indication to proceed with a TAH/BSO.

A 32-year-old G3P1 woman at 37 weeks gestation is admitted to labor and delivery for a scheduled repeat Cesarean delivery. Maternal labs show: HIV positive; blood type B+; RPR non-reactive; HBsAg negative; GBS negative; PPD positive; CXR negative. She received adequate antiretroviral therapy prior to and during the pregnancy. A live male infant is delivered with Apgar scores of 9 and 9 at 1 and 5 minutes, respectively. Which of the following is the most appropriate next step in the management of the newborn? A. Order HIV testing on the infant immediately on admission to the nursery B. Treat the infant with zidovudine (AZT) immediately after delivery C. Encourage breastfeeding D. Start zidovudine at 24 hours of life E. Isolate the infant from the other infants in the nursery

B A usual protocol is to start AZT (Zidovudine) immediately after delivery. HIV testing begins at 24 hours. There is no reason to isolate the infant even though the mother is PPD positive, because her CXR is negative. Breastfeeding would not be encouraged in a mother with HIV.

A 25-year-old G0 woman presents to the clinic for follow-up after having a first trimester spontaneous abortion. She wants to discuss the cause of this event. Which of the following etiologic categories accounts for the majority of first trimester spontaneous abortions? A. Immunologic abnormalities B. Conceptus genetic anomalies C. Maternal genetic anomalies D. Structural/uterine anomalies E. Uterine infections

B Although investigators have implicated all of the listed categories as possible causes of spontaneous abortion, genetic abnormalities involving the conceptus account for the majority. In fact, approximately 50 to 60 percent of embryos and early fetuses that are spontaneously aborted contain chromosomal abnormalities.

A 29-year-old G1P1 woman had an uncomplicated vaginal delivery and breastfed immediately postpartum. She has a significant amount of abdominal soreness secondary to a tubal ligation performed on postpartum day two. She is breastfeeding on her side with the baby lying on her side, well away from the abdomen to prevent pain at the incision site. She developed bleeding and cracked nipples. Which of the following is the most likely cause? A. Feedings not frequent enough B. Poor positioning of infant C. Feedings too frequent D. Not enough milk production E. Irritation from the bra

B Although the side lying position is a good one for breastfeeding, it is important for mother and baby to be belly-to-belly in order for the infant to be in a good position to latch on appropriately, taking a large part of the areola into its mouth. The pain experienced by the patient from her tubal may be interfering with appropriate position and she should be counseled about a different, more comfortable position.

"A 4-year-old girl is taken to see her pediatrician by her father. He states that he has noticed a foul odor in the area of the child's ""private parts."" He brings in a pair of panties with a brown-yellow stain that has a ""funny smell."" The girl is otherwise healthy. Which is the most likely cause of this condition? A. Child abuse B. Foreign body C. Yeast infection D. Pinworm infection E. Urinary tract infection"

B Children will often place foreign objects in any body orifice. Toilet paper is the most common foreign body found.

A 19-year-old G2P1 woman at 28 weeks gestation has been diagnosed with preterm labor. Her physician has chosen to treat her with magnesium sulfate. By what mechanism of action does magnesium sulfate work as a tocolytic? A. Decreases prostaglandin (PG) production B. Competes with calcium for entry into cells C. Increases cAMP in the cell D. Blocks calcium entry into muscle cells E. Inhibits calcium transport

B Magnesium sulfate works by competing with calcium entry into cells. Beta-adrenergic agents work by increasing cAMP in the cell, thereby decreasing free calcium. Prostaglandin synthetase inhibitors, such as Indomethacin, work by decreasing prostaglandin (PG) production by blocking conversion of free arachidonic acid to PG. Calcium channel blockers prevent calcium entry into muscle cells by inhibiting calcium transport.

A 36-year-old woman requests prenatal diagnosis. She is healthy and excited about finally getting pregnant. She is interested in genetic counseling and asks about the advantages of chorionic villus sampling versus amniocentesis. Which of the following is true when chorionic villus sampling is compared to amniocentesis? A. Reduced post-procedure loss rate B. Performed earlier C. More likely to obtain an adequate sample D. Lower rate of procedure related birth defects E. Performed in isoimmunized pregnancies

B Chorionic villus sampling (CVS) is a prenatal test that can detect genetic and chromosomal abnormalities of a fetus. The loss rate with amniocentesis is quoted as 0.5% vs. ~1 to 3% for chorionic villus sampling. CVS is performed between 10 and 12 weeks gestation, while amniocentesis is performed after 15 weeks. Early CVS (<10 weeks gestation) is associated with an increase in rare limb abnormalities. It is more likely that a CVS will involve multiple attempts - a failure to obtain an adequate sample of cells and the woman requiring a repeat test later on - when compared with amniocentesis. Pregnancies complicated by isoimmunization can be followed by serial assessment of the amniotic fluid for bilirubin.

A 4-year-old girl is being evaluated for premature hair growth in the pubic area. She has no breast development and has not had any menstrual bleeding. Laboratory evaluation revealed high DHEA and DHEAS levels and low levels of LH and FSH. Which of the following is the most likely cause of this girl's premature adrenarche? A. Idiopathic isosexual precocious puberty B. Congenital adrenal hyperplasia C. Hypothalamic dysfunction D. Pituitary adenoma E. Polycystic ovarian syndrome

B Congenital adrenal hyperplasia of the 21-hydroxylase type results in the adrenal being unable to produce adequate cortisol as a result of a partial block in the conversion of 17-hydroxyprogesterone to desoxycorticosterone, with the accumulation of adrenal androgens. This leads to precocious adrenarche. Treatment includes steroid replacement. Idiopathic isosexual precocious puberty is GnRH dependent and leads to an appropriate (although early) order of pubertal events. Some girls with premature adrenarche develop polycystic ovarian syndrome in adolescence, but not at this age.

A 50-year-old G3P3 comes to the office due to heavy periods, spotting between menses, fatigue and weakness. Over the past six to eight months, she has noticed a significant increase in the amount of her menstrual bleeding, currently requiring a box of 30 pads for each month. She has noticed an increase in the amount of blood clots and cramping pain during menses. Her previous history is significant for hypertension for 10 years, controlled with hydrochlorothiazide and a postpartum bilateral tubal ligation 20 years ago. She has no history of abnormal Pap smears and no sexually transmitted infections. Blood pressure is 138/84; pulse 82; respirations 20; weight 220 pounds; height 68 inches. On pelvic exam, uterus is approximately 10-week size. She is nontender on bimanual exam and no adnexal masses are appreciated. A Pap smear one month ago was normal, and her hematocrit is 29 and she is asymptomatic. What is the next best step in the management of this patient? A. Hysterectomy B. Endometrial biopsy C. Erythropoietin D. Blood transfusion E. Endometrial ablation

B Endometrial biopsy is typically an office procedure which does not cause extreme discomfort for the patient. It results in information necessary to tailor the patient's care, such as presence of endometritis, endometrial polyps or endometrial carcinoma. In a patient with significant risk factors for endometrial carcinoma, this should be done prior to a hysterectomy or ablation, if at all possible. A hysterectomy or endometrial ablation would be incorrect, as further workup is needed prior to taking the patient for one of these procedures. In addition, this patient potentially has a treatable condition, such as endometritis, an endometrial polyp, endometrial hyperplasia or an enlarging submucosal fibroid, which could all be treated with either medical therapy or a less radical procedure. With stable vital signs and a hematocrit of 29, erythropoietin and a blood transfusion would not be indicated for this patient. Her anemia could be corrected using ferrous sulfate over a period of a few months, along with targeted therapy to decrease her vaginal bleeding (therapy would be based on the endometrial biopsy result).

Thirty-six hours ago a 23 year-old G1P1 delivered vaginally and sustained a 2nd-degree laceration. She had a prolonged first stage of labor, ruptured membranes for 26 hours and received penicillin for group B Strep prophylaxis. She now complains of increasing abdominal pain, cramping and heavy foul smelling lochia. Her vital signs reveal a temperature of 100.0° F, 37.8° C; pulse 80; blood pressure 120/60; and respirations 18. She has a tender uterine fundus that measures at the umbilicus. Her extremities reveal mild bilateral edema; no erythema or tenderness. Blood work reveals a white count of 12.2; hematocrit of 34%; and normal chemistries. Her urinalysis is positive for blood and negative for WBCs, leukocyte esterase and nitrites. In addition to ampicillin, which of the following would be the best antibiotic choice? A. Erythromycin B. Gentamicin C. Doxycycline D. Vancomycin E. Ciprofloxacin

B Endomyometritis is a common complication of prolonged labor, prolonged rupture of membranes and multiple vaginal examinations. The infection is polymicrobial, mostly anaerobic and requires broad spectrum antibiotics for treatment until the patient is afebrile for 24 hours. By adding Gentamicin, you are covering the spectrum of gram-negative organisms. Erythromycin provides good coverage for upper respiratory infections. Vancomycin provides good coverage for S. aureus and penicillin-resistant gram-positive bacteria. Ciprofloxacin provides excellent coverage for gram-negative pathogens, including Pseudomonas.

A 49-year-old G2P2 status post hysterectomy at age 45 for fibroids presents to your office complaining of severe vasomotor symptoms for three months. Hot flashes are affecting her quality of life and she would like to discuss options for treatment. What treatment option for hot flashes associated with menopause do you recommend as the most effective? A. Lifestyle modifications such as dressing in layers B. Estrogen C. Selective estrogen receptor modulator (SERMs) D. Selective serotonin reuptake inhibitors (SSRIs) E. Treatment with phytoestrogen (soy)

B Except for estrogen receptor modulator therapy, all of the above treatment options will improve hot flash symptoms. Treatment with estrogen is most effective, and the current recommendation is for the lowest dose for the shortest duration of time. Hot flashes will resolve completely in 90% of patients receiving this therapy. Raloxifene, a selective estrogen receptor modulator, may actually cause hot flashes to worsen in a patient who has not stopped having these symptoms completely. SSRI antidepressants, some anti-seizure medications and alternative treatments, such as soy products and herbs, have not been shown to be as effective as estrogen.

A 37-year-old G3P0 woman at 29 weeks gestation presents with uterine contractions every five minutes. Her cervix is 1 cm dilated and 50% effaced. Fetal fibronectin test is negative. The patient stops having contractions after bedrest and hydration. What is the strength of using a fetal fibronectin test in patients with preterm contractions? A. Positive predictive value B. Negative predictive value C. High sensitivity D. Low false positive rate E. High false positive rate

B Fibronectin is an extracellular matrix protein that is thought to act as an adhesive between the fetal membranes and underlying decidua. It is normally found in cervical secretions in the first half of pregnancy. Its presence in the cervical mucus between 22 and 34 weeks is thought to indicate a disruption or injury to the maternal-fetal interface. Fetal fibronectin is FDA approved for use in women with symptoms of preterm labor from 24 to 35 weeks and during routine screening of asymptomatic patients from 22 to 30 weeks gestation. Fetal fibronectin has a negative predictive value of 99.2% in symptomatic women — 99 out of every 100 patients with a single negative test result will not deliver in the next 14 days. The positive predictive value in symptomatic women is 16.7% — 17 out of 100 women with a positive test will deliver within 14 days. In asymptomatic women, a negative fetal fibronectin test has a negative predictive value of 96.7% for delivery before 35 weeks.

A 25-year-old G1P1 woman comes in for her annual health maintenance examination. She has intermittent left lower quadrant discomfort. She has regular menses every 30 days and uses a diaphragm for birth control. Her last menstrual period was approximately three weeks ago. Her physical examination is notable for a 3 x 5 cm left adnexal mass. Ultrasound shows a unilocular simple cyst. Which of the following is the most likely diagnosis in this patient? A. Endometrioma B. Functional ovarian cyst C. Mucinous cystadenoma D. Serous cystadenoma E. Dermoid

B Functional ovarian cysts are a result of normal ovulation. They may present as an asymptomatic adnexal mass or become symptomatic. Ultrasound characteristics include a unilocular simple cyst without evidence of blood, soft tissue elements or excrescences. An endometrioma is an isolated collection of endometriosis involving an ovary. This would not classically appear as a simple cyst on ultrasound. Serous cystadenomas are generally larger than functional cysts and patients may present with increasing abdominal girth. Mucinous cystadenomas tend to be multilocular and quite large. Dermoid tumors usually have solid components or appear echogenic on ultrasound, as they may contain teeth, cartilage, bone, fat and hair.

A 17-year-old G1P0 woman at 37 weeks gestation presents for a routine visit. She has no complaints, but is found to have an initial blood pressure of 138/89, and 144/91 on a repeat reading. This reading is noted to be a change from her previous visits, as her blood pressures have been in the 90-100/50-60 range since initiating care at eight weeks gestation. Her urinalysis is negative for protein and her comprehensive metabolic panel and the complete blood count are normal. Which of the following is the most likely diagnosis in this patient? A. Chronic hypertension B. Gestational hypertension C. Mild preeclampsia D. Severe preeclampsia E. Normotensive

B Gestational hypertension is the correct diagnosis, since she lacks other defining criteria for preeclampsia. She has normal labs, liver function tests, no proteinuria and no symptoms. The only abnormal finding is mild thrombocytopenia, which may be a finding in gestational hypertension. Since her blood pressures were normal prior to this visit, chronic hypertension can be excluded. Up to one quarter of women with gestational hypertension will go on to develop preeclampsia.

A 72-year-old woman presents to the office reporting a history of vulvar itching that has been worsening over the last six months. She has a long history of lichen sclerosus, for which she has not been receiving treatment. On exam, you find an irregular-shaped lesion which measures 3.5 cm in greatest dimension, suspicious for malignancy. You perform a punch biopsy at the edge of the lesion and send it for pathologic evaluation. The pathologist reports an invasive moderately differentiated squamous cell carcinoma. Which of the following is the most appropriate treatment for this patient? A. Steroid treatment B. Radical vulvectomy and groin node dissection C. Excisional biopsy D. Laser vaporization of the lesion E. Cryotherapy

B Given the findings of obvious, moderately differentiated carcinoma, definitive treatment can be recommended with radical vulvectomy and groin node dissection. Only microinvasive squamous cell carcinoma of the vulva can be treated by wide local excision, but it is a diagnosis that is only made after pathology evaluation of a small (<2 cm), well-differentiated lesion, with invasion <1.0 mm. Excisional biopsy is not indicated given the larger lesion and confirmed finding of cancer. It would be inappropriate to laser a malignant lesion. Squamous cell carcinoma is the most common vulvar malignancy and may arise in the setting of chronic irritation from lichen sclerosus. Steroids would treat the lichen sclerosus, but would only result in needless delay in treatment of the malignancy. Cryotherapy is not an acceptable treatment for squamous cancer of the vulva.

A 32-year-old G2P1 woman at 36 weeks gestation presents with preterm premature rupture of the membranes. She denies labor. She takes prenatal vitamins and iron. She denies substance abuse, smoking or alcohol use. Her prior pregnancy delivered vaginally at 34 weeks after spontaneous rupture of membranes. Her blood pressure is 110/70; pulse 84; temperature 98.6°F (37.0°C). The estimated fetal weight is 2700 grams. She is having one contraction per hour and fetal heart tracing is category I. Which of the following is the most appropriate next step in the management of this patient? A. Observation until spontaneous onset of labor B. Augmentation of labor C. Magnesium sulfate D. Nifedipine E. Corticosteroids

B In this patient, the benefits for delivery outweigh the risk of expectant management, so the patient should undergo augmentation of labor. Expectant management at 36 weeks poses a large risk to the development of chorioamnionitis. The role of tocolytics in the setting of preterm premature rupture of membranes is controversial and is contraindicated at 36 weeks gestation. Steroid administration after 32 weeks is controversial.

The main source of alfa-fetoprotein (AFP) in the maternal blood at 16 weeks' gestation is: A. maternal liver. B. fetal liver. C. fetal adrenal glands. D. placenta. E. yolk sac.

B. fetal liver.

A 25-year-old G1P0 woman presents to labor and delivery with contractions. She is at 40 weeks gestation. Her cervix is 6 cm dilated and 100% effaced. The fetus is in the occiput anterior presentation at +1 station. Fetal heart tones are reassuring with a baseline in the 140s, multiple accelerations and no decelerations. The patient had a fetal ultrasound three days ago which reported an EFW of 2900 grams. The patient's older sister had a forceps assisted vaginal delivery and has anal incontinence. The patient would like to avoid having this same complication. Which of the following management plans is most appropriate for this patient? A. Cesarean delivery B. Vaginal delivery with no episiotomy C. Vaginal delivery with a small, controlled midline episiotomy D. Forceps assisted delivery with no episiotomy E. Vacuum assisted delivery with no episiotomy

B Historically, the purpose of performing an episiotomy was to facilitate completion of the second stage of labor to improve both maternal and neonatal outcomes. Maternal benefits were thought to include a reduced risk of perineal trauma, subsequent pelvic floor dysfunction and prolapse, urinary incontinence, fecal incontinence, and sexual dysfunction. Current data does not demonstrate these theoretical maternal and fetal benefits and there are insufficient objective evidence-based criteria to recommend episiotomy, and especially routine use of episiotomy. The risk of incontinence increases with increasing degrees of pelvic trauma. One study of extended episiotomies demonstrated that the occurrence of a fourth-degree extension was more highly associated with anal incontinence. Performance of a median episiotomy is the single greatest risk factor for third- or fourth-degree lacerations. Avoiding the use of episiotomies may be the best way to minimize the risk of subsequent extensive damage to the perineum. This patient is in active labor and has a high chance of having a vaginal delivery. A cesarean delivery is not indicated. There is no indication to perform a forceps or vacuum assisted vaginal delivery in this patient at this time.

A 17-year-old G1P1 woman delivered a term infant two days ago. She is not interested in breastfeeding and she asks for something to suppress lactation. Which of the following is the safest method of lactation suppression in this patient? A. Bromocriptine B. Breast binding, ice packs and analgesics C. Medroxyprogesterone acetate D. Oral contraceptives E. Manual milk expression

B Hormonal interventions for preventing lactation appear to predispose to thromboembolic events, as well as a significant risk of rebound engorgement. Bromocriptine, in particular, is associated with hypertension, stroke and seizures. The safest method to suppress lactation is breast binding, ice packs and analgesics. The patient should avoid breast stimulation or other means of milk expression, so that the natural inhibition of prolactin secretion will result in breast involution.

A 25-year-old G2P1 woman at 38 weeks gestation presents to labor and delivery with spontaneous onset of labor and spontaneous rupture of membranes. Cervical examination was 5 cm at presentation and 5 cm at last check, two hours ago. Presently, the patient is uncomfortable and notes strong contractions. You decide to place an intrauterine pressure catheter (IUPC). On placement, approximately 300 cc of frank blood and amniotic fluid flow out of the vagina. What is the most appropriate next step in the management of this patient? A. Emergent Cesarean delivery B. Withdraw the IUPC, monitor fetus and then replace if tracing reassuring C. Begin amnioinfusion D. Begin Pitocin augmentation E. Keep IUPC in position and connect to tocometer

B If an intrauterine pressure catheter is placed, and a significant amount of vaginal bleeding is noted, the possibility of placenta separation or uterine perforation should be considered. In this case, withdrawing the catheter, monitoring the fetus and observing for any signs of fetal compromise would be the most appropriate management. If the fetal status is found to be reassuring, then another attempt at placing the catheter may be undertaken.

A 16-year-old G1P0 woman at 39 weeks gestation presents to labor and delivery reporting a gush of blood-tinged fluid approximately five hours ago and the onset of uterine contractions shortly thereafter. She reports contractions have become stronger and closer together over the past hour. The fetal heart rate is 140 to 150 with accelerations and no decelerations. Uterine contractions are recorded every 2-3 minutes. A pelvic exam reveals that the cervix is 4 cm dilated and 100 percent effaced. Fetal station is 0. After walking around for 30 minutes the patient is put back in bed after complaining of further discomfort. She requests an epidural. However, obtaining the fetal heart rate externally has become difficult because the patient cannot lie still. What is the most appropriate next step in the management of this patient? A. Place the epidural B. Apply a fetal scalp electrode C. Perform a fetal ultrasound to assess the fetal heart rate D. Place an intrauterine pressure catheter (IUPC) E. Recommend a Cesarean delivery

B If the fetal heart rate cannot be confirmed using external methods, then the most reliable way to document fetal well-being is to apply a fetal scalp electrode. Putting in an epidural without confirming fetal status might be dangerous. Although ultrasound will provide information regarding the fetal heart rate, it is not practical to use this to monitor the fetus continuously while the epidural is placed. An intrauterine pressure catheter will provide information about the strength and frequency of the patient's contractions, but will not provide information regarding the fetal status. Closer fetal monitoring via a fetal scalp electrode should be performed.

A 44-year-old nulliparous woman returns to the office for a post operative check following tumor debulking for stage IIIB endometrioid adenocarcinoma of the ovary. Her medical history is significant for diabetes, hypertension, obesity, hypercholesterolemia and major depression. Which of the following is the most appropriate next step in the management of this patient? A. Hospice B. Chemotherapy C. Surveillance D. Pelvic radiation E. Second look laparotomy

B In all patients with advanced ovarian cancer, post-operative chemotherapy with a combination of a taxane and platinum adjunct is considered standard of care in the United States. Women who undergo surgical cytoreduction, followed by chemotherapy, have a better overall survival rate than those who undergo surgery alone. The overall response rate in women with advanced ovarian cancer following surgery and 4-6 cycles of combination chemotherapy with a taxane and platinum adjunct is 60-80%. The overall five-year survival for women with stage III and IV disease is approximately 30%. Second look laparotomy is no longer considered standard of care.

"A mother brings in her 6-year-old daughter for an examination. She states that she thinks her boyfriend may be ""fooling around"" with the child. The girl runs and hides when the boyfriend comes to their home. Examination reveals a normal 6-year-old girl. There is no evidence of sexual abuse. The mother wants nothing done as she is dependent on her boyfriend for support. Which of the following is the most appropriate course of action? A. Advise return visit in one month B. Notify police C. Advise family counseling D. Inform mother that the child may be jealous E. Perform colposcopic examination"

B In all suspected cases of child abuse, the proper authorities must be notified. Often victims of child abuse have no physical findings. Colposcopic examination is often inconclusive. The child may be acting out, but it is better to err on the side of safety.

A 27-year-old G1P0 at 34 weeks gestation is brought in by ambulance after a motor vehicle accident. Although restrained in the car with a safety belt, she suffers a significant head laceration. When she arrives in the emergency room, her initial trauma survey is completed. On her secondary survey, there is bright red blood coming from the vagina. Her abdomen is noted to be tense. Subsequent documentation of the fetal heart tones reveals fetal tachycardia. Abruption is suspected and the patient is rushed to the operating room for an emergent Cesarean section. After delivery, the nurse notes that an informed surgical consent was never signed. Which of the following is true? a. Informed consent is valid if the doctor-patient discussion occurred soon after the patient received intravenous morphine for pain relief b. Informed consent is unnecessary in an emergency situation if a delay in treatment would risk the patient's health/life c. Informed consent is only required for invasive procedures d. Informed consent would not have been valid anyway because the patient sustained a head laceration e. In an emergency situation, informed consent documents can be signed after the procedure is over and the patient is stable

B Informed consent needs to be obtained for all procedures while patient is fully alert and has not received any narcotics or other medications that may affect her decision-making. The only exception is in true emergency situations that would risk the patient's life. Obtaining informed consent does not necessarily protect the provider from lawsuits and should never be signed after a procedure is already completed.

A 24-year-old G0 presents with a one-year history of introital and deep thrust dyspareunia. She has a two-year history of severe dysmenorrhea, despite the use of oral contraceptives. She also reports significant urinary frequency, urgency, and nocturia. A recent urine culture was negative. She underwent a diagnostic laparoscopy six months ago that showed minimal endometriosis with small implants in the posterior cul de sac only, which were ablated with a CO2 laser. What is the most likely diagnosis in this patient? A. Acute cystitis B. Interstitial cystitis C. Acute urethral syndrome D. Acute urethritis E. Salpingitis

B Interstitial cystitis (IC) is a chronic inflammatory condition of the bladder, which is clinically characterized by recurrent irritative voiding symptoms of urgency and frequency, in the absence of objective evidence of another disease that could cause the symptoms. Pelvic pain is reported by up to 70% of women with IC and, occasionally, it is the presenting symptom or chief complaint. Women may also experience dyspareunia. The specific etiology is unknown, but IC may have an autoimmune and even hereditary component.

A 16-year-old girl comes to the doctor to discuss contraception. She recently became sexually active and states she has never had a menstrual cycle. She regularly attends school and participates in the band. On physical examination, she is 5 feet 3 inches tall and weighs 130 pounds. She has no secondary sexual characteristics with normal appearing external genitalia. The physician suspects Kallmann syndrome. Which of the following diagnostic tests will help confirm the diagnosis? A. An MRI of the pituitary B. Olfactory challenge C. Measurement of testosterone levels D. Pelvic ultrasound E. Cortisol levels

B Kallmann syndrome is characterized by olfactory tract hypoplasia and the arcuate nucleus does not secrete GnRH. Therefore, these females have no sense of smell and do not develop secondary sexual characteristics. The diagnosis is often one of exclusion found during the workup of delayed puberty. The presence of anosmia with delayed puberty should suggest Kallmann syndrome. Treatment is pulsatile GnRH therapy.

A 19-year-old G0 presents with severe menstrual pain that causes her to miss school. She takes 600 mg of ibuprofen every four to six hours to control the pain, but this does not relieve the discomfort. You started oral contraceptives, but her symptoms persisted. She also tried Depo-Provera for three months without much improvement. She still has menstrual pain and continues to miss some classes. What is the most appropriate next step in the management? A. Transdermal narcotic for pain relief B. Diagnostic laparoscopy C. Presacral neurectomy D. Prescribe a selective serotonin reuptake inhibitor E. Prescribe GnRH agonist

B Laparoscopy is recommended to confirm the diagnosis of endometriosis and exclude other causes of secondary dysmenorrhea. Some authors suggest that a course of GnRH agonists are appropriate, with laparoscopy reserved for those women who have pain during or after completion of a three-month course.

A 64-year-old G2P2 woman presents with a 12-month history of severe vulvar pruritus. She has applied multiple over-the-counter topical therapies without improvement. She has no significant vaginal discharge. She has severe introital dyspareunia and has stopped having intercourse because of the pain. Her past medical history is significant for allergic rhinitis and hypertension. On pelvic examination the external genitalia show loss of the labia minora with resorption of the clitoris (phimosis). The vulvar skin appears thin and pale and involves the perianal area as in the picture below. No ulcerations are present. The vagina is mildly atrophic, but appears uninvolved. Which of the following is the most likely diagnosis in this patient? A. Squamous cell hyperplasia B. Lichen sclerosus C. Lichen planus D. Candidiasis E. Vulvar cancer

B Lichen sclerosus is a chronic inflammatory skin condition that most commonly affects Caucasian premenarchal girls and postmenopausal women. The exact etiology is unknown, but is most likely multifactorial. Patients typically present with extreme vulvar pruritus and may also present with vulvar burning, pain and introital dyspareunia. Early skin changes include polygonal ivory papules involving the vulva and perianal areas, waxy sheen on the labia minora and clitoris, and hypopigmentation. The vagina is not involved. More advanced skin changes may include fissures and erosions due to a chronic itch-scratch-itch cycle, mucosal edema and surface vascular changes. Ultimately, scarring with loss of normal architecture, such as introital stenosis and resorption of the clitoris (phimosis) and labia minora, may occur. Treatment involves use of high-potency topical steroids. There is less than a 5% risk of developing squamous cell cancer within a field of lichen sclerosus.

A 25-year-old G1P1 comes to the office due to left breast pain and fever. She is breast feeding her 2 ½-week-old infant. The symptoms began earlier in the day and are not relieved by acetaminophen. Blood pressure 120/60; pulse 64; temperature 99.9° F (37.7° C). On exam, she has erythema on the upper outer quadrant of the left breast which is tender to touch. There are no palpable masses. In addition to starting oral antibiotics, what is the most appropriate next step in the management of this patient? A. Discontinue breastfeeding B. Add ibuprofen for pain relief C. Obtain a breast ultrasound D. Use a topical antifungal E. Ice packs and binders

B Mastitis that accompanies pregnancy or nursing is the most common breast infection. Puerperal mastitis most commonly occurs during the second to fourth week after delivery. Patients are treated with oral or IV antibiotics, depending on the severity of infection. Patients may use ibuprofen in addition to acetaminophen for pain relief, and are encouraged to continue breastfeeding or expressing their milk during treatment. Mastitis is usually treated as an outpatient. Ice packs and breast binders are used to decrease breast discomfort in women who do not desire to breastfeed. Cold compresses may reduce inflammation but are not indicated in the management of mastitis. A breast ultrasound is not indicated if there is no suspicion of a breast abscess.

A 32-year old G1 woman at 29 weeks gestation presents with preterm labor. She is started on indomethacin. What is a possible adverse fetal effect associated with indomethacin treatment? A. Polyhydramnios B. Premature constriction of the ductus arteriosus C. Fetal growth restriction D. Hypoxia E. Chorioamnionitis

B Maternal indomethacin exposure can result in premature constriction of the ductus arteriosus, especially if used after 32 weeks gestation. Polyhydramnios is not associated with indomethacin. In fact, indomethacin is associated with oligohydramnios. Fetal hypoxia and decreased uteroplacental blood flow have been associated with the use of calcium channel blockers, such as Nifedipine. Indomethacin should not cause chorioamnionitis.

A 23-year-old female college student presents with amenorrhea for 10 months. She had menarche at age 14 and normal regular menses every 28 days until this year. She is in good health and not taking any medications. She is 5 feet 4 inches tall and weighs 130 pounds. Her examination, including a pelvic exam, is normal. Beta-hCG is < 5 mIU/mL, and TSH is 5.0 mU/L (normal 0.35-6.7 mU/L). What is the most appropriate next diagnostic test to help determine the cause of amenorrhea in this patient? A. Serum 17-hydroxyprogesterone level B. Serum prolactin level C. Pelvic ultrasound D. Serum LH and FSH levels E. Brain MRI

B Measurement of serum prolactin level is part of the initial laboratory assessment for a patient with amenorrhea and no other symptoms or findings on physical exam. A prolactinoma is the most common pituitary tumor causing amenorrhea. Galactorrhea may be present when hyperprolactinemia is the cause of anovulation and amenorrhea. A pelvic ultrasound is not typically helpful in a young patient with a normal pelvic examination. A brain MRI might be indicated if prolactin levels return elevated. 17-hydroxyprogesterone, LH and FSH levels might need to be obtained in the work-up of this patient, if other tests return normal.

A 36-year-old G1 woman presents in active labor. Her past medical history and prenatal course were complicated by chronic hypertension and superimposed preeclampsia. She received magnesium sulfate for seizure prophylaxis and oxytocin augmentation. She undergoes an uneventful spontaneous vaginal delivery. Postpartum, she has a 1000 ml hemorrhage due to uterine atony. Her blood pressure is 130/80; pulse 96; and she is afebrile. Which of the following uterotonic agents is contraindicated in this patient? A. Oxytocin B. Methylergonovine C. Prostaglandin F2-alpha D. Prostaglandin E2 E. Misoprostol

B Methergine, prostaglandins and oxytocin are all uterotonics and used to increase uterine contractions and decrease uterine bleeding. Methylergonovine is an ergot alkaloid, which is a potent smooth muscle constrictor. It is also a vasoconstrictive agent and should be withheld from women with hypertension and/or preeclampsia. Misoprostol, non-FDA approved, is used for cervical ripening and labor induction.

A 29-year-old G1 woman at 31 weeks gestation presents with watery discharge from the vagina commencing several hours ago. Her prenatal course has been uncomplicated and she takes prenatal vitamins and iron. She denies substance abuse, smoking or alcohol use. On examination, her blood pressure is 110/70; pulse 84; temperature 98.6°F (37.0°C). Which of the following is the most appropriate next step in the management of this patient? A. Nitrazine testing of mucus swabbed from cervix B. Examination of vaginal fluid for ferning C. Digital examination of cervix D. Determination of amniotic fluid index (AFI) E. Non-stress test

B Methods to confirm rupture of membranes include testing the vaginal fluid for ferning and nitrazine testing. It is important to test the fluid from the vagina and not to test cervical mucus because of false positive ferning patterns. A digital exam should be avoided in a patient you suspect might have preterm rupture of membranes because of the risk of introducing bacteria into the uterine cavity and increasing risk for chorioamnionitis. Determination of AFI with ultrasound may reveal oligohydramnios and support the diagnosis of rupture of membranes, but does not confirm this diagnosis. Similarly, a non-stress test may reveal variable decelerations, which may be present in the setting of rupture of membranes.

A 19 year-old G1P0 woman at 18 weeks gestation presents with a 3-month history of palpitations and intermittent chest pain. Physical examination reveals a pulse of 96 and grade II/VI systolic ejection murmur with a click. The ECG shows normal rate and rhythm and an echocardiogram is ordered. Which of the following is the best treatment in the management of this patient? A. Anxiolytics B. β-blockers C. Calcium-channel blockers D. Digitalis E. No treatment needed at this time

B Most women with mitral valve prolapse are asymptomatic and diagnosed by routine physical examination or as an incidental finding at echocardiography. A small percentage of women with symptoms have anxiety, palpitations, atypical chest pain, and syncope. For women who are symptomatic, -blocking drugs are given to decrease sympathetic tone, relieve chest pain and palpitations, and reduce the risk of life-threatening arrhythmias. Because she is symptomatic, the option of no treatment is not correct.

A 20-year-old nulliparous college student presents with a one-month history of profuse vaginal discharge and mid-cycle vaginal spotting. She uses oral contraceptives and she thinks her irregular bleeding is due to the pill. She is sexually active and has had a new partner within the past three months. She reports no fevers or lower abdominal pain. She has otherwise been healthy. On pelvic examination, a thick yellow endocervical discharge is noted. Saline microscopy reveals multiple white blood cells, but no clue cells or trichomonads. Potassium hydroxide testing is negative. Vaginal pH is 4.0. No cervical motion tenderness or uterine/adnexal tenderness is present. Testing for gonorrhea and chlamydia is performed, but those results will not be available for several days and the student will be leaving for Europe tomorrow. Which of the following is the most appropriate treatment for this patient? A. Metronidazole and erythromycin B. Ceftriaxone and azithromycin C. Ampicillin and doxycycline D. Azithromycin and doxycycline E. No treatment is necessary until all tests results are known

B Mucopurulent cervicitis (MPC) is characterized by a mucopurulent exudate visible in the endocervical canal or in an endocervical swab specimen. MPC is typically asymptomatic, but some women have an abnormal discharge or abnormal vaginal bleeding. MPC can be caused by Chlamydia trachomatis or Neisseria gonorrhoeae; however, in most cases neither organism can be isolated. Patients with MPC should be tested for both of these organisms. The results of sensitive tests for C. trachomatis or N. gonorrhoeae (e.g. culture or nucleic acid amplification tests) should determine the need for treatment, unless the likelihood of infection with either organism is high or the patient is unlikely to return for treatment. Antimicrobial therapy should include coverage for both organisms, such as azithromycin or doxycycline for chlamydia and a cephalosporin or quinolone for gonorrhea. Uncomplicated cervicitis, as in this patient, would require only 125 mg of Ceftriaxone in a single dose. Ceftriaxone 250 mg is necessary for the treatment of upper genital tract infection or pelvic inflammatory disease (PID).

A 32 year-old delivered a 9-pound baby and sustained a 4th-degree laceration two days ago. The delivery was complicated by a shoulder dystocia. The laceration was repaired in layers in the customary fashion. She now complains of increasing perineal pain, fever chills and weakness. Her vital signs are: blood pressure 90/50; pulse 120; and temperature 102.2°F 39°C. Her abdomen is soft, nontender and her uterine fundus is firm and nontender. Her perineum is erythematous, swollen and the laceration edges are grey. The laceration site is nontender and without feeling but there is tenderness of the surrounding tissue. What is the most likely etiology for her fevers? A. Endomyometritis B. Necrotizing Fascitis C. Cellulitis D. Hematoma E. Proctocolitis

B Necrotizing fascitis, a dangerous infection caused by gas forming organisms, such as Clostridium, can quickly cause sepsis and death. Classic clinical manifestations include fever, pain and induration of the wound. Treatment involves early recognition, antibiotics and debridement of the necrotic tissue. Classic clinical findings for endomyometritis include fever and maternal tachycardia, uterine tenderness and no other localizing signs of infection. Cellulitis presents as swollen, erythematous, tender and warm area, but without grey necrotic edges. Hematoma findings include a swollen, tender and painful area that may be expanding.

A 45-year-old G2P2 woman underwent an abdominal hysterectomy for a large fibroid uterus via a low transverse skin incision. Her postoperative course was significant for new onset right lower quadrant pain and numbness, radiating into the right inguinal area and medial thigh. Her pain was exacerbated by adduction of her right thigh. On abdominal examination, there is a well-healed low transverse incision. Her pain is reproduced with adduction of the right thigh. There is decreased sensation to light touch and pinprick over the right inguinal area and right medial thigh. Patellar reflexes are 2+ and symmetric. Entrapment of which of the following nerves is the most likely cause of her pain? A. Obturator nerve B. Ilioinguinal nerve C. Lateral femoral cutaneous nerve D. Femoral nerve E. Iliohypogastric nerve

B Nerve entrapment syndrome is a commonly misdiagnosed neuropathy that can complicate pelvic surgical procedures performed through a low transverse incision. The nerves at risk are the iliohypogastric nerve (T-12, L-1) and the ilioinguinal (T-12, L-1) nerve. These two nerves exit the spinal column at the 12th vertebral body and pass laterally through the psoas muscle before piercing the transversus abdominus muscle to the anterior abdominal wall. Once at the anterior superior iliac spine, the iliohypogastric nerve courses medially between the internal and external oblique muscles, becoming cutaneous 1 cm superior to the superficial inguinal ring. The iliohypogastric nerve provides cutaneous sensation to the groin and the skin overlying the pubis. The ilioinguinal nerve follows a similar, although slightly lower, course as the iliohypogastric nerve where it provides cutaneous sensation to the groin, symphysis, labium and upper inner thigh. These nerves may become susceptible to injury when a low transverse incision is extended beyond the lateral border of the rectus abdominus muscle, into the internal oblique muscle. Symptoms are attributed to suture incorporation of the nerve during fascial closure, direct nerve trauma with subsequent neuroma formation, or neural constriction due to normal scarring and healing. Damage to the obturator nerve, which can occur during lymph node dissection would result in the inability of the patient to adduct the thigh.

A 62-year-old G0 postmenopausal woman is being referred to your gynecologic oncology colleague after an office endometrial sample demonstrated a FIGO grade 1 endometrioid adenocarcinoma. The patient has no significant medical, surgical or other gynecologic history. She does not smoke and drinks only occasionally at social events. She takes a multivitamin. Her physical exam is unremarkable. Which of the following additional tests is indicated for this patient? A. Pelvic ultrasound B. Chest x-ray C. Pelvic MRI D. CA-125 E. Serum estrogen level

B Once a pathologic diagnosis is confirmed by biopsy, a basic clinical assessment should ensue in all patients to help define the extent of the disease. If a careful history and clinical gynecologic exam suggests that the carcinoma is likely of an early stage, minimal pre-treatment evaluation is necessary. Routine evaluation in this setting should include a chest x-ray as the lungs are the most common site of distant spread. A pelvic ultrasound is not indicated once a pathologic diagnosis has been established, although one may have been obtained as part of the initial evaluation of postmenopausal bleeding. When there is a low suspicion for advanced disease, a CT scan, MRI, PET scan, and other invasive and costly tests are not indicated. A CA-125 may be helpful in predicting those patients that may have extrauterine spread, but is not absolutely necessary.

A 32-year-old G2P2 is concerned about symptoms associated with her menstrual cycle. During the second half of her cycle, she feels anxious, sad and has difficulty sleeping. She has done research on the Internet and believes she suffers from premenstrual dysphoric disorder (PMDD). Which of the following symptoms of the patient is most consistent with this diagnosis? A. Cyclic constellation of symptoms during the follicular phase B. Cyclic occurrence of a minimum of described symptoms and interference in social functioning C. Chronic, mild depressive symptoms that have been present for many years D. Depressed mood or the loss of interest or pleasure in activities E. Anxiety/nervousness interfering in social functioning

B PMDD is a psychiatric diagnosis, describing a severe form of premenstrual syndrome in which the diagnostic criteria include five out of 11 clearly defined symptoms, functional impairment and prospective charting of symptoms. All three areas of symptoms need to be represented for the diagnosis of PMDD.

An 18-year-old woman, gravida 1, para 0, at 41 weeks of gestation has had rupture of membranes for approximately 17 hours. She is in early labor with contractions every 5 minutes with cervical dilation at 5-6 cm. She received an epidural for pain relief 12 hours ago. Her temperature is 38.5°C (101°F). Her pulse is 124 beats per minute (bpm) and the fetal heart rate is 175 bpm with accelerations. She does not have any shortness of breath, dyspnea, or headaches. Her physical examination is notable for fundal tenderness. On admission, her white blood cell count was 18,000 cells/mm and her urinalysis had negative leukocyte esterase and nitrites. The most likely cause of her fever is . A) pyelonephritis B) chorioamnionitis C) pneumonia D) epidural fever E) meningitis

B) chorioamnionitis

A 32-year-old G0 presents with irregular menses occurring every six to eight weeks for the past eight months. The bleeding alternates between light and heavy. Her irregular menses were treated successfully with Medroxyprogesterone Acetate (MPA), 10 mg every day, taken for 10 days each month. By which mechanism does the MPA control her periods? A. Stimulates rapid endometrial growth and regeneration of glandular stumps B. Converts endometrium from proliferative to secretory C. Promotes release of Prostaglandin F2α D. Regenerates functional layer of the endometrium E. Decreases luteal phase inhibin production

B Patients with anovulatory bleeding have predominantly proliferative endometrium from unopposed stimulation by estrogen. Progestins inhibit further endometrial growth, converting the proliferative to secretory endometrium. Withdrawal of the progestin then mimics the effect of the involution of the corpus luteum, creating a normal sloughing of the endometrium. Stimulation of rapid endometrial growth, conversion of proliferative to secretory endometrium, and regeneration of the functional layer describe effects of estrogen on the endometrium. Inhibin is increased in the luteal phase.

A 38-year-old G5P4 woman with a history of four Cesarean deliveries is at 36 weeks gestation with a singleton pregnancy. She presents to labor and delivery with complaints of vaginal bleeding for the last hour. Prenatal care has been unremarkable except for a second trimester ultrasound discovering an anterior placenta, which partially covers the cervical os. Follow up ultrasound exams have confirmed these findings. The patient denies uterine contractions and abdominal pain. She feels the baby moving. Her blood pressure is 110/60, pulse 110, and she is afebrile. Her abdomen and uterus are non-tender and soft. Fetal heart tones have a baseline of 140 and are reassuring. This patient is at greatest risk for which of the following complications? A. Vasa previa B. Placenta accreta C. Placental abruption D. Uterine rupture E. Preterm labor

B Placenta accreta occurs when the placenta grows into the myometrium. This patient is at risk for this condition due to her history of four previous Cesarean deliveries, and the low anterior placenta. The scar tissue from the previous surgery prevents proper implantation of the placenta and it subsequently grows into the muscle. Vasa previa is a rare condition where the umbilical cord inserts into the membranes. Placental abruption is the premature separation of the normally implanted placenta. Risk of uterine rupture could be as high as 5% in this case, and the risk of placenta accreta with four prior c-sections approaches 50%. The patient is not experiencing contractions at the present time, so preterm labor is unlikely.

An 81-year-old G3P3 presents to your office with a history of light vaginal spotting. She states this has occurred recently and in association with a thin yellow discharge. She never experienced any vaginal bleeding since menopause at the age of 52, and denies ever having been on hormone replacement therapy. She is otherwise reasonably healthy, except for osteoporosis, well-controlled hypertension, and diabetes. She is physically active and still drives to all her appointments. She is no longer sexually active since the death of her husband two years ago. On examination, she is noted to have severe atrophic changes affecting her vulva and vagina. A small Pederson speculum allows for visualization of a normal multiparous cervix, and the bimanual examination is notable for a small, mobile uterus. Rectovaginal exam confirms no suspicious adnexal masses or nodularity. Which of the following is the most appropriate management for this patient? A. Pelvic transvaginal ultrasound B. Office endometrial biopsy C. Reassurance and observation for further bleeding D. Vaginal estrogen therapy E. Clindamycin vaginal cream

B Postmenopausal bleeding or discharge accounts for the presenting symptom in 80-90% of women with endometrial cancer. However, the most common causes of postmenopausal bleeding are atrophy of the endometrium (60-80%), hormone replacement therapy (15-25%), endometrial cancer (10-15%), polyps (2-12%), and hyperplasia (5-10%). Any history of vaginal bleeding requires a thorough history, physical/pelvic examination, and assessment of the endometrium. This is ideally done via office endometrial sampling as part of the initial work-up. The use of pelvic transvaginal ultrasound can provide useful information as to the presence of any structural changes (polyps, myomas, endometrial thickening), and for which a diagnosis of endometrial cancer would be less likely if the endometrial thickness is < 5 mm. Although this patient is likely to have atrophy as the cause of her spotting, a thin endometrial stripe does not exclude the possibility of a non-estrogen dependent carcinoma of the atrophic endometrium. Vaginal estrogen or clindamycin are not indicated.

A 22-year-old G1P0 woman at 39-weeks gestation presents in active labor. Her pregnancy is complicated by diet controlled gestational diabetes. She has a history of uterine fibroids. On examination, she is found to be 4 cm dilated in breech presentation. An ultrasound confirms the breech presentation, amniotic fluid index is 5, and the estimated fetal weight is 3900 g. Which of the following is the most likely cause of the breech presentation in this patient? A. Gestational diabetes B. Uterine fibroids C. Oligohydramnios D. Macrosomia E. Gravidity

B Prematurity, multiple gestation, genetic disorders, polyhydramnios, hydrocephaly, anencephaly, placenta previa, uterine anomalies and uterine fibroids are all associated with breech presentation.

A 40-year-old patient presents for her first health maintenance examination. She denies any new complaints or symptoms. She has no history of any gynecologic problems. Family history is significant for a father with hypertension and a mother, deceased, with breast cancer diagnosed at age 56. A paternal aunt has ovarian cancer which was diagnosed at age 83. A physical exam is unremarkable. What screening test should be offered to this patient next? A. Breast MRI B. Mammogram C. Transvaginal pelvic ultrasound D. Breast ultrasound E. BRCA-1/BRCA-2 testing

B Presently, there are limitations in the ability to screen for cancer. The patient's family history of a first-degree relative with postmenopausal breast cancer means that she could be at increased risk and genetic counseling can be suggested for evaluation for BRCA testing. A combination of first and second-degree relatives on the same side of the family diagnosed with breast and ovarian cancer (one cancer type per person) increases the risk of BRCA mutation in non-Ashkenazi women. At present, transvaginal sonography and CA-125 testing are not recommended as first-line screening tools for the general population for ovarian cancer. Ultrasound and MRI are generally used as adjunctive studies when a patient has an abnormal or unsatisfactory mammogram.

A 28-year-old nulliparous woman has a high-grade squamous intraepithelial lesion (HSIL) on a Pap smear. She has no complaints. She smokes one pack of cigarettes per day. Her pelvic exam is normal. Colposcopy is performed. The cervix is noted to have an ectropion and there is abundant acetowhite epithelium. Mosaicism, punctations and several disorderly atypical vessels are noted. Three biopsies are obtained and sent to pathology. Which of the findings on this patient's colposcopy is most concerning? A. Ectropion B. Acetowhite epithelium C. Mosaicism D. Punctations E. Disorderly atypical vessels

B Punctations and mosaicism represent new blood vessels on end and on their sides, respectively. Atypical vessels usually represent a greater degree of angiogenesis and, thus, usually a more concerning lesion. An ectropion is an area of columnar epithelium that has not yet undergone squamous metaplasia. It appears as a reddish ring of tissue surrounding the external os. Acetowhite epithelium can represent dysplasia but, in most cases, is less concerning than the above vascular changes.

A 32-year-old woman is preparing for infertility treatment. You counsel her that she can reduce her risk of having a fetus with a neural tube defect (NTD) by taking a supplement that contains the micronutrient: A) thiamine B) folic acid C) pyridoxine D) beta carotene

B) folic acid

A 28-year-old Rh negative G1P0 woman at eight weeks gestation presents to the clinic for a first prenatal visit. Which of the following is the current recommendation for RhoGAM administration to prevent Rh isoimmunization? A. Routine administration for every Rh-sensitized woman at term B. Administration for Rh-negative patients with no Rh antibodies at 28 weeks C. Administration for every Rh-negative woman who delivers an Rh-negative infant D. Routine administration for all Rh-negative patients during first trimester E. Routine administration for all Rh-negative patients during each trimester

B RhoGAM (Anti-D-immunoglobulin) is administered to Rh-negative women to prevent isoimmunization. Each dose provides 300 micrograms of D-antibody and is given to the D-negative non-sensitized mother to prevent sensitization after any pregnancy-related events that could result in fetal-maternal hemorrhage. Up to 2 percent of women with a spontaneous abortion and 5 percent of those undergoing elective termination become isoimmunized without D-immunoglobulin. The current recommendations for Rh-negative women without evidence of Rh immunization is prophylactically at 28-weeks gestation (after an indirect Coombs' test) and within 72 hours of delivering an Rh-positive baby, following spontaneous or induced abortion, following antepartum hemorrhage and following amniocentesis or chorionic villus sampling. If the father of the fetus is known to be Rh-negative, RhoGAM is not necessary, since the fetus will be Rh-negative and not at risk for hemolytic disease.

An 18-year-old G0 comes in for a health maintenance examination with her mother. The mother had severe PMS symptoms in her twenties and thirties and would like to know if her daughter would inherit this as well. Which of the following has the strongest association with premenstrual syndrome? A. Obesity B. Positive family history C. History of early menarche D. Insulin dependent diabetes mellitus E. Vitamin K deficiency

B Risk factors for PMS include a family history of premenstrual syndrome (PMS) and Vitamin B6, calcium, or magnesium deficiency. PMS becomes increasingly common as women age through their 30s, and symptoms sometimes get worse over time. Previous anxiety, depression or other mental health problems are significant risk factors for developing premenstrual dysphoric disorder (PMDD). There is no known association between premenstrual syndrome and obesity or insulin dependent diabetes mellitus.

A 28-year-old G1P0 presents to the emergency room at 2 am stating that, while having sex with her husband, she started to have severe vaginal bleeding. She has minimal pain, but is concerned that the bleeding won't stop. On examination, she is mildly anxious and appears embarrassed. Her vital signs are notable for a blood pressure of 118/62 and pulse of 104. Her abdominal exam is benign, and her pelvic exam is notable for a laceration along the posterior vaginal wall extending into the fornix. Active bleeding is occurring. When further questioned, she admitted to using a new sexual toy that she and her husband wanted to try out during intercourse. Which of the following is the most appropriate next step in management? A. Transfuse the patient with packed red blood cells immediately B. Repair the laceration under anesthesia C. Obtain cultures for sexually transmitted infections D. Provide emergency contraception E. Notify the police

B Sexual trauma and vaginal lacerations can occur during consensual sexual behavior and do not necessarily indicate sexual abuse. However, aggressive sexual activity can result in serious injuries, especially if mechanical or foreign objects are not safely used. Exploration and repair of this injury should be done in the operating room under general anesthesia. A blood transfusion may be indicated, depending on her blood count and estimated blood loss, but may not be necessary. Testing and treatment for sexually transmitted infections or emergency contraception should be discussed with the patient, as not all cases of sexual trauma are due to abuse/assault.

A 26-year-old African-American G1P0 woman presents to your office at seven weeks gestation with her husband, who is also African-American. The patient's brother has sickle cell anemia, and has been hospitalized on numerous occasions with painful crises requiring narcotic pain medication and blood transfusions. What are the odds that this couple will have a child with sickle cell anemia, if the carrier rate for sickle cell disease in the African American population is 1/10? A. 1 in 15 B. 1 in 60 C. 1 in 100 D. 1 in 160 E. 1 in 400

B Sickle cell anemia is an autosomal recessive condition that occurs in 1/500 births in the African-American population. The carrier state, or sickle-cell trait, is found in approximately 1/10 African-Americans. Since the patient's brother is affected, both of their parents have to be carriers. Each time two carrier parents for an autosomal recessive condition conceive there is a 1/4 chance of having either an affected or an unaffected child and a 1/2 chance of having a child who is a carrier. Since the patient is unaffected, she has a 1/3 chance of not being a carrier and a 2/3 chance of being a carrier. The patient's husband has a 1/10 chance of being a carrier (the general population risk for African-Americans). Thus, the chance that this couple will have a child with sickle cell anemia is: 2/3 X 1/10 X 1/4 = 1/60.

A 30-year-old G2P0 woman at 38 weeks gestation has just delivered a male infant. She has a history of type 1 diabetes since age 11. Maternal labs show: blood type B+; RPR non-reactive; HBsAg negative; HIV negative; and GBS negative. She had moderate control of blood sugar during her pregnancy. Which of the following would be the most likely finding in the newborn? A. Large and hypoglycemic B. Small and hypoglycemic C. Large and hyperglycemic D. Small and hyperglycemic E. Normal size and euglycemic

B Small babies are more common with type 1 diabetes than with gestational diabetes, and the blood sugar level of all newborns of diabetic mothers should be monitored closely after delivery, as they are at increased risk for developing hypoglycemia. Macrosomic (large) infants are typically associated with gestational diabetes. (DM1→ lack of insulin-like GFs → small baby; gDM → insulin-like GFs → big baby)

An 84-year-old G2P2 widow is seen in the Emergency Department because of a pain in her right arm. Her daughter, with whom she lives, states that her mother is always falling down and is becoming forgetful. She has a history of breast cancer treated by mastectomy 15 years ago and took tamoxifen for five years. The patient is on zolpidem (Ambien) 5 mg for sleep. She has been unable to afford the bisphosphonates needed for bone loss. Examination reveals a cachectic white woman, 5 feet 2 inches tall, weighing 94 pounds. She has tenderness in her right forearm with swelling. She is also noted to have old and new bruises on her chest and arms. An x-ray of the right arm reveals a spiral fracture. Which of the following is the most likely cause of the bone fracture? A. Osteoporosis B. Elder abuse C. Bone metastasis from occult breast cancer D. Malnutrition E. Falls due to unsteadiness

B Spiral fractures rarely occur unless violence has taken place. It is due to a twisting of the arm causing the almost pathognomonic finding. Care for the elderly is difficult, but there is no excuse for violence against the elderly.

A 21-year-old G0 woman presents for her first pelvic examination. She is completely asymptomatic, healthy, and reports having only one sexual partner. She uses condoms for contraception. On examination, the patient has a normal appearing cervix except for minimal, non-malodorous vaginal discharge. You perform chlamydia and gonorrhea screening and obtain a Pap smear. The Pap smear is read as ASCUS (atypical squamous cells of undetermined significance), HPV was not performed, and her cultures are negative. Which of the following is the most appropriate management strategy for this patient? A. Pap smear in six weeks B. Pap smear in one year C. Colposcopy with endocervical curettage and directed biopsies D. Cryotherapy E. Cervical conization

B The ASCCP (American Society of Colposcopy and Cervical Pathology) recommends that management options for ASCUS include performing HPV DNA testing or repeat cytology at 12 months following the abnormal Pap test result. If the HPV testing is negative (as was reported in this case), then routine screening can be resumed at three years. If HPV is positive, or if repeat cytology at 12 months reveals ASCUS or higher, then colposcopy should be performed. For women ages 21 - 24, if HPV is positive, then repeat cytology at 12 months is recommended with colposcopy performed only if the repeat cytology reveals ASC-H (atypical squamous cell - cannot rule out high grade squamous intraepithelial lesion), AGC (atypical glandular cells) or HSIL (high-grade squamous intraepithelial lesion). The presence of an underlying infection does not affect the triage of an abnormal Pap smear but may explain the presence of ASCUS. See ASCCP guidelines: http://www.asccp.org/Portals/9/docs/Algorithms%207.30.13.pdf

A 42-year-old G5P4 woman at eight weeks gestation presents for her first prenatal appointment. She has glycosuria noted on urine dipstick in the office. She has a history of four prior vaginal deliveries at full-term with birth weights ranging from 9 to 10.5 pounds. Family history is positive for type 2 diabetes in her mother and two siblings. Weight is 265 pounds and height is 5 feet 4 inches (BMI is 45.5 kg/m2). Which of the following recommendations concerning weight gain during this pregnancy is most appropriate? A. Maintain current weight B. Gain 11 - 20 pounds C. Gain 15 - 25 pounds D. Gain 25 - 35 pounds E. Gain 28 - 40 pounds

B The Institute of Medicine (IOM) has developed guidelines (2009) on weight gain in pregnancy. Historical data show that women who gained within the IOM guidelines experienced better outcomes of pregnancy than those who did not. The recommendations are: underweight (BMI 30 kg/m2) total weight gain 11 - 20 pounds.

A 21-year-old G0 presents with severe menstrual pain. She takes 600 mg of ibuprofen every four to six hours to control the pain, but this does not relieve the discomfort. She is sexually active with one present partner and has four lifetime partners. She uses condoms for contraception. Past medical history is unremarkable, except for breast cancer in her father's sister. Examination is normal. In addition to a Pap smear, what is the most appropriate additional test needed for this patient? A. Baseline mammography B. Chlamydia testing C. HIV testing D. HPV DNA typing E. Lipid profile

B The US Preventive Services Task Force recommends chlamydia and gonorrhea screening for all sexually active patients, age 25 and younger. Since pelvic inflammatory disease is a cause of secondary dysmenorrhea, it needs to be evaluated as a potential cause of her symptoms. Although HPV screening is common, it can be used as an adjunct to cytology in primary screening in women 30 years or older, and is not indicated in a 21-year-old unless her Pap smear shows ASCUS. Lipid screening is not necessary in a 21-year-old otherwise healthy woman with no risk factors.

A 41-year-old G3P2 woman presents with cramping, vaginal bleeding and right lower quadrant pain for five days which has progressively worsened. Her last normal menstrual period occurred seven weeks ago. Her surgical history is notable for a bilateral tubal ligation following her last delivery. On physical exam, vital signs are: blood pressure 110/74; pulse 82; respirations 18; temperature 98.6°F (37.0°C). On abdominal exam, she has right lower quadrant tenderness, with rebound and bilateral guarding in the lower quadrants. On pelvic exam, she has scant old blood in the vagina and a normal appearing cervix. Her uterus is normal size and slightly tender. She has cervical motion tenderness on bimanual examination, and marked tenderness on rectal examination. Her quantitative Beta-hCG is 4000 mIU/ml; progesterone 6.2 ng/ml; hematocrit 34%; and WBC 15,400/mcL, with 88% segmented neutrophils and no bands. The transvaginal ultrasound shows an empty uterus with endometrial thickening, a mass in right ovary measuring 3.8 x 2 cm, and a small amount of free fluid in the pelvis. What is the most likely diagnosis in this patient? A. Pelvic inflammatory disease B. Ectopic pregnancy C. Heterotopic pregnancy D. Missed abortion E. Ruptured corpus luteum cyst

B The diagnosis of ectopic pregnancy is made when either: 1) a fetal pole is visualized outside the uterus on ultrasound; 2) the patient has a Beta-hCG level over the discriminatory zone (the level at which an intrauterine pregnancy should be seen on ultrasound, usually 2000 mIU/ml) and there is no intrauterine pregnancy (IUP) seen on ultrasound; or 3) the patient has inappropriately rising Beta-hCG level (less than 50% increase in 48 hours) and has levels which do not fall following diagnostic dilation and curettage. This patient meets criteria #2, as her Beta-hCG is >2000 mIU/ml with no intrauterine pregnancy seen on ultrasound. The history, physical exam and lab data are not consistent with pelvic inflammatory disease, ovarian torsion, appendicitis or a ruptured corpus luteum cyst. With a heterotopic pregnancy, there should be a visible pregnancy in the uterus. With a missed abortion there should also be some visible tissue or a fetal pole within the uterus.

A 35-year-old African-American G0 woman has a family history of ovarian cancer. Her mother was diagnosed with ovarian cancer at age 50 and is in remission. The patient had onset of menarche at age 14. She has used oral contraceptives for a total of 10 years. She smokes one to two packs of cigarettes per week. The patient had a LEEP for treatment of cervical dysplasia. Which of the following places the patient at greatest risk for developing ovarian cancer? A. African American race B. Family history of ovarian cancer C. Gravidity D. Late age at menarche E. Smoking

B The events leading to the development of ovarian cancer are unknown. Epidemiologic studies, however, have identified endocrine, environmental and genetic factors as important in the carcinogenesis of ovarian cancer. The established risk factors include nulliparity, family history, early menarche and late menopause, white race, increasing age and residence in North America and Northern Europe. Smoking has not been demonstrated to be associated with an increased risk of ovarian cancer.

A 54-year-old woman presents with a breast mass she noticed two months ago. She has no family history of breast cancer. On exam, there is a 2 cm mass palpable in the upper outer quadrant of the left breast. There are no other masses noted and no palpable lymphadenopathy. A fine needle aspiration returns bloody fluid and reduces the size of the mass to 1 cm. In addition to obtaining a mammogram, what is the most appropriate next step in the management of this patient? A. Repeat exam in two months B. Excisional biopsy of the mass C. Obtain a breast MRI D. Perform a lumpectomy and lymph node dissection E. Follow-up in one year if mammogram is normal

B The first noticeable symptom of breast cancer is typically a lump that feels different from the rest of the breast tissue. More breast cancer cases are discovered when the woman feels a lump. Breast cancer can also present with a spontaneous bloody nipple discharge. Even though the mass decreased in size after aspiration, the bloody discharge obtained obligates an excisional biopsy be performed to rule out breast cancer. If clear discharge is obtained on aspiration and the mass resolves, reexamination in two months is appropriate to check that the cyst has not recurred. An MRI is not the appropriate next step and lumpectomy with lymph node dissection is not yet indicated in this case. A normal mammogram does not rule out breast cancer, especially in the presence of bloody discharge.

A feature common for the Mayer-Rokitansky-Kustner-Hauser syndrome and complete androgen insensitivity syndrome is: A. XY karyotype. B. XX karyotype. C. lack of the uterus and vagina. D. defect of the androgen receptor gene. E. presence of testicles in the abdominal cavity

C. lack of the uterus and vagina.

A 37-year-old G3P1 woman presents with elevated blood pressure and 2+ proteinuria. She is 37 weeks gestation and the estimated fetal weight is 2500 grams. The patient is diagnosed with preeclampsia and is treated with MgS04. Magnesium level is 7.2 mEq/L. Maternal labs show: blood type B+; RPR non-reactive; HBsAg negative; HIV negative; and GBS negative. She is currently pushing during the second stage of labor and the delivery of the infant is imminent. What is the most likely complication to be encountered in this infant? A. Meconium aspiration syndrome B. Respiratory distress C. Hypertension D. Hypoglycemia E. Sepsis

B The first objective after delivery is to assess respiratory effort due to use of magnesium and make sure that the neonate is being oxygenated adequately, which might require a bag mask. Hypoglycemia would not be the first concern in this case and is usually associated with poorly controlled maternal diabetes and infection. Magnesium and maternal hypertension are not risk factors for neonatal hypertension.

A 61-year-old G3P3 woman is diagnosed with stage IIIA papillary serous adenocarcinoma of the ovary. She is concerned about her long-term prognosis. Which of the following factors would be most helpful in determining this patient's prognosis? A. Volume of residual disease B. Tumor stage C. Presence of non-malignant ascites D. Tumor grade E. Ovarian tumor diameter

B The five-year survival of patients with epithelial ovarian cancer is directly correlated with the tumor stage. The volume of residual disease following cytoreductive surgery is also directly correlated with survival. Patients who have been optimally debulked (generally <2 cm or <1 cm maximal residual tumor diameter) have a significant improvement in there median survival. Histologic grade of tumor is important. Women with poorly differentiated tumors or clear-cell carcinomas typically have a worse survival than those with well to moderately differentiated tumors. This is especially important in early-stage disease. Tumor size, bilaterality and ascites without cytologically positive cells, are not considered to be of prognostic importance.

A 39-year-old G1P1 comes to see you because of increased bleeding due to her known uterine fibroids, especially during her menses. She reports that her bleeding is so heavy that she has to miss two days of work every month. She has been using oral contraceptives and NSAIDs. Her most recent hematocrit was 27%. She is undecided about having more children. You discuss with her short and long-term options to decrease her bleeding. What is the next best step in the management of this patient? A. Blood transfusion B. Gonadotropin-releasing hormone agonists C. Endometrial ablation D. Uterine artery embolization E. Hysterectomy

B The goals of medical therapy are to temporarily reduce symptoms and to reduce myoma size. The therapy of choice is treatment with a GnRH agonist. The mean uterine size decreases 30-64% after three to six months of GnRH agonist treatment. Even though she is anemic, she is asymptomatic and able to work so a blood transfusion will not be indicated. Although uterine artery embolization and endometrial ablation effectively reduce bleeding, pain and fibroid size, they are contraindicated in a patient who desires future fertility. The failure rate is about 10-15%. A hysterectomy would obviously take care of her bleeding but would not be performed if she desires future fertility.

A 30-year-old G2P1 woman with last menstrual period 10 weeks ago presents for her first prenatal care visit. She is healthy and takes no medications. Her previous pregnancy was an uncomplicated vaginal delivery at 39 weeks. On examination, her vital signs are normal. Her exam is notable for a uterus measuring 14 weeks gestation. Ultrasound shows a diamniotic monochorionic twin gestation at 10 weeks. Which of the following obstetrical complications is more likely in this pregnancy compared to her previous pregnancy? A. Low maternal weight gain B. Congenital anomalies C. Induction after 40 weeks D. Macrosomia E. Rh isoimmunization

B The incidence of congenital anomalies is increased in twins, particularly monozygotic twins, compared to singletons. The majority of twin pairs in which an anomaly is present will be discordant for the anomaly. Twin gestations tend to deliver earlier than singleton gestations, with the average length of twin gestation being 35-37 weeks. The optimal length of twin gestation is a matter of some controversy. An observational study comparing perinatal mortality among twin and singleton gestations showed that perinatal mortality reached a nadir at 37-38 weeks in twins and then increased. There have been no prospective studies to demonstrate that induction of labor after 38 weeks in twin gestations improves perinatal outcome. Twins typically weigh less than singletons of the same gestational age, but their weights usually remain within the normal range. Macrosomia is, therefore, uncommon.

A 22-year-old G1P1 with no prenatal care delivered a 2100 g male infant who looks like an old man. The pediatricians estimate the gestational age to be greater than 43 weeks. Of the following, which finding is not associated with the dysmature postdate infant? A. Peeling skin B. Smooth skin C. Meconium-stained D. Long nails E. Fragile

B The incidence of infants with dysmaturity approaches 10% when the gestational age exceeds 43 weeks. Infants are described as withered, meconium stained, long-nailed, fragile and have an associated small placenta. These infants are at great risk for stillbirth.

A 24-year-old G4P2 woman at 34 weeks gestation complains of a cough and whitish sputum for the last three days. She reports that everyone in the family has been sick. She reports a high fever last night up to 102°F (38.9°C). She denies chest pain. She smokes a half-pack of cigarettes per day. She has a history of asthma with no previous intubations. She uses an albuterol inhaler, although she has not used it this week. Vital signs are: temperature 98.6°F (37°C); respiratory rate 16; pulse 94; blood pressure 114/78; peak expiratory flow rate 430 L/min (baseline documented in the outpatient chart = 425 L/min). On physical examination, pharyngeal mucosa is erythematous and injected. Lungs are clear to auscultation. White blood cell count 8,700; arterial blood gases on room air (normal ranges in parentheses): pH 7.44 (7.36 - 7.44); PO2 103 mm Hg (>100), PCO2 26 mm Hg (28 - 32), HCO3 19 mm Hg (22 - 26). Chest x-ray is normal. What is the correct interpretation of this arterial blood gas? A. Acute metabolic acidosis B. Compensated respiratory alkalosis C. Compensated metabolic alkalosis D. Hypoventilation E. Hyperventilation

B The increased minute ventilation during pregnancy causes a compensated respiratory alkalosis. Hypoventilation results in increased PCO2 and the PO2 would be decreased if she was hypoxic. A metabolic acidosis would have a decreased pH and a low HCO3. The patient's symptoms are most consistent with a viral upper respiratory infection.

A 31-year-old infertile woman has menses every 45-120 days and reports excessive facial hair and acne. She has a normal hysterosalpingogram and her male partner has a normal semen analysis. The medication that is most likely to lead to ovulation and pregnancy is: A) letrozole (Femara) B) metformin hydrochloride (Glucophage) C) clomiphene citrate (Clomid, Serophene) D) dexamethasone E) bromocriptine mesylate (Parlodel)

C) clomiphene citrate (Clomid, Serophene)

A 50-year-old G2P2 has a history of menorrhagia, pelvic pain, dyspareunia, dysmenorrhea, constipation and occasional spotting in between periods. She has a three-year history of urinary urgency and frequency. The patient is concerned that she has fibroids, as her close friend was recently diagnosed with fibroids. What is the symptom most commonly associated with leiomyomas? A. Intermenstrual spotting B. Menorrhagia C. Dyspareunia D. Dysmenorrhea E. Urinary symptoms

B The major symptom associated with myomas is menorrhagia, thought to be secondary to: 1) an increase in the uterine cavity size that leads to greater surface area for endometrial sloughing; and/or 2) an obstructive effect on uterine vasculature that leads to endometrial venule ectasia and proximal congestion in the myometrium/endometrium resulting in hypermenorrhea. Other relatively frequent symptoms include pain and pressure symptoms related to the size of the tumors filling the pelvic cavity, as well as causing pressure against the bladder, bowel and pelvic floor.

A 50-year-old G3P3 complains of menorrhagia. Physical examination is notable for a 14-week size irregularly shaped uterus. Her hematocrit is 35%. Which of the following is the next most appropriate step in this patient's management? A. Hysteroscopy B. Endometrial sampling C. Treatment with GnRH analogue D. Hysterectomy E. Myomectomy

B The majority of patients with uterine fibroids do not require surgical treatment. If patients present with menstrual abnormalities, the endometrial cavity may be sampled to rule out endometrial hyperplasia or cancer. This is most important in patients in their late reproductive years or postmenopausal years. If the patient's bleeding is not heavy enough to cause iron deficiency anemia, reassurance and observation may be all that are necessary. Treatment with GnRH analogues to inhibit estrogen secretion may be used as a temporizing measure. This is helpful in premenopausal women who are likely to be anovulatory with relatively more endogenous estrogen. Treatment with GnRH analogues can be used for three to six months prior to a hysterectomy to decrease the uterine size and increase a patient's hematocrit. This may also lead to technically easier surgery and decreased intraoperative blood loss. Treatment with GnRH analogue can also be used in perimenopausal women as a temporary medical therapy until natural menopause occurs. Myomectomy may be an appropriate treatment for a younger patient who desires future fertility. Hysteroscopy is not indicated at this point prior to endometrial sampling. Hysterectomy is a definitive treatment for women who have completed childbearing. Particularly in a perimenopausal woman, it is important to first rule out an underlying endometrial malignancy with endometrial sampling.

A 10-year-old girl goes to the doctor's office for her regular check-up. She is healthy, active in school sports and gets good grades. On examination she is 5 feet tall and weighs 80 pounds. She has Tanner stage II breasts and no pubic hair. She asks you when she might have her first menstrual period. You explain to her that appropriate sexual development occurs in which of the following orders? A. Thelarche-Menarche-Adrenarche-Growth spurt B. Thelarche-Adrenarche-Growth spurt-Menarche C. Thelarche-Growth spurt-Menarche-Adrenarche D. Thelarche-Growth spurt-Adrenarche-Menarche

B The normal and predictable sequence of sexual maturation proceeds with breast budding, then adrenarche (hair growth), a growth spurt and then menarche. Breast development begins around the age of 10 and average age of menarche is 12.7 years for Caucasian girls and 12.1 for Black girls. Menarche also occurs earlier for heavier girls and later for thinner, physically active girls.

A 19-year-old G1P0 woman notes vaginal spotting. Her last normal menstrual period occurred six weeks ago. She began having spotting early this morning and it has increased only slightly. She has no pain and denies other symptoms. Her medical history is noncontributory. Vital signs are: blood pressure 120/68; pulse 68; respirations 20; and temperature 98.6°F (37.0°C). On pelvic exam, her cervix is normal; uterus is small and nontender; and no masses are palpable. Initial labs show quantitative Beta-hCG 2000 mIU/ml and hematocrit 38%. A repeat Beta-hCG level 48 hours later is 2100 mIU/ml. A transvaginal ultrasound shows an empty uterus with a thin endometrial stripe and no adnexal masses. What is the next best step in the management of this patient? A. Dilation and curettage B. Treat with methotrexate C. Exploratory laparotomy D. Repeat Beta-hCG level in 48 hours E. Repeat ultrasound in 24 hours

B The patient clearly has an abnormal pregnancy, as demonstrated by the slowly increasng Beta-hCG levels. Since the Beta-hCG level is above 2000 mIU/ml, and she has a thin endometrial stripe, this rules out an intrauterine pregnancy and the diagnosis is an ectopic pregnancy. She is a good candidate for medical treatment with methotrexate. Criteria to consider for medical treatment include hemodynamic stability, non-ruptured ectopic pregnancy, size of ectopic mass <4 cm without a fetal heart rate or <3.5 cm in the presence of a fetal heart rate, normal liver enzymes and renal function, normal white cell count, the ability of the patient to follow up rapidly if her condition changes (reliable transportation, etc.). Dilation and curettage and exploratory laparotomy are invasive procedures that can be avoided in this patient. She does not need another Beta-hCG level because the diagnosis is clear. There is no indication for a repeat ultrasound in this case.

A 60-year-old G2P2 woman presents with complaints of urinary frequency and urge incontinence. Past medical history is unremarkable. She is on no medications. Pelvic exam shows no evidence of pelvic relaxation. Post void residual is normal. Urine analysis is negative. A cystometrogram revealed uninhibited detrusor contractions upon filling. Which of the following is the best treatment for this patient? A. Amitriptyline B. Oxybutynin C. Topical (vaginal) estrogen D. Pseudoephedrine E. Kegel exercises

B The patient has the diagnosis of detrusor instability. The parasympathetic system is involved in bladder emptying and acetylcholine is the transmitter that stimulates the bladder to contract through muscarinic receptors. Thus, anticholinergics are the mainstay of pharmacologic treatment. Oxybutynin is one example. Although the tricyclic antidepressant, amitriptyline, has anticholinergic properties, its side effects do not make it an ideal choice. Vaginal estrogen has been shown to help with urgency, but not urge incontinence. Pseudoephedrine has been shown to have alpha-adrenergic properties and may improve urethral tone in the treatment of stress incontinence. Kegel exercises or pelvic muscle training are used to strengthen the pelvic floor and decrease urethral hypermobility for the treatment of stress urinary incontinence.

The diagnosis of endometriosis should be considered in a patient with the following symptoms: 1) periodic pain, beginning just before the menstrual period lasting throughout the whole time of menstruation; 2) the temperature during the menstrual period > 39°C; 3) dyspareunia (pain related to sexual intercourse); 4) pain during defecation or micturition; 5) a stinking discharge from the vagina, occurring cyclically after menstruation. The correct answer is: A. 2,3,4. B. 1,3,4. C. 1,3,4,5. D. 2,5. E. all the above.

B. 1,3,4.

A 34-year-old G2P1 woman at 40 weeks gestation, with a history of one prior vaginal delivery, strongly desires an induction of labor, as she is unable to sleep secondary to severe back pain. Her cervical exam is closed, 20% effaced and -2 station. The cervix is firm and posterior. Which of the following is the most appropriate next step in the management of this patient? A. Wait until 42 weeks for induction B. Administer cytotec C. Insert a foley bulb in the cervix D. Perform artificial rupture of membranes E. Perform a Cesarean delivery

B The patient is multiparous at term and waiting until she reaches 42 weeks may increase the risk of perinatal mortality. Since she is uncomfortable with back pain, it is reasonable to induce labor. Her cervix is unfavorable; therefore, cytotec administration is appropriate prior to pitocin induction. A foley bulb or artificial rupture of membranes cannot be achieved in a patient with a closed cervix. At this time, there are no indications to perform a Cesarean delivery in this patient.

A 57-year-old G2P2 is seen for a routine visit. She states she and her 75-year-old husband stopped having sexual intercourse three years ago when he had an operation for prostate cancer. Menopause occurred at age 50 and she denies taking hormones. Her husband now wishes to resume intercourse and is able to get an erection with sildenafil (Viagra). Attempts at intercourse have been unsuccessful due to the pain she experiences when insertion is attempted. Examination is normal except for a narrowed vagina with atrophic mucosa. Which of the following is the most appropriate recommendation at this time? A. Progesterone cream B. Estrogen cream C. Oral combined hormone therapy D. Petroleum jelly E. Vaginal dilators

B The patient is not on hormone replacement therapy. With decreased estrogen production after menopause, the vaginal mucosa and other estrogen-dependent tissues can become atrophic. Topical estrogen therapy can restore the integrity of the vaginal epithelium, as well as the support tissues around the vagina. Long-term use may require addition of a progestin due to potential systemic absorption and effect on the endometrium. Dyspareunia (pain with intercourse) can often be improved with the use of estrogen cream used vaginally. Water based lubricants may be helpful, but petroleum jelly is contraindicated because it can cause irritation of the vaginal mucosa. This patient does not have any other menopausal symptoms that would be treated by oral combination HRT. Oral progesterone may help to decrease hot flushes, but progesterone cream would not alleviate vaginal atrophy.

A 28-year-old patient presents to the office for a health maintenance examination. She is currently on oral contraceptive pills, but reports a history of irregular menses prior to starting them. Her past medical history is otherwise non-contributory. On physical examination, she is 5 feet 2 inches tall and weighs 180 pounds. She has an area of velvety, hyperpigmented skin on the back of her neck and under her arms. What is the next best step in the management of this patient? A. Complete blood count B. Diabetes screen C. Biopsy pigmented area D. Pelvic ultrasound E. Lipid profile

B The patient is obese, with a BMI of 33. In addition, the skin changes are consistent with acanthosis nigricans, which is closely associated with insulin resistance. Given these risk factors, she should be tested for diabetes.

A 32-year-old G1P0 woman comes to your office for her first prenatal care visit. She has recently read an article about the rising Cesarean section rate in the United States and asks you about the rate in your hospital. What do you explain as the major cause of higher Cesarean delivery rates? A. The rate of breech presentations has increased B. Less women are having vaginal births after Cesarean C. Obstetricians' reluctance to perform forceps delivery D. Increased rate of fetal macrosomia due to uncontrolled gestational diabetes E. Rate of twins has increased

B The rate of vaginal birth after Cesarean (VBAC) has decreased in recent years due to studies that showed an increased risk of complications, especially uterine rupture. This is one factor that has led to the increased Cesarean section rate. In addition, although the rate of breech presentation is stable, there are significantly fewer obstetricians who are willing to perform vaginal breech deliveries. Many obstetricians do not perform instrumental vaginal deliveries, such as forceps and vacuum extractions, further contributing to the rising rate. Gestational diabetes is a well-known pregnancy complication with clear clinical guidelines.

A 30-year-old G3P1 woman undergoes an uncomplicated dilation and curettage for a first-trimester miscarriage. Pathology reveals a complete mole. The patient's medical history is significant for chronic hypertension. She has a history of a previous uncomplicated term pregnancy and a termination of a pregnancy at 16 weeks gestation with trisomy 18. What is the risk of her developing a recurrent molar pregnancy? A. No increase risk over general population B. Less than 2% C. 5% D. 10% E. More than 20%

B The recurrent risk for molar pregnancies ranges from 1 to 2%, which is a 20-fold increase from background risk. The risk of recurrence after two molar pregnancies is 10%.

A 28-year-old G1P0 woman has recovered from suction curettage for a complete molar pregnancy. She understands that her Beta-HCG will be monitored closely, but is eager to have a baby. Which of the following most accurately describes this patient's chances of successfully conceiving and delivering a viable infant? A. Low, because of the high risk of developing another mole B. High, because of the low risk of developing another mole C. Low, because of the poor success rates in curing gestational trophoblastic disease D. High, because of the effective use of chemotherapy in treating molar pregnancies E. Low, because molar pregnancies are seen more frequently in women who have difficulty conceiving

B The risk of developing another molar pregnancy is approximately 1-2% (higher than compared to women who have never had a molar pregnancy). Therefore, full-term pregnancy is considered very plausible, even in women with repeated molar pregnancies. GTD is considered the most curable gynecologic malignancy and preservation of fertility allows for successful future pregnancy. Although chemotherapy is very effective in treating the higher risk GTD diagnoses (choriocarcinoma, invasive moles, post-molar GTD), it is not indicated for treatment of molar pregnancy. Even with chemotherapy treatment, successful pregnancy can ensue. There is no known association between molar pregnancy and infertility.

A 35-year-old G3P0 woman presents at 11 weeks gestation for chorionic villus sampling (CVS). She has had two prior first trimester losses. What is the risk of miscarriage associated with CVS in this patient? A. 0.1% B. 1% C. 5% D. 10% E. 15%

B The risk of fetal loss associated with CVS is approximately 1% and is not related to her prior miscarriage history.

Neoplastic ovarian tumours which produce steroid hormones include: 1) thecoma; 2) mature teratoma; 3) endometrioid carcinoma; 4) granulosa cell tumour. The correct answer is: A. 1,2. B. 1,4. C. 2,3. D. 2,4. E. 3,4.

B. 1,4.

A 29-year-old G0 woman presents to your office for a routine visit. She has been trying to conceive for the last six months unsuccessfully. She requests fertility medications and hopes to get pregnant with twins. What counseling do you tell her regarding the risks of multifetal gestation? A. The morbidity with twin gestations is similar to triplet pregnancies B. The twin infant death rate is five times higher than that of singletons C. The rate of cerebral palsy is double in twin infants D. The incidence of abnormal fetal growth is similar to singleton pregnancies E. The incidence of prematurity is similar to singleton pregnancies

B The twin infant death rate is five times higher than that of singletons. The epidemic of multiple gestations resulting from assisted reproductive techniques is of great significance to individual parturients and to society because of the major morbidities associated with twinning as well as with triplets and higher order multiples. The risk for development of cerebral palsy in twin infants is five to six times higher than that of singletons. One study, with dichorionic twins, monochorionic twins and singletons, showed that twins had a higher incidence of IUGR (intrauterine growth restriction) than singletons. Fifty-eight percent of twins deliver prematurely, with an average gestational age at delivery of 35 weeks. Twelve percent of twins deliver very prematurely.

An 18-year-old G1P0 woman is seen in the clinic for a routine prenatal visit at 28 weeks gestation. Her prenatal course has been unremarkable. She has not been taking prenatal vitamins. Her pre-pregnancy weight was 120 pounds. Initial hemoglobin at the first visit at eight weeks gestation was 12.3 g/dL. Current weight is 138 pounds. After performing a screening complete blood count (CBC), the results are notable for a white blood count 9,700/mL, hemoglobin 10.6 g/dL, mean corpuscular volume 88.2 fL (80.8 - 96.4) and platelets 215,000/mcL. The patient denies vaginal or rectal bleeding. Which of the following is the best explanation for this patient's anemia? A. Folate deficiency B. Relative hemodilution of pregnancy C. Iron deficiency D. Beta thalassemia trait E. Alpha thalassemia trait

B There is normally a 36% increase in maternal blood volume; the maximum is reached around 34 weeks. The plasma volume increases 47% and the RBC mass increases only 17%. This relative dilutional effect lowers the hemoglobin, but causes no change in the MCV. Folate deficiency results in a macrocytic anemia. Iron deficiency and thalassemias are associated with microcytic anemia.

A 44-year-old G2P2 presents with six months of intermittent vulvar itching. She denies any bleeding, but does have a whitish discharge. She has not felt any obvious lumps or sores. She was diagnosed with lupus over 10 years ago and is on prednisone, mycophenolate mofetil (CellCept), and hydroxychloroquine (Plaquenil). Her periods are irregular and her Pap smears have been normal, but infrequently done (last one was two to three years ago). She reports treatment in the past for warts when she was first diagnosed with lupus, but denies any other sexually transmitted infections. Examination of the vulva is notable for diffuse, erythematous labia, with a thin white filmy discharge. There appears to be subtle but multi-focal, flat, whitish lesions measuring 0.5 - 1 cm on the labia bilaterally. Her remaining pelvic exam is unremarkable. A saline and potassium hydroxide wet prep is performed and is negative. What is the most appropriate next step in the management of this patient? A. Treatment with oral fluconazole and a topical imidazole B. Colposcopy and directed biopsies of the vulva C. Treatment of the lesions with topical trichloroacetic acid D. Wide local excision of the vulva E. Adjustment of her immunosuppressive therapies

B These lesions most likely represent an HPV-related condition such as condyloma or vulvar dysplasia. Women who are on immunosuppressive therapy are at higher risk of such conditions and require close surveillance. Although, her history of prior treatment for warts suggests these could be condyloma again, their flat, subtle appearance raises a concern that they may be dysplastic lesions, and further investigation with colposcopy and biopsy are indicated. Treatment for presumed yeast infection is reasonable, given her susceptibility, but would not be the sole treatment in this setting of new clinically evident lesions especially in light of a negative wet prep. Wide excision is not indicated at this time without a diagnosis.

A 24-year-old G3P0 woman at 26 weeks gestation was brought to the hospital by paramedics. Her husband found her shivering and barely responsive. Two days prior, the patient noted that she was feeling sick, with a slight cough. She was having back pain at the time, but thought it was probably normal for pregnancy. Her pregnancy has been uncomplicated except for the recent diagnosis of gestational diabetes. On exam, vital signs are: temperature 100.2°F (37.9°C); pulse 160; and blood pressure 68/32; respiratory rate 32. Oxygen saturation is 82% on room air. There is no apparent fundal tenderness, although the patient exhibits pain with percussion of the right back. Fetal heart tones are not audible. There is no evidence of vaginal bleeding. Extremities are cool to touch. White blood cell count 24,000; hemoglobin 9.5; hematocrit 27%. Urine microscopic analysis shows many white blood cells. What is the most likely etiology for this patient's disease? A. Abruptio placentae B. Pyelonephritis C, Diabetic ketoacidosis D. Chorioamnionitis E. Pneumonia

B This is a patient in septic shock. The most common cause of sepsis in pregnancy is acute pyelonephritis. Given the absence of bleeding, the clinical picture is not suggestive of placental abruption. Diabetic ketoacidosis is unusual in gestational diabetic patients. Chorioamnionitis and pneumonia may both lead to sepsis, but are less important causes than is pyelonephritis, and are not suggested by the clinical picture.

A 23-year-old nulliparous woman presents with a painful nodule in her axilla for three days. She is healthy and has no personal or family history of breast disease. On exam, no abnormalities are seen on inspection and no breast mass is palpated. In the axillary area, shaved skin is noted and an erythematous raised 1 cm lesion is palpated and is slightly tender to touch. What is the most likely diagnosis? A. Paget's disease B. Folliculitis C. Fibroadenoma D. Supernumerary nipple E. Duct obstruction

B This is a typical presentation for folliculitis which can occur with shaving the axillary hair. Paget's disease is a malignant condition of the breast that has the appearance of eczema and does not typically present in the axillary area. Fibroadenomas are common, but are usually firm, painless and freely movable. A supernumerary nipple is a congenital variation and is typically located in the nipple line and not tender. A clogged milk duct can be present in the axillary region, but it is typically present in a woman who is breastfeeding.

A 26-year-old G2P1 woman at 26 weeks gestation presents for a routine 50-gram glucose challenge test. After receiving a one-hour blood glucose value of 148 mg/dl, the patient has a follow up 100-gram three-hour oral glucose tolerance test with the following plasma values: Fasting 102 mg/dl (normal ≤95 mg/dl) 1-hour 181 mg/dl (normal ≤180 mg/dl) 2-hour 162 mg/dl (normal ≤155 mg/dl) 3-hour 139 mg/dl (normal ≤140 mg/dl) What is the most appropriate next step in the management of this patient? A. Repeat the glucose tolerance test at 28 weeks gestation B. Begin a diabetic diet and blood glucose monitoring C. Begin a diabetic diet, an oral hypoglycemic agent, and blood glucose monitoring D. Begin a diabetic diet, insulin, and blood glucose monitoring E. Reassurance and routine prenatal care

B This patient has three values on the three-hour glucose tolerance test that were abnormal. Initial management should include teaching the patient how to monitor her blood glucose levels at home on a schedule that would include a fasting blood sugar and one- or two-hour post-prandial values after all three meals, daily. Goals for blood sugar management would be to maintain blood sugars when fasting below 90 and one- and two-hour post-meal values below 120. A repeat glucose tolerance test would not add any value, as an abnormal test has already been documented. Oral hypoglycemic agents and insulin are not indicated at this time, as the patient may achieve adequate glucose levels with diet modification alone. Gestational diabetes varies in prevalence. The prevalence rate in the United States has varied from 1.4 to 14% in various studies. Risk factors for gestational diabetes include: a previous large baby (greater than 9 lb), a history of abnormal glucose tolerance, pre-pregnancy weight of 110% or more of ideal body weight, and member of an ethnic group with a higher than normal rate of type 2 diabetes, such as American Indian or Hispanic descent.

A 17-year-old G2P0 female has severe right lower quadrant pain. Her last normal menstrual period seven weeks ago. She notes that last night she began having suprapubic pain that radiated to her right lower quadrant. This morning, the pain awoke her from sleep. She has had no vaginal bleeding, no nausea or vomiting. The patient's history is notable for two first trimester elective abortions and a history of Chlamydia treated twice. Vital signs are: blood pressure 90/60; pulse 99; respirations 22; and temperature 98.6°F (37°C). On physical exam, the patient is noted to be curled on a stretcher in a fetal position and says she hurts too much to move. She has rebound and voluntary guarding on abdominal examination. She has profound cervical motion tenderness and rectal tenderness. Her Beta-hCG level is 2500 mIU/ml; hematocrit 24%; and urinalysis negative. Ultrasound shows no intrauterine pregnancy, a right adnexal mass that measures 6 x 2 cm, and a moderate amount of free fluid. Which of the following is the most appropriate next step in the management of this patient? A. Admit for serial examinations B. Exploratory surgery C. Recheck Beta-hCG level in 48 hours D. Administer methotrexate E. Dilation and curettage

B This patient has a ruptured ectopic pregnancy until proven otherwise. Her vital signs, examination and anemia are consistent with an intra-abdominal bleed. Exploratory laparoscopy/laparotomy is indicated at this point. Conservative management with observation, serial examinations or repeat Beta-hCG testing could be dangerous in a patient suspected of having a ruptured ectopic pregnancy. Medical management (methotrexate) is not used in a patient with an acute surgical abdomen. Dilation and curettage would not be the next step in management and might only be considered in this scenario after the patient's abdomen was explored.

A 29-year-old G3P0 woman presents for prenatal care at eight weeks gestation. Her two prior pregnancies ended in spontaneous losses at 19 and 18 weeks, respectively. Records corroborate the patient's history of an uncomplicated gestation until the evening of the losses, when she experienced a pink-tinged discharge that prompted her to call her obstetrician. In both cases, the obstetrician noted that her cervix had dilated completely with the amnionic sac bulging into the vagina to the level of the introitus. The patient was afebrile without other complaints and there was no uterine tenderness on exam. She spontaneously delivered the fetus and placenta in both cases. A sonohysterogram confirmed normal uterine anatomy several weeks later. What is the most appropriate next step in the management of this patient? A. Begin weekly fetal fibronectin testing B. Placement of a cervical cerclage at approximately 14 weeks gestation C. Immediate placement of a cervical cerclage D. Administer low dose aspirin and heparin E. Administration of prophylactic progesterone

B This patient has an incompetent cervix and should have a cervical cerclage at 14 weeks. A positive fetal fibronectin does not indicate incompetent cervix and is used later in pregnancy as a negative predictor of preterm delivery. Pregnancy loss in the late second trimester is not usually related to genetic abnormality of the conceptus and most clinicians delay placement of a cerclage until after the first trimester, given the high background prevalence of first trimester pregnancy wastage. Although some clinicians strongly support the existence of an antiphospholipid antibody syndrome, the term most commonly refers to pregnancy loss or demise, rather than the clinical scenario of silent cervical dilation with delivery described. The patient would, therefore, not need aspirin or heparin. Although some clinicians use prophylactic progesterone to prevent recurrent abortion, as well as preterm labor, no controlled trials support the use of prophylactic progesterone in the treatment of cervical incompetence.

A 19-year-old G0 woman presents with lower abdominal cramping. The pain started with her menses and has persisted, despite resolution of the bleeding. She thinks she may have a fever, but has not taken her temperature. No urinary frequency or dysuria are present. Her bowel habits are regular. She denies vomiting, but has mild nausea. A yellow blood-tinged vaginal discharge preceded her menses. No pruritus or odor was noted. She is sexually active, uses oral contraceptives and states that her partner does not like condoms. On examination: temperature is 100.2°F (37.9°C); pulse 90; blood pressure 110/60. She is well-developed and nourished and in mild distress. No flank pain is elicited. Her abdomen has normal bowel sounds, but is very tender with guarding in the lower quadrants. No rebound is present. Pelvic examination reveals a moderate amount of thick yellow discharge. The cervix is friable with yellow mucoid discharge at the os. Cervical motion tenderness is present. Uterus and the adnexa are tender without masses. Urine dip is negative for nitrates. Urine pregnancy test is negative. What is the most likely diagnosis? A. Vulvovaginal candidiasis B. Acute salpingitis C. Trichomonas vaginitis D. Cervicitis E. Bacterial vaginosis

B This patient has findings suggestive of acute salpingitis (pelvic inflammatory disease) including lower abdominal pain, adnexal tenderness, fever, cervical motion tenderness, and vaginal discharge. Mucopurulent cervicitis with exacerbation in the symptoms during and after menstruation is classically gonorrhea. Chlamydia is frequently associated with gonorrhea and also causes cervicitis and pelvic inflammatory disease. Cervicitis alone would not explain this patient's constellation of findings. Trichomonas may cause a yellow frothy discharge, and Candida may cause a thick white cottage cheese like discharge, but neither would cause fever and abdominal pain.

A 24-year-old G1P0 woman at 28 weeks gestation reports difficulty breathing, cough and frothy sputum. She was admitted for preterm labor 24 hours ago. She is a non-smoker. She has received 6 liters of Lactated Ringers solution since admission. She is receiving magnesium sulfate and nifedipine. Vital signs are: 100.2°F (37.9°C); respiratory rate 24; heart rate 110; blood pressure 132/85; pulse oximetry is 97% on a non-rebreather mask. She appears in distress. Lungs reveal bibasilar crackles. Uterine contractions are regular every three minutes. The fetal heart rate is 140 beats/minute. Labs show white blood cell count 17,500/mL with 94% segmented neutrophils. Potassium and sodium are normal. Which of the following has most likely contributed to this patient's respiratory symptoms? A. Increased plasma osmolality B. Use of tocolytics C. Chorioamnionitis D. Preterm labor E. Increased systemic vascular resistance

B This patient has pulmonary edema. Plasma osmolality is decreased during pregnancy which increases the susceptibility to pulmonary edema. Common causes of acute pulmonary edema in pregnancy include tocolytic use, cardiac disease, fluid overload and preeclampsia. Use of multiple tocolytics increases the susceptibility of pulmonary edema, especially with the use of isotonic fluids. Systemic vascular resistance is decreased during pregnancy. Women with chorioamnionitis are also more likely to develop pulmonary edema, but this is not usually the main cause unless the patient is in septic shock and this patient does not have chorioamnionitis.

A normal fetal heart rate is in the range of: A. 160-180 beats/minute. B. 110-160 beats/minute. C. 100-120 beats/minute. D. 80-100 beats/minute. E. 60-80 beats/minute.

B. 110-160 beats/minute.

"A 27-year-old G0 presents to the clinic because of concerns that she has not been able to get pregnant for the last year. She has been married for two years and was using oral contraceptives, which she stopped a year ago to start a family. She is in good health and her only medication is a prenatal vitamin. She was hospitalized at age 19 for a ""pelvic infection."" Her periods are regular, every 28 days with normal flow; her last period was two weeks ago. She has no history of abnormal Pap smears. Her husband is also healthy with no medical problems. She is 5 feet 4 inches tall and weighs 130 pounds. Her examination, including a pelvic exam, is completely normal. Which of the following is the most likely diagnostic test to find out the cause of her infertility? A. Hysteroscopy B. Hysterosalpingogram C. Progesterone level mid-cycle D. Clomiphene citrate challenge test E. Cervical mucous monitoring"

B This patient is having difficulty conceiving after trying for one year. Based on her history, the most likely underlying factor is tubal disease, as she has a history of being hospitalized for a pelvic infection, most likely pelvic inflammatory disease. This can cause adhesions and blockage of the tubes, which is best assessed with a hysterosalpingogram to evaluate the uterine cavity and tubes. After a single episode of salpingitis, 15% of patients experience infertility. Hysteroscopy will assess the uterine cavity and while sometimes used during a work up for infertility, it does not provide sufficient information about tubal patency. Progesterone levels, a Clomiphene challenge test or cervical mucous monitoring are used at times with infertility workups, but, in a young patient of normal BMI and with normal cycles, it is unlikely to find major ovulatory dysfunction.

A 21-year-old G1 woman at 36 weeks gestation presents with sudden onset of abdominal pain and bleeding. She smokes a pack of cigarettes a day, but otherwise her pregnancy has been uncomplicated. She takes no medications other than prenatal vitamins. Her blood pressure is 150/90, pulse 90 and she is afebrile. Her uterus is tense and very tender. Pelvic ultrasound shows the placenta to be posterior and fundal, with a cephalic presentation of the fetus. Cervical examination reveals no lesions, blood coming through the os and is one centimeter dilated. Fetal heart tones have a baseline of 150, with a category III fetal heart rate tracing. Tocometer reveals contractions every 30-45 seconds. Which of the following is the most appropriate next step in the management of this patient? A. Amniotomy B. Cesarean delivery C. Induction of labor D. Tocolysis E. A double set-up examination

B This patient is undergoing a placental abruption, with a deteriorating fetal condition. An emergent Cesarean delivery is necessary. The mother risks excessive blood loss, DIC and possible hysterectomy. The fetus risks neurological injury from anoxia or death. Risk factors for abruption include smoking, cocaine use, abdominal trauma, chronic hypertension, multiparity and prolonged premature rupture of membranes. Since immediate delivery is needed, amniotomy, induction, or tocolysis are not appropriate. A double set-up examination (performed in the operating room with a Cesarean section team scrubbed and ready) is not indicated, since the ultrasound determined the location of the placenta to be fundal.

A 36-year-old G0 presents due to increasing facial hair growth and irregular menstrual cycles. She has gained 40 pounds over the last three years. Her symptoms began three years ago and have gradually worsened. She has never been pregnant and is not currently on any medications. On physical exam, she is overweight with dark hair growth at the sideburns and upper lip. The pelvic exam is normal. Which of the following would you expect to find in this patient? A. Decreased luteinizing hormone levels B. Elevated free testosterone C. Decreased prolactin level D. Increased ovarian estrogen production E. Elevated 17-hydroxyprogesterone

B This patient likely has polycystic ovarian syndrome (PCOS). PCOS patients have testosterone levels at the upper limits of normal or slightly increased. Free testosterone (biologically active) is elevated often because sex hormone binding globulin is decreased by elevated androgens. LH is increased in response to increased circulating estrogens fed by an elevation of ovarian androgen production. Insulin resistance and chronic anovulation are hallmarks of PCOS. Prolactin levels may be elevated in amenorrhea but are not elevated in patients with PCOS.

A 32-year-old G0 comes to the office due to the inability to conceive for last two years. She reports having been on oral contraceptives for eight years prior. She had menarche at age 14 and has had irregular cycles about every three months until she started oral contraceptives, which regulated her cycles. In the last year, she has had about five cycles in total; her last menstrual period was six weeks ago. She is otherwise in good health and has not had any surgeries. She has no history of abnormal Pap smears or sexually transmitted infections. She is 5 feet 4 inches tall and weighs 165 pounds. On general appearance, she seems to be hirsute on the face and the abdomen. The rest of her exam is otherwise normal. Which of the following is most likely to help identify the underlying cause of this woman's infertility? A. Luteinizing hormone levels B. Testosterone levels C. Follicle stimulating hormone levels D. Thyroid function tests E. Progesterone levels

B This patient most likely has PCOS (polycystic ovarian syndrome) based on her history of irregular cycles, her body habitus and hirsutism. Having normal cycles on the birth control pills (OCPs) supports the diagnosis as other causes, such as hypothyroidism, will not normalize the cycles on OCPs. Testosterone levels will be helpful to confirm the diagnosis, especially in the presence of hirsutism. Once a diagnosis is established, progesterone levels are helpful during medical treatment to check if the woman is ovulating. An increased LH/FSH ratio is observed to be elevated in PCOS patients but each test separately will not aid in the diagnosis.

A 48-year-old G0 woman comes to the office for a health maintenance exam. She is healthy and not taking any medications. She has no history of abnormal Pap smears or sexually transmitted infections. Her menstrual cycles are normal and her last cycle was three weeks ago. Her mother was diagnosed with endometriosis and had a hysterectomy and removal of the ovaries at age 38. She is 5 feet 4 inches tall and weighs 130 pounds. On pelvic examination, the patient has a palpable left adnexal mass. An ultrasound was obtained, which showed a 4 cm complex left ovarian cyst and a 2 cm simple cyst on the right ovary. What is the most likely diagnosis in this patient? A. An endometrioma B. A hemorrhagic cyst C. Ovarian carcinoma D. A mature teratoma E. Polycystic ovaries

B This patient most likely has a hemorrhagic cyst, considering her history and where she is in her menstrual cycle. Her mother's history of endometriosis does increase her risk; however, it is unlikely since she has never had any symptoms herself. Ovarian carcinoma would need to be ruled out, but it is unlikely in an otherwise asymptomatic premenopausal patient. A mature teratoma would have more pathognomonic findings on ultrasound. This patient does not have typical symptoms, body habitus or ultrasound findings for patients with polycystic ovaries.

A 28-year-old G2P1 woman presents at 20 weeks gestation for a routine prenatal care visit. This pregnancy has been complicated by scant vaginal bleeding at seven weeks and an abnormal maternal serum alpha fetoprotein (MSAFP), with increased risk for Down syndrome, but had a normal amniocentesis: 46, XX. Her previous obstetric history is significant for a Cesarean delivery at 34 weeks due to placental abruption and fetal distress. Prenatal labs at six weeks showed blood type A negative, antibody screen positive: anti-D 1:64. Which of the following is the most likely cause of the Rh sensitization? A. ABO incompatibility B. Placental abruption C. Amniocentesis D. Abnormal maternal serum alpha fetoprotein (MSAFP) E. First trimester bleeding

B This patient was sensitized during her first pregnancy that was complicated by abruption and required Cesarean delivery. Transplacental hemorrhage of fetal Rh-positive red blood cells into the circulation of the Rh-negative mother may occur following a number of obstetric procedures and complications, such as amniocentesis, chorionic villus sampling, spontaneous/threatened abortion, ectopic pregnancy, dilation and evacuation, placental abruption, antepartum hemorrhage, preeclampsia, cesarean section, manual removal of the placenta and external version.

A 31-year-old G3P0 woman at 27 weeks gestation is being managed expectantly for severe preeclampsia remote from term. Her blood pressure is 155/100 on methyldopa (Aldomet) 500 mg three times a day. Her recent 24-hour urine had 6.6 grams of protein. An ultrasound revealed a fetus with adequate growth, having an estimated fetal weight in the 10th percentile. Her labs are normal, except for a uric acid of 8.0 mg/dL; hematocrit 42% (increased from 37%); and platelet count 97,000. Which of these findings necessitates delivery at this time? A. Elevated uric acid B. Thrombocytopenia C. Proteinuria D. Poorly controlled blood pressures E. Hemoconcentration

B Thrombocytopenia <100,000 is a contraindication to expectant management of severe preeclampsia remote from term (<32 weeks). Other contraindications include: inability to control blood pressure with maximum doses of two antihypertensive medications, non-reassuring fetal surveillance, liver function test elevated more than two times normal, eclampsia, persistent CNS (central nervous system) symptoms and oliguria. Delivery should not be based on the degree of proteinuria. Although elevated, uric acid and hemoconcentration are markers of preeclampsia, they are not part of the diagnostic or management criteria.

A 19-year-old G2P1 woman presents with vaginal spotting and uterine cramping. Her last normal menstrual period was six weeks ago and she began spotting three days ago. She has no history of sexually transmitted infections. Her vital signs are: blood pressure 120/70; pulse 78; respirations 20; and temperature 98.6 °F (37.0°C). On pelvic examination, she has no cervical motion tenderness, her uterus is normal size and nontender; no adnexal masses are palpable. Quantitative Beta-hCG 48 hours ago was 1500 mIU/ml; current beta-hCG is 3100 mIU/ml; progesterone 26 ng/ml; hematocrit 38%; and urinalysis is normal. What is the most likely finding on transvaginal ultrasound? A. Debris in uterus B. Viable intrauterine pregnancy C. Adnexal mass, empty uterus D. No adnexal mass, empty uterus E. Non-viable intrauterine pregnancy

B Transvaginal ultrasound will most likely show an intrauterine pregnancy. The Beta-hCG level is above the discriminatory zone for ultrasound (2000 mIU/ml), and the level has doubled in 48 hours. Additionally, the progesterone level is within expected range for a normal pregnancy (>25 ng/ml suggests healthy pregnancy) and up to 30% of all normal pregnancies experience first trimester spotting/bleeding. The findings of debris in the uterus, an empty uterus, with or without an adnexal mass, or free fluid (suggesting hemoperitoneum) would not be anticipated.

An 8-year-old has been diagnosed with precocious puberty due to presence of menarche, Tanner stage III breasts and otherwise normal work-up for brain, adrenal and ovarian abnormalities. What is the most appropriate next step in the management of this patient? A. Depo-Provera B. GnRH agonist C. Danazol D. Estradiol E. Observation

B True precocious puberty is manifested by premature secretion of GnRH hormone in a pulsatile manner. Once other causes of hormone production are ruled out, treatment would include GnRH agonist to suppress pituitary production of follicular-stimulating hormone and luteinizing hormone. Observation is acceptable if the precocious puberty is within a few months of the routinely expected puberty. The process should be treated if the bone age or puberty is advanced by several years.

A 35-year-old G3P3 woman comes to the office to discuss tubal ligation as she desires permanent sterilization. The non-contraceptive health benefits of female sterilization reduce the risk of which of the following? A. Breast cancer B. Ovarian cancer C. Endometrial cancer D. Cervical cancer E. Menorrhagia

B Tubal ligation has not been shown to reduce the risk of breast, cervical, or endometrial cancers, nor is there a decrease in menstrual blood flow in women who have undergone a tubal ligation. There is a slight reduction in the risk of ovarian cancer, but the mechanism is not yet fully understood.

A 35-year-old G1P0 woman with a known twin gestation undergoes an ultrasound evaluation at 18 weeks. She would like to know if her twins are identical or fraternal. Which ultrasound marker is suggestive of dizygotic (fraternal) twins? A. Increased amniotic fluid volume in one of the twins B. Two separate placentas (anterior and posterior) C. Dividing membranes less than 1 mm D. Concordant growth of the twins E. Twin-twin transfusion syndrome

B Ultrasound markers suggestive of dizygotic (non-identical) twins include a dividing membrane thickness greater than 2 mm, twin peak (lambda) sign, different fetal genders and two separate placentas (anterior and posterior). The two different placental types in twin gestation are monochorionic and dichorionic. Dizygotic conceptions always have dichorionic placentas. Monozygotic conceptions may have either monochorionic or dichorionic placentation, depending upon the time of division of the zygote. Diamniotic dichorionic placentation occurs with division prior to the morula state (within three days post fertilization). Diamniotic monochorionic placentation occurs with division between days four and eight post-fertilization. Monoamniotic, monochorionic placentation occurs with division between days eight and 12 post fertilization. Division at or after day 13 results in conjoined twins. The ultrasound markers for determination of chorionicity described above have been used to assess risk for complications of pregnancy, most notably the twin-twin transfusion syndrome.

The examiner places his hands flat and parallel on both sides of the abdomen of a pregnant woman, at the level of the umbilicus. The described maneuver allows to determine the fetal position and is called: A. 1 Leopold's maneuver. B. 2 Leopold's maneuver. C. 3 Leopold's maneuver. D. 4 Leopold's maneuver. E. 5 Leopold's maneuver (Zangemeister maneuver).

B. 2 Leopold's maneuver.

A 29-year-old G4P2 woman was diagnosed with twin-twin transfusion syndrome when an ultrasound was performed at 24 weeks gestational age. Which of the following is a complication of twin-twin transfusion syndrome? A. Fetal macrosomia in the donor twin B. Neurologic sequelae in the surviving twin C. Tricuspid regurgitation in the donor twin D. Heart failure in the donor twin only E. High perinatal mortality for donor twin only

B Untreated severe twin-twin transfusion syndrome has a poor prognosis, with perinatal mortality rates of 70-100%. Death in utero of either twin is common. Surviving infants have increased rates of neurological morbidity, with increased risk of cerebral palsy for the surviving twin. Excessive volume can lead to cardiomegaly, tricuspid regurgitation, ventricular hypertrophy and hydrops fetalis for the recipient twin. Although the recipient twin is plethoric, it is not macrosomic. The donor twin becomes anemic and hypovolemic, and growth is retarded. The recipient twin becomes plethoric, hypervolumic and macrosomic. Either twin can develop hydrops fetalis. The donor twin can become hydropic because of anemia and high-output heart failure.

A 76-year-old G3P3 woman presents to the office with worsening urinary incontinence for the last three months. She reports an increase in urinary frequency, urgency and nocturia. On examination, she has a moderate size cystocele and rectocele. A urine culture is negative. A post-void residual is 50 cc. A cystometrogram shows two bladder contractions while filling. Which of the following is the most likely diagnosis in this patient? A. Stress incontinence B. Urge incontinence C. Overflow incontinence D. Functional incontinence E. Mixed incontinence

B Urge incontinence is due to detrusor instability. Though the testing may be simple (using a Foley catheter and attached large syringe without the plunger, filling with 50-60 cc of water at a time) or complex (using computers and electronic catheters), the uninhibited contraction of the bladder with filling makes the diagnosis. Genuine stress incontinence (GSI) is the loss of urine due to increased abdominal pressure in the absence of a detrusor contraction. The majority of GSI is due to urethral hypermobility (straining Q-tip angle >30 degrees from horizon). Some (<10%) of GSI is due to intrinsic sphincteric deficiency (ISD) of the urethra. Overflow incontinence is associated with symptoms of pressure, fullness, and frequency, and is usually a small amount of continuous leaking. It is not associated with any positional changes or associated events. Mixed incontinence occurs when increased intra-abdominal pressure causes the urethral-vesical junction to descend causing the detrusor muscle to contract.

"A 24-year-old woman comes to her physician for help with her premenstrual syndrome symptoms. She complains of ""not being herself"" for three to four days before her period and has episodes of crying and irritability. She denies depressive symptoms and notes she is a stay-at-home mother for her three children. After a complete history and physical examination, the patient is prescribed a selective serotonin reuptake inhibitor but, after three months, she returns as there is no change in her symptoms. Upon further discussion, the patient admits that her husband has a bad temper at times. Physical examination is normal with the exception of some bruising on the patient's arms. She claims that she fell and that she is often clumsy. Which of the following is the most appropriate next step in the management of this patient? A. Report the injuries to the police B. Offer domestic violence resources to the patient C. Refer the patient for a psychiatric consultation D. Refer the patient to the emergency department for further evaluation E. Arrange to move her children to a home of a close friend or family member"

B Victims of domestic violence frequently present with vague physical and emotional complaints that are consistent with PMS. Victims will rarely volunteer information on first physician encounters, but subsequent visits give opportunities to offer resources for assistance. It is the patient's choice as to whether she would like to involve the police. Social work consultation, family counseling and relocation are all reasonable options that fall under the resources that may be discussed with the patient, but must be arranged at the patient's request and with her consent. Emergency department evaluation or physchiatric consultation are not indicated at this point.

A 42-year-old G2P2 woman presents with a two-week history of a thick, curdish white vaginal discharge and pruritus. She has not tried any over-the-counter medications. She is currently single and not sexually active. Her medical history is remarkable for recent antibiotic use for bronchitis. On pelvic examination, the external genitalia show marked erythema with satellite lesions. The vagina appears erythematous and edematous with a thick white discharge. The cervix appears normal and the remainder of the exam is unremarkable except for mild vaginal wall tenderness. Vaginal pH is 4.0. Saline wet prep reveals multiple white blood cells, but no clue cells or trichomonads. Potassium hydroxide prep shows the organisms. Which of the following is the most appropriate treatment for this patient? A. Clindamycin B. Azole cream C. Metronidazole D. Doxycycline E. Ciprofloxacin

B Vulvovaginal candidiasis (VVC) usually is caused by C. albicans, but is occasionally caused by other Candida species or yeasts. Typical symptoms include pruritus and vaginal discharge. Other symptoms include vaginal soreness, vulvar burning, dyspareunia and external dysuria. None of these symptoms are specific for VVC. The diagnosis is suggested clinically by vulvovaginal pruritus and erythema with or without associated vaginal discharge. The diagnosis can be made in a woman who has signs and symptoms of vaginitis when either: a) a wet preparation (saline or 10% KOH) or Gram stain of vaginal discharge demonstrates yeasts or pseudohyphae; or b) a vaginal culture or other test yields a positive result for a yeast species. Microscopy may be negative in up to fifty percent of confirmed cases. Treatment for uncomplicated VVC consists of short-course topical Azole formulations (1-3 days), which results in relief of symptoms and negative cultures in 80%-90% of patients who complete therapy.

A 23-year-old G1P0 patient at 24 weeks gestation requires treatment for depression. She has no other pregnancy complications. In addition to counseling, she begins therapy with fluoxetine (Prozac). Which of the following symptoms is the most common side effect of her therapy? A. Fatigue B. Sleep disturbance C. Headache D. Irritability E. Agitation

B While all the side effects listed are reported in patients on fluoxetine, an SSRI antidepressant medication, the most common side effect is insomnia. Significant insomnia may affect one in five patients taking SSRIs. In addition to sleep disturbances, sexual dysfunction, such as decreased libido and delayed or absent orgasm, are common.

Indicate the true sentences regarding ovarian reserve: 1) it is lower in Afro-Americans than in Caucasian women; 2) it is determined genetically; 3) it decreases after ovarian surgery; 4) it decreases as a consequence of ovarian disorders; 5) it increases with woman's age; 6) it decreases after antibiotic treatment. The correct answer is: A. 1,2,3. B. 2,3,4. C. 3,4,5. D. 4,5,6. E. 1,4,5.

B. 2,3,4.

A 35-year-old G3P2 woman is at 18 weeks gestation. Her obstetrical history is significant for two previous low transverse Cesarean deliveries. Her first one was performed secondary to arrest of dilation in the active phase at 7cm. She delivered a healthy 3500-gram infant. Her second Cesarean delivery was an elective repeat. She delivered a healthy 3400-gram infant. The patient strongly desires to attempt a VBAC (vaginal birth after cesarean). Which of the following statements is correct? A. The likelihood of a uterine rupture after two Cesarean sections is is approximately 10% B. The likelihood of a successful VBAC is lower in patients with two previous Cesarean deliveries than in women with one prior Cesarean delivery C. The likelihood of a successful VBAC is not affected by the indication of the previous Cesarean delivery D. The likelihood of a successful VBAC after two Cesarean sections is approximately 30%. E. She can safely undergo a prostaglandin induction of labor at term

B Women attempting a vaginal birth after Cesarean (VBAC) after one previous low transverse Cesarean delivery have a 70-80% chance of having a successful VBAC and approximately 70% with two previous cesarean sections. The risk of uterine rupture with a history of one previous low transverse Cesarean section is approximately 1 percent or less. There are no data to demonstrate the exact increased risk of uterine rupture with a history of two previous Cesarean deliveries. The indication for the previous Cesarean delivery may affect the success rate of a future VBAC. Patients who had a prior Cesarean delivery for a nonrecurring indication, such as placenta previa or breech presentation are more likely to have a successful VBAC compared to patients whose previous Cesarean delivery was performed secondary to cephalopelvic disproportion. Prostaglandin induction in this patient would is contraindicated.

A 3-year-old girl is taken to see her primary care physician by her father, secondary to vaginal discharge and swelling of the vulvar area for the past week. She has been scratching the area and says it hurts. She was on an antibiotic two weeks ago for an ear infection. The child has had normal growth and development. She has two male siblings, ages 8 and 6. Her father is a stay-at-home dad and her mother is an attorney. Examination reveals a red and swollen vulva with linear abrasions over the area. Which of the following is the most appropriate next step in treatment? A. Check a complete blood count B. Begin antifungal treatment C. Perform nasal speculum vaginal examination D. Refuse to treat until mother is present E. Notify police

B Yeast infections are common after antibiotic therapy. Scratching can appear like abuse is taking place. To examine a child, even with a nasal speculum, is traumatic. If a foreign body is suspected, an exam under anesthesia may be necessary, if ultrasound is not successful or indicated.

A 32-year-old G0 woman comes to the office due to the inability to conceive for the last two years. She reports having been on oral contraceptives for eight years prior. She had menarche at age 14 and had irregular cycles about every three months until she started birth control pills, which made her cycles regular. In the last year, she has had about five cycles in total; her last menstrual period was six weeks ago. She is otherwise in good health and has not had any surgeries. She has no history of abnormal Pap smears or sexually transmitted infections. Her husband of four years is 35 years old, and has a 10-year-old son from a previous marriage. She is 5 feet 4 inches tall and weighs 165 pounds. On general appearance, she is hirsute on the face and the abdomen. The rest of her exam is normal. In addition to recommending weight loss and starting metformin, what is the most appropriate treatment for this patient's infertility problem? A. Luteal phase progesterone B. Ovulation induction agents C. Intrauterine insemination D. In vitro fertilization E. Donor egg

B his patient has PCOS (polycystic ovarian syndrome) based on her history, signs and symptoms. It is most important for her to try to lose weight. Metformin and ovulation induction agents are the first-line of treatment for ovulatory dysfunction in PCOS patients. Since there is known ovulatory dysfunction and there is no reason to believe there are problems with semen analysis, IVF is not justified. There is no role for the laparoscopy in this patient. Although oral contraceptives would regulate her cycles, it is not indicated because she is trying to get pregnant.

In performing preconceptual counseling to young couple, you inform them that the most susceptible time period for teratogen exposure to the human embryo is at which embryonic week (week after conception)? A) 2 weeks B) 6 weeks C) 10 weeks D) 14 weeks E) 18 weeks

B) 6 weeks

During pregnancy, lactation is suppressed by the action of: A) Insulin B) Estrogen C) Thyroid hormone (T4) D) Human placental lactogen E) Inhibin

B) Estrogen

True statements concerning the diagnosis of an adnexal mass are the following, EXCEPT: A) The diagnosis varies with the age of the patient. B) In patients in the reproductive age period, a cystic mass larger than 5 cm should be explored immediately. C) In premenarchal patients, most neoplasms are germ cell in origin and require surgical exploration. D) In postmenopausal women, enlargement of the ovary is abnormal and should be considered malignant until proven otherwise. E) In patients in the reproductive age period, a solid mass larger than 8 cm should be explored.

B) In patients in the reproductive age period, a cystic mass larger than 5 cm should be explored immediately.

During the midluteal phase, progesterone secretion occurs immediately after pulsatile secretion of: A) Follicle stimulating hormone (FSH) B) Luteinizing hormone (LH) C) Estradiol D) Inhibin E) Activin

B) Luteinizing hormone (LH)

Engagement of the fetal head is defined by: A) The leading edge of the fetal head is at the ischial spines. B) The biparietal diameter is through the pelvic inlet. C) The occipitofrontal diameter is through the pelvic inlet. D) The lowest part of the head is at a plane between the ischial spines. E) The leading edge of the fetal head is at the vaginal introitus.

B) The biparietal diameter is through the pelvic inlet.

A 22-year-old woman comes to the emergency department complaining of sudden onset of severe cramping in the right lower quadrant. Her temperature is 37.4°C (99.4°F), pulse is 90 bpm, and blood pressure is 100/70 mm Hg. The abdomen is tender to palpation in the right lower quadrant, and peritoneal signs are present. Pelvic examination reveals an exquisitely tender 8-cm right adnexal mass. Urine pregnancy test is negative. She continues to complain of unbearable pain. The most likely diagnosis is: A) Appendicitis B) Torsion of ovary C) Ectopic pregnancy D) Rupture of corpus luteum E) Rupture of tuboovarian abscess

B) Torsion of ovary

Which of the following are risk factors for exercise-induced urinary incontinence? 1) young age; 2) BMI < 30 kg/m 3) chronic constipation 4) taking diuretics or anti-hypertensive medications; 5) chronic diseases with coughing. The correct answer is: A. 1,2. B. 3,4,5. C. 1,3,5. D. 2,4,5. E. all the above.

B. 3,4,5

A 38-year-old woman, gravida 4, para 2, is seen at 29 weeks of gestation for generalized intense itching for the past week. The itching is worst on her hands and feet. She does not report any infectious contacts or new skin care products. Her temperature is 37.0°C (98.6°F), blood pressure is 135/85 mm Hg, and pulse is 90 beats per minute. She appears in no acute distress. Her sclere are nonicteric and her oropharynx is normal. She has no rash. Her cervix is closed and long. The fetal heart rate is 135 beats per minute and reassuring. Urine dipstick test result is negative for protein. Laboratory evaluation reveals her white blood count is 9.0 x 109/L, hematocrit is 33%, and platelet count is 240,000/mm'. Metabolic panel shows the serum alanine transaminase level is 55 units/L and the total bilirubin level is 0.7 mg/dL. The most appropriate diagnostic test is: A) serum alkaline phosphatase B) serum ammonia C) serum bile acid levels D) serum glucose

B) serum ammonia

A 30-year-old patient with infertility has just completed a cycle of in vitro fertilization (IVF utilizing leuprolide acetate (Lupron) for prevention of a luteinizing hormone (LH) surge. A total of 20 oocytes were retrieved and a single blastocyst was transferred. Her husband travels frequently and is currently out of town for 2 weeks. You advise her that her best choice for luteal phase support may be: A) oral micronized progesterone B) vaginal progesterone C) human chorionic gonadotropin (hCG) D) intramuscular progesterone

B) vaginal progesterone

Adaptive changes in the circulatory system of a pregnant woman consist of: 1) increase in the circulating blood volume, including 45% of the plasma volume and 20% of the erythrocyte mass, with a greater increase of the circulating blood volume in obese women than in slim women; 2) increase in cardiac output; 3) acceleration of heart rate by 10-15 beats per minute; 4) increased peripheral arterial resistance by 20%; 5) increase in central venous pressure. The correct answer is: A. 1,5. B. 1,2,3. C. 1,3,4. D. only 1. E. 4,5.

B. 1,2,3.

The composition of the amniotic fluid includes the following proteins produced by the foetus: 1) transferrin; 2) alpha-fetoprotein; 3) beta-microglobulin; 4) IgG immunoglobulins. (receives from mother at 6 months of gestation) The correct answer is: A. 1,2,4. B. 1,2,3. C. 2,3,4. D. only 2. E. all the above.

B. 1,2,3.

The normal age of initial menstruation in Poland is defined as: A. 8-9 years. B. 9-16 years. C. 16-17 years. D. >18 years. E. no norm exists.

B. 9-16 years.

A 30-year-old nulligravid woman has been trying to conceive for the past year. She is unsure as to when she is ovulating and would like to know how she can best detect ovulation. The most reliable indicator of impending ovulation is an increase in: A) estradiol B) follicle-stimulating hormone C) luteinizing hormone D) progesterone E) basal body temperature

C) luteinizing hormone

Case where patient has proliferative endometrium and irregular menstrual cycles?

a. Anovulation

Indicate findings present in normal cardiotocographic non-stress test (NST) in pregnancy: 1) baseline fetal heart rate - 100/min; 2) baseline fetal heart rate - 150/min; 3) marked variability of the fetal heart rate; 4) absence of accelerations; 5) absence of decelerations. The correct answer is: A. 1,3. B. 2,4. C. 2,5. D. 2,3. E. 3,5.

C. 2,5.

Pregnant woman with HIV. What to do to protect them? a. Antiretroviral therapy for both mom and baby b. Antiretroviral therapy for mom c. Cesarean section

a. Antiretroviral therapy for both mom and baby

Case where patient had clue cells?

a. Bacterial vaginosis

Case where patient had pseudohyphae?

a. Candida

Labor begins with regular contractions of the uterus causing the cervix to shorten and open. Which of the following concerning the first stage of labor is true? 1) it is the time from full cervical dilation to the birth of a child; 2) it is the time of postpartum content expulsion; 3) it is the time from the beginning of labor to the full cervical dilation; 4) it is divided into two phases: latent - from the labor beginning to the opening around 3-4 cm and active - from the 3-4 cm of the dilation to the complete dilation; 5) maximal time of the first stage of labor is 2 hours. The correct answer is: A. only 1. B. only 2. C. 3,4. D. 3,4,5. E. none of the above.

C. 3,4

A 29-year-old woman, gravida 2, para 2, had her annual preventive health care examination 10 days ago. At the time of her visit, she reported no vaginal symptoms. You are now reviewing her Pap test report that states normal cytology and bacterial vaginosis. Your next step in management is: A) perform a wet mount B) treat with metronidazole C) treat with clindamycin hydrochloride D) manage her expectantly

D) manage her expectantly

A 32-year-old childless patient has a history of 4 miscarriages in the first trimester. The evaluation of the possible causes of miscarriages in this patient should include: 1) diagnostics of Toxoplasma gondii infection; 2) diagnostics of Human Papilloma Virus infection; 3) determination of anti-cardiolipin antibodies; 4) diagnostics of Cytomegalovirus infection; 5) determination of lupus anticoagulant. The correct answer is: A. 1,2. B. 2,3. C. 3,5. D. 2,4,5. E. 1,4,5.

C. 3,5

Complete androgen insensitivity syndrome is characterized by: A. female phenotype, no axillary hair, male external genitalia. B. male phenotype, no pubic hair, male external genitalia. C. female phenotype, pubic an axillary hair, female external genitalia. D. 46 XY karyotype with estrogen-secreting ovaries. E. female phenotype, no pubic and axillary hair, female external genitalia.

E. female phenotype, no pubic and axillary hair, female external genitalia.

A contraindication to sexual activity in pregnancy is not: A. multiple pregnancy. B. cervical incompetence. C. placenta previa. D. inflammatory urinary tract. E. fetal breech presentation.

E. fetal breech presentation.

Case where patient had urinary symptoms. Recently received nitrofurantoin, but this didn't help. Still symptoms. Leukocytes found in urine, but no bacteria. Culture is sent for analysis, but is not back yet? a. Candida b. Trichomonas

a. Candida

Azoospermia is defined as: A. absence of semen. B. absence of sperm in semen C. absence of sperm mobility. D. sperm with abnormal morphology. E. number of sperm in semen lower than 1 million/ml.

B. absence of sperm in semen

Indicate the true statement concerning vulvar cancer: A. most often occurs before the age of 40. B. accounts for 3-8% of all female genital malignancies. C. more than 90% of vulvar cancers are glandular cancers. D. the treatment of choice is a systemic treatment. E. five-year survival for women in FIGO stage I is 31%.

B. accounts for 3-8% of all female genital malignancies.

Which of the following is not an indication for cesarean section? A. Marfan syndrome with aorta dilatation and the coarctation of the aorta that has not been surgically corrected. B. anxiety of a pregnant woman before the delivery. C. significant difference of foetal weights of > 20% in twin pregnancy. D. foetal weight > 3500 g in the longitudinal foetus breech position. E. heart failure or significant arrhythmia in a pregnant woman.

B. anxiety of a pregnant woman before the delivery.

Assessment of the location of the fundal height is an important part of obstetrical examination. The fundus of the uterus at 24 week of gestation is normally located: A. 3 finger widths above the pubic symphysis. B. at the umbiculus. C. 3 finger widths above the umbiculs. D. 3 finger widths below the xiphoid process. E. under the costal margin.

B. at the umbiculus.

Fetal bradycardia is defined as a decrease of the basic fetal heart rate: A. below 100 beats/min. lasting at least 15 minutes. B. below 110 beats/min. lasting at least 10 minutes. C. below 110 beats/min. lasting at least 15 minutes. D. below 120 beats/min. lasting at least 10 minutes. E. below 120 beats/min. lasting at least 15 minutes.

B. below 110 beats/min. lasting at least 10 minutes.

Which of the following markers should be used in the diagnosis of molar pregnancy? A. Ca125. B. beta HCG. C. estradiol. D. inhibin A. E. HE4.

B. beta HCG.

Which of the following is introduced into the peritoneal cavity in order to produce pneumoperitoneum during the laparoscopic procedure? A. saline solution. B. carbon dioxide. C. oxygen. D. nitrogen. E. compressed air.

B. carbon dioxide.

Which of the following should not be used in the treatment of a urinary tract infection in pregnant or breastfeeding woman? A. nitrofurantoin. B. ciprofloxacin. C. cefuroxime. D. amoxicillin. E. ampicillin.

B. ciprofloxacin.

The best method to diagnose fetal hemolytic disease is: A. amniocentesis. B. cordocentesis. C. fetoscopy. D. placenta biopsy. E. biopsy of the fetal liver.

B. cordocentesis.

The secondary prophylaxis of cervical cancer consists in: A. pro-healthy style of life to avoid HPV infection. B. cytological screening. C. cervical conization in the case of CIN2+ cytological outcome. D. radical hysterectomy with lymphadenectomy in the case of invasive cancer. E. brachytherapy in the case of metastatic cancer.

B. cytological screening.

Indicate the true statement concerning ovarian cancer surgery: A. it strives for tumour sample obtaining and fast radical chemotherapy application. B. cytoreduction is performed to the highest possible extent. C. only and always the affected ovary is removed. D. following uterectomy the region of the lesser pelvis may be irradiated. E. neoadjuvant chemotherapy is always used in order to decrease a tumour mass.

B. cytoreduction is performed to the highest possible extent.

Which of the following changes in blood morphology occurs in the second trimester of normal pregnancy? A. increased hematocrit levels. B. decreased hemoglobin concentration. C. increased hemoglobin concentration. D. increased platelet count. E. increased leucocyte count.

B. decreased hemoglobin concentration.

In a pregnant (41 week of pregnancy) CTG test was done. It showed a basic heart rate 145/minute. The heart rate started to drop to 110/minute at the top of each contraction and it has been observed for the last 20 minutes. This record shows: A. utero-placental insufficiency and it is an alarming sign because it precedes intrauterine foetal death. B. foetal head compression and vagus nerve irritation and is not an adverse symptoms unless it correlates with extended basic changes in the heart rate. C. it is an indication for shortening the second stage of labour. D. threat of intrauterine asphyxia and the need to consider a caesarean section. E. none of the above.

B. foetal head compression and vagus nerve irritation and is not an adverse symptoms unless it correlates with extended basic changes in the heart rate.

Which of the following determinations is used to diagnose hyperandrogenism in a 25- year-old woman? A. ratio of LH to FSH. B. free testosterone or index of free androgens. C. dihydrotestosterone. D. SHBG. E. SHBG and dihydrotestosterone.

B. free testosterone or index of free androgens.

The incidence of which of the following diseases does not increase in female patients with Turner syndrome? A. strabismus. B. grass pollen allergy. C. chronic otitis media. D. arterial hypertension. E. autoimmune thyroid disease.

B. grass pollen allergy.

Which of the history or diagnostics information does not confirm the diagnosis of hypothalamic-pituitary insufficiency? A. amenorrhea. B. high FSH levels. C. low FSH levels. D. negative progesterone test. E. positive estrogen-progesterone test.

B. high FSH levels.

In female adolescents without signs of breast development gonadotropins should be measured in order to exclude: A. hyperprolactinemia. B. hypergonadotropic hypogonadism. C. hyperthyroidism. D. precocious puberty. E. polycystic ovarian syndrome.

B. hypergonadotropic hypogonadism.

Sheehan's syndrome - postpartum pituitary gland necrosis - is characterized by: A. hypogonadotropic hypergonadism. B. hypogonadotropic hypogonadism. C. hypergonadotropic hypergonadism. D. hypergonadotropic hypogonadism. E. primary amenorrhoea.

B. hypogonadotropic hypogonadism.

Vaginal bleeding and hypogastric pain occurred in a 25-year-old patient in 8 week of gestation. Gynecological examination showed the thinning of the cervix, and cervical dilation with parts of the gestational sac visible in the internal orifice. Indicate the correct diagnosis and the management of choice: A. missed miscarriage, awaiting for the spontaneous miscarriage for 2 weeks. B. inevitable miscarriage, curettage of the uterine cavity with histopathological examination. C. threatened miscarriage, bed rest. D. anembryonic pregnancy, awaiting for the spontaneous miscarriage for 2 weeks. E. missed miscarriage, intravaginal therapy with progesterone.

B. inevitable miscarriage, curettage of the uterine cavity with histopathological examination.

Which of the following is characteristic of stress urinary incontinence (SUI) in women? A. involuntary voiding that is preceded by strong urge to void and nycturia. B. leakage of urine following increased abdominal pressure. C. involuntary leakage of urine as the bladder is filled over. D. continuous urinary leakage through the fistula. E. feeling of incomplete voiding after emptying the bladder.

B. leakage of urine following increased abdominal pressure.

The lower limit of the reference range of hemoglobin concentration below which anemia is diagnosed in a pregnant woman is: A. higher than in non-pregnant women because the increase in the volume of circulating blood in pregnancy does not keep up with the increase in erythrocyte production. B. lower than in non-pregnant women because the increase in erythrocyte production does not keep up with the increase in the volume of circulating blood. C. the same as in non-pregnant women. D. irrelevant because anemia in pregnancy is diagnosed on the basis of hematocrit. E. dependent on the level of hemoglobin concentration before the pregnancy.

B. lower than in non-pregnant women because the increase in erythrocyte production does not keep up with the increase in the volume of circulating blood.

Ultrasonographic examination performed at 6 week of gestation revealed coexistence of two fetuses and a single yolk sac. That allowed for the diagnosis of: A. monochorionic, diamniotic twin pregnancy. B. monochorionic, monoamniotic twin pregnancy. C. dichorionic twin pregnancy, conjoint twins. D. dichorionic twin pregnancy, with abnormal development of one of the fetuses. E. dichorionic, monoamniotic twin pregnancy.

B. monochorionic, monoamniotic twin pregnancy.

Symptoms of ovarian cancer do not include: A. pain in the lower abdomen or pelvis. B. oliguria. C. increase in the abdominal circumference. D. tympanites, abdominal distention. E. urge incontinence.

B. oliguria.

In a pregnant vegetarian you should expect the insufficiency of vitamin: 1) B 2) B12 3) D 4) C 5) E The correct answer is: A. only 1. B. only 2. C. only 3. D. 1,3,4,5. E. 1,2,4,5.

B. only 2.

Microscopic evaluation of the material obtained at the outer opening of the cervix is called cervical smear (Papanicolaou test) and is used to diagnose precancerous states or cervical cancer. Which of the following should be advised in the case of the abnormal result of cervical smear - HSIL (high-grade squamous intraepithelial lesion)? A. repeat cervical smear - twice every 4-6 months. B. perform colposcopy with the biopsy of the suspected lesions and abrasion of the cervical canal. C. HPV determination. D. repeat cervical smear and determine HPV. E. none of the above.

B. perform colposcopy with the biopsy of the suspected lesions and abrasion of the cervical canal.

Factors increasing the risk of emergency peripartum hysterectomy do not include: A. previous cesarean section. B. pregnancy after in vitro procedure. C. placenta praevia or percreta. D. numerous previous pregnancies. E. age above 35.

B. pregnancy after in vitro procedure.

Indicate the true statement concerning the risk factors for ovarian cancer: A. BRCA2 mutation is more common than BRCA1 mutation in Poland. B. risk of the disease is higher in the bearers of BRCA1 mutation than in the bearers of BRCA2 mutation. C. risk of the disease is similar in the bearers of BRCA1 mutation and the bearers of BRCA2 mutation. D. family history of breast cancer does not increase the risk of the disease. E. long-term use of hormonal contraceptives increases the risk of the disease.

B. risk of the disease is higher in the bearers of BRCA1 mutation than in the bearers of BRCA2 mutation.

Which of the following fetal head circumferences is the smallest? A. occipito-frontal. B. suboccipito-bregmatic. C. occipito-mental. D. suboccipito-hyoid. E. genio-hyoid.

B. suboccipito-bregmatic.

A 65-year-old patient in good medical condition, with arterial hypertension and type 2 diabetes mellitus treated with oral therapy, comes to the gynecologist because of the moderate hemorrhage from the reproductive tract. Her body weight is 87 kg and the height is 163 cm. The result of the endometrial biopsy is: endometrial adenocarcinomaG1. Transvaginal ultrasound imaging suggests minimal invasion of the endometrium, CT imaging shows no intra-abdominal dissemination nor pathological pelvic or paraaortic lymph nodes. Indicate the optimal treatment for this patient: A. surgery: hysteroscopic removal of the tumor. B. surgery: panhysterectomy. C. radiotherapy (brachytherapy + teletherapy). D. chemotherapy (paclitaxel + carboplatin). E. hormonal (megestrol acetate).

B. surgery: panhysterectomy.

The triad of Sabin and Pinkerton (hydrocephalus or microcephaly, chorioretinitis, intracranial calcifications) is typical of the congenital infection with: A. rubella virus. B. toxoplasmosis. C. parvovirus B19. D. cytomegalovirus. E. herpes simplex virus.

B. toxoplasmosis.

Dopamine D receptor agonists (bromocriptine, quinagolide, cabergoline) are used for: A. stimulation of lactation when the amount of milk produced in the postpartum period is too low. B. treatment of pituitary tumors producing prolactin. C. diagnostics of functional hyperprolactinemia. D. controlled ovarian hyperstimulation. E. contraception.

B. treatment of pituitary tumors producing prolactin.

The most common complication in twin monoamniotic pregnancies is: A. preterm birth. B. umbilical cord entanglement. C. intrauterine growth restriction. D. polyhydramnios. E. umbilical cord prolapse

B. umbilical cord entanglement.

Indicate the false statement concerning HPV infection: A. so-called oncogenic HPV may cause cancers of the anus, the perineum, the penis and the oral cavity. B. use of condoms fully eliminates the risk of HPV infection. C. cytological result of LSIL suggests HPV infection. D. vaccination against HPV is recommended for young girls who have not started sexual activity. E. HPV infection does not lead to stable acquired immunity against subsequent infections with HP viruses.

B. use of condoms fully eliminates the risk of HPV infection.

Which of the following is a risk factor for ovarian hyperstimulation syndrome? A. long time of infertility treatment. B. young age. C. hypothyroidism. D. obesity. E. supplementation of progesterone in the luteal phase.

B. young age.

A 24-year-old G1P0 at 32 weeks gestation presents with vaginal bleeding most likely caused by placental abruption. She receives a standard dose of 300 micrograms of RhoGAM. What amount of fetal blood is neutralized by this dose? A. 10 cc B. 20 cc C. 30 cc D. 40 cc E. 50 cc

C 30 cc of fetal blood is neutralized by the 300 micrograms dose of RhoGAM. This is equivalent to 15 cc of fetal red blood cells. At 28-weeks gestation, 300 micrograms of Rh-immune globulin is routinely administered after testing for sensitization with an indirect Coombs' test. Administration is given following amniocentesis at any gestational age.

A 29-year-old G0 woman presents due to the inability to conceive for the last 18 months. She has a known history of endometriosis, which was diagnosed by laparoscopy three years ago. She has pelvic pain, which is controlled with non-steroidal anti-inflammatory drugs. Her cycles are regular. She is otherwise in good health and has been married for five years. Her husband had a semen analysis, which was normal. She had a hysterosalpingogram, which showed patent tubes bilaterally. She is getting frustrated that she has not yet achieved pregnancy and asks to proceed with fertility treatments. What is the most appropriate next step in the management of this patient? A. Reassurance and return in six months B. Administer a GnRH agonist C. Ovarian stimulation with clomiphene citrate D. Intrauterine insemination E. Proceed with in vitro fertilization

C A patient with a known history of endometriosis who is unable to conceive and has an otherwise negative workup for infertility, benefits from ovarian stimulation with clomiphene citrate, with or without intrauterine insemination. Waiting another six months is not appropriate as she has been trying to conceive for 18 months unsuccessfully. A GnRH agonist is used to control pelvic pain in endometriosis patients unresponsive to other hormonal treatments. In vitro fertilization and adoption can be offered if other treatments fail.

A 34-year-old healthy G3P3 woman presents for an health maintenance examination. She has not seen a gynecologist since she had a tubal ligation six years ago. She has been married for 12 years. She has no history of abnormal Pap smears or sexually transmitted infections. The patient's physical examination is normal, except for a small white plaque noted at the 12:00 position on her external cervical os. In addition to obtaining a Pap smear, which of the following is the most appropriate next step in the management of this patient? A. Annual Pap smears B. Pap smear in three years C. Biopsy the lesion D. Perform a wet mount E. Perform cervical conization

C A white plaque found on the cervix is called leukoplakia and should be biopsied directly or under colposcopic guidance as soon as possible, regardless of Pap smear outcome. Pap smears have a false-negative rate as high has 20-30%. If there is no evidence of dysplasia and her Pap smear is normal, then routine screening can be resumed. A wet mount would be indicated if there was evidence of white discharge. Although cervical conization maybe necessary if high grade dysplasia is diagnosed, this is not the most appropriate step in the management of this patient.

An 18-year-old nulliparous woman presents to discuss contraception. Her best friend's mother was just diagnosed with ovarian cancer. The patient is healthy and does not have any significant medical history. She does not have a family history of ovarian, breast or any other malignancies. She uses condoms for birth control. She would like to know what she can do to minimize her risk for developing ovarian cancer. Which of the following recommendations is the most appropriate for this patient? A. Begin childbearing now B. Use an intrauterine device C. Use oral contraceptives until she is ready to have children D. Have a prophylactic oophorectomy once childbearing is complete E. There are no proven means to reduce the risk of ovarian cancer

C A woman's risk for development of ovarian cancer during her lifetime is approximately 1%. Factors associated with development of ovarian cancer include low parity and delayed childbearing. Long-term suppression of ovulation appears to be protective against the development of ovarian cancer. Oral contraceptives that cause anovulation appear to provide protection against the development of ovarian cancer. Five years cumulative use decreases the lifetime risk by one-half. Prophylactic oophorectomy is not a practical choice for this patient with no family history, even once she completes childbearing. This option might be considered for a woman with a strong family history and the BRCA mutation.

Check-up of 19 year old sexually active girl. Everything is fine. What to check on current visit?

a. Chlamydia testing

A 32-year-old G2P2 woman presents for a health maintenance examination. She is in good health and has no concerns. She does not have a history of abnormal Pap smears and her last one was three years ago. Her examination is normal including her pelvic exam. A Pap smear is performed and returns as normal with HPV negative. What is the most appropriate screening recommendation for cervical cancer in this patient? A. Pap smear and HPV testing in one year B. Pap smear and HPV testing in three years C. Pap smear and HPV testing in five years D. HPV testing alone in one year E. HPV testing alone in three years

C According to the American Cancer Society (ACS), the American Society for Colposcopy and Cervical Pathology (ASCCP), and the American Society for Clinical Pathology (ASCP) guidelines for the Prevention and Early Detection of Cervical Cancer, women ages 30 to 65 years should be screened with cytology and HPV testing (''cotesting'') every five years (preferred) or cytology alone every three years (acceptable). Screening by HPV testing alone is not recommended for most clinical settings and there is insufficient evidence to change screening intervals in this age group following a history of negative screens.

A 32 year-old delivered a 9-pound baby and sustained a 4th-degree laceration two days ago. The delivery was complicated by a shoulder dystocia. Her laceration was repaired in layers in the customary fashion. She now complains of increasing pain in her perineal area, fever chills and weakness. Her vital signs are: blood pressure 90/50; pulse 120; and temperature 102.2°F, 39°C. Her abdomen is soft, nontender and her uterine fundus is firm and nontender. Her perineum is erythematous, swollen, but the laceration edges have separated and are grey. The laceration site is nontender and without feeling but there is tenderness of the surrounding tissue. In addition to broad spectrum antibiotics, what is your next step in the management of this patient? A. Sitz baths B. Whirlpool therapy C. Debridement D. Repair of laceration site E. Incision and drainage of perineal laceration

C Aggressive debridement of the necrotic areas is required to prevent further spread of the infection. Debridement should extend until vital tissue with good blood supply is encountered. Repair of the defect should be delayed until the infection has completely resolved. Sitz baths and whirlpool therapy will provide symptomatic relief for her discomfort, but not adequate treatment. Incision and drainage of perineal laceration is appropriate for an uncomplicated abscess.

A 25-year-old G2P1 woman states her gestational age by sure LMP is 16 weeks, 3 days. She reports no complaints and is not yet feeling fetal movement. Her fundal height is 22 cm. The MSAFP (maternal serum alpha fetoprotein) result is elevated. Which of the following is the most likely cause for the abnormal MSAFP result? A. Fetal trisomy B. Polyhydramnios C. Twin gestation D. Fetal abdominal wall defect E. Fetal neural tube defects

C Alpha fetoprotein (AFP) levels in twin gestations are elevated and should be roughly twice that seen in singleton pregnancies. An additional clue to a possible diagnosis of twin gestation is the fundal height exceeding gestational age in weeks. Other causes of elevated maternal serum AFP include neural tube defects, pilonidal cysts, cystic hygroma, sacrococcygeal teratoma, fetal abdominal wall defects, and fetal death. Polyhydramnios is not by itself associated with abnormal MSAFP levels.

An 18-year-old woman comes to your office for her first gynecologic visit. Her 43-year-old stepmother was just diagnosed with breast cancer. She wants to discuss breast cancer screening and wants to have a mammogram. According to the American College of Obstetricians and Gynecologists (ACOG), which of the following is the most appropriate recommendation for this patient? A. Yearly starting at age 35 B. Every one to two years from 35-49, then yearly starting at age 50 C. Yearly starting at age 50 D. Yearly breast ultrasound at age 35, then yearly mammograms at age 50 e. Yearly breast ultrasound now, then yearly mammograms at age 40

C Although the recommendation to initiate breast cancer screening timing with mammograms varies among different professional societies (American Cancer Society: yearly at age 40; American College of Obstetricians and Gynecologists (ACOG): yearly at age 40; United States Preventive Services Task Force [USPSTF]: every two years at age 50), none recommend routine screening prior to age 40 in a patient with no family history of breast cancer.

A 48-year-old G2P2 comes to your office because she has skipped her menstrual period for the past three months. She denies any menopausal symptoms. Review of symptoms and physical exam are unremarkable. What is the most likely diagnosis in this patient? A. Hypothyroidism B. Early pregnancy C. Perimenopause D. Premature ovarian failure E. Autoimmune disorder

C Although there has been a decline in the average age of menarche with the improvement in health and living conditions, the average age of menopause has remained stable. The Massachusetts Women's Health Study reports that the average age of menopause is 51.3. This patient is most likely perimenopausal and will probably have more menstrual periods in the future. Although it is important to consider pregnancy and hypothyroidism, this patient's presentation is most consistent with perimenopause. Premature ovarian failure occurs before age 35.

"A 33-year-old G3P1 woman presents to your office with a positive home pregnancy test. Her last menstrual period was 12 weeks ago. Obstetrical history is notable for a prior full term vaginal delivery and a miscarriage. Ultrasound reveals multiple internal echoes consistent with a ""snow storm"" appearance within the 20-week sized uterus, as well as bilateral 6 cm multicystic ovaries. Beta-hCG level is >200,000 mIU/mL. Dilation and curettage is performed and final pathology reveals a complete molar pregnancy. What is this patient's risk of developing persistent (post-molar) gestational trophoblastic disease? A. Not at risk B. Lower than that of a partial molar pregnancy C. Higher than that of a partial molar pregnancy D. Same as that of a partial molar pregnancy E. High enough to mandate prophylactic treatment with methotrexate"

C Although very effective in evacuating both complete and partial molar pregnancies, suction curettage provides definitive therapy in the vast majority of partial moles (>95%). For complete molar pregnancies, although Beta-hCG levels initially do drop following dilation and curettage, they can plateau and eventually rise in approximately 20% of cases. The risk following partial moles is much less (5%). The development of this post-molar GTD may be due to persistent (retained or invasive) disease in the uterus or metastatic disease (often to the lungs). The constellation of findings described in this patient (large uterus, theca lutein cysts, high Beta-hCG) increases the risk that this molar pregnancy will persist despite complete evacuation, hence the need for close follow-up with serial Beta-hCG levels. Persistent disease can easily be cured with chemotherapy, if it develops, and is therefore not routinely given prophylactically, except in high-risk situations (e.g. non-compliant patient who will be lost to follow-up).

Woman with red and swollen albue/forearm. Boyfriend said it was from a spider bite?

a. Clindamycin

A 28-year-old nulliparous woman presents for an annual examination. She is in good health and not taking any medications. She had a history of normal cycles until six months ago, when she stopped having menses after starting an intense exercise regimen. She is 5 feet 6 inches tall and weighs 120 pounds. Her examination is completely normal. Her urine pregnancy test is negative. What is the underlying pathophysiology of the disease process in this patient? A. Psychogenic amenorrhea B. Premature ovarian failure C. Hypothalamic amenorrhea D. Androgen excess E. Hyperthyroidism

C Amenorrhea associated with exercise falls under the category of hypothalamic amenorrhea, which causes chronic anovulation. Although it may be related to energy requirements, alterations in the hypothalamic-pituitary-ovarian axis have been described in athletic women. The patient's history and physical exam make ovarian failure, androgen excess and hyperthyroidism less likely, although a TSH level would still be recommended.

A 19-year-old G1P0 woman presents to the hospital at 25 weeks gestation with vaginal bleeding for the past hour. She had intercourse earlier without discomfort. Currently she denies cramping or pain and feels the baby moving. Her prenatal course has been uncomplicated. She takes no medication besides prenatal vitamins and denies smoking, alcohol or drug use. Her vital signs are: blood pressure 110/70; pulse 68; and she is afebrile. Her abdomen and uterus are soft and non-tender. Fetal heart tones are in the 150s. Which of the following is the most appropriate next step in the management of this patient? A. Digital cervical examination B. Biophysical profile C. Pelvic ultrasound D. Blood transfusion E. Bed rest

C An ultrasound should be performed to check for abnormal placentation. A biophysical profile is not appropriate in this patient. Placenta previa must be ruled out before proceeding with digital cervical examination because of the risk of injury to the placenta and hemorrhage. Bleeding from placenta previa often is without warning or pain.

A 36-year-old G1P0 Asian woman presents to the office accompanied by her 32-year-old husband. She is thrilled that she is now pregnant with twins after undergoing in vitro fertilization. She has a history of polycystic ovarian syndrome and thought she would not be able to have a baby. Her husband has a twin brother. Which of the following is the most likely cause of twins in this patient? A. Advanced maternal age B. Ethnicity C. Assisted reproductive technology D. Nulliparity E. Paternal family history

C Assisted reproduction has led to an increase number of multiple gestations. The rates of multiple births after IVF (in-vitro fertilization) vary according to maternal age and the number of embryos transferred. Transfer of multiple embryos is more likely to result in multiple gestations in younger women than in older women. The frequency of multiple gestations increases with the use of ovulation inducing drugs. The risk is in the 5-6% range, but varies depending on the drug and dosage regimen used. Dizygous twinning results from the ovulation of multiple follicles and the rate of multiple gestation increases with advancing maternal age. Elevated follicle-stimulating hormone correlates with dizygous multiple births. A higher number of prior pregnancies and previous history of multiple births increases the chance of having a multiple gestation. These do not apply here because the patient's obstetrical history does not support this. Dizygous twinning appears to have a genetic component and rates of dizygous twins vary according to ethnicity, but are not related to paternal family history.

A 24-year-old G2P1 woman at 18 weeks gestation with a history of asthma presents to the office with worsening symptoms, needing to use her inhaler more frequently. The symptoms began with the pregnancy and have gradually increased. She is using her albuterol inhaler as needed, recently three times a day. She denies any illness or fever. She has had asthma since she was a child. On exam, the patient appears comfortable. Her temperature is 100.2°F (37.9°C) and respiratory rate is 18. Auscultation of the lungs shows good air movement with mild scattered end expiratory wheezes. There are no rales or bronchial breath sounds. Which of the following is the most appropriate next step in the management of this patient? A. Oral theophylline B. Subcutaneous terbutaline C. Inhaled corticosteroids D. Oral zafirlukast (leukotriene inhibitor) E. Antibiotic treatment

C Asthma generally worsens in 40% of pregnant patients. One of the indications for moving to the next line of treatment includes the need to use beta agonists more than twice a week. The appropriate choice for her treatment would be inhaled corticosteroids or cromolyn sodium. Theophylline would be used in more refractory patients. Subcutaneous terbutaline and systemic corticosteroids would be used in acute cases. Zafirlukast, a leukotriene receptor antagonist, is not effective for acute disease. There is little experience with their use in pregnancy, thus the safety of zafirlukast in pregnancy is not well established. Antibiotic treatment is only used when a pulmonary infection is diagnosed.

A 22-year-old G1 woman is undergoing treatment with magnesium sulfate for severe preeclampsia. She was delivered 10 hours ago via Cesarean section for a non-reassuring fetal heart rate tracing. She has oliguria and appears lethargic. On exam, no deep tendon reflexes can be appreciated. Her magnesium level is 11 mEq/L. Which of the following conditions is most likely to occur in this patient? A. Seizures B. Paralysis C. Respiratory depression D. Pulmonary edema E. Cardiac arrest

C At a magnesium level of 11 mEq/L, respiratory depression is most likely to occur. A therapeutic magnesium level is between 4-7 mEq/L. Seizures are prevented by the use of magnesium. Loss of deep tendon reflexes occurs at a level of 7-10 mEq/L. Cardiac arrest may occur at a level of 15 mEq/L. Pulmonary edema can occur with magnesium therapy, but is not related to toxicity from the drug.

A 25-year-old G0 woman presents for a refill on oral contraceptives. She has a history of recurrent ovarian cysts. She has no significant medical or surgical history. Her grandmother was recently diagnosed with ovarian cancer and her mother is undergoing treatment for metastatic breast cancer. The patient is interested in assessing her risk for ovarian cancer susceptibility. Which of the following is the most appropriate test to offer this patient? A. Annual CA125 levels B. Annual pelvic ultrasound C. Genetic testing of BRCA 1 and 2 mutations on the patient's mother D. Genetic testing of BRCA 1 and 2 mutations on the patient's grandmother E. Check the patient for p53 mutation

C BRCA1 and BRCA2 mutations are typically seen in cases of hereditary ovarian cancers. Overall, it has been estimated that inherited BRCA1 and BRCA2 mutations account for 5 to 10 percent of breast cancers and 10 to 15 percent of ovarian cancers among white women in the United States. Given this family history, it is highly likely that a mutation is present, and the affected individual (proband) should be tested if still alive. Because breast cancers are part of the BRCA mutation, the affected mother should be tested. Routine screening for ovarian cancer has not been established.

A 32-year-old nulliparous woman with a last menstrual period three weeks ago, presents with a three-month history of a malodorous vaginal discharge. She reports no pruritus or irritation. She has been sexually active with a new partner for the last four months. Her past medical history is unremarkable. Pelvic examination reveals normal external genitalia without rash, ulcerations or lesions. Some discharge is noted on the perineum. The vagina reveals only a thin, gray homogeneous discharge. The vaginal pH is 5.0. A wet prep is shown in the image below. Which of the following is the most appropriate treatment for this patient? A. Ceftriaxone B. Doxycycline C. Metronidazole D. Azithromycin E. Penicilli

C Bacterial vaginosis is the most common cause of vaginitis. The infection arises from a shift in the vaginal flora from hydrogen peroxide-producing lactobacilli to non-hydrogen peroxide-producing lactobacilli, which allows proliferation of anaerobic bacteria. The majority of women are asymptomatic; however, patients may experience a thin, gray discharge with a characteristic fishy odor that is often worse following menses and intercourse. Modified Amsel criteria for diagnosis include three out of four of the following: 1) thin, gray homogenous vaginal discharge; 2) positive whiff test (addition of potassium hydroxide releases characteristic amine odor); 3) presence of clue cells on saline microscopy; and 4) elevated vaginal pH >4.5. Treatment consists of Metronidazole 500 mg orally BID for seven days, or vaginal Metronidazole 0.75% gel QHS for five days.

A 90-year-old G7P7 woman presents with severe vaginal prolapse. The entire apex, anterior and posterior wall are prolapsed beyond the introitus. She cannot urinate without reduction of the prolapse. Hydronephrosis was noted on ultrasound of the kidneys and it is thought to be related to the prolapse. She has a long-standing history of diabetes and cardiac disease. She has failed a trial of pessaries. Which of the following is the next best step in the management of this patient? A. Do nothing and observe B. Anterior and posterior repair C. Colpocleisis D. Sacrospinous fixation E. Sacrocolpopexy

C Because of the hydronephrosis due to obstruction, intervention is required. Colpocleisis is a procedure where the vagina is surgically obliterated and can be performed quickly without the need for general anesthesia. Anterior and posterior repairs provide no apical support of the vagina. She will be at high risk of recurrent prolapse. The sacrospinous fixation (cuff to sacrospinous-coccygeus complex) or sacrocolpopexy (cuff to sacral promontory using interposed mesh) require regional or general anesthesia and is not the best option for this patient with high surgical morbidity.

A 32-year-old G3P3 woman had a Pap smear that showed a high-grade squamous intraepithelial lesion (HSIL). She smokes one pack of cigarettes per day. She has a history of three vaginal deliveries and a tubal ligation. On colposcopic examination, at 12:00 there is an acetowhite lesion with punctations that extends into the endocervical canal. Endocervical speculum is unsuccessful at visualizing the entire lesion. Endocervical curettage and biopsy of this area is negative. Which of the following would be the most appropriate next step in the management of this patient? A. Repeat colposcopy in two months B. Cryotherapy ablation of the transformation zone C. Cervical conization D. HPV typing E. Repeat Pap smear in six months

C Because the entire lesion cannot be visualized, this colposcopy is unsatisfactory. Severe dysplasia and even invasive cancer cannot be ruled out. Endocervical curettage has a relatively low sensitivity (i.e. a high amount of false negatives) and, therefore, you cannot rule out endocervical disease. The endocervical canal must be histologically examined. A cervical conization should be performed to obtain a pathologic specimen. Cryotherapy may serve to ablate part of the canal, but will not provide a pathologic sample to assess for dysplasia or to rule out cancer.

A 23-year-old G1P0 woman at 38 weeks gestation, with an uncomplicated pregnancy, presents to labor and delivery with the complaint of lower abdominal pain and mild nausea for one day. Fetal kick counts are appropriate. Her review of symptoms is otherwise negative. Vital signs are: temperature 98.6°F (37.0°C); blood pressure 100/60; pulse 79; respiratory rate 14; fetal heart rate 140s, reactive, with no decelerations; tocometer shows irregular contractions every 2-8 minutes; fundal height 36 cm; cervix is firm, long, closed and posterior. A urine dipstick is notable for 1+ glucose with negative ketones. Which of the following is the most likely diagnosis in this patient? A. Appendicitis B. Gestational diabetes C. Braxton-Hicks contractions D. First stage of labor E. Dehydration

C Braxton Hicks contractions are characterized as short in duration, less intense than true labor, and the discomfort as being in the lower abdomen and groin areas. True labor is defined by strong, regular uterine contractions that result in progressive cervical dilation and effacement. This patient's history does not suggest she is in the first stage of labor. Patients with appendicitis usually present with fever, decreased appetite, nausea and vomiting. Gestational diabetes is diagnosed based on glucose challenge tests. The first test with a 50 gram load is typically performed at 24-28 weeks gestation. It is not abnormal for patients to have glucosuria. This finding is not diagnostic for gestational diabetes. Patients with dehydration frequently present with maternal tachycardia and have ketonuria.

A 25-year-old G1 woman at 41 weeks gestation presents to labor and delivery with painful contractions every four minutes. Her cervix is 5 cm dilated, 90% effaced. On cervical exam, you are able to feel a fetal body part but it is not the head. Which of the following is the most likely body part you were palplating? A. Foot B. Hand C. Buttocks D. Back E. Shoulder

C Breech presentation occurs in approximately 3-4% of women in labor overall, and occurs more frequently in preterm deliveries. Frank breech is the most common type, occurring in 48-73% of cases and the buttocks are the presenting part. Complete breech is found in approximately 5-12% of cases and incomplete breech (footling breech) occurs in approximately 12-38% of cases.

A 37-year-old G3P2 woman presents with her husband at 11 weeks gestation for genetic counseling due to advanced maternal age. The patient and her husband are interested in chorionic villus sampling (CVS). In addition to obtaining a karyotype, which of the following can be detected with this procedure? A. Spina bifida B. Fetal omphalocele C. Cystic fibrosis D. Anencephaly E. Fetal cardiac anomaly

C CVS is generally performed at 10-12 weeks gestation. The procedure involves sampling of the chorionic frondosum, which contains the most mitotically active villi in the placenta. CVS can be performed using a transabdominal or transcervical approach. The sampled placental tissue may be analyzed for fetal chromosomal abnormalities, biochemical, or DNA-based studies including testing for the mutations associated with cystic fibrosis. CVS cannot be used to detect neural tube defects. Omphaloceles and neural tube defects are generally diagnosed using prenatal ultrasound.

A 17-year-old G1 woman at 24 weeks gestation presents with vaginal bleeding. She denies any pain, cramping or dysuria. She reports last having intercourse three weeks ago. Prenatal care and labs have been unremarkable. Her vital signs are normal and she is afebrile. Pelvic ultrasound reveals a fundal placenta and viable fetus. Abdominal examination is unremarkable. Vaginal examination reveals a uniformly friable cervix with a small amount of blood in the vault. Digital examination reveals a firm, closed cervix. What is the most likely diagnosis that explains the bleeding? A. Trauma B. Cervical cancer C. Cervicitis D. Bloody show E. Threatened abortion

C Cervicitis caused by chlamydia, gonorrhea, trichomonas or other infections can present with vaginal bleeding. The cervix is much more vascular during pregnancy and inflammation can lead to bleeding. Evaluation for other causes of bleeding must be completed and then treatment for the infection should be initiated. The patient does not give any history of trauma and cancer is unlikely because of her age. She is not in labor, and a bloody show associated with cervical dilatation is not consistent with the history provided. Threatened abortion occurs during the first trimester.

A 17-year-old G0 female presents with a three-year history of severe dysmenorrhea shortly after menarche at age 14. Her menstrual cycles are regular with heavy flow. She has been treated with ibuprofen and oral contraceptives for the last year without significant improvement. She misses 2-3 days of school each month due to her menses. She has never been sexually active. Physical examination is remarkable for Tanner Stage IV breasts and pubic hair. Pelvic examination is normal, as is a pelvic ultrasound. Both the patient and her mother are concerned. What is the next best step in the management of this patient? A. Sonohysterogram B. CT scan of the pelvis C. Diagnostic laparoscopy D. MRI of the pelvis E. Hysterosalpingogram

C Chronic pelvic pain is the indication for at least 40% of all gynecologic laparoscopies. Endometriosis and adhesions account for more than 90% of the diagnoses in women with discernible laparoscopic abnormalities, and laparoscopy is indicated in women thought to have either of these conditions. Often, adolescents are excluded from laparoscopic evaluation on the basis of their age, but several series show that endometriosis is as common in adolescents with chronic pelvic pain as in the general population. Therefore, laparoscopic evaluation of chronic pelvic pain in adolescents should not be deferred based on age. Laparoscopy can be both diagnostic and therapeutic in this patient in whom you suspect endometriosis. None of the other imaging modalities listed will help in the further workup of this patient.

A 25-year-old G1 woman at term presents in active labor. Her cervix rapidly changes from 7 centimeters to complete dilation in 1 hour. She has been pushing for two hours. The fetal station has changed from -1 to +1. Fetal heart tracing is category I. The patient is feeling strong contractions every three minutes. Which of the following is the most appropriate next step in the management of this patient? A. Cesarean delivery B. Forceps delivery C. Continued monitoring of labor D. Augmentation with oxytocin E. Ultrasound for estimated fetal weight

C Continued monitoring of labor is appropriate if clinical evaluation indicates that the fetus is not macrosomic or there is no obvious fetopelvic disproportion. If either were the case, then a Cesarean delivery would be indicated. At this time, there is no fetal or maternal indication to perform a forceps delivery because the station is +1. Augmentation would be indicated if the contractions were inadequate in intensity or frequency. An ultrasound at this stage of labor is inaccurate and one relies on clinical estimates of weight.

A 60-year-old postmenopausal woman has an appointment for a health maintenance examination. She wants to discuss bone density screening for osteoporosis. Her medical history is significant for hypertension for the last 10 years that is well controlled with antihypertensive medications. Which of the following is the most appropriate recommendation for this patient regarding initial bone mineral density screening? A. Obtain a DEXA scan now B. Obtain a DEXA scan now only if her blood pressure is not well controlled C. Obtain a DEXA scan at age 65 D. Bone density screening is only necessary if she presents with a fracture E. Recommend bone density screening five years after the onset of menopause

C Correct answer is C. Bone density screening is recommended for women beginning at age 65 unless they have pre-existing risk factors which warrant earlier screening. Risk factors for osteoporosis are early menopause, glucocorticoid therapy, sedentary lifestyle, alcohol consumption, hyperthyroidism, hyperparathyroidism, anticonvulsant therapy, vitamin D deficiency, family history of early or severe osteoporosis, or chronic liver or renal disease. These factors would institute early screening in a patient for osteoporosis. A postmenopausal patient presenting with fractures should alert you to suspect osteoporosis.

A 17-year-old G0 high school student is brought in by her mother for her first gynecologic examination. She began her menses at age 12 and has had regular periods for the past three years. Her last menstrual period was one week ago. For privacy, you ask to examine the patient without her mother. Further history is obtained in the examination room. She admits that she has been sexually active with her boyfriend for the past three years. She uses condoms occasionally and is fearful about possible pregnancy. She requests that her mother not be informed about her sexual activity. On physical examination, she is anxious, but normally developed. Her pelvic examination reveals no vulvar lesions, minimal non-malodorous discharge, and a nulliparous appearing cervix. The bimanual examination reveals a normal size uterus, and her adnexa are non-tender and not enlarged. Urine pregnancy test is negative. In addition to discussing contraception. What is the next best step in the management of this patient? A. Obtain a serum Beta-hCG level B. Obtain a Pap smear C. Obtain DNA probes for gonorrhea and chlamydia D. Initiate treatment with doxycycline and ceftriaxone E. Order a pelvic ultrasound

C Counseling about and screening for sexually transmitted infections is the best next step. This patient does not require treatment due to a lack of diagnostic criteria. A serum Beta-hCG is not indicated in the setting of normal menstrual cycles with last menstrual period a week ago and a negative urine pregnancy test. Guidelines for initiation of cervical cancer screening is recommended at age 21 regardless of coitarche. A pelvic ultrasound would not be indicated at this time especially since the pregnancy test is negative and given her lack of menstrual or pelvic symptoms.

A 14-year-old G0 adolescent reports menarche six months ago, with increasingly heavy menstrual flow causing her to miss several days of school. Three months ago, her pediatrician started her on oral contraceptives to control her menstrual periods, but she continues to bleed heavily. Her previous medical history is unremarkable. The patient has a normal body habitus for her age. Appropriate breast and pubic hair development is present. Her hemoglobin is 9.1 mg/dl, hematocrit 27.8%, urine pregnancy test negative. Which of the following etiologies for menorrhagia is most likely the cause of her symptoms? A. Uterine leiomyoma B. Thyroid disorder C. Coagulation disorder D. Endometrial hyperplasia E. Chronic endometritis

C Disorders of clotting may present with menstrual symptoms in young women, with Von Willeberand disease being most common. Leiomyomas typically present in women in their 30s and 40s. Endometrial hyperplasia can occur in younger anovulatory patients, but the short duration of this patient's symptoms makes this less likely. She does not have any signs of infection or thyroid disease.

A 32-year-old G3P2 woman presents at 40 1/7 weeks gestation because of regular uterine contractions every five minutes for the last two hours. Her prenatal course was unremarkable. She states the baby is moving, but she has had a bright red, bloody discharge for the last 30 minutes. She does not think she has ruptured her membranes. Her blood pressure is 120/70, pulse 80 and she is afebrile. Her abdomen is soft and she has regular contractions of moderate intensity. Fetal heart tones have a baseline of 130 with a category I fetal heart rate tracing. Pelvic ultrasound reveals a fundal placenta and cephalic presentation of the fetus. Cervical examination reveals a friable cervix that bleeds easily and is 5 centimeters dilated and completely effaced. Membranes are confirmed to be intact. Which of the following is the most likely source of bleeding? A. Placental abruption B. Placenta previa C. Bloody show D. Cervical cancer E. Cervicitis

C During pregnancy the cervix is extremely vascular, and with dilation a small amount of bleeding may occur. This bloody show is not of clinical significance and often occurs with normal labor. Serious causes of bleeding, such as placental abruption and placenta previa, need to be ruled out in order to make the proper delivery plans. Cervical cancer and cervicitis are very unlikely causes for the bleeding in this situation.

A 45-year-old G2P2 comes to the office because of heavy and irregular menstrual periods. The heavy periods started three years ago and have gradually worsened in amount of flow over time. The periods are interfering with her daily activities. The patient has had two spontaneous vaginal deliveries, followed by a tubal ligation three years ago. On pelvic examination, the cervix appears normal and the uterus is normal in size without adnexal masses or tenderness. A urine pregnancy test is negative. TSH and prolactin levels are normal. Hemoglobin is 12.5 mg/dl. On pelvic sonography, she has a normal size uterus and a 2 cm simple cyst on the right ovary. Endometrial biopsy is consistent with a secretory endometrium; no neoplasia is found. What is the most likely diagnosis in this patient? A. Polycystic ovarian syndrome B. Mid-cycle bleeding C. Dysfunctional uterine bleeding D. Benign cystic teratoma E. Ovarian cancer

C Dysfunctional uterine bleeding is defined as irregular or increased menstrual bleeding without identified etiology. This patient had a complete workup, including TSH, Prolactin, pelvic ultrasound and endometrial biopsy, which were all normal. Mid-cycle bleeding at the time of ovulation is due to the drop in estrogen. Ovarian teratomas are not associated with abnormal menses. They typically present with abdominal or pelvic pain which may be associated with torsion. The 2 cm cyst is a functional cyst and is a common finding in ovulatory patients.

A 19-year-old G0 presents with severe menstrual pain which causes her to miss school. She takes 600 mg of ibuprofen every four to six hours to control the pain, but this does not relieve the discomfort. She is sexually active, with one present partner (two lifetime partners) and uses condoms for contraception. Examination is normal. What is the most appropriate next step in the management of this patient? A. Change NSAID to a COX-2 inhibitor B. GnRH agonist C. Oral contraceptives D. Continuous medroxyprogesterone E. Laparoscopy

C Dysmenorrhea or painful menstrual cramps is often incapacitating. Oral contraceptives will not only relieve primary dysmenorrhea, but also provide more reliable contraception. COX-2 inhibitors have targeted action but have significant side effects and are no longer routinely prescribed. xContinuous oral Medroxyprogesterone may be effective, but will not provide contraception.x Depo-Provera would be a better choice. GnRH agonists are too expensive and have too high a side effect profile to be used for this purpose.

A 19-year-old G1P0 at 41 weeks with spontaneous rupture of membranes for 13 hours presented to labor and delivery. She had an uncomplicated prenatal course. Her vital signs are: blood pressure 120/70; pulse 72; afebrile; fundal height 36 cm; and estimated fetal weight of 2700 gm. Cervix is dilated to 4 cm, 100% effaced, + 1 station. What does the fetal heart rate tracing seen below show? A. Late deceleration B. Variable decelerations C. Early decelerations D. Sinusoidal rhythm E. Normal fetal heart rate pattern

C Early decelerations are physiologic caused by fetal head compression during uterine contractions, resulting in vagal stimulation and slowing of the heart rate. This type of deceleration has a uniform shape, with a slow onset that coincides with the start of the contraction and a slow return to the baseline that coincides with the end of the contraction. Thus, it has the characteristic mirror image of the contraction. A late deceleration is a symmetric fall in the fetal heart rate, beginning at or after the peak of the uterine contraction and returning to baseline only after the contraction has ended. Late decelerations are associated with uteroplacental insufficiency. Variable decelerations show an acute fall in the FHR with a rapid down slope and a variable recovery phase. They are characteristically variable in duration, intensity, and timing, and may not bear a constant relationship to uterine contractions. The true sinusoidal pattern is a regular, smooth, undulating form typical of a sine wave that occurs with a frequency of two to five cycles/minute and an amplitude range of five to 15 beats/minute. It is also characterized by a stable baseline heart rate of 120 to 160 beats/minute and absent beat-to-beat variability.

A 25-year-old G0 woman presents to her doctor for preconception counseling. She is healthy without significant medical problems. She takes no medications. She smokes one pack of cigarettes per day since age 16 and drinks occasionally. She weighs 140 pounds and her vital signs and examination are normal. The patient is at increased risk of which of the following during her pregnancy? A. Fetal chromosomal abnormality B. Breech presentation C. Placental abruption D. Cerebral palsy E. Neural tubal defect

C Smoking increases the risk of several serious complications of pregnancy, including placental abruption, placenta previa, fetal growth restriction, preeclampsia and infection. Women who smoke should be counseled vigorously to quit smoking prior to conception and to resist restarting after the baby is born.

A 22-year-old G1 at 38 weeks gestation presents in labor. Her prenatal course and past history are uncomplicated. She is having regular contractions and, on examination, the cervix is 5 cm, 100% effaced and fetal head at +1 station. The fetal heart rate tracing is shown below. What is the most likely interpretation? A. Normal reassuring B. Bradycardia C. Early deceleration D. Variable decelerations E. Late decelerations

C Early decelerations are thought to represent the fetal response to head compression during the contraction and the fetal heart rate inversely mirrors the changes noted during the contraction. Variable contractions are thought to be due to cord compression and can occur at any position in relation to a contraction. Generally, they have an abrupt onset and return of the fetal heart rate deceleration to the baseline heart rate. Late decelerations are thought to represent uteroplacental insufficiency. The deceleration of the fetal heart rate occurs at or after the peak of the uterine contraction and returns to baseline after complication of the contraction. Bradycardia is defined as fetal heart rate less than 110 beats perminute.

A 25-year-old G2P1 woman at eight weeks gestation is diagnosed with a spontaneous abortion. Her husband is 40 years old. The patient's past medical history is noncontributory. She gets some exercise regularly and smokes two packs of cigarettes a day. Three years ago, she had a full-term delivery that was complicated by mild preeclampsia. Which of the following factors is most likely the cause of this spontaneous abortion? A. Infection B. Advanced paternal age C. Environmental factors D. Uterine anomaly (i.e. unicornuate uterus) E. History of preeclampsia

C Environmental factors, such as smoking, alcohol and radiation are causes of spontaneous abortion. Although the risk increases with infections, such as listeria, mycoplasma, ureaplasma, toxoplasmosis and syphilis, advancing maternal or paternal age, advancing parity and some mullerian anomalies, the clinical scenario does not support these as possible causes. An isolated history of preeclampsia confers no increase in risk of spontaneous abortion.

A 20 year-old G1P1 delivered her first baby 24 hours ago. Delivery was uncomplicated and she had an epidural placed for analgesia at 5 centimeters of cervical dilation. Earlier in the afternoon, she was complaining of a headache and was given ibuprofen. Three hours later, she complained of increasing headache, photophobia and nausea. She denies heavy bleeding. Vital signs are pulse 110; respirations 20; temperature 101.5°F, 38.6°C; and blood pressure 100/50. Physical examination reveals obvious distress, as she has her eyes covered and pain when she moves her neck. Her lungs are clear and heart has a regular rate. Her abdomen is nontender, and uterine fundus is easily palpable just below the umbilicus and is nontender. Her extremities reveal no erythema, swelling or tenderness. Which of the following would be your next diagnostic step? A. Chest x-ray B. Urinalysis C. Lumbar puncture D. CBC with differential E. Pelvic ultrasound

C Epidurals are used commonly for pain relief during labor. Complications of epidural include spinal headache, localized back pain and meningitis. Symptoms of meningitis progress rapidly and require aggressive treatment with antibiotics. Diagnosis is made with evaluation of the cerebral spinal fluid from a lumbar puncture.

A 28-year-old G0 woman presents with her husband for preconception counseling. Her family is Ashkenazi Jewish from Poland. Her husband is 30 years old and is also Jewish. They seek information about preconception and prenatal screening. Carrier screening should be performed for all of the following conditions except: A. Fanconi anemia B. Tay-Sachs disease C. Beta thalassemia anemia D. Cystic fibrosis E. Niemann-Pick disease

C Fanconi anemia, Tay-Sachs disease, Cystic Fibrosis, and Niemann-Pick disease are all autosomal recessive conditions that occur at an increased incidence in Jews of Ashkenazi descent. The Beta thalassemia is seen mainly in Mediterranean populations.

A 28-year-old G1P0 woman is at 15 weeks gestation. Her husband's cousin has moderate mental retardation. The most common cause of inherited mental retardation in this patient's child would be? A. Undiagnosed phenylketonuria (PKU) B. Neonatal hypothyroidism C. Fragile X syndrome D. Down syndrome E. Autism

C Fragile X syndrome is the most common form of inherited mental retardation. The syndrome occurs in approximately 1 in 3,600 males and 1 in 4,000 to 6,000 females. Down syndrome is genetic but the majority of cases are not inherited.

A 36-year-old woman comes to the office due to hair loss. She delivered a healthy infant girl three months ago. She is currently on a progestin-only oral contraceptive pill since she is breastfeeding. In the last month, she has noticed a large amount of hair on her brush each morning. Her father has male pattern baldness and her mother, who is postmenopausal, has had some thinning of her hair, as well. Testosterone and TSH levels are within the normal range. Which of the following is the most likely underlying cause for alopecia in this patient? A. Genetic predisposition B. Progesterone only pills C. High estrogen levels during pregnancy D. Stress during pregnancy and delivery E. Breastfeeding

C High estrogen levels in pregnancy increase the synchrony of hair growth. Therefore, hair grows in the same phase and is shed at the same time. Occasionally, this can result in significant postpartum hair loss. In the non-pregnant state, asynchronous hair growth occurs such that a portion of hair is in one of the three hair growth cycles at all times.

A 26-year-old G2P1 woman at 41 weeks gestation is brought in by ambulance. The emergency medical technician reports that a pelvic examination performed 20 minutes ago when the patient had a severe urge to push revealed that she was fully dilated and the fetal station was +2. Fetal heart tones were confirmed to be in the 150s, with no audible decelerations. When the patient is placed on the fetal monitor, the heart rate is noted to be in the 60s. The maternal heart rate is recorded as 100. Without pushing, the fetal scalp is visible at the introitus. A repeat pelvic exam shows that the infant is in the left occiput anterior position. What is the most appropriate next step in the management of this patient? A. Emergent Cesarean delivery B. Amnioinfusion C. Assisted operative vaginal delivery D. Confirm the fetal heart rate with an internal fetal scalp electrode E. Use ultrasound to assess the fetal heart rate

C If the patient cannot deliver the infant with one or two pushes, the next best choice given the fetal station and presentation is to perform an emergent outlet forceps or vacuum-assisted delivery. None of the other options offer an expedient mode of delivery. Since the patient's heart rate is distinct from the fetal heart rate, it is not necessary to check the fetal heart rate with an ultrasound. This will potentially delay the time until delivery of the fetus. Amnioinfusion is not indicated given the imminent delivery.

An 18-year-old college student is brought to your office by her roommate who found her sobbing in their room. The student states that she was sitting on a couch with her new boyfriend. He gave her something to drink. She thinks that she passed out and awoke two hours later and was undressed. She is afraid that she may have had intercourse. She is not on any form of contraceptive. Her last menstrual period was two weeks ago. Examination reveals no obvious injuries and a vaginal swab shows motile sperm. In addition to providing counseling and screening for sexually transmitted infections, which of the following is the most appropriate next step in management? A. Discuss her responsibility to report this to her college B. Admit to the hospital for intravenous antibiotics C. Recommend emergency contraception D. Notify her college advisor E. Obtain more information from her roommate

C In a case of date rape, the best course of action is to ensure that the patient does not get pregnant. The patient should also have screening for sexually transmitted diseases, with consideration to being offered antibiotic prophylaxis. Although the risk of infection is unknown among victims of sexual assault, it may be higher compared to consensual sexual encounters.

A 35-year-old G4P3 woman comes in for a postpartum visit. She had a normal uncomplicated vaginal delivery two weeks ago. She has a history of postpartum depression, which required treatment with antidepressants with her last pregnancy. Which of the following signs or symptoms of postpartum depression are most useful to distinguish it from postpartum blues and normal changes that occur after delivery? A. Anhedonia B. Crying spells C. Ambivalence toward the newborn D. Sleeplessness E. Weight loss

C In addition to the more common symptoms of depression, the postpartum patient may manifest a sense of incapability of loving her family and manifest ambivalence toward her infant. Anhedonia is an inability to experience pleasure from normally pleasurable life events such as eating, exercise, and social or sexual interaction.

A 19-year-old G1 woman at 28 weeks gestation comes to labor and delivery because of the onset of contractions. The patient describes the contractions as progressively becoming more painful, each lasting 40 seconds and now occurring every five minutes. She reports good fetal movement and does not have any bleeding or leakage of fluid. On evaluation in triage, it is noted that she is having regular contractions, approximately every five minutes, has intact membranes and her cervical exam is 3 cm dilated and 50% effaced. What is the most frequent cause of this condition? A. Dehydration B. Fetal anomalies C. Idiopathic D. Uterine fibroids E. Cervical incompetence

C In most cases, preterm labor is idiopathic (i.e. no cause can be identified). Dehydration and uterine distortion (from uterine fibroids or structural malformations) can be associated with preterm labor. In some cases, preterm labor is due to iatrogenic causes; for example, when a physician induces a preterm patient who has severe preeclampsia. Fetal anomalies typically do not cause preterm labor. Cervical incompetence is usually diagnosed earlier in the second trimester.

A 28-year-old G1 woman at 31 weeks gestation presents with complaints of fluid leaking from the vagina. Preterm premature rupture of membranes is diagnosed. The patient has mild uterine tenderness concerning for early chorioamnionitis. An amniocentesis is performed. Which of the following amniotic fluid results is indicative of an intra-amniotic infection? A. Presence of leukocytes B. Low Interleukin-6 C. Amniotic glucose less than 20 mg/dl D. Elevated level of bilirubin E. Lecithin/sphingomyelin (L/S) ratio <2

C In some cases of preterm rupture of the membranes, amniocentesis may be performed to detect intra-amniotic infection. The presence of amniotic leukocytes has the lowest predictive value for the diagnosis of chorioamnionitis. Interleukin-6 would be increased in the setting of chorioamnionitis. A low amniotic fluid glucose is an indication of intra-amniotic infection. L/S ratio is a marker for fetal lung maturity.

A 17-year-old nulliparous female presents to your office with vaginal spotting for the last three days. Her last menstrual period was six weeks ago. Vitals signs are normal. Abdominal and pelvic examination reveals a 10-week sized uterus. Beta-HCG is 80,000 mIU. What is the best next step in the management of this patient? A. Repeat Beta-HCG in 24 hours B. Repeat Beta-HCG in 48 hours C. Perform a pelvic ultrasound D. Perform dilation and curettage E. Routine prenatal care

C In the face of discrepancy between dates and uterine size, a pelvic ultrasound in indicated to confirm dates, exclude multiple gestation, uterine abnormalities, and molar pregnancy. There is no single Beta-hCG value that is diagnostic for a molar pregnancy. A quantitative Beta-hCG, though, is crucial at determining whether or not a pelvic (transvaginal) ultrasound will confirm a very early gestation. With a Beta-HCG above the discriminatory zone (>1500), an IUP should be easily identified on transvaginal ultrasound. If an IUP is not seen, the ultrasound findings (in conjunction with the Beta-hCG level) should identify a mole (multiple internal echoes) or an ectopic (absence of intra-uterine gestation). Additional Beta-HCG levels are not indicated at this time. Suction curettage will provide a pathologic specimen that can distinguish between a normal and molar pregnancy, but it is used only as a therapeutic intervention. Routine prenatal care would be appropriate only after establishing a normal pregnancy.

A 29-year-old G2P1 woman at 36 weeks gestation is seen for management of her gestational diabetes. Despite diet modification, the patient has required insulin to control her serum glucose levels. She has gained 25 pounds with the pregnancy. She is at risk for all the following complications, except: A. Polyhydramnios B. Neonatal hypoglycemia C. Intrauterine growth restriction D. Preeclampsia E. Fetal macrosomia

C Intrauterine growth restriction is typically seen in women with pre-existing diabetes and not with gestational diabetes. Shoulder dystocia, metabolic disturbances, preeclampsia, polyhydramnios and fetal macrosomia are all associated risks of gestational diabetes.

A 36-year-old G2P0 woman at 11 weeks gestational age requests a surgical termination of pregnancy. She had a manual vacuum aspiration last year and would like to undergo the same procedure again. She has chronic hypertension and diabetes well controlled on medications. Vital signs reveal a blood pressure of 120/80 and fasting blood glucose of 100. Which of the following is a contraindication for manual vacuum aspiration of this patient? A. Age B. Parity C. Gestational age D. Chronic hypertension E. Diabetes

C Manual vacuum aspiration is more than 99% effective in early pregnancy (less than eight weeks). Age, parity and medical illnesses are not contraindications for manual vacuum aspiration. Although the risk of Asherman's syndrome increases with each subsequent pregnancy termination, this patient may still undergo surgical termination as long as she understands risks and benefits. Complications of pregnancy termination increase with increasing gestational age.

A 48-year-old G4P4 woman with last menstrual period four weeks ago presents with a one-year history of non-cyclical pelvic pain, dysmenorrhea and dyspareunia. She has a past history of endometriosis, diagnosed 10 years ago by laparoscopy. She had previously been on oral contraceptives for birth control and menstrual cycle regulation, but elected for permanent laparoscopic sterilization 14 months ago. Minimal endometriosis was noted at the time of laparoscopy. She now has recurrent symptoms and desires definitive treatment. Which of the following is the most appropriate surgical option for this patient? A. Hysteroscopy and dilation and curettage B. Diagnostic laparoscopy C. Hysterectomy with bilateral salpingo-oophorectomy D. Endometrial ablation E. Wedge resection of the ovaries

C It is estimated that chronic pelvic pain is the principal preoperative indication for 10-12% of hysterectomies. Since the patient had a tubal ligation and does not desire any more children, the best option is removal of ovaries with or without a hysterectomy. Repeat laparoscopy with treatment of endometriosis and adhesions can be helpful; however, the patient will continue to be at increased risk of recurrent disease. An endometrial ablation or wedge resection of ovaries alone would not be very helpful in the setting of non-cyclical pain.

A 32-year-old G3P0 woman presents to the clinic for preconception counseling. Her prior three pregnancies resulted in first trimester losses. Which of the following tests should be ordered for this patient? A. Adrenal stimulation test B. Clomiphene citrate-FSH challenge test C. Lupus anticoagulant test D. Pelvic MRI E. CT scan of the pelvis

C It is important to rule out systemic disease in a patient with recurrent abortion (three successive first trimester losses). Testing for lupus anticoagulant, diabetes mellitus and thyroid disease are commonly performed. Maternal and paternal karyotypes should also be obtained. Infectious causes should also be considered. Uterine imaging to exclude a septum or other anomaly is routinely done using hysteroscopy or hysterography and not CT or MRI scanning. There is no role for clomiphene citrate-FSH challenge in the evaluation of this patient.

A 26-year-old G0 presents to the reproductive endocrinology clinic seeking an infertility evaluation for failing to conceive after 14 months of unprotected intercourse with her boyfriend, who has fathered two other children. She works as a janitor in a nearby elementary school and currently has Medicaid for her health insurance. The physician discourages her from pursuing treatment because she will likely have to pay for her visit with cash, check or charge, and is told that treatment for infertility often involves procedures and technology that are very expensive. She is also informed that, in most states, many of these therapies are not paid by insurance or Medicaid. This situation violates which of the following ethical principles? a. Patient autonomy b. Beneficence c. Justice d. Physician autonomy e. A patient's right to privacy

C Justice requires that we treat like cases alike. It is the physician's duty to educate the patient about all her treatment options in a non-judgmental way regardless of the nature of the treatment and her socioeconomic status.

A 29-year-old G1P0 at 42 weeks gestation presents in labor. She denies ruptured membranes. Her prenatal course was complicated by chronic hypertension. Her vital signs are: blood pressure 130/80; pulse 72; afebrile; fundal height 36 cm; and estimated fetal weight of 2100 gm. Cervix is dilatedto 4 cm, 100% effaced, +1 station. The fetal heart rate tracing is shown below. What is the most likely diagnosis? A. Normal fetal heart rate pattern B. Sinusoidal rhythm C. Late deceleration D. Variable decelerations E. Early decelerations https://www.apgo.org/student/uwise2/unit1intro/unit-1?quiz_id=12

C Late decelerations are a symmetric fall in the fetal heart rate, beginning at or after the peak of the uterine contraction and returning to baseline only after the contraction has ended. Late decelerations are associated with uteroplacental insufficiency. Variable decelerations show an acute fall in the FHR with a rapid downslope and a variable recovery phase. They are characteristically variable in duration, intensity, and timing, and may not bear a constant relationship to uterine contractions. Early decelerations are physiologic caused by fetal head compression during uterine contraction, resulting in vagal stimulation and slowing of the heart rate. This type of deceleration has a uniform shape, with a slow onset that coincideswith the start of the contraction and a slow return to the baseline that coincides with the end of the contraction. Thus, it has the characteristic mirror image of the contraction. The true sinusoidal pattern is a regular, smooth, undulating form typical of a sine wave that occurs with a frequency of two to five cycles/minute and an amplitude range of five to 15 beats/minute. It is also characterized by a stable baseline heart rate of 120 to 160 beats/minute and absent beat-to-beat variability.

A 52-year-old nulliparous woman presents with long-standing vulvar and vaginal pain and burning. She has been unable to tolerate intercourse with her husband because of introital pain. She had difficulty sitting for prolonged periods of time or wearing restrictive clothing because of worsening vulvar pain. She recently noticed that her gums bleed more frequently. She avoids any topical over-the-counter therapies because they intensify her pain. Her physical examination is remarkable for inflamed gingiva and a whitish reticular skin change on her buccal mucosa. A fine papular rash is present around her wrists bilaterally. Pelvic examination reveals white plaques with intervening red erosions on the labia minora as shown in below picture. A speculum cannot be inserted into her vagina because of extensive adhesions. The cervix cannot be visualized. Which of the following is the most likely diagnosis in this patient? A. Squamous cell hyperplasia B. Lichen sclerosus C. Lichen planus D. Genital psoriasis E. Vulvar cancer

C Lichen planus is a chronic dermatologic disorder involving the hair-bearing skin and scalp, nails, oral mucous membranes and vulva. This disease manifests as inflammatory mucocutaneous eruptions characterized by remissions and flares. The exact etiology is unknown, but is thought to be multifactorial. Vulvar symptoms include irritation, burning, pruritus, contact bleeding, pain and dyspareunia. Clinical findings vary with a lacy, reticulated pattern of the labia and perineum, with or without scarring and erosions as well. With progressive adhesion formation and loss of normal architecture, the vagina can become obliterated. Patients may also experience oral lesions, alopecia and extragenital rashes. Treatment is challenging, since no single agent is universally effective and consists of multiple supportive therapies and topical superpotent corticosteroids.

The true conjugate is defined as: A. line connecting the lower edge of the pubic symphysis with the iliococcygeus joint. B. line connecting the promontory with the upper edge of the pubic symphysis. C. line connecting the promontory with the most inwardly facing part of the pubic symphysis. D. line connecting a cavity between the spinous processes of L3 and L4 with the upper edge of the pubic symphysis. E. line connecting the promontory with the lower edge of the pubic symphysis.

C. line connecting the promontory with the most inwardly facing part of the pubic symphysis.

A 24-year-old G2P1 at 42 weeks gestation presents in early labor. At amniotomy, there is thick meconium. Her sister-in-law had a procedure to dilute the meconium and she asks if this would be of benefit to her. Which of the following statements is the most accurate regarding the benefits of amnioinfusion? A. Decreases admissions to the neonatal intensive care unit B. Decreases post maturity syndrome C. Decreases repetitive variable decelerations D. Decreases the risk for Cesarean section E. Decreases meconium below the vocal cords

C Meconium staining of the amniotic fluid is three to four times more common in the postterm pregnancy. This is likely due to two reasons: 1) greater length of time in utero allows for activation of a more mature vagal system; and 2) fetal hypoxia. Amnioinfusion is a procedure where normal saline is infused into the intrauterine cavity. Routine prophylactic amnioinfusion for thick meconium does not appear to decrease the incidence of meconium aspiration syndrome or have an impact on neonatal outcomes. Based on current literature, routine prophylactic amnioinfusion for meconium-stained amniotic fluid is not recommended. Amnioinfusion remains a reasonable approach in the treatment of repetitive variable decelerations, regardless of amniotic fluid meconium status.

A 19-year-old G1 woman at 40 weeks gestation has an uncomplicated vaginal delivery followed by a brisk hemorrhage. Her past medical history is significant for steroid-dependent asthma. Her blood pressure is 110/70; pulse 84; and she is afebrile. Which of the following uterotonic agents should not be used in this patient? A. Intramuscular oxytocin B. Intravenous oxytocin C. Prostaglandin F2-alpha D. Prostaglandin E1 (Misoprostol) E. Methylergonovine

C Methergine, prostaglandins and oxytocin are all uterotonics and used to increase uterine contractions and decrease uterine bleeding. Prostaglandin F2-alpha (Hemabate) is a potent smooth muscle constrictor, which also has a bronchio-constrictive effect. As such, it should be used with caution in any patient with a reported history of asthma. It is absolutely contraindicated in patients with poorly controlled or severe asthma. Misoprostol, non-FDA approved for this purpose, is often used for cervical ripening and labor induction.

A 30-year-old G0 woman presents with her husband for preconception counseling. The patient is of Jewish Ashkenazi descent. Her husband is Irish. The patient has a brother who has a child diagnosed with attention deficit hyperactivity disorder. Which of the following genetic diseases is the most likely to affect their future children? A. Canavan disease B. Bloom syndrome C. Cystic fibrosis D. Tay-Sachs disease E. Gaucher's disease

C Non-Hispanic white individuals, including Ashkenazi Jews, are at increased risk for being carriers for cystic fibrosis. The carrier frequency is approximately 1/25 in the non-Hispanic white population. Since the patient's husband is not of Ashkenazi Jewish or French Canadian descent, he is not at increased risk for being a carrier for Tay-Sachs disease. The carrier frequency for Tay-Sachs disease is estimated at 1/30 for Ashkenazi Jews. The gene occurs at a much lower frequency (1 in 300) in most other populations. Canavan disease, Bloom syndrome and Gaucher's disease occur at an increased incidence in the Ashkenazi Jewish population. The carrier frequency for Gaucher's disease is approximately 1/15 for Ashkenazi Jews. The frequency of the disease is 1/900 in this population.

A 15-year-old adolescent discusses with her doctor that she has never had a menstrual cycle. She is healthy, active in school activities and eats a normal diet. She denies ever being sexually active. On physical examination, she has Tanner stage II breast and pubic hair growth, and average weight and height. Vaginal opening is present and appears normal. What is the most appropriate next step in her management? A. Pelvic ultrasound B. Oral contraceptive pills C. Reassurance D. MRI of sella turcica E. Cortisol challenge test

C Normal age for menarche is between nine and 17. Since this patient has secondary sexual characteristics and normal anatomy, she should be offered reassurance that she is normal and her menses will probably start soon.

A 22-year-old G2P1 is at 42 weeks gestation dated by an ultrasound performed five weeks ago. Her cervix is long and closed. She does not report contractions and states there is good fetal movement. She would like to wait until she goes into labor spontaneously. Which of the following treatment options is optimal at this time? A. Allow the patient to go into labor spontaneously B. Perform an ultrasound to determine gestational age C. Perform a non-stress test (NST) and amniotic fluid index (AFI) twice a week, with induction of labor for a nonreactive NST or oligohydramnios. D. Patient should perform daily fetal movement counts and proceed with induction for decreased fetal movement. E. Perform daily biophysical profiles and deliver if 4 or less

C Optimal management for the patient with an unfavorable cervix at an uncertain 42 weeks gestation is arguable. Given the uncertainty of her dates, it is reasonable to follow this patient with antepartum fetal testing, such as twice weekly non-stress tests with amniotic fluid index. The risk of fetal death is 1-2/1000 high-risk pregnancies with a reassuring NST, contraction stress test or biophysical profile. The addition of amniotic fluid assessment may improve the predictive value of a reactive NST and reduce the risk of antepartum fetal demise to even lower levels. Ultrasound for gestational age determination in the third trimester is not useful since the measurement error is +/- 3 weeks. Allowing spontaneous onset of labor is okay, but not without some type of antepartum testing.

Which of the following factors does not predispose to the breech presentation? A. oligohydramnios. B. polyhydramnios. C. localization of the placenta on the posterior wall of the uterus. D. hydrocephalus. E. placenta praevia.

C. localization of the placenta on the posterior wall of the uterus.

Case where patient had endocrinology issues. Hyperpigmentation a. Cushing's disease b. Addisons disease c. Premature ovarian failure

a. Cushing's disease

A 23-year-old nulliparous woman presents to the office because she has not had any menses for four months. She has a long history of irregular menstrual cycles since menarche at age 14. She is in good health and is not taking any medications. She is sexually active with her partner of six months, and uses condoms for contraception. She is 5 feet 4 inches tall and weighs 170 pounds. On exam, she has noticeable hair growth on her upper lip and chin. The rest of her examination including a pelvic exam is normal. Her Beta-hCG is < 5 mIU/mL, and her prolactin and TSH levels are normal. In addition to recommending weight loss, what is the most appropriate next step in the management of this patient? A. Treatment with gonadotropin releasing hormone level (GnRH) agonist B. Treatment with clomiphene citrate C. Treatment with oral contraceptives D. Check progesterone levels E. Check cortisol levels

C Oral contraceptives (OCPs) are the most appropriate treatment for this patient who most likely has the diagnosis of polycystic ovarian syndrome (PCOS). The constellation of findings support this clinical diagnosis (irregular cycles, obesity, and hirsutism). Because she is using condoms for contraception and is sexually active, OCPs would help regulate her cycles and further provide effective contraception. When she desires pregnancy, however, she will most likely need treatment for ovulation induction due to the anovulatory cycles as the leading cause of her oligomenorrhea. Clomiphene citrate is not indicated at this time.

A 26-year-old G0 woman returns for a follow-up visit regarding endometriosis. She has been using NSAIDs to manage her pelvic pain, but had to miss four days of work in the last two months. She is sexually active with her husband of two years, although it has been more painful recently. She has regular menstrual cycles and is using condoms for contraception. On pelvic exam, she has localized tenderness in the cul de sac and there were no palpable masses. What is the most appropriate next step in the management of this patient? A. GnRH agonist B. Danazol C. Oral contraceptives D. Laparoscopy and ablation of endometriosis E. Progesterone intrauterine device

C Oral contraceptives will be the next best choice for this patient. They provide negative feedback to the pituitary-hypothalamic axis which stops stimulation of the ovary to produce sex hormones, such as estrogen, which stimulates endometrial tissue located outside of the endometrium and uterus. GnRH agonists also exert negative feedback, but can be used short term only and have more side effects. Danazol is a synthetic androgen used to treat endometriosis, but due to its androgenic side effects (weight gain, increased body hair and acne, and adverse affect on blood lipid levels) it is not usually the first choice of treatment. Laparoscopy is indicated in the patient who fails medical treatment and/or is planning pregnancy in the near future. A progesterone intrauterine device might potentially help alleviate some of her symptoms but is not the best management for endometriosis.

A 76-year-old G3P3 woman presents to your office with worsening urinary incontinence for the past three months. She reports increased urinary frequency, urgency and nocturia. Her exam shows mild cystocele and rectocele. A urine culture is negative. A post-void residual is 400 cc. Which of the following is the most likely diagnosis in this patient? A. Genuine stress incontinence B. Detrusor instability C. Overflow incontinence D. Functional incontinence E. Mixed incontinence

C Overflow incontinence is characterized by failure to empty the bladder adequately. This is due to an underactive detrusor muscle (neurologic disorders, diabetes or multiple sclerosis) or obstruction (postoperative or severe prolapse). A normal post-void residual (PVR) is 50-60 cc. An elevated PVR, usually >300 cc, is found in overflow incontinence. Stress incontinence occurs when the bladder pressure is greater than the intraurethral pressure. Overactive detrusor contractions can override the urethral pressure resulting in urine leakage. The mixed variety includes symptoms related to stress incontinence and urge incontinence.

A 68-year-old G2P2 woman who has recently moved in with her daughter (a long-standing patient of yours) comes in for a health maintenance examination. A vaginal hysterectomy was done in her fifties for uterine prolapse. She is not sure if her ovaries were removed. She has never had an abnormal mammogram or Pap smear and has had yearly exams. She stopped hormone replacement therapy 10 years ago. She was recently widowed after being married for 50 years. She does not smoke or drink. Her diabetes is well-controlled with Metformin; she takes a daily baby aspirin and is on a lipid-lowering agent. On examination, she is a thin elderly woman with a dowager's hump. Her breast exam is unremarkable. Her lower genital tract is notable for atrophy. No masses are noted on bimanual and recto-vaginal exam. A fecal occult blood test is negative. Which of the following tests is not necessary? A. Bone density B. Colonoscopy C. Pap smear D. Mammogram E. Annual bimanual and recto-vaginal exam

C Pap smear screening is not indicated in patients who have had a hysterectomy, unless it was done for cervical cancer or a high-grade cervical dyspalsia. Patients with a uterus can discontinue cervical cancer screening between the ages of 65 - 70 if they have had three consecutive negative smears or two negative consecutive cotesting in the last 10 years and no history of high-grade cervical intraepithelial neoplasia or cancer. Patients still need yearly bimanual and rectovaginal exam. Mammograms are done annually, as breast cancer increases with age. Colon cancer screening is recommended at age fifty. The patient has an exaggerated thoracic spine curvature, termed a dowager's hump, likely secondary to thoracic compression fractures secondary to osteoporosis. If this is confirmed on a bone density test, she may benefit from the addition of bisphosphonates.

A 25-year-old G3P2, who had recently undergone a primary Cesarean section, had her HIV status revealed to her mother when a nurse left her chart open in the recovery room. She speaks to patient relations and is thinking about seeking damages through legal avenues. When trying to explain the concept of patient privacy, which of the following statements is correct? a. Patient privacy is based on the ethical principle of justice b. Patient privacy is protected by federal law, primarily with the Federal Emergency Medical Treatment and Labor Act (EMTALA) statute c. Patient privacy is the responsibility of physicians; physicians may be fined and/or assessed criminal penalties for violating the privacy of a patient's protected health information d. The patient cannot win a lawsuit in this case because the mother should not have looked at the open record e. Patient privacy is based on the ethical principal of beneficence

C Patient privacy is the responsibility of physicians. Physicians may be fined and/or assessed criminal penalties for violating the privacy of a patient's protected health information. It was the responsibility of the physicians and the other health care providers in this case to make sure the chart is not left open so someone walking by sees the information.

Postmenopausal woman, bleeding from vagina?

a. Endometrial biopsy

Case where patient got pain and fever 72 hours after cesarean section?

a. Endometritis

A 65-year-old G3P3 woman presents with symptoms of vaginal pressure and heaviness, which seem to worsen towards the end of the day. She has a history of three vaginal deliveries. Her surgical history is significant for hysterectomy for abnormal vaginal bleeding at age 45. On exam, she is found to have a large pelvic prolapse. Which of the following is the most appropriate initial treatment of this patient's prolapse? A. Sacrospinous ligament suspension B. Transvaginal tape C. Pessary fitting D. Anterior repair E. Topical vaginal estrogen

C Pessary fitting is the least invasive intervention for this patient's symptomatic prolapse. Although a sacrospinous ligament suspension would be an appropriate procedure for this patient, it is invasive and not an appropriate first step. Transvaginal tape is used for urinary incontinence and has no role in the management of this patient. An anterior repair can potentially help with her symptoms, depending on what is contributing most to her prolapse but, again, it is invasive. Topical estrogen is unlikely to properly treat her prolapse and related symptoms.

"A 34-year-old G3P1 woman at 26 weeks gestation reports ""difficulty catching her breath,"" especially after exertion for the last two months. She is a non-smoker. She does not have any history of pulmonary or cardiac disease. She denies fever, sputum, cough or any recent illnesses. On physical examination, her vital signs are: blood pressure 108/64, pulse 88, respiratory rate 15, and she is afebrile. Pulse oximeter is 98% on room air. Lungs are clear to auscultation. Heart is regular rate and rhythm with II/VI systolic murmur heard at the upper left sternal border. She has no lower extremity edema. A complete blood count reveals a hemoglobin of 10.0 g/dL. What is the most likely explanation for this woman's symptoms? A. Pulmonary embolism B. Mitral valve stenosis C. Physiologic dyspnea of pregnancy D. Peripartum cardiomyopathy E. Anemia"

C Physical examination findings are not consistent with pulmonary embolus (e.g tachycardia, tachypnea, hypoxia, chest pain, signs of a DVT) or mitral stenosis (diastolic murmur, signs of heart failure). Physiologic dyspnea of pregnancy is present in up to 75% of women by the third trimester. Peripartum cardiomyopathy is an idiopathic cardiomyopathy that presents with heart failure secondary to left ventricular systolic function towards the end of pregnancy or in the several months following delivery. Symptoms include fatigue, shortness of breath, palpitations, and edema. The history and physical do not suggest a pathologic process, nor does her hemoglobin level.

28-year-old G3P2 woman presents in labor at 39 weeks gestation and delivers a 3500 gram infant spontaneously after oxytocin augmentation of labor. Thirty minutes later, the placenta has not delivered. Her past medical history is significant for leiomyoma uteri. Her prenatal course was uncomplicated. What is the most likely risk factor for retained placenta in this case? A. Placental abruption B. Labor augmentation C. Leiomyomas D. Multiparity E. Circumvallate placenta

C Placental abruptions, labor augmentation, degree of parity and circumvallate placenta have no impact on the risk of retained placenta. The following are associated with retained placenta: prior Cesarean delivery, uterine leiomyomas, prior uterine curettage and succenturiate lobe of placenta.

A 45-year-old G2P2 woman presents for a six-week post partum check. She reports crying spells, loss of appetite, difficulty sleeping and a feeling of low self-worth that began one week after her delivery. She denies any suicidal or homicidal ideations. She is frustrated because she has not been able to breastfeed and feels that she is a bad mother. She has a previous history of anxiety. Which of the following is the most likely diagnosis in this patient? A. Normal puerperium B. Postpartum blues C. Postpartum depression D. Anxiety disorder E. Bipolar disorder

C Postpartum depression is a common condition estimated to affect approximately 10-15% of women and often begins within two weeks to six months after delivery. Signs and symptoms of depression which last for less than two weeks are called postpartum blues. It occurs in 40-85% of women in the immediate postpartum period. It is a mild disorder that is usually self-limited. This patient does not have signs/symptoms of anxiety disorder or bipolar disorder.

A 28-year-old G3P3 woman status post an uncomplicated spontaneous vaginal delivery of 4150 gram infant experiences profuse vaginal bleeding of 700 cc. Prior obstetric history was notable for a previous low uterine segment transverse Cesarean section, secondary to transverse fetal lie. The patient had no antenatal problems. The placenta delivered spontaneously without difficulty. Which of the following is the most likely cause of this patient's hemorrhage? A. Vaginal lacerations B. Cervical lacerations C. Uterine atony D. Uterine dehiscence E. Uterine rupture

C Postpartum hemorrhage (PPH) is an obstetrical emergency that can follow vaginal or Cesarean delivery. Uterine atony is the most common cause of PPH and occurs in one in every twenty deliveries. It is important to detect excessive bleeding quickly and determine an etiology and initiate the appropriate treatment as excessive bleeding may result in hypovolemia, with associated hypotension, tachycardia or oliguria. The most common definition of PPH is an estimated blood loss of greater than or equal to 500 ml after vaginal birth, or greater than or equal to 1000 ml after Cesarean delivery.

A 38-year-old G4P2 woman was diagnosed with triplets when an ultrasound was performed at 12 weeks gestational age. Which of the following is the most concerning complication for this multiple gestation? A. Preeclampsia B. Intrauterine growth restriction C. Preterm birth D. Gestational diabetes E. Abnormal placentation

C Preterm delivery increases the risk of morbidity and mortality, increasing with higher orders of multiples. Preterm birth occurs in over 50% of twin pregnancies, 90% of triplet pregnancies, and almost all quadruplet pregnancies. While all the choices may occur with a multiple gestation, prematurity has the most significant consequences as it is associated with an increased risk of respiratory distress syndrome (RDS), intracranial hemorrhage, cerebral palsy, blindness, and low birth weight. Intrauterine growth restriction, intrauterine death of one or more fetuses, miscarriage and congenital anomalies are all more common with multiple gestations, as are the complications of preeclampsia, diabetes and placental abnormalities.

A 17-year-old G0 was kissing her 18-year-old boyfriend in a parked car. She refused to have sexual intercourse and he overpowered her, hit her on the face and raped her. He did not use a condom and ejaculated inside her. Which of the following made this case a rape? A. Failure to use effective barrier contraception B. Sexual intercourse C. Lack of consent D. Legal age of the woman E. Legal age of the boyfriend

C Rape is characterized by lack of consent or inability to give consent. Most definitions include the use of physical force, deception, intimidation or the threat of bodily harm.

Picture with huge mass protruding out of vagina of old patient. Previously done hysterectomy?

a. Enterocele

A 19-year-old G1P0 woman with a desired pregnancy notes vaginal spotting early this morning and it has slightly increased. Her last normal menstrual period occurred six weeks ago. She has no pain or other symptoms. Her medical history is noncontributory. Vital signs are: blood pressure 120/68; pulse 68; respirations 20; and temperature 98.6°F (37.0°C). On pelvic exam, her cervix is normal; her uterus is small and nontender; there are no masses palpable. Labs show Quantitative Beta-hCG 750 mIU/ml; progesterone 3.8 ng/ml; hematocrit 38%. Which of the following is the most appropriate next step in the management of this patient? A. Order a transvaginal ultrasound B. Repeat Beta-hCG level in 24 hours C. Repeat Beta-hCG level in 48 hours D. Dilation and curettage E. Bed rest

C Repeating the Beta-hCG level will show whether the pregnancy is viable or failing. The appropriate time interval for repeating the initial level is 48 hours, since during the first 42 days of gestation levels increase by approximately 50% every 48 hours in most viable pregnancies. Ordering an ultrasound would not be helpful, since the patient's Beta-hCG level is lower than the discriminatory zone (the level at which an intrauterine pregnancy should be seen on ultrasound, usually 2000 mIU/ml). There is no need to repeat the progesterone level. Dilation and curettage or treatment with methotrexate are both inappropriate without a diagnosis, since both could interrupt a viable pregnancy.

A 22 year-old delivered her first baby five days ago after a prolonged labor and subsequent Cesarean section for arrest of cervical dilation at 7 centimeters. Fever was noted on postoperative day 2 and, despite broad spectrum antibiotics, she continues to have temperature spikes above 101.3°F, 38.5°C. She is eating a normal diet and ambulating normally. On physical examination, her breasts have no erythema and nipples are intact. Her abdomen is soft, uterine fundus is firm and nontender, and her incision is healing without induration or erythema. She has normal lochia and her urinalysis is normal. Pelvic examination reveals a firm nontender uterus and no adnexal masses or tenderness. Which of the following is the most likely cause of her fevers? A. Endometritis B. Mastitis C. Septic pelvic thrombophlebitis D. Cystitis E. Ovarian abscess

C Septic thrombophlebitis involves thrombosis of the venous system of the pelvis. Diagnosis is often one of exclusion of other causes, but sometimes a CT scan will reveal thrombosed veins. Treatment requires addition of anticoagulation to antibiotics and resolution of fevers is rapid. Anticoagulation treatment is short-term. Classic clinical findings for endometritis include fever and maternal tachycardia, uterine tenderness and no other localizing signs of infection. The clinical manifestations of cystitis include lower abdominal pain, frequency, urgency and dysuria. The clinical findings in patients with mastitis include fever, tenderness, induration and erythema of the affected breast.

A 38-year-old G0 presents for a preconception evaluation. She has a history of long-time anxiety and depression, and is interested in continuing her medications in pregnancy, which includes sertraline (Zoloft). In what FDA pregnancy category is this drug? A. Category A B. Category B C. Category C D. Category D E. Category X

C Sertraline is a Category C drug. Depression is more common in women than men. Appropriate treatment, including during the antepartum period, is a component of good medical care. As in all cases, when considering treatments, the benefits should outweigh the risks. With Category A drugs, there are adequate, well-controlled studies in pregnant women that have not shown an increased risk of fetal abnormalities to the fetus in any trimester of pregnancy. With Category B, animal studies have revealed no evidence of harm to the fetus; however, there are no adequate and well-controlled studies in pregnant women or animal studies that have shown an adverse effect, but adequate and well-controlled studies in pregnant women have failed to demonstrate a risk to the fetus in any trimester. Category C drugs have animal studies that show an adverse effect and there are no adequate and well-controlled studies in humans, but potential benefits may warrant use of the drug in pregnant women despite potential risks. Category D drugs have adequate well-controlled or observational studies in pregnant women and are known risks to the fetus. Category X drugs should not be used in pregnancy, because adequate well-controlled or observational studies in animals or pregnant women have demonstrated positive evidence of fetal abnormalities or risks.

A 21-year-old G1P1 woman presents to the office with amenorrhea since the birth of her one-year old daughter. She reports extreme fatigue, forgetfulness, and depression. She was unable to breastfeed because her milk never came in. She notes hair loss including under her arms and in her pubic area. Her delivery was complicated by a postpartum hemorrhage, hypovolemic shock, requiring aggressive resuscitation. She is afebrile. Vital signs are: blood pressure 90/50; pulse 84. The patient appears tired. Her exam is normal but she is noted to have dry skin. A urine pregnancy test is negative. Which of the following is the most likely diagnosis in this patient? A. Hyperprolactinemia B. Hyperthyroidism C. Sheehan Syndrome D. Asherman Syndrome E. Major depressive disorder

C Sheehan Syndrome is a rare occurrence. When a patient experiences a significant blood loss, this can result in anterior pituitary necrosis, which may lead to loss of gonadotropin, thyroid-stimulating hormone (TSH) and adrenocorticotropic hormone (ACTH) production, as they are all produced by the anterior pituitary. Signs and symptoms of Sheehan syndrome may include slow mental function, weight gain, fatigue, difficulty staying warm, no milk production, hypotension and amenorrhea. Sheehan's syndrome frequently goes unnoticed for many years after the inciting delivery. Treatment includes estrogen and progesterone replacement and supplementation with thyroid and adrenal hormones.

A 38-year-old G1P1 woman who delivered by a Cesarean section three weeks ago presents to the clinic with concerns that the baby is not getting enough milk and is fussy. She reports that she is feeding on demand and not supplementing. Which of the following is indicative that the baby is getting adequate milk? A. Sleeps through the night B. Spits up a small amount of milk after feeding C. 3-4 stools in 24 hours D. 3-4 diapers wet with urine in 24 hours E. Coverage of the entire areola with his mouth when he breastfeeds

C Signs that a baby is getting sufficient milk include 3-4 stools in 24 hours, six wet diapers in 24 hours, weight gain and sounds of swallowing.

"A 17-year-old G1P0 woman at 32 weeks gestation complains of right flank pain that is ""colicky"" in nature and has been present for two weeks. She denies fever, dysuria and hematuria. Physical examination is notable for moderate right costovertebral angle tenderness. White blood cell count 8,800/mL, urine analysis negative. A renal ultrasound reveals no signs of urinary calculi, but there is moderate (15 mm) right hydronephrosis. Which of the following is the most likely cause of these findings? A. Smooth muscle relaxation due to declining levels of progesterone B. Smooth muscle relaxation due to increasing levels of estrogen C. Compression by the uterus and right ovarian vein D. Elevation of the bladder in the second trimester E. Iliac artery compression of the ureter"

C Some degree of dilation in the ureters and renal pelvis occurs in the majority of pregnant women. The dilation is unequal (R > L) due to cushioning provided by the sigmoid colon to the left ureter and from greater compression of the right ureter due to dextrorotation of the uterus. The right ovarian vein complex, which is remarkably dilated during pregnancy, lies obliquely over the right ureter and may contribute significantly to right ureteral dilatation. High levels of progesterone likely have some effect but estrogen has no effect on the smooth muscle of the ureter.

A 27-year-old G2P0 woman is diagnosed with an early first trimester spontaneous abortion. She has a history of type I diabetes mellitus, mild chronic hypertension and one prior termination of pregnancy. Which of the following is the most likely cause of this spontaneous abortion? A. Prior termination of pregnancy B. Chronic hypertension C. Diabetes mellitus D. Intrauterine adhesions E. Infection

C Systemic diseases such as diabetes mellitus, chronic renal disease and lupus are associated with early pregnancy loss. In women with insulin-dependent diabetes, the rates of spontaneous abortion and major congenital malformations are both increased. The risk appears related to the degree of metabolic control in the first trimester. There are many other causes of spontaneous abortion, including genetic factors, endocrine abnormalities, reproductive tract abnormalities, immunologic factors and environmental factors. The patient's history of mild chronic hypertension and one prior termination of pregnancy do not increase her risk of a first trimester loss. Additionally, an uncomplicated termination of pregnancy, intrauterine adhesions and infection are not likely causes in this scenario.

A 68-year-old woman with a history of breast cancer presents for evaluation of endometrial cancer risk. She was treated with lumpectomy and axillary node dissection and radiation therapy. She has been on tamoxifen therapy for the past year. She denies any vaginal bleeding or discharge. She is 5 feet 3 inches tall and weighs 140 pounds. Her pelvic examination is notable only for severe vulvovaginal atrophy. What is the next best step in the management of this patient? A. Endometrial biopsy now to obtain a baseline B. Annual endometrial biopsy C. Annual exams D. Annual pelvic ultrasound E. Endometrial biopsy upon completion of five years of tamoxifen therapy

C Tamoxifen is known to increase the risk of endometrial cancer. However, diagnostic studies, such as endometrial biopsy, are reserved for when the patient develops symptoms of bleeding or abnormal vaginal discharge. Ultrasound is not helpful because Tamoxifen is known to cause changes to the endometrium, including thickening. Endometrial biopsy is not indicated as a screening tool for endometrial cancer.

A 35-year-old G1 woman at 30-weeks gestation is transferred from an outside hospital in preterm labor. Her cervix is 3 cm dilated, 50% effaced and the vertex is at 0 station. She is having contractions every five minutes and has no signs consistent with an intra-amniotic infection (chorioamnionitis). She was initially treated with terbutaline prior to her transfer. Which of the following side effects would you expect? A. Premature constriction of the ductus arteriosus B. Respiratory depression C. Tachycardia D. Tachypnea E. Headache

C Terbutaline is a beta-adrenergic agent. Side effects include tachycardia, hypotension, anxiety and chest tightening or pain. Tachypnea and headaches are not usual side effects. The FDA made a formal announcement in 2011 warning against using terbutaline to stop preterm labor stating that terbutaline is both ineffective and dangerous if used for longer than 48 hours. The drug may still be used on a short-term basis in patients with active contractions, such as those being transferred to another hospital for tertiary care. Alternative tocolytic agents should be used for longer term treatment of preterm labor.

A 33-year-old G2P1 woman with a singleton pregnancy at 10 weeks gestation asks about weight gain during pregnancy. Her body mass index is 42. What is the recommended weight gain for this patient? A. Maintain current weight; no weight gain B. 10 pounds for the entire pregnancy C. 11-20 pounds for the entire pregnancy d. 25-35 pounds for the entire the pregnancy e. Maintain current weight or lose up to 10 pounds as long as the fetal growth is appropriate

C The Institute of Medicine (2009 guidelines) and the 2013 ACOG Committee Opinion recommend that obese women with a singleton pregnancy gain 11-20 pounds. The majority of weight gain occurs in the second half of pregnancy.

A representative from a domestic violence outreach program asks for your help in distributing information, to include a hot line to call, if needed. Where is the best place for this information to be made available? A. Office waiting room B. Distribution to all patients at time of check in C. Office restroom D. Letters sent to all your patients E. Posters in front of your office

C The best place to have literature is where there is the most privacy such as an office restroom. In the other areas cited, an abuser may see the information and prevent his/her spouse or partner from obtaining it.

A 49-year-old G5P5 woman presents for her first health maintenance examination since she had her last child 10 years ago. She has no health complaints. She has had two sexual partners. She smokes three to five cigarettes per day, and has been smoking for the past 15 years. Last month, her mother underwent a radical hysterectomy for Stage 1B cervical carcinoma. Her pelvic examination is normal, except for mucopurulent discharge and vaginal condyloma. Which of the following is the patient's greatest risk for developing cervical cancer? A. Family history of cervical cancer B. Smoking history C. Vaginal condyloma D. Multiparity E. Age

C The majority of risk factors for cervical cancer are related to HPV exposure and include early-onset sexual activity, multiple sexual partners, a sexual partner with multiple partners, history of HPV or other sexually transmitted diseases, immunosuppression, smoking, low socioeconomic status and a lack of regular Pap smears. In this patient with multiple risk factors, the presence of an HPV-related condition (vaginal condyloma) already indicates infection with HPV. Although the HPV type associated with condyloma is typically a low risk strain (e.g. types 6 and 11), she is also at risk of having been exposed to high-risk types that are typically associated with high-grade dysplasia and cervical cancer (e.g. types 16 and 18).

A 25-year-old G1P0 woman is seen for an initial obstetrical appointment at eight weeks gestation. She has had a small ventricular septal defect (VSD) since birth. She has no surgical history and no limitations on her activity. Vital signs are: respiratory rate 12; heart rate 88; blood pressure 112/68. On physical examination: her skin appears normal; lungs are clear to auscultation; heart is a regular rate and rhythm. There is a grade IV/VI coarse pansystolic murmur at the left sternal border, with a thrill. Chest x-ray and ECG are normal. Which of the following is the correct statement regarding cardiovascular adaptation in this patient? A. Approximately 2% of women will normally have a diastolic murmur B. Maternal pulmonary vascular resistance is normally less than systemic vascular resistance C. The maternal cardiac output will increase up to 33% during pregnancy D. Maternal systemic vascular resistance increases throughout pregnancy E. The increase in cardiac output is only due to the increase in the maternal stroke volume

C The cardiac output increases up to 33% due to increases in both the heart rate and stroke volume. The SVR falls during pregnancy. Up to 95% of women will have a systolic murmur due to the increased volume. Diastolic murmurs are always abnormal. The systemic vascular resistance (SVR) is normally greater than the pulmonary vascular resistance. If the pulmonary vascular resistance exceeds the SVR, right to left shunt will develop in the setting of a VSD, and cyanosis will develop.

"A 12-year-old girl is brought to the office by her mother who complains that her daughter has never been interested in dolls and pretty dresses, but prefers to play with tools and mechanical things. The mother also divulges that her brother is gay and is worried that her daughter will grow up as a lesbian and be stigmatized. A private conversation with the girl reveals that she is starting to show an interest in boys, and even has a ""boyfriend."" Examination reveals a normal pre-pubertal phenotype. Which of the following is the most appropriate course of action at this time? A. Encourage mother to support more gender-appropriate activities including dressing like and playing with other girls B. Refer the mother to a family counselor C. Reassure the mother that her daughter's behavior is normal D. Inform mother that her daughter may ultimately develop an alternative sexual lifestyle E. Have the patient return for a repeat assessment once she has begun menarche"

C The division of interests based on gender is inappropriate. Children should be encouraged to follow their own interests and desires. The girl has a normal physical exam; therefore, diagnostic studies are not indicated.

A 35-year-old G1 woman with an IVF conceived 12 weeks gestation has a slightly elevated fetal nuchal translucency (2.5 multiples of the median), but her integrated first trimester screen shows no increased risk for Down syndrome or Trisomy 18. Still concerned about the increased nuchal translucency, the patient requests non-invasive testing to exclude other abnormalities. Which of the following is the next best step in the management of this patient? A. Reassurance B. Monthly ultrasound to assess for fetal growth C. Detailed ultrasound and fetal echocardiogram at approximately 18 - 20 weeks gestation D. Repeat first trimester screening E. Amniocentesis

C The first trimester screen alone yields an 85% detection rate. The NT is the measurement of the fluid collection at the back of the fetal neck in the first trimester. A thickened NT may be associated with fetal chromosomal and structural abnormalities as well as a number of genetic syndromes. Patients who desire non-invasive assessment of their risk for aneuploidy can have first trimester screen (a fetal nuchal translucency (NT) measurement and a maternal serum PAPP-A) and a second trimester quadruple screen. The sequential screen yields a 95% detection rate for Down syndrome at a 5% false-positive rate. Since the fetus in this case had a thickened NT, this patient should be scheduled to have a detailed fetal ultrasound and echocardiogram at 18-20 weeks to rule out anomalies. Amniocentesis would detect other chromosomal abnormalities, but is an invasive test. Of note, the American Congress of Obstetrics and Gynecology (ACOG) recommends that all patients be offered aneuploidy screening and invasive prenatal diagnosis as indicated.

An obese 30-year-old G3P1 Asian woman undergoes an uncomplicated dilation and curettage for a first-trimester miscarriage. Pathology reveals a molar pregnancy. The patient's medical history is significant for chronic hypertension. She has a history of a previous uncomplicated term pregnancy, and termination of a pregnancy at 16 weeks gestation for trisomy 18. What aspect of the patient's history places her at increased risk for a molar pregnancy? A. Obesity B. Previous history of fetal aneuploidy C. Asian race D. Chronic hypertension E. Prior term pregnancy

C The incidence of molar pregnancy is approximately 1 per 1,500-2,000 pregnancies among Caucasians in the United States. There is a much higher incidence among Asian women in the United States (1/800.) Molar pregnancy occurs more frequently in women less than 20 or older than 40 years of age. The incidence is higher in areas where people consume less beta-carotene and folic acid. There is no known association between molar pregnancy and obesity, a previous history of fetal aneuploidy, chronic hypertension and parity. The risk of having a molar pregnancy is increased in women with two or more miscarriages.

A 23-year-old G0 with severe dysmenorrhea that is unresponsive to non-steroidal anti-inflammatory agents and oral contraceptives is taken to the operating room for a laparoscopy. Blue-black powder burn lesions are seen in the pelvis. A biopsy is performed and sent to pathology. Which of the following pathologic lesions would you expect to see in this patient? A. Blue-domed cysts greater than 3 mm B. Decidual effect in the endometrium C. Endometrial glands/stroma and hemosiderin-laden macrophages D. Invasion of endometrial glands into the myometrium E. Well-circumscribed, non-encapsulated myometrium

C The lesions described are classic for endometriosis. One would therefore expect to see endometrial glands/stroma with hemosiderin-laden macrophages.

A 23-year-old G1P1 woman develops a fever on the third day after an uncomplicated Cesarean delivery that was performed secondary to arrest of descent. The only significant finding on physical exam is moderate breast engorgement and mild uterine fundal tenderness. What is the most likely diagnosis in this patient? A. Urinary tract infection B. Mastitis C. Endometritis D. Wound cellulitis E. Septic pelvic thrombophlebitis

C The most common cause of postpartum fever is endometritis. The differential diagnosis includes urinary tract infection, lower genital tract infection, wound infections, pulmonary infections, thrombophlebitis, and mastitis. Endometritis appearing in a postpartum period is most closely related to the mode of delivery and occurs after vaginal delivery in approximately 2 percent of patients and after Cesarean delivery in about 10 to 15 percent. Factors related to increased rates of infection with a vaginal birth include prolonged labor, prolonged rupture of membranes, multiple vaginal examinations, internal fetal monitoring, removal of the placenta manually and low socioeconomic status. Uterine fundal tenderness is commonly observed in patients with endometritis.

"An 88-year-old G2P2 nursing home resident is brought in for evaluation of blood found in her diapers. She has a long-standing history of incontinence. This is the first time that her caregivers have noted blood. They describe it as ""quarter size."" Her nurses think that she may have been itching, as they frequently find her scratching through the diaper. On review of her medical record, biopsy-documented lichen sclerosus of the vulva was diagnosed fifteen years ago. She has not been on any therapy for this condition for years. Examination of the external genitalia reveals an elevated, firm irregular lesion arising from the left labia. The lesion measures 2.5 cm in greatest dimension. The remainder of the external genitalia shows evidence of excoriation of thin, white skin with a wrinkled parchment appearance. The vagina and cervix are atrophic. No masses are noted on bimanual or rectovaginal exam and a sample of her stool is negative for blood. No nodularity is noted in her groin. Which of the following is the most appropriate next step in the management of this patient? A. Begin steroids B. Obtain cultures of the lesion C. Biopsy the lesion D. Perform a Pap smear (cytology) of the lesion E. Complete vulvectomy and lymph node dissection"

C The most important step is to first biopsy the lesion. It would be inappropriate to treat the lichen sclerosus first, as the lesion is suspicious for malignancy. Diagnostic studies such as cultures and cytology of such a lesion are not appropriate given the exophytic, nodular lesion seen on examination. A biopsy should be performed to make a definitive diagnosis and rule out malignancy. It would also be inappropriate to treat the patient with a vulvectomy and lymph node dissection before obtaining a clear diagnosis.

30-year-old G1P1 woman who underwent an urgent vacuum extraction of a baby girl two months ago is experiencing persistent depressive symptoms suggestive of postpartum depression. She is recently divorced and has no immediate family or close friends. She works as a mechanic in a local garage and is planning on going back to school. She contemplated terminating the pregnancy but ultimately decided to have the baby despite no support from her ex-husband. She has a history of depression in the past but has not required any medications for the last three years. Which of the following is her most significant risk factor for postpartum depression? A. Single parent B. Consideration to terminate the pregnancy C. Personal history of depression D. Urgent delivery E. Social isolation

C The most significant risk factor for developing postpartum depression is the patient's prior history of depression. Other risk factors for postpartum depression include marital conflict, lack of perceived social support from family and friends, having contemplated terminating the current pregnancy, stressful life events in the previous twelve months, and a sick leave in the past twelve months related to hyperemesis, uterine irritability or psychiatric disorder.

A 72-year-old G3P2 postmenopausal woman is referred by her internist after work-up for abdominal bloating revealed a large pelvic mass on transvaginal ultrasound and an elevated CA-125. The patient had a normal colonoscopy and mammography two months ago. The patient's greatest complaint is early satiety and upper abdominal discomfort. Her physical exam is notable for moderate abdominal distension and a significant fluid wave. Which of the following tests would be most helpful in assessing the extent of disease? A. Barium enema B. PET scan C. CT scan of abdomen and pelvis D. Chest X-ray E. Intravenous pyelogram

C The most useful radiologic tool for evaluating the entire peritoneal cavity and the retroperitoneum is computerized tomography. Specifically in this patient, it would be important to look for significant involvement of the omentum. A chest x-ray provides adequate evaluation of the chest, unless it is abnormal. If there is a suspicion for chest involvement on the chest film, then a chest CT is necessary. With a normal colonoscopy and no symptoms suggestive of colonic obstruction, a barium enema would not be useful. PET scan, to date, has not been shown to play a role in the initial evaluation of women with a suspected ovarian malignancy. However, PET scan may play a role in evaluating women with a known diagnosis of ovarian cancer who have a suspected recurrence. An IVP would be useful if there was suspected ureteral obstruction, but otherwise is quite limited in assessing the entire abdominal/pelvic cavity.

"A 47-year-old G2P2 comes to see you because she is concerned that she has uterine fibroids, as she recently gained about 20 pounds around her waist. Her mother had a hysterectomy for large fibroids that ""made her look like she was 40 weeks pregnant."" She has smoked one pack of cigarettes a day for the last 35 years. She reports normal menstrual cycles. Her weight is 216 pounds and she is 5 feet 4 inches tall. Her exam is extremely limited by her body habitus. A beta-hCG is negative. A pelvic ultrasound shows a 4 cm intramural fibroid. What is the next best step in the management of this patient? A. Obtain a pelvic MRI B. Perform laparoscopic myomectomy C. Counsel her on diet and exercise D. Perform a hysterectomy E. Recommend bariatric surgery"

C The mostly likely cause of this patient's weight gain is excessive dietary intake and lack of exercise. She should be counseled on healthy habits and quitting smoking. The treatment of asymptomatic relatively small fibroid is not indicated. She does not qualify for bariatric surgery based on her BMI.

"A 23-year-old G0 woman comes to the office to discuss contraception. Her past medical history is remarkable for hypothyroidism and mild hypertension. She has a history of slightly irregular menses. Her best friend recently got a ""patch,"" so she is interested in using a transdermal system (patch). Her vital signs are: blood pressure 130/84; weight 210 pounds; height 5 feet 4 inches. What is the most compelling reason for her to use a different method of contraception? A. Age B. Hypothyroidism C. Weight D. Unpredictable periods E. Hypertension"

C The patch has comparable efficiency to the pill in comparative clinical trials, although it has more consistent use. It has a significantly higher failure rate when used in women who weigh more than 198 pounds. The patch is a transdermal system that is placed on a woman's upper arm or torso (except breasts). The patch (Ortho Evra) slowly releases ethinyl estradiol and norelgestromin, which establishes steady serum levels for seven days. A woman should apply one patch in a different area each week for three weeks, then have a patch-free week, during which time she will have a withdrawal bleed.

A 30-year-old G3P2 woman, whose last normal menstrual period was eight weeks ago, began spotting three days ago and developed cramping this morning. She has a history of a chlamydia infection with a previous pregnancy. She smokes one pack of cigarettes per day and denies alcohol or drug use. On physical exam: blood pressure 120/70; pulse 82; respirations 20; and temperature 98.6°F (37.0°C). Abdominal examination is normal. Pelvic examination reveals old blood in the vaginal vault, closed cervix without lesions, slightly enlarged uterus and no adnexal tenderness. Pertinent labs: Quantitative Beta-hCG is 1000 mIU/ml; urinalysis normal; hematocrit = 32%. Transvaginal ultrasound shows no intrauterine pregnancy, no adnexal masses, no free fluid in pelvis. Which of the following is the most appropriate next step in the management of this patient? A. Treat with methotrexate B. Exploratory surgery C. Repeat Beta-hCG in 48 hours D. Repeat Beta-hCG in one week E. Admit the patient to the hospital for observation

C The patient first needs to have an accurate diagnosis before a treatment plan is entertained. She has risk factors for ectopic pregnancy. Repeating the Beta-hCG is the next step in this patient's management. Inappropriately rising Beta-hCG levels (less than 50% increase in 48 hours) or levels that either do not fall following diagnostic dilation and curettage would be consistent with the diagnosis of ectopic pregnancy. Alternatively, a fetal pole must be visualized outside the uterus on ultrasound. The patient would need a Beta-hCG level over the discriminatory zone (the level where an intrauterine pregnancy can be seen on ultrasound) with an empty uterus. The level commonly used is 2000 mIU/ml. Treatment with methotrexate may be appropriate, but only after a definitive diagnosis is made. The patient does not yet have this level and is stable. She is, therefore, not a candidate for exploratory surgery. If she had unstable vital signs or an acute abdomen, a diagnostic laparoscopy/laparotomy would be indicated. Repeating the ultrasound in one week is not recommended because a delay in diagnosis could result in a ruptured ectopic pregnancy and increased risk to the patient. The patient is hemodynamically stable, therefore, she does not need to be admitted to the hospital.

A 42-year-old G5P4 woman is exclusively breastfeeding her two-month-old baby when she develops a fever and a red tender wedge-shaped area on the outer quadrant of her left breast. Which of the following is the most appropriate treatment for this condition? A. Cessation of breastfeeding for 48 hours B. Cessation of breastfeeding until afebrile C. Antibiotics D. Warm compresses E. Incision and drainage

C The patient has a classic picture of mastitis that is usually caused by streptococcus bacteria from the baby's mouth. Mastitis is easily treated with antibiotics. The initial choice of antimicrobial is influenced by the current experience with staphylococcal infections at the institution. Most are community-acquired organisms, and even staphylococcal infections are usually sensitive to penicillin or a cephalosporin. If the infection persists, an abscess may ensue which would require incision and drainage. However, this patient's presentation is that of simple mastitis. There is no need for the mother to stop breastfeeding because of the mastitis.

A 22-year-old G4P1 woman at 26 weeks gestation presents with a postcoital musty odor and increased milky, gray-white discharge for the last week. This was an unplanned pregnancy. She had her first pregnancy at age 15. She reports that she has no new sex partners, but the father of the baby may not be monogamous. On examination, there is a profuse discharge in the vaginal vault, which covers the cervix. Pertinent labs: wet mount pH >4.5 and whiff test positive. Microscopic exam reveals clue cells, but no trichomonads or hyphae. Which of the following is the most appropriate next step in the management of this patient? A. Delay treatment until postpartum B. Treat her now and again during labor C. Treat her now D. Treat her and her partner E. No treatment necessary

C The patient has bacterial vaginosis. All symptomatic pregnant women should be tested and treatment should be not be delayed because treatment has reduced the incidence of preterm delivery. The optimal regimen for women during pregnancy is not known, but the oral metronidazole regimens are probably equally effective. Once treated antepartum, there is no need to treat during labor unless she is reinfected.

A 29-year-old G0 woman presents due to the inability to conceive for the last year. Her cycles are regular every 28 days, but she has very painful periods, occasionally requiring that she miss work despite the use of non-steroidal anti-inflammatory drugs (NSAIDs). She also reports painful intercourse, which is becoming a problem as she now tries to avoid intercourse, even though she would like to conceive. She is otherwise in good health and has been married for five years. She is 5 feet 4 inches tall and weighs 130 pounds. She has a history of pelvic inflammatory disease at age 19, for which she was hospitalized. Her mother had a history of ovarian cancer at age 49. On physical examination, she has abdominal and pelvic lower quadrant tenderness. Uterus is normal in size, but there is a slightly tender palpable left adnexal mass. A pelvic ultrasound shows a 5 cm left complex ovarian cyst and two simple cysts measuring 2 cm in the right ovary. What best explains the underlying pathophysiology of the disease process in this patient? A. Chronic pelvic inflammatory disease B. Family history of ovarian cancer C. Endometrial glands outside the uterine cavity D. Polycystic ovarian syndrome E. Functional hemorrhagic cysts

C The patient has typical signs of endometriosis which is characterized by the presence of endometrial glands and stroma outside of the uterus. Endometriosis is present in about 30% of infertile woman. She does not have the signs and symptoms of chronic pelvic inflammatory disease. She also does not have the signs and symptoms of polycystic ovarian syndrome, which typically presents with oligomenorrhea in overweight patients. The complex ovarian cyst is most likely an endometrioma. The duration of her symptoms makes functional hemorrhagic cyst a less likely option.

A 20-year-old G1 woman at 40 weeks gestation presents to labor and delivery complaining of painful contractions every 3-4 minutes since midnight. Her examination on admission was 2 centimeters dilated, 90% effaced and 0 station. Three hours later, her exam is unchanged. The patient is still having contractions every 3-4 minutes. She is discouraged about her lack of progress. Which of the following is the most appropriate next step in the management of this patient? A. Laminaria placement B. Artificial rupture of membranes C. Counseling about latent phase of labor and rest D. Manual cervical dilation E. Cesarean section for arrest of labor

C The patient is in the latent phase of labor and has not yet reached the active phase (more than 4 cm). A prolonged latent phase is defined as >20 hours for nulliparas and >14 hours for multiparas, and may be treated with rest or augmentation of labor. Artificial rupture of membranes is not recommended in the latent phase as it places the patient at increased risk of infection. Cervical dilation or laminaria placement are not indicated.

A 33-year-old G3P3 woman presents to the office complaining of a new onset vaginal discharge of 4 days duration. The discharge is thick and white. She has noted painful intercourse and itching since the dischage began. Her vital signs are: blood pressure 120/76 and pulse 78. The pelvic examination reveals excoriations on the perineum, thick white discharge, and is otherwise non-contributory. What is the most likely diagnosis in this patient? A. Herpes simplex virus B. Primary syphilis C. Candida vaginalis D. Bacterial vaginosis E. Trichomonas

C The patient is most likely has candida vaginalis. Clinically women have itching and thick white cottage cheese like discharge. They may also have burning with urination and pain during intercourse. Herpes simplex viral infections are characterized by viral like symptoms preceding the appearance of vesicular genital lesions. A prodrome of burning or irritation may occur before the lesions appear. With primary infection, dysuria due to vulvar lesions can cause significant urinary retention requiring catheter drainage. Pain can be a very significant finding as well. Treatment is centered on care of the local lesions and the symptoms. Sitz baths, perineal care and topical Xylocaine jellies or creams may be helpful. Anti-viral medications, such as acyclovir, can decrease viral shedding and shorten the course of the outbreak somewhat. These medications can be administered topically or orally. Syphilis is a chronic infection caused by the Treponema pallidum bacterium. Transmission is usually by direct contact with an infectious lesion. Early syphilis includes the primary, secondary, and early latent stages during the first year after infection, while latent syphilis occurs after that and the patient usually has a normal physical exam with positive serology. In primary syphilis, a painless papule usually appears at the site of inoculation. This then ulcerates and forms the chancre, which is a classic sign of the disease. Left untreated, 25% of patients will develop the systemic symptoms of secondary syphilis, which include low-grade fever, malaise, headache, generalized lymphadenopathy, rash, anorexia, weight loss, and myalgias. Bacterial vaginosis is due to an overgrowth of anaerobic bacteria and characterized by a grayish / opaque foul-smelling discharge. Trichomonas is a protozoan and is transmitted via sexual contact. It typically presents with a non-specific vaginal discharge. It does not have a systemic manifestation.

A 29-year-old G3P0 woman presents for evaluation and treatment of pregnancy loss. Her past medical history is remarkable for three early (<16 weeks gestation) pregnancy losses and a deep vein thrombosis two years ago. Her work up includes: prolonged dilute Russell viper venom test; elevated anticardiolipin antibodies; normal thyroid function; normal prolactin; and normal MRI of the pelvis. She wishes to get pregnant soon. In addition to aspirin, which of the following treatments is appropriate for this patient? A. No additional treatment B. Corticosteroid C. Heparin D. 17-OH progesterone E. Bromocriptine

C The prolonged dilute Russell viper venom time leads one to suspect that the etiology of recurrent pregnancy loss is due to antiphospholipid antibody syndrome. The treatment is aspirin plus heparin. There is roughly a 75% success rate with combination therapy versus aspirin alone. There is conflicting evidence regarding steroid use for treatment. 17-OH progesterone is used for the prevention of preterm delivery and not recurrent pregnancy loss.

A 45-year-old G5P5 premenopausal woman was initially seen in your office for work-up and evaluation of a FIGO grade 3 endometrial cancer that was diagnosed by her gynecologist. Which of the following is the most appropriate treatment for this patient? A. Chemotherapy B. Radiation treatment C. Total abdominal hysterectomy, bilateral salpingo-oophorectomy, bilateral pelvic and para-aortic lymphadenectomy D. Supracervical abdominal hysterectomy with ovarian preservation E. Medroxprogesterone (Megace)

C The recommended components of the surgical approach to an early endometrial cancer are the extrafascial total abdominal hysterectomy, bilateral salpingo-oophorectomy, and pelvic and para-aortic lymphadenectomy. Alternative surgical approaches to early endometrial cancer include a total vaginal hysterectomy with or without a bilateral salpingo-oophorectomy in women who are medically unstable or have contraindications to major abdominal surgery. Ideally, this approach would only be utilized in patients with well-differentiated endometrioid adenocarcinomas and avoided in patients with high-grade lesions or aggressive cell types, such as clear cell or papillary serous carcinomas. A total laparoscopic hysterectomy, BSO, with or without staging is being utilized more and more in lieu of the traditional open approach for select patients in many centers, and is a reasonable alternative. Although chemotherapy, radiation, and hormonal therapy may be utilized, it is usually in an adjuvant setting.

A 24-year-old G1P0 woman presents at 32 weeks gestation with vaginal bleeding due to placental abruption. What is the most appropriate method to determine the correct RhoGAM dose to give to the patient? A. Determine delta OD 450 (optical density) B. Measure fetal hemoglobin levels C. Perform a Kleihauer-Betke test D. Measure maternal hemoglobin levels E. Administer routine dose of RhoGAM at time of inciden

C The routine dose of RhoGAM neutralizes 30 cc of fetal blood. The Kleihauer-Betke test is an accurate and sensitive acid elution test. It has great value in determining the incidence and size of fetal transplacental hemorrhage. In this test, using acid elution, the mother's red blood cells become very pale, while fetal cells, which contain a different form of hemoglobin, remain stained. Simple comparative counts allow an estimate of whether a significant fetal maternal transfusion has occurred.

A 16 year-old G1P0 African-American woman presents at 8 weeks gestation for prenatal care. She reports occasional spotting but denies pain or fever. The laboratory reports hemoglobin of 8 g/dL and a peripheral smear reveals hypochromia and microcytosis. Which of the following is the most likely diagnosis for this patient? A. sickle cell anemia B. folate deficiency C. iron deficiency D. β-thalassemia E. acute blood loss

C The two most common causes of anemia during pregnancy and the puerperium are iron deficiency and acute blood loss. Classical morphological evidence of iron-deficiency anemia is erythrocyte hypochromia and microcytosis. Serum ferritin levels are lower than normal and there is no stainable bone marrow iron on examination of a bone marrow aspirate. The spotting she reports would not lead to anemia due to blood loss.

An 18-year-old G1P0 woman presents for prenatal care at 14 weeks gestation. Her medical, surgical, gynecologic, social and family history are unremarkable. Her dietary history includes high carbohydrate intake with no fresh vegetables. Her physical examination is within normal limits except that she is pale and has a BMI of 42. Nutritional counseling should include the following: A. 25-30 grams of protein in her diet every day B. A strict diet to maintain her current weight C. Folic acid supplementation D. Intake of 1200 calories a day E. Initiation of a vigorous weight loss exercise program

C There should be folic acid supplementation, as well as evaluation for deficiencies in her iron, protein and other nutrient stores. In general, a patient needs approximately 70 grams of protein a day, along with her other nutrients. It would be prudent to caution her that, though aerobic exercise is recommended and would be a benefit to her, it is not advisable to initiate a vigorous program in a woman who has not been routinely working out. Women should gain weight during their pregnancy, and 1200 calories a day is not sufficient for a pregnant woman.

A 20-year-old G1P0 woman at 18 weeks gestation with a history of epilepsy has conceived while taking valproic acid. She is scheduled for an ultrasound. What is the most common anomaly associated with prenatal exposure to valproic acid? A. Cardiac defects B. Caudal regression syndrome C. Neural tube defects D. Cleft lip and palate E. Holoprosencephaly

C Valproic acid use during pregnancy is associated with a 1 to 2% incidence of neural tube defects, specifically lumbar meningomyelocele. Fetal ultrasound examination at approximately 16 to 18 weeks gestation is recommended to detect neural tube defects. Other malformations have been reported in the offspring of women being treated with valproic acid and a fetal valproate syndrome has been described which includes spina bifida, cardiac defects, facial clefts, hypospadius, craniosynostosis, and limb defects, particularly radial aplasia. Case reports have associated prenatal exposure to valproic acid with omphalocele and lung hypoplasia. Caudal regression syndrome is a rare syndrome observed in offspring of poorly controlled diabetics.

A 32-year-old G1 is at 36 weeks gestation. Ultrasound reveals limited fetal growth over the past three weeks. Biometry is consistent with 30-5/7, EFW 1900 g, less than 10th percentile. Which of the following is LEAST likely to be associated with this pregnancy? A. Fetal demise B. Perinatal demise C. Polyhydramnios D. Meconium aspiration E. Polycythemia

C This fetus had intrauterine growth restriction and, with the exception of polyhydramnios, all of the morbidities listed above may complicate intrauterine growth restriction. In general, the causes of polyhydramnios relate to amniotic fluid production (abnormalities of the fetal urinary tract) and removal (abnormalities of fetal swallowing and intestinal reabsorption of fluid). Some investigators report an increase in fetal urinary output when there is hyperglycemia and increased renal osmotic load, thus resulting in polyhydramnios. Abnormal fetal swallowing may be a result of a CNS or gastrointestinal tract abnormalities, such as anencephaly, esophageal or duodenal atresia, diaphragmatic hernia or primary muscular disease. Typically, polyhydramnios is not associated with asymmetric growth restriction (the most common form of IUGR), since an asymmetric growth pattern reflects poor uterine blood flow and limited substrate availability. In fact, oligohydramnios is frequently identified in pregnancies complicated by fetal growth restriction.

A 41-year-old G2P2 presents with menstrual pain, menorrhagia, irregular periods and intermenstrual bleeding. She describes the pain as pressure and cramps. Ibuprofen improves the pain, but does not entirely eliminate the discomfort. Pelvic examination reveals a 14-week size uterus with irregular masses within the uterus. Pelvic ultrasound confirms the diagnosis of fibroids. What is the most appropriate next step in the management of this patient? A. CA125 assay B. CT scan of the pelvis C. Endometrial biopsy D. GnRH agonist E. Hysterectomy

C This patient has classic symptoms of leiomyomata, including menorrhagia. An endometrial biopsy should be performed on all women over age 40 with irregular bleeding to rule out endometrial carcinoma. The CA125 assay measures the level of CA125 in the blood and is increased in some types of cancer, including ovarian cancer or other conditions. This non-specific marker is not indicated in this patient. A CT scan of the pelvis is also not indicated. A simple pelvic ultrasound could be used to help confirm the clinical diagnosis. GnRH agonist and hysterectomy are not used until the diagnosis of leiomyomata uteri is confirmed. Her desire for future fertility should be discussed.

A 37-year-old G2P1 comes to the clinic with her husband due to the inability to conceive for the last year. She reports being in good health and not having problems with her prior pregnancy two years ago, except for some postpartum depression for which she was placed on Imipramine and which she continues to take. She took birth control pills after her pregnancy and stopped one year ago, when she began trying to conceive. Her periods were regular on the pills, but have been irregular since she stopped taking them. She has no history of sexually transmitted infections or abnormal Pap smears. Her husband is also healthy and he fathered their first child. Her physical examination is completely normal. Laboratory tests show: Results Normal Values TSH 2.1 mIU/ml 0.5-4.0 mIU/ml Free T4 1.1 ng/dl 0.8-1.8 ng/dl Prolactin 60 ng/ml <20 ng/ml FSH 6 mIU/ml 5-25 mIU/ml LH 4 mIU/ml 5-25 mIU/ml What is the most appropriate next step in the management of this patient's subfertility? A. Begin bromocriptine B. Ovulation induction with clomiphene citrate (Clomid) C. Wean off imipramine D. Perform a visual field examination E. Obtain a brain MRI

C This patient has hyperprolactinemia due to imipramine. The patient has to be weaned off imipramine and placed on a more appropriate medication. Once she is off imipramine and the cause of her elevated prolactin levels is confirmed, her normal menses should resume. Although MRI of the brain would be a reasonable step, it would be premature, and visual field examination does not aid in the diagnostic work-up. It would be premature to obtain an MRI or begin bromocriptine without this intermediate step. An endometrial biopsy is not indicated at this point, especially since the patient had normal cycles on OCPs. Although Clomid is used to help with ovulatory dysfunction, the hyperprolactinemia must be addressed first.

A 24-year-old G2P1 woman who underwent an elective termination two days ago presents to the emergency room with abdominal and pelvic pain. She has been feeling nauseated and reports a fever at home. On presentation, her blood pressure is 100/60; pulse 100; respiration 16; temperature 102°F (38.9°C). Physical examination reveals diffuse abdominal tenderness and, on pelvic examination, she has marked cervical motion tenderness. In addition to sending a CBC and cultures, which of the following is the most appropriate next step in the management of this patient? A. Obtain a Beta-hCG level B. Order a hysterosonogram C. Begin IV antibiotics D. Proceed with a dilation and curettage E. Proceed with a laparoscopy

C This patient has postoperative endometritis that could be due to introduction of bacteria into the uterine cavity at the time of dilation and curettage. It is important to begin antibiotics immediately. After starting antibiotics, an ultrasound should be obtained to look for products of conception. If found, the patient would then require a repeat dilation and curettage. A Beta-hCG level would not be helpful 2 days after the termination. Hysterosonogram is contraindicated when infection is present. There are no indications for laparoscopy in this patient.

A 30-year-old G2P1 woman at 38 weeks gestation presents to labor and delivery with contractions every 2-3 minutes. Her membranes are intact. Her cervical examination is 5 centimeters dilated, 100% effaced, and -1 station. The fetal heart rate tracing is category I. Two hours later, she progresses to 7 cm and 0 station and receives an epidural for pain. Four hours after that, her exam is unchanged (7/100/0). Fetal heart rate tracing remains category I. Which of the following is the most appropriate next step in the management of this patient? A. Allow her to ambulate and return when she is ready to push B. Perform a contraction stress test C. Perform an amniotomy D. Perform a Cesarean delivery E. Place an internal fetal scalp electrode

C This patient has secondary arrest of dilation, as she has not had any further cervical change in the active phase for over four hours. Amniotomy is often recommended in this situation. After it is performed, if the patient is still not in an adequate contraction pattern, augmentation with oxytocin can be attempted after careful evaluation. Although the patient requires close monitoring, it is too early to proceed with a Cesarean delivery. An internal scalp electrode is not necessary, since the fetal heart monitoring is reassuring.

In patients with severe preeclampsia the treatment of choice, both for the cessation of seizures as well as for their prevention, includes: A. atosiban administered intravenously. B. atosiban administered intramuscularly. C. magnesium sulfate administered intravenously. D. magnesium sulfate administered intramuscularly. E. calcium gluconate administered intravenously.

C. magnesium sulfate administered intravenously.

A 32-year-old G0 woman presents with a one-month history of profuse vaginal discharge with mild odor. She has a new sexual partner with whom she has had unprotected intercourse. She reports mild to moderate irritation, pruritus and pain. She thought she had a yeast infection, but had no improvement after using an over-the-counter antifungal cream. She is concerned about sexually transmitted infections. Her medical history is significant for lupus and chronic steroid use. Pelvic examination shows normal external genitalia, an erythematous vagina with a copious, frothy yellow discharge and multiple petechiae on the cervix. Vaginal pH is 7. Which of the following findings on a wet prep explains the etiology of this condition? A. Hyphae B. Clue cells C. Trichimonads D. Lactobacilli E. Normal epithelial cells

C This patient has signs and symptoms of trichomoniasis, which is caused by the protozoan, T. vaginalis. Many infected women have symptoms characterized by a diffuse, malodorous, yellow-green discharge with vulvar irritation. However, some women have minimal or no symptoms. Diagnosis of vaginal trichomoniasis is performed by saline microscopy of vaginal secretions, but this method has a sensitivity of only 60% to 70%. The CDC recommended treatment is metronidazole 2 grams orally in a single dose. An alternate regimen is metronidazole 500mg orally twice daily for seven days. The patient's sexual partner also should undergo treatment prior to resuming sexual relations.

An 18-year-old G1 woman presents for prenatal care at 16 weeks gestation without complaints. The patient denies any history of sexually transmitted disease, although admits to a history of multiple sex partners, with irregular use of condoms. She is allergic to penicillin, which causes anaphylaxis. Physical exam is unremarkable. Pertinent labs: rapid plasma reagin test (RPR) positive (titer = 32); fluorescent treponemal antibody absorption test (FTA-ABS) is positive. Which of the following is the best treatment for this patient? A. Oral erythromycin B. Oral doxycycline C. Desensitization and penicillin D. Intravenous erythromycin E. Intravenous cefazolin

C This patient has syphilis, and the fluorescent treponemal antibody absorption test (FTA-ABS) confirms the diagnosis. The transmission rates for primary and secondary disease are approximately 50-80%. There are no proven alternatives to penicillin therapy during pregnancy and penicillin G is the therapy of choice to treat syphilis in pregnancy. Women with a history of penicillin allergy can be skin tested to confirm the risk of immunoglobulin E (IgE)-mediated anaphylaxis. If skin tests are reactive, penicillin desensitization is recommended and is followed by intramuscular benzathine penicillin G treatment. Erythromycin has an 11% failure rate. Doxycycline is contraindicated in pregnancy. Cefazolin is commonly used to treat urinary tract infections and is not effective in the treatment of syphillis.

A 27-year-old G0 woman presents due to a one-year history of dysmenorrhea and dyspareunia. Pain, when present, is 7/10 in strength and requires that she miss work. She now avoids intercourse and no longer finds it pleasurable. She is otherwise in good health. Her last menstrual period was 17 days ago and her menses are typically 28 days apart. She had chlamydia once, at age 19. Physical examination is notable for some tenderness on abdominal and pelvic exams in the lower quadrants. Uterus is normal in size and there is uterosacral ligament nodularity. What is the most likely diagnosis in this patient? A. Adenomyosis B. Chronic pelvic inflammatory disease C. Endometriosis D. Endometritis E. Premenstrual dysphoric disorder

C This patient has typical symptoms of endometriosis, including dysmenorrhea and dyspareunia. In addition, the nodularity on the back of the uterus is suggestive of endometriosis. Chronic pelvic inflammatory disease would not present this far out from a known infection. Adenomyosis is endometrial glands embedded in the wall of the uterus. Endometritis is an infection of the endometrium. Premenstrual dysphoric disorder (PMDD) is a condition in which a woman has severe depressive symptoms, irritability, and tension before menstruation.

A 57-year-old G2P2 woman presents with a six-month history of urinary incontinence, urgency, and nocturia. She describes the amount of urine loss as large and lasting for several seconds. The urine loss occurs when she is standing or sitting and is not associated with any specific activity. What is the most likely cause of this patient's symptoms? A. Stress incontinence B. Overflow incontinence C. Urge incontinence D. Mixed incontinence E. Vesicovaginal fistula

C This patient has urge incontinence, which is caused by overactivity of the detrusor muscle resulting in uninhibited contractions, which cause an increase in the bladder pressure over urethral pressure resulting in urine leakage. Stress incontinence is caused by an increase in intra-abdominal pressure (coughing, sneezing) when the patient is in the upright position. This increase in pressure is transmitted to the bladder that then rises above the intra-urethral pressure causing urine loss. Associated structural defects are cystocele or urethrocele. Overflow incontinence is associated with symptoms of pressure, fullness, and frequency, and is usually a small amount of continuous leaking. It is not associated with any positional changes or associated events. Mixed incontinence occurs when increased intra-abdominal pressure causes the urethral-vesical junction to descend causing the detrusor muscle to contract. A vesicovaginal fistula typically results in continuous loss of urine.

A 22-year-old G1P0 woman presents to the emergency department at eight weeks gestation experiencing heavy vaginal bleeding. Pelvic examination demonstrates brisk bleeding through a dilated cervical os. The patient's hemoglobin is 7 g/dL (hematocrit 21%). Which of the following is the most appropriate next step in the management of this patient? A. Administration of intravaginal misoprostol B. Administration of oral misoprostol C. Dilation and suction curettage D. Endometrial ablation E. Expectant care to permit spontaneous abortion

C This patient is actively bleeding and is anemic. She, therefore, requires immediate surgical treatment consisting of dilation and suction curettage. Although clinicians increasingly utilize both expectant management and various drug regimens to treat spontaneous abortion, a prerequisite for either is that the patient is hemodynamically stable and reliable for follow-up care. Endometrial ablation will not work in this case, as the products of conception need to be evacuated to control the bleeding.

A 34-year-old married woman, gravida 3. para 3, comes to your office for an annual examination She has no significant past medical history and is a nonsmoker. She had a negative Pap test 2 years ago during routine prenatal care. You order human papillomavirus (HPV) testing along with cervical cytology. The cytology test results are negative for intraepithelial neoplasia, but the HPV test result is positive for high-risk HPV subtypes. The best next step in management is . A) cervical cytology in 3 years B) HPV vaccine C) cervical cytology and HPV testing in 1 year D) colposcopy

C) cervical cytology and HPV testing in 1 year

Postmenopausal woman. Sexually active, but painful. How to treat? a. Estrogen cream b. Progesterone

a. Estrogen cream

A 23-year-old G2P1 woman at 36 weeks gestation presents with her third episode of heavy vaginal bleeding. She has normal prenatal labs and a known placenta previa. She denies uterine contractions or abdominal pain and reports good fetal movement. Her vital signs are: blood pressure 100/60; pulse 110; and she is afebrile. Her abdomen and uterus are nontender. Fundal height measures 35 centimeters and fetal heart tones reveal a baseline of 140 and are reassuring. Pelvic ultrasound confirms a placenta previa and the fetus is in the cephalic presentation. Hematocrit is 29%. Which of the following is the most appropriate next step in the management of this patient? A. Tocolysis B. Induction of labor C. Cesarean delivery D. Amniocentesis E. Administer steroids

C This patient is near term with a third episode of active bleeding from a placenta previa. The appropriate next step would be to move towards delivery via Cesarean section. The patient is not experiencing contractions, so tocolysis is not necessary and would not be used with heavy vaginal bleeding. Catastrophic bleeding could occur due to disruption of blood vessels as the cervix dilates if a vaginal delivery is pursued, and induction of labor would therefore be contraindicated. An amniocentesis is not indicated in this situation. Although the patient is not yet at term, delivery is appropriate due to the third episode of heaving bleeding at near term. Administering steroids is not appropriate at this gestational age.

A 45-year-old G3P3 comes to the office because she has been unable to conceive for the last two years. She is healthy and has three children, ages 10, 12 and 14, whom she conceived with her husband. She used a copper IUD after the birth of her last child and had it removed two years ago, hoping to have another child. She has no history of sexually transmitted infections or abnormal Pap smears. Her cycles are regular every 28 to 32 days. She is not taking any medications. She has been married for the last 16 years, and her husband is 52-years-old and in good health. Her physical examination, including a pelvic exam, is completely normal. What is the most appropriate next step in the management of this patient? A. Perform a hysteroscopy B. Order a hysterosalpingogram C. Order clomiphene challenge test D. Order a sperm penetration assay E. Basal body temperatures for six months

C This patient, most likely, has decreased ovarian reserve due to her age. A clomiphene challenge test, which consists of giving clomiphene citrate days five to nine of the menstrual cycle and checking FSH levels on day three and day 10, will help determine ovarian reserve. This will help counsel the patient on appropriate options to have a child, as most women will not be able to conceive at this age and would not be good candidates for ovarian stimulation or IVF. Even though this patient had an IUD in the past, there is no reason to believe that this contributed to her inability to conceive, as IUDs do not cause infertility and she has no risk factors for tubal disease. Having her keep basal body temperatures for six months would be a waste of time for this patient, who is already 45. A semen analysis or sperm penetration assay is not necessary as a first step as she was able to conceive from her husband previously without problems. Most likely this patient will have to use a donor egg if she wants to carry the pregnancy herself.

A 38-year-old G3P3 presents to the office because she has noted dark spots on her vulva. She states that the lesions have been present for at least two years and are occasionally itchy. She has a history of laser therapy for cervical intraepithelial neoplasia ten years ago, and has not had a pelvic exam since then. She has had multiple partners and uses condoms. Her menses are regular and she had a tubal ligation. She has a history of genital herpes, but has only one or two recurrences a year. She has smoked since age 14. On examination, multicentric brown-pigmented papules are noted on the perineum, perianal and labia minora. No induration or groin nodularity is noted. The vagina and cervix are normal in appearance. Which of the following is the most likely diagnosis? A. Hidradenitis suppurativa B. Molluscum contagiosum C. Vulvar intraepithelial neoplasia D. Melanoma E. Paget's disease

C This presentation is classic for human papilloma virus (HPV) related VIN 3. Melanoma would be unlikely to be multifocal and warts have a characteristic verrucous appearance, although pigmentation can occur. Molluscum, a poxvirus, is characterized by multiple shiny non-pigmented papules with a central umbilication. Paget's disease, although multicentric, does not have brown pigmentation. Hidradenitis is a chronic, unrelenting skin infection causing deep, painful scars and foul discharge.

A 25-year-old G4P1 woman with a previous term delivery presents to the emergency department at 7 1/2 weeks gestation with vaginal bleeding. She has not passed any tissue. On examination, there is blood in the vault and cervical os is closed. Ultrasound confirms an intrauterine pregnancy and fetal cardiac activity is noted. Which of the following diagnoses most accurately describes her condition? A. Incomplete abortion B. Missed abortion C. Threatened abortion D. Complete abortion E. Recurrent abortion

C Threatened abortion refers to vaginal bleeding before 20 weeks without the passage of any products. Incomplete abortions have passed some, but not all, of the products of conception. Missed abortions have experienced fetal demise without cervical dilatation or passage of products of conception. Recurrent abortion refers to three successive spontaneous abortions.

A 17-year-old nulligravid woman comes to you for an annual examination and to refill her oral contraceptives. She has recently noted vaginal discharge. She has been sexually active for 3 years She uses a combination oral contraceptive regularly but is inconsistent with condom use. Her most recent Pap test was at age 15 years. In addition to discussing human papillomavirus (HPV) vaccination with her, the test that her gynecologic evaluation should include is: A) cervical HPV DNA testing B) cervical cytology screening C) chlamydial infection testing D) colposcopy

C) chlamydial infection testing

What hormone is highest during the first 3 days after a normal menstruation ?

a. FSH

A 7-year-old is undergoing evaluation for vaginal bleeding. On physical examination, she has Tanner stage III breasts, tall stature and an otherwise normal examination. An MRI of the brain and a pelvic ultrasound are normal. LH and FSH levels are in the pre-pubertal levels and she has normal DHEAS and androgen levels. What is the most likely diagnosis in this patient? A. Pituitary adenoma B. Congenital adrenal hyperplasia C. True precocious puberty D. McCune Albright Syndrome E. Ovarian neoplasm

C True precocious puberty is a diagnosis of exclusion where the sex steroids are increased by the hypothalamic-pituitary-gonadal axis, with increased pulsatile GnRH secretion. CNS abnormalities associated with precocious puberty include the following: tumors (e.g., astrocytomas, gliomas, germ cell tumors secreting human chorionic gonadotropin [hCG]); hypothalamic hamartomas; acquired CNS injury caused by inflammation, surgery, trauma, radiation therapy, or abscess; or congenital anomalies (e.g. hydrocephalus, arachnoid cysts, suprasellar cysts). These conditions are not likely in the presence of a normal work-up in this patient. McCune Albright Syndrome is characterized by premature menses before breast and pubic hair development. An ovarian neoplasm is unlikely with a normal pelvic ultrasound.

A 29-year-old G4P2 woman with no previous prenatal care presents at 24 weeks gestation with signs and symptoms of preterm labor. Her cervix is 3 cm dilated and 80% effaced. Fundal height is 30 cm and an ultrasound examination reveals a twin gestation. Estimated fetal weights on the twins are 850 gm and 430 gm. The maximum vertical amniotic fluid pocket around the smaller twin is 1 cm; the maximum vertical amniotic fluid pocket around the larger twin is 8 cm. Which of the following is the most likely associated with these ultrasound findings? A. Dichorionic diamniotic twins B. Monochorionic monoamniotic twins C. Monochorionic diamniotic twins D. Superfecundation E. Rh-isoimmunization

C Twin-twin transfusion syndrome is the result of an intrauterine blood transfusion from one twin to the other. It most commonly occurs in monochorionic, diamniotic twins. The donor twin is often smaller and anemic at birth. The recipient twin is usually larger and plethoric at birth. Clues to the presence of the twin-twin transfusion syndrome include the large weight discordance (although this is not necessary for diagnosis), polyhydramnios around the larger (recipient) twin, and oligohydramnios around the smaller (pump) twin. The two different placental types in twin gestation are monochorionic and dichorionic. Monozygotic conceptions may have either monochorionic or dichorionic placentation, depending upon the time of division of the zygote. Dizygotic conceptions always have dichorionic placentas. Diamniotic dichorionic placentation occurs with division prior to the morula state (within three days post fertilization). Diamniotic monochorionic placentation occurs with division between days four and eight post fertilization. Monoamniotic, monochorionic placentation occurs with division between days eight and 12 post fertilization. Superfecundation is the fertilization of two different ova at two separate acts of intercourse in the same cycle. Isoimmunization is associated with polyhydramnios and fetal hydrops and does not cause twin-twin transfusion.

A 37-year-old nulliparous woman presents with a one-week history of a painful vulvar ulcer. She reports no fevers, malaise or other systemic symptoms. She recently started use of a topical steroid ointment for a vulvar contact dermatitis. She is married and has no prior history of sexually transmitted infections. She reports no travel outside the United States by her husband or herself. Her last Pap smear, six months ago, was normal. A vulvar herpes culture later returns positive for herpes simplex virus type 2. A Rapid Plasma Reagin (RPR) is nonreactive, and HIV testing is negative. Which of the following is the most likely diagnosis in this patient? A. Primary HSV episode B. Recurrent HSV-1 episode C. Recurrent HSV-2 episode D. Atypical HSV episode E. Contact dermatitis

C Two serotypes of HSV have been identified: HSV-1 and HSV-2. Most cases of recurrent genital herpes are caused by HSV-2. Up to 30% of first-episode cases of genital herpes are caused by HSV-1, but recurrences are much less frequent for genital HSV-1 infection than genital HSV-2 infection. Genital HSV infections are classified as initial primary, initial nonprimary, recurrent and asymptomatic. Initial, or first-episode primary genital herpes is a true primary infection (i.e. no history of previous genital herpetic lesions, and seronegative for HSV antibodies). Systemic symptoms of a primary infection include fever, headache, malaise and myalgias, and usually precede the onset of genital lesions. Vulvar lesions begin as tender grouped vesicles that progress into exquisitely tender, superficial, small ulcerations on an erythematous base. Initial, nonprimary genital herpes is the first recognized episode of genital herpes in individuals who are seropositive for HSV antibodies. Prior HSV-1 infection confers partial immunity to HSV-2 infection and thereby lessens the severity of type 2 infection. The severity and duration of symptoms are intermediate between primary and recurrent disease, with individuals experiencing less pain, fewer lesions, more rapid resolution of clinical lesions and shorter duration of viral shedding. Systemic symptoms are rare. Recurrent episodes involve reactivation of latent genital infection, most commonly with HSV-2, and are marked by episodic prodromal symptoms and outbreaks of lesions at varying intervals and of variable severity. Clinical diagnosis of genital herpes should be confirmed by viral culture, antigen detection or serologic tests. Treatment consists of antiviral therapy with acyclovir, famciclovir or valacyclovir.

A 28-year-old G3P3 woman experiences profuse vaginal bleeding of 700 cc in one hour following an uncomplicated spontaneous vaginal delivery of a 4150 gram infant. The placenta delivered spontaneously without difficulty. Prior obstetric history is notable for a previous low transverse Cesarean section, secondary to transverse fetal lie. The patient had no antenatal complications. Which of the following is the most likely cause of this patient's hemorrhage? A. Vaginal or cervical lacerations B. Uterine inversion C. Uterine atony D. Uterine dehiscence E. Uterine rupture

C Uterine atony is the most common cause of postpartum hemorrhage. Risk factors for uterine atony include precipitous labor, multiparity, general anesthesia, oxytocin use in labor, prolonged labor, macrosomia, hydramnios, twins and chorioamnionitis. Patients at risk for genital tract lacerations are those who have a precipitous labor, macrosomia or who have an instrument-assisted delivery or manipulative delivery i.e. breech extraction. Factors that lead to an over-distended uterus are risk factors for uterine inversion. Grand multiparity, multiple gestation, polyhydramnios and macrosomia are all risk factors. The most common etiology of uterine inversion, however, is excessive (iatrogenic) traction on the umbilical cord during the third stage of delivery. Although the patient is at risk for uterine dehiscence and uterine rupture because of her history of a prior Cesarean section, these are infrequent occurrences so the most likely cause of postpartum hemorrhage in this patient is uterine atony.

A 45-year-old G3P3 presents to the office because of a large dark spot on her vulva. She states that the lesion has been present for at least two years and is occasionally itchy. She has smoked since age 20. She has a history of genital herpes, but only has one or two recurrences a year. On examination, a 2.5 cm lesion is noted. No induration or groin nodularity is noted. The vagina and cervix appear normal. There are no additional lesions noted on colposcopic examination of the vulva. A biopsy of the lesion returns as vulvar intraepithelial neoplasia grade 3 (VIN 3). What is the most appropriate next step in the management of this patient? A. Imiquimod (Aldara) treatment B. Trichloroacetic acid (TCA) treatment C. Wide local excision D. Cryotherapy E. Radical vulvectomy

C VIN III should be treated with local superficial excision. Even with complete removal of all disease, the likelihood is very high for recurrence and the patient will need close surveillance. It is inappropriate to do radical surgery in this setting as cancer has not been diagnosed. Treatment with TCA and Aldara are reserved for condyloma, although some studies have shown utility in the use of Aldara in treating low grade VIN. Cryotherapy is primarily used to treat cervical dysplasia.

A 24-year-old G2P1 woman has a fetus that is affected by Rh disease. At 30 weeks gestation, the delta OD450 (optical density deviation at 450 nm) results plot on the Liley curve in Zone 3 indicating severe hemolytic disease. Which of the following is the most appropriate next step in the management of this patient? A. Immediate Cesarean delivery B. Induction of labor C. Intrauterine intravascular fetal transfusion D. Umbilical blood sampling E. Maternal plasmapheresis

C Values in Zone 3 of the Liley curve indicate the presence of severe hemolytic disease, with hydrops and fetal death likely within 7-10 days, thus demanding immediate delivery or fetal transfusion. At 30 weeks gestation, the fetus would benefit from more time in utero. An attempt should be made to correct the underlying anemia. Intravascular transfusion into the umbilical vein is the preferred method. Intraperitoneal transfusion is used when intravascular transfusion is technically impossible. If fetal hydrops is present, the reversal of the fetal anemia occurs much more slowly via intraperitoneal transfusion. Percutaneous umbilical blood sampling should not be used as a first-line method to evaluate fetal status. Maternal plasmapheresis is used in severe disease when intrauterine transfusions are not possible.

A 27-year-old nulliparous woman presents with a three-year history of dyspareunia. She reports a history of always having painful intercourse, but she is now unable to tolerate intercourse at all. She has avoided sex for the last six months. She describes severe pain with penile insertion. On further questioning, she reports an inability to use tampons because of painful insertion. She also notes a remote history of frequent yeast infections while she was on antibiotics for recurrent sinusitis that occurred years ago. Her medical history is unremarkable, and she is on no medications. Pelvic examination is remarkable for normal appearing external genitalia. Palpation of the vestibule with a Q-tip elicits marked tenderness and slight erythema. A normal-appearing discharge is noted. Saline wet prep shows only a few white blood cells, and potassium hydroxide testing is negative. Vaginal pH is 4.0. The cervix and uterus are unremarkable. Which of the following is the most likely diagnosis in this patient? A. Vaginal cancer B. Genital herpes infection C. Vulvar vestibulitis D. Contact dermatitis E. Chlamydia infection

C Vulvar vestibulitis syndrome consists of a constellation of symptoms and findings limited to the vulvar vestibule, which include severe pain on vestibular touch or attempted vaginal entry, tenderness to pressure and erythema of various degrees. Symptoms often have an abrupt onset and are described as a sharp, burning and rawness sensation. Women may experience pain with tampon insertion, biking or wearing tight pants, and avoid intercourse because of marked introital dyspareunia. Vestibular findings include exquisite tenderness to light touch of variable intensity with or without focal or diffuse erythematous macules. Often, a primary or inciting event cannot be determined. Treatment includes use of tricyclic antidepressants to block sympathetic afferent pain loops, pelvic floor rehabilitation, biofeedback, and topical anesthetics. Surgery with vestibulectomy is recommended for patients who do not respond to standard therapies and are unable to tolerate intercourse.

A 28-year-old G0 comes to the office for preconception counseling and the inability to conceive for one year. She and her husband of three years are both in good health. She has normal cycles every 28-33 days. She has intercourse about once a month, depending on her schedule. She is an airline pilot and travels a lot. Her examination is normal. She asks about when to best have intercourse during her cycle to maximize her chances of pregnancy. What is the most appropriate advice to give her? A. Keep basal body temperatures and try to attempt intercourse immediately after the rise in body temperature B. Best to attempt intercourse after she is done with her menses C. Use ovulation predictor kits and attempt intercourse after it turns positive D. Take a leave from her work so she can have intercourse three times a week until she gets pregnant E. Attempt intercourse on day 18 of her cycle

C Women are most fertile during the middle of their cycle when they are ovulating. Assuming normal cycles every 28 days, a woman is most likely to ovulate on day 14. Since sperm can live for up to three days, intercourse up to three days before ovulation can still result in pregnancy. Since this patient has cycles that vary in length, she can best tell when she is ovulating by using an ovulation predictor kit. The basal body temperature charts tell when a patient ovulated retrospectively, so it cannot be used to time intercourse to conceive, as the egg is only viable for about 24 hours. Although having intercourse more frequently will increase her likelihood of conceiving, it is not a practical solution for a working person to stop their work in order to conceive.

A 50-year-old G4P4 presents for her health maintenance examination. She states her menses are of normal flow every 32 days, with minimal cramping and five-day duration. Review of systems is negative. She has no medical problems. She had a bilateral tubal ligation following her last child, and a laparoscopic cholecystectomy five years ago. She has a history of LSIL Pap smear with colposcopy and cryotherapy 15 years ago; her Paps have been normal since then. She does not smoke, drink alcohol or use any drugs. She is sexually active with one partner with no problems. Her general exam, including a breast exam and pelvic exam, is normal. In addition to performing a Pap smear, which of the following is the most appropriate screening test for this patient? A. Pelvic ultrasound B. Endometrial biopsy C. Mammogram D. DEXA scan E. Colposcopy

C Women should be offered mammograms yearly starting at age 50 and at least every two years between age 40 and 50 according to American College of Obstetricians and Gynecologists (ACOG). Ultrasound is not a good screening modality for pelvic pathology. It may be used along with yearly examination if the patient's habitus does not allow full evaluation of the adnexa; this patient had a normal exam, so it is not indicated. An endometrial biopsy is indicated if a patient is experiencing irregular bleeding. A DEXA scan is not indicated in a pre-menopausal patient. A colposcopy is not indicated as she has not had a recent history of abnormal Pap smears

A 36-year-old G3P2 presents in active labor at full term with a known placenta previa. She reports brisk vaginal bleeding. Evaluation shows that fetus and patient are currently hemodynamically stable. She has had two normal vaginal deliveries in the past. She declines your recommendation to undergo Cesarean section. Which of the following is not advisable during your initial management of this patient? a. Soliciting her reasons for not undergoing a Cesarean section b. Obtaining hospital Ethics Committee recommendation c. Proceeding with an emergency Cesarean section d. Explaining your reasons for recommending a Cesarean section e. Informing risk management of the situation that has developed

C You should not perform any procedure on the patient without her consent. It is best in these situations to explain your reasons for the recommended Cesarean section and elicit the patient's reasons for not wanting to undergo the procedure. A court order should only be obtained as a last resort.

Of the following, which is the most appropriate initial antibiotic treatment for a tuboovarian abscess? . A) Clindamycin and ampicillin B) Tetracycline and penicillin C) Clindamycin and gentamicin D) Ciprofloxacin and gentamicin E) Ampicillin and gentamicin

C) Clindamycin and gentamicin

At the time of her annual examination, you find an 11-week-sized irregular uterus on an asymptomatic 40-year-old woman. Her last exam 1 year prior was normal. Your next step in the management of this patient is: . A) Hysterectomy B) Endometrial biopsy C) Reexamination in 6 months D) Fractional dilation and curettage E) Gonadotropin releasing hormone agonist therapy

C) Reexamination in 6 months

A woman with severe preeclampsia suddenly develops respiratory depression after the administration of hydralazine and magnesium sulfate. Respiratory depression is due to: A. impending eclampsia. B. hydralazine toxicity. C. magnesium sulfate toxicity. D. pulmonary embolism. E. heart attack.

C. magnesium sulfate toxicity.

A 32-year-old nulligravid woman comes to your office reporting fatigue and weight gain after knee surgery 1 year ago for a torn meniscus. She says she gained 18 kg (40 lb) and presently weighs 95.3 kg (210 lb). She is 1.57 m (62 in.) tall with a body mass index (weight in kilograms divided by height in meters squared [kg/m2]) of 38.4. Her waist measures 109 cm (43 in.). Her menstrual cycles occur every 28-30 days with moderate flow. You note excess hair on her forearms, cheeks, and chin, and fullness in the thyroid region. She has increased pigmentation on her upper lip, malar prominences consistent with chloasma, and velvety dark pigmentation at the nape of her neck. Fasting glucose level is 108 mg/dL and total cholesterol level is 270 mg/dL. Her blood pressure is 135/90 mm Hg. The most likely diagnosis is: A) late onset congenital adrenal hyperplasia B) polycystic ovarian disease C) metabolic syndrome D) Cushing syndrome E) Hashimoto thyroiditis

C) metabolic syndrome

A 47-year-old premenopausal woman receives a diagnosis of pneumonia. You see her in consultation because she has an elevated CA 125 level of 90 units/mL. She appears well, reports no symptoms other than a persistent cough, and is taking proper antibiotics for community-acquired pneumonia. Physical examination findings, including pelvic examination, are normal, except for egophony and dullness to percussion in the chest. Her mother had postmenopausal ovarian cancer. After resolution of her pneumonia, you recommend . A) laparoscopy B) laparotomy C) repeat CA 125 assessment D) pelvic ultrasonography

C) repeat CA 125 assessment

A 55-year-old postmenopausal woman had a 6-cm simple right ovarian cyst diagnosed by transvaginal ultrasonography. She is asymptomatic and does not have a family history of breast cancer or ovarian cancer. The next step in management is. A) ultrasonographically guided cyst aspiration B) laparoscopy C) repeat ultrasonography in a year D) computed tomography (CT) scan of pelvis E) observation

C) repeat ultrasonography in a year

Which of the following are indications for first-trimester complex prenatal diagnosis? 1) advanced maternal age (at least 35 years at the expected time of delivery); 2) giving birth to a child with genetic disorders in the past; 3) increased nuchal translucency (NT) values; 4) preeclampsia in the previous pregnancy; 5) oligohydramnios. The correct answer is: A. 1,4,5. B. 2,4,5. C. 1,2,3 D. 4,5. E. 3,4.

C. 1,2,3

The complications typical of monochorionic-monoamniotic twin pregnancies include: 1) acute type of twin-to-twin transfusion syndrome (TTTS); 2) chronic type of twin-to-twin transfusion syndrome (TTTS); 3) impairment of the fetus whose twin died in the utero; 4) conjoint twins; 5) premature labor after the end of 37 week of gestation. The correct answer is: A. 2,3,4,5. B. 1,3,4,5. C. 1,2,3,4. D. all the above. E. 1,2.

C. 1,2,3,4.

The surgical treatment of cervical incompetence consists in a cervical cerclage. Which Of the following conditions should be met before putting on a cerclage? 1) normal fetal development; 2) alive fetus; 3) absence of uterine contractions; 4) normal vaginal biocenosis; 5) gestational age over 28 weeks. The correct answer is: A. 2,3,4,5. B. 1,3,4,5. C. 1,2,3,4. D. all the above. E. 1,2.

C. 1,2,3,4.

High FSH serum levels accompanied by low estradiol levels are characteristic of: 1) Turner syndrome; 2) premature ovarian failure (POF); 3) pure gonadal dysgenesis; 4) polycystic ovary syndrome (PCOS); 5) postmenopausal period. The correct answer is: A. 1,2,3. B. 1,2,5. C. 1,2,3,5. D. only 3. E. all the above.

C. 1,2,3,5.

Indicate true statements concerning male reproductive system: 1) cycle of the seminiferous epithelium takes approx. 74 days; 2) in the testes FSH acts mainly on Sertoli and LH mainly on Leydig cells; 3) after gonadotropin stimulation Leydig cells produce testosterone; 4) estrogens do not participate in the regulation of male reproductive system. The correct answer is: A. all the above. B. 2,4. C. 1,2,3. D. 2,3,4. E. 3,4.

C. 1,2,3.

Bacterial vaginitis: 1) is asymptomatic in most women, and in symptomatic cases manifests with a grey or white vaginal discharge; 2) is related to dysuria and dyspareunia; 3) is diagnosed on the basis of reported-by-the-patient fishy smell that aggravates after the intercourse; 4) is related to the increase in the number of Lactobacilli; 5) is characterized by the presence of clue cells in the smear. The correct answer is: A. 1,3. B. 1,5. C. 1,3,5. D. 2,3,4. E. 2,3,5.

C. 1,3,5.

What percentage of the patients with diagnosed ovarian cancer are BRCA1/BRCA2 germinal mutation carriers? A. 1%. B. 5%. C. 10%. D. 30%. E. 50%.

C. 10%.

Fetal heart auscultation is an important part of obstetrical examination and it helps safe delivery management. During normal delivery the basic fetal heart rate should be in the range of: A. 70 - 90 bits per minute. B. 100 - 170 bits per minute. C. 110 - 150 bits per minute. D. 110 - 190 bits per minute. E. above 170 bits per minute.

C. 110 - 150 bits per minute.

Which of the following are among factors increasing the risk of hyperemesis gravidarum? 1) cigarette smoking; 2) bulimia; 3) obesity; 4) pregnancy in a nullipara; 5) lipid metabolism disorders. The correct answer is: A. 1,3,5. B. 2,3,5. C. 2,3,4,5. D. 1,2,3,4. E. all the above.

C. 2,3,4,5.

Which class in the Quigley scale corresponds to complete androgen insensitivity syndrome (CAIS)? A. from 1 to 3. B. 4. C. 7. D. 6. E. 5.

C. 7.

When, during pregnancy, the concentration of the β-subunit of hCG in blood is the highest? A. 5-6 week. B. 6-7 week. C. 9-12 week. D. 14-16 week. E. 16-18 week.

C. 9-12 week.

Intermittent amenorrhea denotes: A. lack of menstruation under 16 years of age. B. menstrual bleeding lasting 5-7 days. C. menstrual bleeding occurring every 43 days - 6 months. D. menstrual bleeding occurring with gaps shorter than 21 days. E. alternative name for coitus interruptus.

C. menstrual bleeding occurring every 43 days - 6 months.

Which of the following medicines is not indicated in endometriosis treatment? A. danazol. B. goserelin. C. mirabegron. D. dienogest. E. progesteron.

C. mirabegron.

Which of the following does not affect the result of the triple test? A. body weight of the pregnant woman. B. tobacco smoking. C. number of pregnancies. D. race. E. type I diabetes.

C. number of pregnancies.

A young female patient gave birth to her child about a year ago through the vaginal delivery, after which curettage was necessary because of fragments of the placenta did not separate from the uterus. She stopped breastfeeding 4 months ago, but the periods have not reoccurred yet. Her medical history may suggest: A. Kallmann syndrome. B. Sheehan's syndrome. C. Asherman's syndrome. D. Mayer-Rokitansky-Küster-Hauser syndrome. E. Couvelaire syndrome.

C. Asherman's syndrome.

Which of the following manoeuvres is not recommended in the case of shoulder dystocia: A. suprapubic pressure. B. Zavanelli's manoeuvre. C. Bracht's manoeuvre. D. McRoberts manoeuvre. E. Woods manoeuvre.

C. Bracht's manoeuvre.

Old woman in automobile accident. Used a cane and had calcified mass in right breast, with bruise on overlying skin?

a. Fat necrosis?

Young woman with breast lump. Freely mobile?

a. Fibroadenoma

Patient is pregnant with pruritus. What to check to establish dx? a. Increased bile acids in serum b. Increased acetylcholinesterase

a. Increased bile acids in serum

Case that showed pregnant woman with low Hct etc. This is due to?

a. Increased plasma

A 64-year-old obese female reports postmenopausal vaginal bleeding. A cytological smear was obtained and in a transvaginal ultrasound endometrial hyperplasia was found. The mucosa biopsy was performed and the histopathological diagnosis of adenocarcinoma was obtained. Pap smear was normal. The uterus with appendage was completely resected and the lymph nodes were sampled. In postoperative examination adenocarcinoma cells were found in the uterus region (no evidence of serous membrane infiltration) and in the cervical canal near the internal orifice. No evidence of adnexal infiltration or metastases to the lymph nodes was found. Indicate tumor advancement according to FIGO classification: A. IA. B. IB. C. II. D. IIIA. E. IIIB.

C. II.

Which stage of endometriosis according to the classification of the American Society for Reproductive Medicine should be diagnosed in a patient with an endometrial cyst of the left ovary of 4 cm in diameter, partial obliteration of the recto-uterine pouch and superficial lesions of the peritoneal membrane of the diameter < 1 cm? A. I. B. II. C. III. D. IV. E. V.

C. III.

A 45-year-old woman with the diagnosis of cervical cancer went to see her gynecologist. Which of the following data from her medical history is not a risk factor for this cancer? A. 5 deliveries. B. numerous viral and bacterial infections of the cervix. C. obesity. D. smoking. E. frequent changing of sexual partners.

C. obesity.

Patient is pregnant. Which vaccines can she take? a. Influenza b. MMR c. Diphtheria-tetanus etc.

a. Influenza

Treatment for endometritis?

a. Intravenous clindamycin and amoxicillin

Which of the following are characteristic of a granuloma cell tumor? 1) most often occurs on both sides; 2) it is a germ cell tumor; 3) it is secreting estrogens and inhibin; 4) it is often accompanied by ascites; 5) most often occurs before puberty. The correct answer is: A. 1,2,4. B. 2,4,5. C. only 3. D. 1,3,5. E. 2,3,5.

C. only 3.

Which of the following heart diseases in a pregnant woman are absolute indications for cesarean delivery? 1) uncorrected ASD; 2) uncorrected VSD; 3) uncorrected coarctation of the aorta; 4) Marfan syndrome. The correct answer is: A. 2,3,4. B. 1,2,3. C. only 4. D. 3,4. E. all the above

C. only 4.

Indicate the true statement concerning the uterine artery: A. originates from the ovarian artery. B. gives the branch running through the round ligament. C. originates from the internal iliac artery. D. crosses the ureter from the rear. E. provides blood to the uterus only.

C. originates from the internal iliac artery.

Patient had been living at a homeless shelter during the last year. Pregnant. No cravings. Blood labs showed microcytic anemia. Whats her deficiency? a. Iron b. Folic acid c. Vitamin B12 d. Pica

a. Iron

Pregnant patient had pain right lower quadrant. Acute pain with absent bowel sounds (peritonitis I think). What to do?

a. Laparoscopy

Which of the following is characteristic of the vaginal discharge in a woman with bacterial vaginitis? A. pH < 4.5, yellow-greenish foamy discharge with clue cells present. B. pH < 4.5, grey or milky homogenous discharge with clue cells present. C. pH > 4.5, grey or milky homogenous discharge with clue cells present. D. pH > 4.5, white, clotty, serous discharge with clue cells present. E. pH < 4.5, white, clotty, serous discharge without any cells present.

C. pH > 4.5, grey or milky homogenous discharge with clue cells present.

Which of the following is not included in the fetal biophysical profile (Manning's score)? A. fetal heart rate. B. fetal muscle tone. C. placenta development stage. D. amniotic fluid volume. E. fetal respiratory movements.

C. placenta development stage.

The pharmacological treatment of endometriosis does not include: A. danazol. B. gonadoliberin analogues. C. preparations which contain 17-b estradiol only. D. progestogens. E. estrogen-progestogen formulations.

C. preparations which contain 17-b estradiol only.

Case where patient was bleeding from the vagina. Well demarcated mass found on a stalk in cervix (?) a. Leiomyoma b. Adenomyosis

a. Leiomyoma

Low molecular weight heparin during pregnancy: A. has tocolytic effect. B. accelerates foetal lung maturation. C. prevents miscarriage in women with hereditary thrombophilia. D. decreases the risk of premature placental abruption. E. should not be used due to its teratogenic effect.

C. prevents miscarriage in women with hereditary thrombophilia.

Endometriosis should be taken into account in the case of: A. endometrial hyperplasia on ultrasound, the presence of symptoms of hirsutism. B. abnormal postmenopausal bleeding from the uterine cavity. C. reduced fertility, pelvic pain syndrome, dysmenorrhea. D. rare menstruation with the image of polycystic ovaries on ultrasound. E. presence of endometrial polyps in the uterine cavity.

C. reduced fertility, pelvic pain syndrome, dysmenorrhea.

Woman had just delivered. Breastfeeding + fever. Fatigue? a. Mastitis b. Mastodynia c. Breast abscess d. Milk fever

a. Mastitis

Fetal lie is: A. relation of the particular parts of the fetus to each other. B. relation of the particular parts of the fetus to the particular parts of the uterus. C. relation of the long fetal axis to the long axis of the uterus. D. relation of the particular parts of the fetus to the long D. relation of the particular parts of the fetus to the long axis of the uterus. E. relation of the long fetal axis to the particular parts of the uterus.

C. relation of the long fetal axis to the long axis of the uterus.

The most common uterine anomaly is: A. unicorn uterus with a rudimentary horn. B. uterus didelphys. C. septate uterus. D. bicornuate uterus. E. unicorn uterus.

C. septate uterus.

A single-handed maneuver performed during palpation of the pregnant woman's abdomen that allows to determine the leading part of the fetus and its location in relation to the inlet is called: A. first Leopold's maneuver. B. second Leopold's maneuver. C. third Leopold's maneuver. D. forth Leopold's maneuver. E. none of the above.

C. third Leopold's maneuver.

Obstetrical complications that may lead to disseminated intravascular coagulation (DIC) do not include: A. amniotic fluid embolism. B. premature placental abruption of the normally located placenta. C. HELLP syndrome. D. eclampsia. E. fetal hypotrophy caused by placental insufficiency.

E. fetal hypotrophy caused by placental insufficiency.

Marsupialization is a surgical method of: A. primary treatment of stress urinary incontinence. B. treatment of recurrent stress urinary incontinence. C. treatment of a recurrent Bartholin gland abscess. D. treatment of impaired statics of the reproductive organ. E. creation of the vagina in patients with Mayer-Rokitansky-Küster-Hauser syndrome

C. treatment of a recurrent Bartholin gland abscess.

Which of the following parameters is not included in the assessment of the fetal biophysical profile? A. fetal movement. B. fetal tone. C. umbilical artery blood flow. D. amniotic fluid volume. E. fetal heart rate.

C. umbilical artery blood flow.

Indicate the correct sequence of fetal head movements in the birth canal during labor: A. flexion, extension, rotation, rotation. B. extension, rotation, flexion, rotation. C. flexion, rotation, rotation, extension. D. rotation, flexion, extension, rotation. E. flexion, rotation, extension, rotation.

E. flexion, rotation, extension, rotation.

A 25-year-old man with normal male phenotype is examined because of infertility. Apart from a decreased sperm count in the semen analysis, gynecomastia was found and laboratory tests showed decreased follicle-stimulating hormone levels and increased testosterone levels. The most probable cause of the above abnormalities is: A. chemotherapy in the past. B. radiotherapy in the past. C. use of anabolic steroids. D. Kallmann syndrome. E. Sertoli cell-only syndrome.

C. use of anabolic steroids.

Pregnancy in a woman with Swyer's syndrome may be possible: A. after stimulation of ovulation with gonadotropins. B. after performing patient's oocyte retrieval by the aspiration of ovarian follicles. C. when a donor oocyte is used. D. after surgical recreation of the vagina. E. after the uterus transplantation.

C. when a donor oocyte is used.

Which of following is not an advantage of Pfannenstiel incision? A. good cosmetic effect. B. rare cases of wound dehiscence. C. wide access to the operative field. D. rare cases of incisional hernia. E. better blood supply to the wound.

C. wide access to the operative field.

A 29-year-old G1P0 is at 11 weeks gestation. She has a history of depression which has been well controlled with fluoxetine (Prozac). Although the medication is very helpful in controlling her depression, she is concerned about potential side effects on her neonate. Which of the following conditions in the neonate is associated with maternal use of Fluoxetine during pregnancy? A. Necrotizing enterocolitis B. Intracranial hemorrhage C. Agitation and poor feeding D. Temperature instability E. Persistent pulmonary hypertension

C. Third trimester maternal use of SSRIs including Fluoxetine has been associated with abnormal muscle movements (extrapyramidal signs or EPS) and withdrawal symptoms which may include agitation, abnormally increased or decreased muscle tone, tremor, sleepiness, severe difficulty breathing, and difficulty in feeding. In some newborns, the symptoms subside within hours or days and do not require specific treatment; other newborns may require longer hospital stays. SSRI use during pregnancy is not associated with newborn seizures, intracranial hemorrhage or temperature instability. The FDA has concluded that, given the conflicting results from different studies, it is premature to reach any conclusion about a possible link between SSRI use in pregnancy and persistent pulmonary hypertension.

Chronic alcohol abuse may lead to disorders of spermatogenesis as the result of: A. increased concentration of levonorgestrel. B. decreased adrenal production of androstendione. C. increased concentrations of estrogens. D. increased synthesis of LH. E. increased synthesis of testosterone.

C. increased concentrations of estrogens.

What is the role of estrogens in the combined contraceptive pill? A. fallopian tube peristalsis reduction. B. thickening of cervical mucus. C. inhibition of selection and growth of the dominant follicle. D. suppression of the LH secretion. E. A and C are correct.

C. inhibition of selection and growth of the dominant follicle.

Which of the following is the correct management of a 30-year-old patient in 37 weeks of her first pregnancy who was diagnosed with the minor type of placenta previa confirmed by repeated US examination and reports minor bleeding from the reproductive tract? A. transvaginal palpation, transvaginal ultrasound examination, conservative treatment (pregnancy continuation up to 38 weeks). B. transvaginal palpation, CTG monitoring, vaginal delivery. C. assessment of the general condition of the mother, fetal pulse monitoring, preparations and performing caesarean section. D. assessment of the general condition of the mother, fetal pulse monitoring, vaginal delivery. E. speculum examination, intravenous fenoterol infusion, pregnancy continuation up to38 weeks.

C. assessment of the general condition of the mother, fetal pulse monitoring, preparations and performing caesarean section.

So called clue cells are typical of: A. vaginal candidiasis. B. vaginal trichomoniasis. C. bacterial vaginosis. D. normal vaginal biocenosis. E. vaginal actinomycosis

C. bacterial vaginosis.

Premature thelarche in contrast to precocious puberty is characterized by the isolated: A. development of axillary hair. B. development of pubic hair. C. breast development. D. occurrence of ovulation. E. occurrence of voice change.

C. breast development.

A 41-year-old patient came to gynecologist asking for advice on contraception during perimenopause. She has two children, and has diabetes mellitus type I diagnosed at the age of 4. The current level of glycated haemoglobin HbA C is 8.9%, urine albumin excretion - 40 mcg/mg creatinine, her blood pressure is about 130/80 mmHg, ophthalmoscopy showed pre-proliferative retinopathy. Which of the following methods is contraindicated in this patient? A. cervical mucus-body temperature method. B. progestin-only pill. C. combined oral contraceptive pills with ethinylestradiol and levonorgestrel. D. barrier method. E. intrauterine device with levonorgestrel.

C. combined oral contraceptive pills with ethinylestradiol and levonorgestrel.

Which of the following is not considered in the assessment of the cervix according to the Bishop score? A. dilation. B. consistency. C. distance from the sacral bone. D. effacement. E. fetal station.

C. distance from the sacral bone.

Routine obstetric ultrasounds between 11 weeks and 14 weeks of gestational age do not include: A. evaluation of the number of gestational sacs in the uterine cavity. B. measurement of the crown-rump length in the sagittal plain. C. evaluation of the blood flow in the umbilical artery. D. evaluation of the fetal heart rate (FHR). E. determination of the number of chorionic sacs for multiple gestations.

C. evaluation of the blood flow in the umbilical artery.

A 28-year-old woman, with menstrual cycle length of about 35 days, comes to the admission room with pains in the hypogastrium and vaginal bleeding lasting for 3 days. The last menstrual flow occurred 45 days before. The patient also complains of frequent urination and tenderness of the breasts. What diagnosis should be considered first? A. hemorrhagic ovarian cyst. B. ovarian torsion. C. extrauterine pregnancy. D. adnexitis. E. ruptured ovarian cyst.

C. extrauterine pregnancy.

Which statement regarding the placenta is false? A. forms the afterbirth together with the umbilical cord and the amniotic sac. B. its mass is approximately 500g. C. fully formed placenta is not observed until the second half of the second trimester. D. cotyledon is the morphological and structural unit of the placenta. E. right umbilical vein undergoes atrophy in the early stage of foetal development.

C. fully formed placenta is not observed until the second half of the second trimester.

Indicate the false statement concerning hormonal contraception: A. progestogen-only pills can be used during breastfeeding. B. in the case of progestogen-only pills irregular bleedings or secondary amenorrhea can occur. C. hormonal contraception decreases the risk of ovarian, cervical and endometrial cancers. D. migraine is a contraindication to the use of oral hormonal contraception. E. so-called breakthrough bleedings are more intense in smokers.

C. hormonal contraception decreases the risk of ovarian, cervical and endometrial cancers.

A contraindication to the safe delivery of the vaginal birth with the breech presentation of the fetus is: A. multiparity. B. pregnancy term 36-40 weeks. C. fetal frank breech presentation. D. fetal weight from 2000 to 3700 g. E. footling breech presentation.

E. footling breech presentation.

A 13-year-old girl is brought to the physician for increasingly severe abdominal pain. The pain is now a constant low discomfort, but every month she has a week when it is more severe. She has Tanner stage II breasts and pubic hair development. On genital examination, there is a bluish mass pushing the labia open. What is the most likely cause of this patient's abdominal pain? A. Turner's syndrome B. Transverse vaginal septum C. Isolated atresia of the cervix D. Imperforate hymen E. Synechiae of the uterine cavity

D

Patient 8 weeks pregnant. Got infected by Rubella from a child. Antibody titer is 1:10. What to do? a. Measure again later b. Give MMR vaccine c. The patient is fully immunized

a. Measure again later

A 19-year-old G1P0 at 42 weeks gestation presents to labor and delivery with spontaneous rupture of membranes for 13 hours and spontaneous onset of labor. Her vital signs are: blood pressure 120/70; pulse 72; afebrile; fundal height 36 cm; and estimated fetal weight of 2700 gm. Cervix is dilated to 4 cm, 100% effaced, +1 station. Which statement best describes the tracing seen below?

D (0 contractions) Variable decelerations show an acute fall in the FHR, with a rapid down slope and a variable recovery phase. They are characteristically variable in duration, intensity, and timing, and may not bear a constant relationship to uterine contractions. They are typically associated with cord compression, especially in the setting of low amniotic fluid volume. Early decelerations are physiologic caused by fetal head compression during uterine contraction, resulting in vagal stimulation and slowing of the heart rate. This type of deceleration has a uniform shape, with a slow onset that coincides with the start of the contraction and a slow return to the baseline that coincides with the end of the contraction. Thus, it has the characteristic mirror image of the contraction. A late deceleration is a symmetric fallin the fetal heart rate, beginning at or after the peak of the uterine contraction and returning to baseline only after the contraction has ended. Late decelerations are associated with uteroplacental insufficiency. The true sinusoidal pattern is a regular,smooth, undulating form typical of a sine wave that occurs with a frequency of two to five cycles/minute and an amplitude range of five to 15 beats/minute. It is also characterized by a stable baseline heart rate of 120 to 160 beats/minute and absentbeat-to-beat variability.

A 42-year-old G2P2 woman complains of bloating, mood swings and irritability the week prior to her menses. She is convinced that something is wrong with her hormone levels. In addition to a complete physical examination, which of the following diagnostic tools would provide information to accurately determine the diagnosis? A. Pelvic ultrasound B. Estradiol level C. CAGE questionnaire D. Prospective symptom calendar E. Mini mental status examination

D A calendar of symptoms can clarify if there is a cyclic or constant nature of the symptoms. Often women will mistakenly attribute their symptoms to their menstrual cycle. Different self-reporting scales have been written to assist patients track their symptoms. Because she is menstruating regularly, there is no role for obtaining serum hormone levels. The CAGE questionnaire is a screening test for alcohol dependence.

A 42-year-old G0 woman with stage IV endometriosis presents with chronic pelvic pain. For the past three years, the patient has had worsening cyclic pelvic pain along with dyspareunia. Her pain has been unresponsive to continuous and cyclic contraceptive pills and depot medroxyprogesterone acetate injections. It has been slightly responsive to leuprolide acetate injections. She is currently taking ibuprofen and hydrocodone for pain relief. She does not desire fertility. Pelvic exam is notable for uterine and bilateral adnexal tenderness with uterosacral nodularity. Uterus is normal size and retroverted. A pelvic ultrasound is normal. The patient desires definitive treatment. What is the next best step in the management of this patient? A. Laparoscopy with excision of endometriosis B. Radical hysterectomy C. Levonorgestrel IUD D. Hysterectomy with bilateral salpingo-oophorectomy E. Endometrial ablation

D A hysterectomy with bilateral salpingo-oophorectomy is the definitive treatment for a patient with pelvic pain due to endometriosis. In 60% of cases, when a patient with endometriosis undergoes a simple hysterectomy without bilateral salpingo-oophorectomy for pelvic pain, re-operation for continued pain will be necessary. Even if the patient requires hormone replacement therapy postoperatively, her pain is unlikely to return. A laparoscopy is indicated in the workup of pelvic pain in order to determine the etiology of the pain. If endometriosis is noted, it may be excised, fulgurated or burned by laser. This may offer some relief of the patient's pain; however, relief is usually temporary in a pre-menopausal female. In addition, this patient had a previous laparoscopy with only temporary relief. A radical hysterectomy, usually used to treat cervical cancer, is too invasive for the treatment of endometriosis. It is very unlikely in this scenario to decrease the patient's pain simply by changing her oral contraceptive pill. An endometrial ablation is an acceptable treatment for menorrhagia and will likely not be helpful in this patient. Although Levonorgestrel IUD may effectively relieve endometriosis, it is unlikely to do so in this patient who has been on progestins and oral contraceptives without relief.

A 33-year-old G5P4 woman just delivered her fourth baby without complications. She had gained 50 pounds during this pregnancy and would like to begin a weight loss program as soon as possible. She desires long-term effective contraception, because she doubts she wants more children. She also desires to breastfeed exclusively for six months and has had trouble with this in the past. Which of the following is the most appropriate contraceptive choice for this patient? A. Depot medroxyprogesterone B. Combined estrogen-progestin contraceptives C. Tubal ligation D. Intrauterine device (IUD) E. Essure (Bilateral occluding tubal coils)

D A paucity of data exists regarding the effect of hormonal contraception on breastfeeding. There are concerns that hormones, especially estrogen, may have a negative impact on the quantity or quality of breast milk. Although Depot medroxyprogesterone is a progesterone only contraceptive, it is known to cause weight gain and would not be a good choice in this patient. The IUD is the best choice because it is long term but reversible, and does not affect milk production. Tubal ligation and Essure are permanent sterilization and would not be best for a patient who may desire more children.

A 41-year-old G3P3 reports heavy menstrual periods occurring every 26 days lasting eight days. The periods have been increasingly heavy over the last three months. She reports soaking through pads and tampons every two hours. She has a history of three uncomplicated spontaneous vaginal deliveries and a tubal ligation following the birth of her last child. On pelvic examination, the cervix appears normal and the uterus is normal in size. Which of the following tests or procedures would be most useful in further evaluation of this patient's complaint? A. Follicle stimulating hormone level B. Prolactin level C. Coagulation studies D. Pelvic ultrasound E. Hysteroscopy

D A pelvic ultrasound would image the endometrium and assess for endometrial pathology such as polyps or submucosal fibroids. In the absence of menopausal symptoms, FSH is unlikely to be helpful. The patient is unlikely to have a coagulation disorder, as she has had three spontaneous vaginal deliveries without postpartum hemorrhage. Hysteroscopy would not be helpful if the cause of abnormal bleeding is myometrial pathology such as intramural and subserosal fibroids or adenomyosis. Hyperprolactinemia is found with prolactin-secreting adenomas associated with amenorrhea.

A 42-year-old G3P3 comes to the office after noticing a breast mass while performing a breast self-exam. She is in good health and has normal menstrual cycles. Physical exam is significant for a 2 cm dominant breast mass. The remainder of the exam is normal. A mammogram obtained today shows no abnormalities. A fine needle aspiration was negative, and the mass persisted. What is the most appropriate next step in the management of this patient? A. Reassurance and observation B. Obtain an MRI of the chest C. Breast ultrasound D. Perform an excisional biopsy E. Repeat mammogram in two months

D A specimen obtained on fine-needle aspiration is examined both histologically and cytologically. An excisional biopsy should be performed when the results are negative, due to the possibility of a false-negative result. It can, however, prevent the need for other diagnostic testing and is the appropriate next step. Breast ultrasound can be used to distinguish between a cyst and a solid mass. Fine needle aspiration under ultrasound guidance can help distinguish a fibroadenoma from a cyst and exclude cancer in certain situations. A normal mammogram does not rule out breast cancer and there is no need to repeat it in two months. There are no indications for obtaining an MRI of the chest in the initial diagnosis of this patient.

A 23-year-old G1P0 woman presents in labor at term. Her prenatal course was uncomplicated. She delivers a 3500 gram infant spontaneously after oxytocin augmentation of labor. Immediately postpartum, there is excessive bleeding greater than 2000 cc. There are no lacerations and the uterus is found to be boggy. Her blood pressure is 90/40; pulse is 120. Conservative and medical management have failed and you proceed with an exploratory laparotomy. Which of the following is the most appropriate next step in the management of this patient? A. Cervical artery ligation B. Ovarian artery ligation C. External iliac artery ligation D. B-Lynch suture E. Hysterectomy

D A uterine compression suture such as a B-Lynch has been shown to be effective in the management of unresponsive uterine atony. Ligation of a number of pelvic vessels can lead to reduction in the vascular pressure in the pelvis thus controlling hemorrhage. This is especially true with internal iliac artery (hypogastric artery) ligation. However, ligation of the ovarian arteries should not be undertaken as a primary approach. Ligation of the external iliac artery results in devascularization of the leg and, therefore, should not be performed. If these more conservative maneuvers fail, hysterectomy may be necessary but should be a last resort considering the age and parity of the patient.

"A 25-year-old woman presents complaining of a ""wart"" on her vulva for one year. The area has been treated several times in the past year with several applications of trichloroacetic acid in the office, as well as a 12-week course of Imiquimod cream at home. The lesion never fully resolved with either treatment. On examination, an area of verrucous lesions approximately 1 cm x 1.5 cm is noted on the left labia majora near the fourchette. Upon application of 5% acetic acid, the area becomes acetowhite. There are no other colposcopic abnormalities. The remainder of the exam is normal. A Pap smear six months ago is normal. What is the most appropriate next step in the management of this patient? A. Continued treatment with Imiquimod B. Repeat Pap smear today C. Laser vaporization of the lesion D. Vulvar biopsy of lesion E. Intra-lesional Interferon injection"

D A vulvar lesion unresponsive to treatment needs a biopsy. In addition to testing for invasive cancer, the biopsy can also ensure that your diagnosis and treatment are correct. If the initial diagnosis of condyloma is unsure, a biopsy should be performed prior to initiating therapy. Imiquimod would not be recommended, as this patient previously had a full treatment without total response. Prior to initiating treatment again, a tissue diagnosis is recommended. A repeat Pap is not indicated for a vulvar lesion, and, prior to using laser vaporization to destroy the lesion, a biopsy should be done to ensure that the lesion is not cancer. Interferon is not effective in the treatment of HPV.

A 23 year-old G1P1 delivered vaginally a 42-week infant after a prolonged induction of labor. She had an epidural, with an indwelling catheter for 36 hours and three IV sites for her intravenous medications. She now complains of lower abdominal pain, frequency and dysuria. Her vital signs are temperature 98.6°F , 37°C; pulse 70; blood pressure 100/60; and respirations 12. On examination, her lungs are clear, cardiac exam is normal, abdomen is soft, uterine fundus is firm and nontender, and she has mild suprapubic tenderness. Which of the following organisms is most likely causing her discomfort? A. Group A streptococcus B. Gardnerella vaginalis C. Chlamydia trachomatis D. Escherichia coli E. Group B Streptococcus

D Acute cystitis is a common complication after vaginal delivery and the risk increases with the use of an indwelling catheter. The most common cause of acute cystitis infection is gram-negative bacteria. The major pathogens are E. coli (75%), P. mirabilis (8%), K. pneumoniae (20%), S. faecalis (<5%), and S. agalactiae.

A 68-year-old G3P3 comes to the office due to breast tenderness. She is in good health and not taking any medications. Family history is significant for her 70-year-old sister recently diagnosed with breast cancer. On breast examination, her breasts have no lesions; there are no palpable masses, nodules or lymphadenopathy. Her last mammogram was four months ago and was normal. What is the most appropriate next step in the management of this patient? A. Order a mammogram B. Order a breast ultrasound C. Obtain genetic testing (BRCA-1 and BRCA-2 mutations) D. Reassurance E. Order a breast MRI

D Age and gender are the greatest risk factors for developing breast cancer. Having one first-degree relative with breast cancer does increase the risk. A women's risk of developing breast cancer before menopause is increased if she is BRCA1 or BRCA2 positive; however, these genetic mutations occur in a low percentage of the general population. There is no indication for a mammogram since the patient's last mammogram was normal four months ago. Ultrasound and MRI would not add valuable information especially in the setting of a normal mammogram and no masses on physical examination. Genetic testing is not indicated in this case as there is no strong family history and the sister with breast cancer was postmenopausal at time of diagnosis.

A 24-year-old G2P2 woman with a history of two prior Cesarean deliveries desires a tubal ligation for permanent sterilization. She has two daughters, who are 3 and 1 years old. She is very sure she does not desire any more children. She is happily married and is a stay-at-home-mom. What is the strongest predictor of post-sterilization regret for this patient? A. Not working outside the home B. Parity C. Marital status D. Age E. Children's gender

D Approximately 10% of women who have been sterilized regret having had the procedure with the strongest predictor of regret being undergoing the procedure at a young age. The percentage expressing regret was 20% for women less than 30 years old at the time of sterilization. For those under age 25, the rate was as high as 40%. The regret rate was also high for women who were not married at the time of their tubal ligation, when tubal ligation was performed less than a year after delivery, and if there was conflict between the woman and her partner.

A 30-year old G2P1 woman is at 11 weeks gestation. She had a triple screen with her last pregnancy and would like to have aneuploidy screening with the current pregnancy. Which of the following screening tests will provide the highest detection rate for trisomy 21 for this patient? A. First trimester combined test B. Triple screen C. Quad screen D. Sequential screen E. Serum integrated screen

D All of the tests screen for trisomy 21 and trisomy 18. Detection rates provided at a 5% false positive screen rate. • First trimester combined test: first trimester nuchal translucency, PAPP-A (pregnancy associated plasma protein A) and Beta-hCG - 85% Detection Rate • Triple screen: second trimester AFP (alpha fetoprotein), Beta-hCG, uE3 (unconjugated estriol) - 69% Detection Rate • Quad screen: (second trimester Triple screen + inhibin A) - 81% Detection Rate • Sequential screen: (first trimester NT and PAPP-A + second trimester quad screen) - 93% Detection Rate • Serum integrated screen, when unable to obtain nuchal translucency: (first trimester PAPP-A + second trimester quad screen) - 85-88% Detection Rate

A 28-year-old G1 presents for prenatal care. Her periods have been irregular and she does not recall when the last one occurred. She is healthy and denies any medical problems. The uterus is 10 weeks in size and there are no adnexal masses. At this point in time, what is the best way to date the pregnancy? A. Qualitative serum hCG B. Quantitative serum hCG C. Ultrasound measurement, gestational sac D. Ultrasound measurement, crown-rump length E. Uterine size on pelvic exam

D All the above can potentially be used to help date a pregnancy; however, ultrasound measurement of crown-rump length is considered the most reliable (+/- 4 to 5 days) in the first trimester. Other means to date the pregnancy include: fetal heart tones that have been documented for 20 weeks by a non-electronic fetoscope or for 30 weeks by Doppler; it has been 36 weeks since a positive serum or urine hCG pregnancy test was performed by a reliable laboratory;

A 17-year-old nulliparous sexually active female presents to the emergency room with acute right lower quadrant pain and nausea for 12 hours. Her periods have always been irregular, with her last one six weeks ago. She is otherwise completely healthy. She appears in mild distress. Physical examination: temperature 100.2°F (37.9°C); blood pressure 110/60; heart rate 108 beats/min. She has moderate abdominal tenderness with right greater than left pelvic tenderness. Pelvic exam reveals normal external genitalia and pink-tinged discharge is noted on speculum examination. Bimanual/rectovaginal exam confirms mild cervical motion tenderness and fullness in the right adnexa with moderate tenderness and some voluntary guarding. What is the single most important test to obtain? A. Pelvic ultrasound B. CT scan of the abdomen and pelvis C. GC and chlamydia DNA probe D. Beta-hCG E. CBC with differential

D Although all of the tests listed above may be considered, it is imperative to obtain a Beta-HCG to rule out an ectopic pregnancy.

A 36-year-old G5P4 woman with no prenatal care presented in active labor with a blood pressure of 170/105 and 3+ proteinuria. Fundal height is 28 cm. Fetal heart tones were found to be in the 170s with decreased variability and a sinusoidal pattern. Resting uterine tone was noted to be increased and she was having frequent contractions (every 1-2 minutes). The patient complained of bright red vaginal bleeding for the past hour. Based on this history, what is the most likely etiology of her vaginal bleeding? A. Uterine rupture B. Placenta previa C. Bloody show D. Abruption placenta E. Cervical trauma

D Although all the options above can result in third trimester vaginal bleeding, the most likely cause in this patient is placental abruption. This diagnosis goes along with the tachysystole on tocometer and evidence of fetal anemia (tachycardia and sinusoidal heart rate pattern) on the heart rate tracing. Hypertension and preeclampsia are risk factors for abruption. She has no history of cervical trauma.

A 65-year-old G2P2 postmenopausal woman with a remote history of stage I, grade 1 endometrial cancer treated with surgery 15 years ago returns to your office for a health maintenance examination. During a review of systems, the patient reports several months of a dry cough, progressive dyspnea on exertion, and swelling in her legs. She is a non-smoker, but her now deceased husband smoked heavily. She saw her family physician, who initially treated her with a short course of antibiotics; however, because of persistent symptoms a chest x-ray was obtained and revealed a bilateral pleural effusion and a suspicious pulmonary nodule. Her examination is notable for decreased breath sounds at the lung bases, a normal abdominal exam, and a pelvic exam without any suspicious masses or nodularity. She has pitting edema in both of her lower extremities. What is the most appropriate next step in the management of this patient? A. Obtain a Doppler of her lower extremities B. Refer to oncologist C. Refer to palliative care D. Refer to pulmonologist E. Repeat chest x-ray in three months

D Although recurrent endometrial cancer can present as multiple pulmonary nodules, this patient is unlikely to have a recurrence of her endometrial cancer given the initial early stage and remote timing of her cancer diagnosis. The most appropriate next step is to refer her to a pulmonologist (or cardiologist) for a thorough work-up. The finding of pleural effusions and lower extremity edema point towards a cardiopulmonary etiology; however, the finding of a solitary lung nodule in a patient exposed to second hand smoke certainly suggests the possibility of a primary lung cancer. Referral to palliative care would be premature at this point. A Doppler ultrasound to rule out a deep venous thrombosis is reasonable, but typically of more utility in the setting of unilateral edema, and still would not address the need to evaluate her lung findings

A 42-year-old G3P3 woman comes to the office after noticing a breast mass while performing a breast self-exam. She is in good health and has normal menstrual cycles. Family history is significant for multiple first- and second-degree relatives having breast cancer. Physical exam reveals a 2 cm dominant breast mass. The remainder of the exam is normal. A mammogram obtained today shows no abnormalities. What is the most appropriate next step in the management of this patient? A. Reassurance and observation B. Obtain genetic testing for BRCA1 and BRCA2 C. MRI of the breast D. Fine needle aspiration E. Repeat mammogram in two months

D Any solid dominant breast mass on exam should be evaluated cytologically, with a fine needle aspiration (FNA), or histologically, with an excisional biopsy. In this scenario, an MRI should not be part of the initial work-up for the patient's palpable breast mass. Testing for genetic mutations is indicated for patients with a strong family history of breast cancer, but diagnosis is the most important next step in the management of this patient. A normal mammogram does not rule out the presence of cancer, and there is no reason to repeat the mammogram in two months, especially considering that the first one was normal.

Case with what treatment for bacterial vaginosis ? a. Metronidazole for patient b. Metronidazole for patient and partner

a. Metronidazole for patient

A 36-year-old G0 woman presents to the emergency department accompanied by her female partner. The patient notes severe abdominal pain. She states that this pain began 2-3 days ago and was associated with diarrhea as well as some nausea. It has gotten progressively worse and she has now developed a fever. Neither her partner, nor other close contacts, report any type of viral illness. She had her appendix removed as a teenager. On examination, her temperature is 102.0°F (38.9°C), her abdomen is tender with mild guarding and rebound, and she has an elevated white count. On pelvic examination, she is exquisitely tender, such that you cannot complete the examination. Pelvic ultrasound demonstrates bilateral 3-4 cm complex masses. What is the most likely underlying pathogenesis of her illness? A. Diverticulitis B. Gastroenteritis C. Reactivation of an old infection D. Ascending infection E. Pyelonephritis

D Although salpingitis is most often caused by sexually transmitted agents such as gonorrhea and chlamydia, any ascending infection from the genitourinary tract or gastrointestinal tract can be causative. The infection is polymicrobial consisting of aerobic and anaerobic organisms such as E. coli, Klebsiella, G. vaginalis, Prevotella, Group B streptococcus and/or enterococcus. Although diverticulitis and gastroenteritis should be part of the differential diagnosis initially, the specific findings on examination and ultrasound are more suggestive of bilateral tubo-ovarian abscesses. Even though this patient does not have the typical risk factors for salpingitis, the diagnosis should be considered and explained to the patient in a sensitive and respectful manner. The patient should also be questioned separate from her partner regarding the possibility of other sexual contacts.

A 16-year-old nulliparous female presents to the emergency department with a two-day history of abdominal pain, nausea and vomiting. She is sexually active with a new partner and is not using any form of contraception. On examination, her temperature is 100.2°F (37.9°C), and she has bilateral lower quadrant pain, with slight rebound and guarding. On pelvic examination, she has purulent cervical discharge and cervical motion tenderness. Her white count is 14,000/mcL. What is the most appropriate next step in the management of this patient? A. Oral amoxicillin clavunate and doxycycline B. Oral metronidazole and doxycycline C. IV metronidazole and doxycycline D. IV cefotetan and doxycycline E. No treatment until culture results are back

D Although some patients can be treated with an outpatient regimen, this patient should be hospitalized for IV treatment, as she has nausea and vomiting so she might not be able to tolerate oral medications. She is also at risk for non-compliance with an outpatient treatment regimen. It is important to treat aggressively in order to prevent the long-term sequelae of acute salpingitis. You would not wait for culture results before initiating treatment. Her recent sexual contacts should also be informed (by her and/or with her consent) and treated. According to the 2010 CDC treatment guidelines, there are two options for parenteral antibiotics covering both gonorrhea and chlamydia. Cefotetan or cefoxitin PLUS doxycycline or clindamycin PLUS gentamicin. For outpatient treatment, the 2010 CDC guidelines recommend ceftriaxone, cefoxitin, or other third-generation cephalosporin (such as ceftizoxime or cefotaxime) PLUS doxycycline WITH or WITHOUT metronidazole. There are alternative oral regimens as well. http://www.cdc.gov/std/treatment/2010/pid.htm

36-year-old G2P1 woman presents for her first prenatal visit at 11 weeks gestation. She has a two-year history of chronic hypertension treated with lisinopril and labetalol. In addition, she has hypothyroidism treated with levothyroxine, and recurrent herpes, for which she is on chronic acyclovir suppressive therapy. She takes amitriptyline for migraine headaches. Which of her medications is contraindicated in pregnancy? A. Levothyroxine B. Labetalol C. Acyclovir D. Lisinopril E. Amitriptyline

D Amitriptyline, levothyroxine, labetalol and acyclovir are medications that are frequently used in pregnancy and generally are felt to have acceptable safety profiles. The use of angiotensin converting enzyme inhibitors, such as Lisinopril, beyond the first trimester of pregnancy has been associated with oligohydramnios, fetal growth retardation and neonatal renal failure, hypotension, pulmonary hypoplasia, joint contractures and death. Amitriptyline is used in pregnancy to treat migraine headaches and has a good safety profile.

A 33-year-old G2P1 woman at eight weeks presents to the clinic. This is an unplanned pregnancy. She had planned a tubal ligation six years ago when she was diagnosed with pulmonary hypertension, but was unable to have the procedure. She states her pulmonary hypertension has been stable, but she gets short of breath when climbing stairs. She sleeps on one pillow at night. What is the concern for her during this pregnancy? A. There are no additional concerns compared to a normal pregnancy B. She will need a Cesarean section at delivery C. Her baby is at increased risk for pulmonary hypoplasia D. The mother's mortality rate is above 25% E. Epidural analgesia is contraindicated

D Among women with cardiac disease, patients with pulmonary hypertension are among the highest risk for mortality during pregnancy, a 25-50% risk for death. Management of labor and delivery is particularly problematic. These women are at greatest risk when there is diminished venous return and right ventricular filling which is associated with most maternal deaths. Similar mortality rates are seen in aortic coarctation with valve involvement and Marfan syndrome with aortic involvement. The baby is not at increased risk of pulmonary hypoplasia or Marfan's due to the mother's condition.

A 24-year-old nulliparous woman comes into the office because she has not had her menses for six months. She is in good health and not taking any medications. She is not sexually active. She does well in graduate school, despite her demanding new program. Her height is 5 feet 6 inches and her weight is 104 pounds. Her vital signs are normal. Her physical examination, including a pelvic examination, is completely normal. What is the most likely reason for her amenorrhea? A. Ovarian dysfunction B. Thyroid disease C. Premature ovarian failure D. Hypothalamic-pituitary dysfunction E. Pregnancy

D Anorexia nervosa or significant weight loss may cause hypothalamic-pituitary dysfunction that can result in amenorrhea. A lack of the normal pulsatile secretion of gonadotropin releasing hormone (GnRH) leads to a decreased stimulation of the pituitary gland to produce follicle stimulating hormone (FSH) and luteinizing hormone (LH). This leads to anovulation and amenorrhea. Although testing for thyroid dysfunction may be indicated, she has no other symptoms to suggest thyroid disease. While ovarian dysfunction/failure, premature ovarian failure and pregnancy cause amenorrhea, they are unlikely in this case.

Indicate the true statement concerning Duncan's method of placental separation: A. it is correlated with lower bleeding than in Schultze's method. B. it begins centrally. C. it occurs in 80% of cases. D. more frequently occurs in twin pregnancy. E. usually does not lead to a hematoma formation.

E. usually does not lead to a hematoma formation.

A 29-year-old G0 woman presents with a 10-month history of pelvic pain. Her pain is constant, rated as five out of 10 and is greater with intercourse especially on deep penetration. She was hospitalized for pelvic inflammatory disease and right tubo-ovarian abscess 14-months ago and was treated with parenteral antibiotics. She has a persistent right adnexal tubular mass on ultrasound. She desires future fertility. Her vital signs are normal. Pelvic examination reveals a retroverted, normal-sized uterus with limited mobility and marked tenderness and fullness in the right adnexa. Findings at the time of laparoscopy include multiple filmy and dense adhesions between the posterior uterus and cul-de-sac, normal left fallopian tube and ovary, and large right hydrosalpinx, with a few filmy adhesions between the normal appearing right ovary and distended fallopian tube. In addition to lysis of adhesions, what is the most appropriate procedure for this patient? A. No further procedure is indicated B. Needle aspiration of right hydrosalpinx C. Right salpingostomy D. Right salpingectomy E. Right salpingo-oophorectomy

D Approximately 18-35% of all women with acute pelvic inflammatory disease (PID) develop chronic pelvic pain. The actual mechanisms by which chronic pelvic pain results from PID are not known and not all women with reproductive organ damage secondary to acute PID develop chronic pelvic pain. Chronic pelvic pain is the indication for at least 40% of all gynecologic laparoscopies. Endometriosis and adhesions account for more than 90% of the diagnoses in women with discernible laparoscopic abnormalities. Given this patient's desire for future fertility, conservative surgical intervention is indicated with lysis of adhesions. Retention of the patient's ovaries is appropriate given the patient's age and their normal appearance. Since the patient has a persistent hydrosalpinx and pelvic pain, the right fallopian tube should be removed. Neither a salpingostomy (an incision in the tube) nor aspiration of the tubal fluid would be adequate treatment for this patient.

A 22-year-old G2P1 has a history of a previous postterm pregnancy. She delivered a 3500 g healthy male infant at 42-½ weeks gestation via a Cesarean section secondary to fetal distress. She is currently 15 weeks pregnant, based on an irregular last menstrual period. What is the most appropriate management at this time? A. Plan for a repeat Cesarean section at 38 completed weeks B. Schedule for a repeat Cesarean section if she does not go into spontaneous labor by 40 weeks gestation C. Plan to admit the patient for an induction of labor (a VBAC) if she does not go into spontaneous labor by 41 weeks gestation D. Obtain a fetal ultrasound to date the pregnancy E. Start weekly non-stress tests and amniotic fluid indexes at 40 weeks gestation and proceed with either induction of labor or Cesarean section for a nonreactive non-stress test or oligohydramnios or if patient has not gone into spontaneous labor by 41 weeks gestation

D Approximately 50% of patients with a history of a postterm pregnancy will experience prolonged pregnancy with the next gestation. The diagnosis of postterm pregnancy is based on the establishment of an accurate gestational age. In a patient with irregular menses, it is important to obtain an ultrasound prior to 20 weeks to accurately date the pregnancy. It is reasonable to allow a patient with reassuring fetal surveillance to go past 41 weeks gestation. However, because of a prior Cesarean birth, consideration should be given to delivery before 41 weeks

A 31-year-old G3P0 presents with amenorrhea for six months. She is otherwise in good health and is not taking any medications. She had a miscarriage seven months ago, which was complicated by an infection and required antibiotics and a dilation and curettage procedure. Her examination is normal. Her laboratory results show a Beta-hCG <5 mIU/mL, and normal TSH and prolactin levels. What is the most likely underlying cause of this patient's amenorrhea? A. Chronic endometritis B. Recurrent miscarriages C. Hypothalamic-pituitary amenorrhea D. Asherman's syndrome E. Sheehan's syndrome

D Asherman's syndrome can be caused by curettage or endometritis. The intrauterine synechiae or adhesions result from trauma to the basal layer of the endometrium, which causes amenorrhea. Chronic endometritis may be associated with abnormal uterine bleeding and not amenorrhea. Hypothalamic amenorrhea is unlikely because of the temporal relationship of her amenorrhea to the procedure. Sheehan's syndrome is typically due to severe postpartum hemorrhage leading to pituitary apoplexy.

"A 62-year-old G3P2 previously healthy postmenopausal woman is referred to your office by her internist after a work-up for abdominal bloating revealed a large pelvic mass on transvaginal ultrasound and an elevated CA-125. She has no significant medical history and only a prior appendectomy. The CT scan showed a large heterogenous pelvic mass measuring 20 x 13 x 10 cm. There was a moderate amount of ascites and likely ""omental caking."" There was no significant pelvic or abdominal lymphadenopathy, and the chest x-ray showed only a trace right pleural effusion. On pelvic examination, there is minimal cul-de-sac nodularity and the mass is readily palpable and somewhat mobile. There is an obvious fluid wave. Which of the following is the most appropriate next step in the management of this patient? A. Thoracentesis B. Paracentesis C. PET scan D. Exploratory laparotomy and tumor debulking E. Neoadjuvant chemotherapy"

D Based on this patient's initial evaluation, it seems highly likely that she has an advanced stage ovarian carcinoma. Given her lack of significant medical co-morbidities, and a gynecologic examination suggestive that the tumor would be potentially resectable, it would be appropriate to offer her a surgical intervention. The surgical staging of an apparent ovarian cancer should include a vertical skin incision, sampling ascites for cytology, inspection of the entire peritoneal cavity, total abdominal hysterectomy, bilateral salpingo-oophorectomy, omentectomy, cytoreduction to microscopic disease or to residual disease <1 cm in maximum diameter, and pelvic and para-aortic lymph node dissection if omental disease is <2 cm or if bulky, lymph node disease is present. Thoracentesis to rule out malignant pleural effusion is part of the staging criteria, but may not be justified if the effusion is too small, and would not impact the need to perform surgery. Neoadjuvant chemotherapy is generally considered when a patient has unresectable disease or is a poor surgical candidate, at which point paracentesis for cytologic confirmation would be needed.

19-year-old G1 at 40 weeks gestation reports that she is postterm and wants to be induced. She has had an uncomplicated pregnancy. Her blood pressure is 124/76 and her pulse is 84. What gestational age would define postterm pregnancy? A. The due date B. 40 completed weeks C. 41 completed weeks D. 42 completed weeks E. 43 completed weeks

D By definition, a postterm pregnancy is a pregnancy that has progressed past 42 completed weeks or 294 days.

A 58-year-old G3P3 has been postmenopausal for five years and is concerned about osteoporosis. She has declined hormone therapy in the past. Her mother has a history of a hip fracture at age 82. A physical exam is unremarkable. In addition to weight bearing exercise and vitamin D supplementation, what optimal daily calcium intake should she take? A. None B. 200-300 mg C. 500-800 mg D. 1000-1200 mg E. 1500-2000 mg

D Calcium absorption decreases with age because of a decrease in biologically active vitamin D. A positive calcium balance is necessary to prevent osteoporosis. Calcium supplementation reduces bone loss and decreases fractures in individuals with low dietary intakes. In order to remain in zero calcium balance, postmenopausal women require a total of 1200 mg of elemental calcium per day.

A 45-year-old G4P3 woman presents with vaginal bleeding. Last week, she performed a home pregnancy test that was positive. She thinks her last menstrual period was four months ago. The last time she saw her doctor was eight years ago, with the birth of her last child. She has no serious medical problems, has smoked a pack of cigarettes a day since the age of 20, occasionally has a beer and does not exercise. Abdominal examination reveals a soft abdomen and the fundus palpable just below the umbilicus. Pelvic ultrasound reveals a fundal placenta and a fetus measuring 18 weeks with normal cardiac activity. Vaginal examination reveals a 3-centimeter lesion arising off the posterior lip of the cervix. It easily bleeds with palpation and is hard in consistency. Which of the following is the most likely cause of the bleeding? A. Cervicitis B. Cervical polyp C. Endometrial polyp D. Cervical cancer E. Nabothian cyst

D Cervical cancer can unfortunately complicate pregnancies and presents with bleeding. She is at risk due to lack of screening. Other causes of bleeding need to be ruled out such as cervical incompetence, infection or trauma. Treatment for cervical cancer during pregnancy requires difficult decisions that consider the stage of cancer, appropriate therapy, maternal welfare and fetal welfare.

A 28-year-old nulliparous woman with a low-grade squamous intraepithelial lesion (LSIL) on a Pap smear presents for evaluation. A colposcopy is performed and is satisfactory. A lesion is seen at 3:00 that turns white with acetic acid, has punctations and mosaicism, and is friable. This lesion is biopsied with a pathology report of CIN 1. The patient's endocervical curettage (ECC) is positive for a high-grade lesion. Which of the following is the most appropriate next step in the management of this patient? A. Follow up Pap smear in six months B. Repeat colposcopy in six months C. Cryotherapy D. Cervical conization E. Hysterectomy

D Cervical conization is indicated in this patient who has a positive ECC. Hysterectomy is the treatment for invasive cancer. Waiting six months can potentially be harmful, as the lesion can progress or a higher-grade lesion might already be present. Cryotherapy will not provide a pathologic specimen to rule out invasive cancer, but can be used to treat cervical dysplasia once cancer has been completely ruled out and the entire lesion can be visualized.

A 19-year-old G1 presents at 41 weeks gestation with a fever, spontaneous ruptured membranes and no contractions. Her temperature is 102.6° F (39.2° C); pulse 126. Ultrasound reveals a singleton with decreased amniotic fluid and placenta partially covering the os. The cervix appears long and closed. Why should this patient be delivered by Cesarean section? A. Chorioamnionitis B. Unfavorable cervix C. Oligohydramnios D. Placenta covering the cervical os E. Spontaneous ruptured membranes not in labor

D Cesarean delivery is indicated in this patient because of a placenta previa (placenta covering the internal os). A vaginal delivery is contraindicated in patients with a placenta previa. Post-term pregnancies, chorioamnionitis, oligohydramnios, and term premature rupture of membranes are all acceptable indications for induction and delivery if the patient is a good candidate for initiation of labor. An unfavorable cervix is not a contraindication for a vaginal delivery.

A mother brings her 16-year-old daughter to the doctor because she has not begun menses. She performs poorly in school because of dyslexia. On physical examination, she is 4 feet 11 inches tall, 100 pounds and has Tanner stage I breast and pubic hair growth. Her forehead is wide and her face tapers to her chin, chest is broad, breast nipples are widely spaced and her neck is thickened. No genital tract abnormalities are noted on exam. Which of the following is the most likely cause of her delayed sexual maturation? A. Partial deletion of the long arm of the X chromosome B. Down Syndrome C. Turner Syndrome D. Noonan's Syndrome E. Rokitansky-Kuster-Hauser Syndrome

D Clinically, patients with Noonan's syndrome may have short stature, webbed neck, heart defects, abnormal faces and delayed puberty. Individuals with Noonan's syndrome have a normal karyotype. Partial deletions of the long arm of the X chromosome also cause premature ovarian failure. The average age of puberty in females with Down syndrome is not significantly different than normal females. The genetic defect of Turner's syndrome is the absence of one of the X chromosomes. These females have failure to establish secondary sexual characteristics, short stature and characteristic physical features: pterygium colli, shield chest and cubitus valgus. Rokitansky-Kuster-Hauser Syndrome causes vaginal and uterine agenesis and is not suspected in this case due to the normal pelvic exam findings.

A 16-year-old patient has a new boyfriend and comes in to discuss contraception. She is well aware of the importance of preventing sexually transmitted infections and specifically wants to know about prevention of pregnancy. Other than abstinence, the most effective method of birth control in this patient is: A. The male condom B. The diaphragm with spermicide C. Oral contraceptives D. Depo-Provera E. The contraceptive ring

D Contraceptive methods with <1% pregnancy rates (typical use) are Depo-Provera, IUD, sterilization (male or female), and Implanon. Oral contraceptives have a 3-5% pregnancy rate with typical use, and the male condom has a 12% pregnancy rate. Eight percent of women will experience an unintended pregnancy after one year of typical use with a contraceptive ring. Of the methods listed, the diaphragm with spermicide has the highest failure rate (18%) with typical use.

A 24-year-old G2P1 woman is undergoing a Cesarean section for placental abruption. She presented to labor and delivery with severe abdominal pain and heavy vaginal bleeding. The fetus was delivered uneventfully. The placenta delivered with a significant clot attached to the maternal surface. The patient continues to bleed from the placental bed. Estimated blood loss is 1500 ml. The operative team decides to give her fresh frozen plasma (FFP) to replace which of the following components? A. Platelets B. Von Willebrand's factor C. Red blood cells D. Fibrinogen E. Factor X

D Correcting coagulation deficiencies requires replacing all necessary blood components. Fresh frozen plasma contains fibrinogen, as well as clotting factors V and VIII. Cryoprecipitate contains fibrinogen, factor VIII and von Willebrand's factor. Neither of these preparations contains red blood cells or platelets, which must be given separately.

A 23-year-old G0 woman reports having a solitary, painful vulvar lesion that has been present for three days. This lesion has occurred twice in the past. She states that herpes culture was done by her doctor during her last outbreak and was negative. She is getting frustrated in that she does not know her diagnosis. She has no significant previous medical history. She uses oral contraceptives and condoms. She has had four sexual partners in her lifetime. On physical examination, a cluster of three irregular erosions with a superficial crust is noted on the posterior fourchette. Urine pregnancy test is negative. You suspect recurrent genital herpes. How do you explain the negative culture? A. Cultures were taken too early B. Oral contraceptives affect the growth of the virus C. The cultures were refrigerated prior to transport to the lab D. Herpes cultures have a 10-20% false negative rate E. The herpes virus cannot be recovered with recurrent infections

D Culture is the gold standard in the diagnosis of herpes. They are highly specific, yet sensitivity is limited. It is best to culture the lesion very early in the course. The blister is unroofed and the base is vigorously scraped. The herpes virus can theoretically be isolated from both primary and recurrent infections. This patient very likely presented too late in the course for a useful culture. Oral contraceptives do not affect the growth of viruses. While serum antibody screening can be performed, it indicates lifetime exposure and would not answer the question as to the etiology of the specific lesion. Alternatively, DNA studies such as the polymerase chain reaction can be done, if available.

"A 35-year-old woman presents for a health maintenance examination. She is present with her husband who appears to be over-bearing and answers all the questions for her. She defers to him without resistance. During the examination, he gladly leaves the room. Her examination is entirely normal without any signs of bruising, trauma, or injuries. When you ask her if she is in a relationship with a person who threatens or physically hurts her, she denies this and quickly dresses and leaves the exam room to meet her husband who is waiting for her outside the room prior to checking out. The following month, she returns again for an appointment alone to discuss ""private matters."" She is worried that her husband will leave her if she refuses to have sex with him even though she doesn't want to, and states he gets angry easily and throws things. She loves him dearly and admits that they do have frequent disagreements, but he has never hit her. She is worried that he might find out that she is seeing the doctor alone, and had to sneak out without him knowing. Which of the following is LEAST likely to be a sign that she is in an abusive relationship? A. Being frightened by threats of violence B. Having objects thrown by partner when angry C. Being restricted from contacting family or friends D. Having frequent disagreements E. Having sex when not wanting to"

D Domestic violence does not always have to manifest in physical abuse. Disagreements and arguments, even heated discussions, are part of a normal relationship. However, physical violence or other abusive behaviors are not. Signs of being involved in an abusive relationship may come in several forms and screening all women at routine ob-gyn visits, during family visits and during preconception visits are recommended by the American College of Ob-Gyn (ACOG). For the pregnant woman, screening should occur at the first prenatal visit, at least once per trimester, and at the postpartum checkup.

A 28-year-old G1P1 woman delivered three days ago and desires to breastfeed her infant, but is having problems since her milk came in with full tender breasts. She is uncomfortable and has engorged breasts. Which of the following strategies may help relieve her discomfort? A. Discontinue breastfeeding for 24 hours to decrease the milk supply B. Cover the breast with cool lettuce leaves C. Increase the interval between breastfeeding sessions to decrease the milk supply D. Nurse every 1.5-3 hours around the clock E. Don't wear a bra until the engorgement subsides

D Engorgement commonly occurs when milk comes in. Strategies that may help include frequent nursing, taking a warm shower or warm compresses to enhance milk flow, massaging the breast and hand expressing some milk to soften the breast, wearing a good support bra and using an analgesic 20 minutes before breast feeding.

A 54-year-old G4P4 who has been menopausal for four years comes to you for a health maintenance examination. She is in good health, eats a balanced diet, exercises regularly, and has never had any menopausal symptoms and wants to know why. You explain to her that some untreated postmenopausal women will have circulating estrogen levels that are adequate to prevent them from experiencing the symptoms of menopause. What is the most likely source of these circulating estrogens? A. Exogenous dietary intake B. Limited adrenal estrogen secretion C. Lingering ovarian estrogen D. Aromatization of circulating androgens E. Continued low-level ovarian production

D Estrogen production by the ovaries does not continue beyond menopause. Estrogen levels in postmenopausal women can be significant, due to the extraglandular conversion of androstenedione and testosterone to estrogen. This conversion occurs in peripheral fat cells and, thus, body weight has been directly correlated with circulating levels of estrone and estradiol.

A 54-year-old G2P2 presents for her health maintenance examination. She has a history of breast cancer treated with mastectomy with reconstruction, chemotherapy, and is currently on tamoxifen. She has been in remission for two years and has been menopausal since the initiation of her chemotherapy. She experiences very mild hot flashes, and is not sleeping well. She appears apprehensive during the examination, although her examination is completely normal except for severe vaginal atrophy. At the conclusion of the office visit, she finally opens up and admits that she has a new boyfriend. She has not had a sexual relationship since her divorce five years earlier, but has been enjoying masturbation. Although excited about initiating sexual activity again, she is obviously concerned. Which of the following is most likely to contribute to sexual dysfunction? A. Sexual desire B. Arousal C. Orgasm D. Dyspareunia E. Body image

D Female sexual dysfunction can be classified as disorders in sexual desire, arousal, orgasm, or sexual pain, and can include any combination of these. In this case, because she states a desire to initiate in sexual activity and has been enjoying masturbation, it is unlikely that she will experience any problems related to desire, arousal, or orgasm. However, in the presence of severe atrophy and lack of estrogen, she may in fact experience pain related to dyspareunia. She should be encouraged to use some form of water-based lubricant to diminish the effects of the vaginal dryness since estrogen is likely contraindicated with her breast cancer diagnosis. Although body image may play a role, it would be classified under the category of sexual desire.

A 24-year-old Rh-negative G2P1 woman is found at 10 weeks gestation to have anti-D antibodies. You follow her closely during this pregnancy and order serial ultrasound examinations. Which of the following fetal ultrasound findings would be most explained by the presence of Rh disease? A. Meconium B. Fetal bladder obstruction C. Oligohydramnios D. Pericardial effusion E. Placenta previa

D Fetal hydrops is easily diagnosed on ultrasound. It develops in the presence of decreased hepatic protein production. It is defined as a collection of fluid in two or more body cavities, such as ascites, pericardial and/or pleural fluid and scalp edema. On occasion, when extramedullary hematopoiesis is extensive, there will be evidence of hepatosplenomegaly. Placentomegaly (placental edema) and polyhydramnios are also seen on ultrasound. Meconium, fetal bladder obstruction, oligohydramnios and placenta previa do not fit the clinical scenario.

A 29-year-old G1P0 woman at 28 weeks gestation who is the wife of basketball player is diagnosed with gestational diabetes. Her mother had a delivery complicated by shoulder dystocia and she is concerned about her own risk. Which of the following is her biggest risk factor for shoulder dystocia? A. Family history B. Tall husband C. Age D. Gestational diabetes E. Parity

D Fetal macrosomia, maternal obesity, diabetes mellitus, postterm pregnancy, a prior delivery complicated by a shoulder dystocia, and a prolonged second stage of labor are all associated with an increased incidence of shoulder dystocia. Although a family history can be indicative of large babies which might place her at additional risk, her gestational diabetes represents her largest risk factor.

A 24-year-old patient complains of cyclic mastalgia since the onset of her period at age 12. The symptoms have increased over the years and were less troublesome when she took oral contraceptives a few years ago. Currently, she takes no medications and is not sexually active. She is a strict vegetarian and eats soy products. She does not smoke and reports she drinks a glass of wine three times a week, and several diet cokes every day. Her mother was diagnosed with breast cancer at age 55. Her breast exam is normal, except for some mild fibrocystic changes. Which of the following elements in her history contributes to her increasing pain? A. Alcohol intake B. Vegetarian diet C. Family history of breast cancer D. Caffeine intake E. Age at menarche

D Fibrocystic breast changes are the most common type of benign breast conditions and occur most often during the reproductive years. There is an increased risk of breast cancer when atypia is present. The changes do not appear distinct histologically (three stages) or mammographically. Fibrocystic disease is often associated with cyclic mastalgia, possibly related to a pronounced hormonal response. Caffeine intake can increase the pain associated with fibrocystic breast changes, so recommending that she decrease her caffeine intake may be helpful.

A 34-year-old G2P2 presents with biopsy-proven vulvar intraepithelial neoplasia, grade 2 (VIN 2). She had undergone routine examination by her primary physician, who performed a Pap smear (normal) and noted multiple warty-type lesions on the labia. The patient describes some mild itching that she self-treated for a yeast infection, with minimal relief. Otherwise, she is completely healthy, except for smoking a half-pack of cigarettes per day. She is sexually active, and is concerned about the impact this will have on her sex life. Examination confirms multiple, whitish raised 0.5 - 1.5 cm papules throughout her labial minora, majora, clitoral hood and perineum. Which of the following is the most appropriate treatment option for this patient? A. Trichloroacetic acid (TCA) B. Skinning vulvectomy C. Observation and expectant management D. CO2 laser ablation of the lesions E. Smoking cessation

D Given the multifocality of the vulvar dysplasia (VIN 2), laser treatment is the best choice. In order to adequately treat these lesions, a complete (skinning) vulvectomy would be the other choice, but would be disfiguring and require removal of the clitoris which would have detrimental effects on her sexual function. Treatment with TCA is recommended for treatment of warts and not VIN. Smoking cessation is strongly recommended regardless, but would not be the sole means of addressing these lesions. Observation is not ideal, given her mild symptoms, moderate grade, and diffuse nature of the lesions.

A 49-year-old G0 reports that her periods have become heavier over the last year. The patient's physical exam is notable for her having a slightly enlarged, irregularly shaped uterus. A pelvic ultrasound confirms the presence of two 2 x 2 cm intramural uterine fibroids. The patient's friend recently had a hysterectomy due to uterine fibroids and menorrhagia. The patient would like to avoid having surgery. She has tried NSAIDs which did not seem to help much. Her endometrial biopsy is negative. She is interested in the medical options for treating symptomatic uterine fibroids. What is the next best step in the management of this patient? A. Aspirin B. Methotrexate C. Estrogen D. Gonadotropin-releasing hormone agonists E. Indomethacin

D Growth of uterine fibroids is stimulated by estrogen. Gonadotropin-releasing hormone agonists inhibit endogenous estrogen production by suppressing the hypothalamic-pituitary-ovarian axis. They can result in a 40-60% reduction in uterine size. This treatment is commonly used for three to six months before a planned hysterectomy in an attempt to decrease the size of the uterus, which may lead to a technically easier surgery and decreased intraoperative blood loss. In patients who are not yet menopausal, once the gonadotropin-releasing hormone agonist therapy is discontinued, the fibroids may grow again with re-exposure to endogenous estrogen. Thus, this therapy may be most useful for women who are close to menopause, as this patient is at age 49. Aspirin and Methotrexate are not effective treatments for fibroids. Methotrexate is used in ectopic pregnancies. Aspirin and Indomethacin will likely not help, as she did not respond to NSAIDs.

A 42-year-old patient presents for a health maintenance examination. Her past medical history, physical exam, and labs are normal. Her body mass index (BMI) is normal. Her family history is significant for hypertension and hypercholesterolemia in her father and diabetes mellitus in her mother. What lifestyle modification is most important for this patient? A. Starting a weight loss diet B. Starting a sugar-free and cholesterol-free diet C. Recording a daily blood pressure D. Starting an aerobic exercise program E. Recording a weekly blood pressure

D Heart disease is the number one killer of women. Lifestyle modifications to reduce her risk, especially considering her family history, are important proactive changes that she can make. Studies show an inverse relationship between the level of physical activity and incidence of death from coronary disease. Exercise would be an appropriate first step with this patient. She does not need to lose weight (normal BMI) and does not need to be on a special diet (normal labs). Recording daily or weekly blood pressures is not necessary, but her blood pressure should be checked once a year.

A 23-year-old G3P2 woman wants to exclusively breastfeed her baby. She is deciding at which hospital she will deliver. Hospital policies that promote breastfeeding include which of the following? A. Uninterrupted sleep for the mother on her first night in the hospital B. Use of a breast pump to help increase the milk supply C. Use of pacifiers to prevent sore nipples D. Unlimited access of mother to baby E. Use of metoclopramide to increase the milk supply

D Hospital policies that promote breastfeeding include getting the baby on the breast within a half hour of delivery and rooming-in for the baby to ensure frequent breastfeeding on demand (i.e. unlimited access).

A 34-year-old G2P2 presents with inter-menstrual bleeding for one year. The bleeding typically occurs two weeks after her menses and last two to three days. The symptoms began one year ago and the bleeding has not changed recently. She is currently taking oral contraceptives. On pelvic examination, the cervix appears normal and the uterus is normal in size and shape. Her urine pregnancy test is negative; an endometrial biopsy is negative for neoplasia. Which of the following tests or procedures would be indicated for further work-up? A. Prolactin level B. Progesterone level C. Hysterosalpingogram (HSG) D. Pelvic ultrasound E. Colposcopy

D Intermenstrual bleeding is frequently caused by structural abnormalities of the endometrial cavity, such as myomas, polyps or malignancy. An ultrasound would be helpful as the next step in diagnosis. Although an HSG might reveal structural abnormalities, it is too invasive for a next step. A colposcopy would not be helpful in the diagnosis, nor would obtaining a Prolactin level, as it would be indicated for the evaluation of anovulatory bleeding. Progesterone levels are not helpful in a patient on oral contraceptives.

A 23-year-old G1P1 woman delivered a healthy infant two days ago. She has had difficulty breastfeeding despite multiple attempts. Her nipples are sore and cracked and she is thinking about exclusively bottlefeeding. The patient's pregnancy was complicated by gestational diabetes and the patient has chronic hypertension and a history of an abnormal Pap. She had a cone biopsy two years ago and had a normal Pap with the current pregnancy. The patient's mother has a history of endometrial and colon cancer and her maternal grandmother and grandfather both had fatal heart attacks in their early sixties. Breastfeeding decreases the risk of which of the following for this patient? A. Type 2 diabetes B. Coronary artery disease C. Cervical cancer D. Ovarian cancer E. Colon cancer

D Human milk is recognized by the American Academy of Pediatrics as an optimal feeding for all infants. The American Academy of Pediatrics recommends exclusive breastfeeding for the first six months after birth. Physicians can influence a patient's feeding choice, and prenatal education is important in the initiation and maintenance of breastfeeding. Nationally representative surveys have noted that women were more likely to initiate breastfeeding if their physicians or nurses encouraged it. Benefits to the mother include increased uterine contraction due to oxytocin release during milk let down and decreased blood loss. Breastfeeding is associated with a decreased incidence of ovarian cancer. Some studies have reported a decreased incidence of breast cancer. Breastfeeding has not been shown to decrease the risk for developing coronary artery disease, cervical dysplasia and cervical cancer or colon cancer in the mother. Breast milk is a major source of Immunoglobulin A which is associated with a decrease of newborn's gastrointestinal infections.

A 26-year-old woman comes to the office due to irregular menses since menarche, worsening for the last six months. The patient has noted increasing hair growth on her chin and most recently hair growth on her chest, requiring that she shave periodically. No one in her family has hirsutism. On exam, you also notice acne on her chin, acanthosis nigricans and temporal balding. Her serum testosterone is elevated. You suspect hyperthecosis. Which of the following might also be associated with this condition? A. Hyperthyroidism B. Hyperprolactinemia C. Atrophic changes of external genitalia D. Deepening of the voice E. Hyperparathyroidism

D Hyperthecosis is a more severe form of polycystic ovarian syndrome (PCOS). It is associated with virilization due to the high androstenedione production and testosterone levels. In addition to temporal balding, other signs of virilization include clitoral enlargement and deepening of the voice. Hyperthecosis is more difficult to treat with oral contraceptive therapy. It is also more challenging to achieve successful ovulation induction. Hyperthyroidism and hyperparathyroidism are not typically associated with hyperthecosis. Hyperprolactinemia is typically associated with amenorrhea and does not cause hirsutism.

A 42-year-old G4P4 presents for management of suspected adenomyosis. She had a tubal ligation four years ago. A pelvic examination shows an enlarged, soft, boggy uterus. A pregnancy test is negative and she is mildly anemic. An ultrasound shows an enlarged uterus with no fibroids. The patient desires definitive treatment for this condition. What is the most appropriate next step in her management? A. Continuous estrogen/progestin therapy B. Endometrial ablation C. GnRH agonist D. Hysterectomy E. Insertion of a levonorgestrel containing intrauterine system

D Hysterectomy is nearly 80% effective in eliminating pain and abnormal bleeding, if she is willing to undergo surgery. Gonadotropin releasing agents are the first choice for medical therapy for the pain, but the problem is that the adenomyosis seems to recur after discontinuing the therapy. Endometrial ablation and insertion of a levonorgestrel-containing intrauterine system are options in women who decline hysterectomy or desire to maintain fertility. For abnormal bleeding problems and desire for uterine conservation, a progesterone intrauterine contraceptive device can also be used to improve irregular bleeding. Hysteroscopic endometrial ablation can be a treatment for adenomyosis.

A 32-year-old G3P2 woman has delivered a previous child with anencephaly. What is the appropriate recommended dose of folic acid for this woman? A. 0.4 mg B. 0.8 mg C. 1.0 mg D. 4 mg E. 8 mg

D In 1991, the Centers for Disease Control and Prevention recommended that all women with a previous pregnancy complicated by a fetal neural tube defect ingest 4 mg of folic acid daily before conception and through the first trimester. In one analysis, this dose of folic acid in women at high risk reduced the incidence of neural tube defects by 85%. The recommended dose for non-high risk patients is 0.4mg/day.

A 38-year-old G2P2 woman is diagnosed with a high-grade squamous intraepithelial lesion (HSIL). Colposcopy is performed, and three biopsies and endocervical curettage are obtained and were normal. Which of the following would be the most appropriate next step in the management of this patient? A. Pap smear in six months B. Colposcopy with directed biopsies in one year C. Cryotherapy D. Cervical conization E. Treat for presumed infection and repeat Pap in 4-6 weeks

D Indications for cervical conization include 1) unsatisfactory colposcopy, including inability to visualize the entire transformation zone, 2) positive endocervical curettage, 3) Pap smear indicating adenocarcinoma in situ, 4) cervical biopsies that cannot rule out invasive cancer, and 5) a substantial discrepancy between Pap smear and biopsy results. The other options are not adequate evaluation. Since there was a significant discrepancy between the HSIL Pap smear result and the normal reading on the biopsies and the ECC, the patient requires a cervical conization such as LEEP or cold knife cone. Cotesting with cytology and HPV can be repeated at 12 and 24 months with further management based on these results. (See ASCCP guidelines: http://www.asccp.org/Portals/9/docs/Algorithms%207.30.13.pdf)

A 23-year-old G1P0 at 39 weeks gestation presents in spontaneous labor. Pregnancy was complicated by gestational diabetes. She delivers a 4200 gram infant with ruddy color and jitteriness. The infant is at immediate risk for which of the following conditions? A. Hyperglycemia B. Anemia C. Thrombocytopenia D. Polycythemia E. Hypercalcemia

D Infants born to diabetic mothers are at increased risk for developing hypoglycemia, polycythemia (hypoxic), hyperbilirubinemia, hypocalcemia and respiratory distress. Thrombocytopenia is not a risk.

A 20-year-old G2P1 is at 41 weeks gestation. Her prenatal course and past history are unremarkable. She has not had any complications with her pregnancy and fetal surveillance is reassuring. Which of the following complications is most likely to occur in this pregnancy? A. Preeclampsia B. Retained placenta C. Postpartum hemorrhage D. Macrosomia E. Placenta abruption

D Postterm pregnancies are associated with macrosomia, oligohydramnios, meconium aspiration, uteroplacental insufficiency and dysmaturity. Gestational diabetes is associated with macrosomia, but is not alone a risk factor for postterm pregnancies. There is no associated risk for preeclampsia in postterm gestations.

A 32-year-old G3P2 woman with a last menstrual period two weeks ago presents with a six-month history of abdominal pain. She has noncyclic intermittent pain, which she describes as crampy and diffuse across the lower abdomen. Her pain is typically relieved with defecation and is associated with loose, watery stools. Onset of the symptoms is associated with a change in stool frequency from once daily to multiple times daily. She also experiences bloating and abdominal distention several times a week. Her medical history is significant for chronic migraines and she denies previous surgery. Her gynecological history is unremarkable. Her abdominal examination is notable for mild tenderness to palpation in the left lower quadrant, and her pelvic examination is normal. What is the most likely diagnosis in this patient? A. Pelvic adhesions B. Diverticulosis C. Endometriosis D. Irritable bowel syndrome E. C. difficile colitis

D Irritable bowel syndrome (IBS) is a common functional bowel disorder of uncertain etiology. It is characterized by a chronic, relapsing pattern of abdominal and pelvic pain, and bowel dysfunction with constipation or diarrhea. IBS is one of the most common disorders associated with chronic pelvic pain. IBS appears to occur more commonly in women with chronic pelvic pain than in the general population. Diagnosis is based on the Rome II Criteria for IBS, which includes at least 12 weeks (need not be consecutive) in the preceding 12 months of abdominal discomfort or pain that has two of three features: 1) relief with defecation; 2) onset associated with a change in frequency of stool; or 3) onset associated with a change in stool form or appearance. The patient's history does not support pelvic adhesions, and diverticulosis (although very common) typically may be asymptomatic unless inflammation/infection develops. In this case, the symptoms for IBS may be indistinguishable from diverticulitis or severe diverticular disease. Although severe endometriosis may affect the lower bowel with constricting and invasive implants, the lack of any gynecologic/menstrual symptoms and the normal pelvic examination essentially excludes this diagnosis. The lack of recent antibiotic exposure essentially rules out the diagnosis of C. difficile.

A 28-year-old G0 woman whose last normal menstrual period was four weeks ago presents with a two-day history of spotting. She awoke this morning with left lower quadrant pain of intensity 4/10. She has no urinary complaints, no nausea or vomiting, and the remainder of the review of systems is negative. She has no history of sexually transmitted infections. She smokes one pack of cigarettes per day and denies alcohol or drug use. Her vital signs are: blood pressure 124/68, pulse 76, respirations 18, and temperature 100.2° F (37.9°C). On examination, she has mild left lower quadrant tenderness, with no rebound or guarding. Pelvic exam is normal except for mild tenderness on the left side. Quantitative Beta-hCG is 400 mIU/ml; progesterone 5 ng/ml; hematocrit 34%. Ultrasound shows a fluid collection in the uterus, with no adnexal masses and no free fluid. What is the most likely diagnosis? A. Ovarian torsion B. Missed abortion C. Early intrauterine pregnancy D. Unable to establish a diagnosis E. Ectopic pregnancy

D It is difficult to establish a definitive diagnosis at this time. When the Beta-hCG level is below the discriminatory zone (2000 mIU/ml), an early intrauterine pregnancy may not be visualized on ultrasound. Missed abortion, early intrauterine pregnancy and ectopic pregnancy could only be confirmed by serial Beta-hCG levels (at least every 48 hours until a trend is established, usually three levels). Ovarian torsion is a possible diagnosis, however, this is more common with an ovarian mass.

A 32-year-old G3P1 woman presents to your office today because of exposure to hepatitis B. She had vaginal and anal intercourse with a new partner three days ago and did not use condoms. The partner informed her today he was recently diagnosed with acute hepatitis B acquired from intravenous drug use and needle sharing. She has no prior history of hepatitis B infection and has not been vaccinated. She is currently aymptomatic and her examination is normal. Her urine pregnancy test is negative. What is the next best step in the mangement of this patient? A. Check AST, ALT, and HBsAg B. Administer HBIG one dose C. Administer HBIG two doses D. Administer HBIG and start hepatitis B vaccine series E. Administer hepatitis B vaccine series only

D It is estimated that 38% of hepatitis B cases worldwide are acquired from sexual transmission. Post-exposure prophylaxis should be initiated as soon as possible but not later than 7 days after blood contact and within 14 days after sexual exposure. In individuals who are unvaccinated but exposed to persons who are HBsAG positive, recommendations are to receive a dose of HBIG (Hepatitis B Immune Globulin) and the HBV (Hepatitis B Vaccine Series). If the source is HBsAG negative or unknown status, then only the HBV series is used. If the exposed individual has been vaccinated and is a responder then no further treatment is necessary. If the exposed individual is vaccinated and a non-responder, then HBIG plus HBV or HBIG times two doses is used. Because the incubation period for the virus is six weeks to six months, checking liver function and immunologic status at this time is not indicated.

A 69-year-old G3P3 comes in for a health maintenance examination. Her younger sister was recently diagnosed with endometrial cancer and she is concerned about her risk. Your patient experienced her last menstrual period at age 49, and she has not had any bleeding since. Her medications include only a multivitamin and supplemental calcium. She has no other significant family history. Her physical examination including a pelvic examination is normal. She is 5 feet 5 inches tall and weighs 120 pounds. What is the most appropriate management for this patient? A. Endometrial biopsy B. CA125 level C. Ultrasound with measurement of the endometrial lining D. Annual exams E. Refer to genetic counselor for risk assessment

D Less than 5% of women diagnosed with endometrial cancer are asymptomatic. Approximately 80-90% of women with endometrial carcinoma present with vaginal bleeding or discharge as their only presenting symptom. Since this patient does not have any symptoms or risk factors for endometrial cancer, she does not need to have any diagnostic testing. Risk factors for endometrial cancer include late menopause, unopposed estrogen therapy, nulliparity, obesity, Tamoxifen therapy and diabetes mellitus. Although sometimes associated with Hereditary Non-polyposis Colorectal Cancer Syndrome (HNPCC, or Lynch II), endometrial cancer is typically not a genetically-inherited malignancy, and so genetic counseling for risk assessment would not be recommended unless a more significant family history existed. Endometrial cancer ranks as the fourth most common cancer detected in women in the US. In 2010, according to the American Cancer Society, there will be an estimated 43,470 new endometrial cancer cases. It is the most common gynecologic malignancy. Top Five Cancers Detected in Women: • Breast 28% • Lung 14% • Colon 10% • Uterine 6% • Ovary 3% Gynecologic Cancers: • Uterine 52% • Ovary 26% • Cervix 14% • Vulva 5% • Vagina 3%

A 25-year-old G1 woman at six weeks gestation comes to the office because of undesired pregnancy. You discuss with her the risks and benefits of surgical versus medical abortion using misoprostol and mifepristone. Compared to surgical abortion, which of the following is increased in a woman undergoing a medical abortion? A. Post abortion pain B. Lower failure rate C. Long-term psychological sequelae D. Blood loss E. Future infertility

D Medical abortion is associated with higher blood loss than surgical abortion. Early in pregnancy (less than 49 days) both medical and surgical procedures can be offered. Mifepristone (an antiprogestin) can be administered, followed by misoprostol (a prostaglandin) to induce uterine contractions to expel the products of conception. This approach has proven to be effective (96%) and safe. A surgical termination is required in the event of failure or excessive blood loss. Medical termination seems to be more desirable by some patients since they do not have to undergo a surgical procedure. It does not affect future fertility. Any termination of pregnancy, whether medical or surgical, can have psychological sequelae.

A 19-year-old nulliparous woman presents to the office with a two-week history of low pelvic pain and cramping. She has a new sexual partner and is on oral contraception and uses condoms. She is one week into her cycle. She has noted no vaginal discharge, itch or odor. She denies fevers or chills. She does note that she is on a new diet and has started drinking lots of water. As such, she notes that she is urinating much more frequently. Her examination is entirely unremarkable. Which of the following is the most appropriate next step in the management of this patient? A. Pelvic ultrasound B. Pap test C. Wet prep D. Urinalysis E. Testing for chlamydia

D Mildly symptomatic or asymptomatic urinary tract infections are common in female patients. Urinary tract infection must be considered in patients who present with low pelvic pain, urinary frequency, urinary urgency, hematuria or new issues with incontinence. A pelvic ultrasound is not indicated at this point.

Child is born with cleft palate. Mother on different medication. Which of the meds caused cleft palate?

a. Phenytoin

A 23 year-old G1P1 is 5 days post-operative from a Cesarean section for arrest of labor at 7 centimeters. She now complains of minimal abdominal pain and drainage from the right side of the incision. Lochia is normal and she has no urinary complaints. Her vital signs are normal and she is afebrile. On physical exam, her lung and cardiac examinations are normal. Her abdomen and uterine fundus are nontender. Her Pfannenstiel incision has erythema extending 3 centimeters from the incision and there is purulent, bloody drainage coming from the right side. What is the next best step in the management of this patient? A. Initiate intravenous antibiotics B. Initiate oral antibiotics C. Occlusive dressing to the wound D. Open drainage of wound E. Tropical antibiotics to the wound

D Mixed bacteria originating from the skin, uterus and vagina cause wound infections after a Cesarean section. Treatment requires opening the wound, checking for fascia dehiscence and drainage of the purulent material. Packing the wound until it has healed from the bottom up prevents persistent infection. Broad spectrum antibiotics are started, but alone will not treat the abscess. Hot packs may relieve some minor symptoms, but is not adequate treatment alone.

A 35-year-old G1P0 woman with last menstrual period one week ago presents with an eight-month history of pelvic pain. She reports regular menstrual cycles with moderate flow and dysmenorrhea, relieved with ibuprofen. She describes her pain as a deep, achy sensation with frequent sharp exacerbations. She has not been sexually active for the last several months because of dyspareunia and some arguments with her new partner of one year. She has no history of sexually transmitted infections. Her medical history is significant for irritable bowel syndrome, managed with a fiber supplement. She is a business executive. She has smoked one pack of cigarettes a day since age 25, and drinks a glass of wine three times a week. She tries to exercise regularly by running three to four times a week. This new pain is distinctly different from her IBS symptoms. Which of the following risk factors can contribute to increased incidence of pelvic pain in this patient? A. Alcohol use B. Smoking habit C. Occupation D. New partner E. Age

D Most published evidence suggests a significant association of physical and sexual abuse with various chronic pain disorders. The arguments with the new partner allude to possible abuse. Studies have found that 40-50% of women with chronic pelvic pain have a history of abuse. Whether abuse (physical or sexual) specifically causes chronic pelvic pain is not clear, nor is a mechanism established by which abuse might lead to the development of chronic pelvic pain. Women with a history of sexual abuse and high somatization scores have been found to be more likely to have non-somatic pelvic pain, suggesting the link between abuse and chronic pelvic pain may be psychologic or neurologic. However, studies also suggest that trauma or abuse may also result in biophysical changes, by literally heightening a person's physical sensitivity to pain.

A 23-year-old G0 woman presents with complaints of a bilateral nipple itchy sensation for six months. There is no nipple discharge or dry skin. She reports her nipple appears to be swollen at times and there is an erythematous fine rash. She had breast implants placed five years ago, but otherwise has no significant medical problems or surgical history. What is the most likely cause of her symptoms? A. Breast adenoma B. Breast cancer C. Rupture of breast implants D. Chemical irritants E. Mastitis

D Nipple itch is a common symptom of allergies, dry skin, inflammation, or even physical irritation. The itch is characterized by tingling and/or uneasy sensation near the skin surface. Perhaps the most common cause is a chemical irritant such as laundry detergents, soaps, and even perfumes. Itching sensations are not associated with adenomas or ruptured breast implants. In rare circumstances, nipple itching may signify an underlying malignancy such as Paget's disease of the breast. Mastitis is most common in postpartum women and usually presents with pain and fever.

A 32 year-old G2P2 delivered five days ago by uncomplicated vaginal delivery. Her postpartum course thus far has been unremarkable and she is breast feeding without difficulty. She woke up in the middle of the night with terrible upper abdominal pain and chills. She admits that she has had pain like this before, but never this severe. Vital signs reveal blood pressure 120/70; pulse 110; and temperature 101.8°F, 38.8°C. On physical examination, she has abdominal pain located in the right upper quadrant with rebound. Her uterine fundus is well below the umbilicus and nontender. Her lochia is normal. Laboratory tests reveal mild anemia, a slightly elevated white count and slightly elevated liver function tests. What is the most likely etiology of her pain? A. Endomyometritis B. Ruptured ovarian abscess C. Gastric ulcer D. Cholecystitis E. Appendicitis

D Non-pregnancy related conditions must be considered when evaluating women in the postpartum period. Pregnancy puts women at risk for cholelithiasis and, therefore, cholecystitis. Classic symptoms include nausea, vomiting, dyspepsia and upper abdominal pain after fatty foods. Treatment would be dependent on the severity of symptoms, but often involves cholecystectomy that is usually performed laparoscopically. Classic clinical findings for endomyometritis include fever and maternal tachycardia, uterine tenderness and no other localizing signs of infection. This patient is unlikely to have an ovarian cyst. A gastric ulcer would most likely have caused some symptoms during pregnancy. Appendicitis presents with nausea, vomiting, anorexia and abdominal pain.

A 30-year-old G4P3 woman at 24 weeks gestation is found to have an anterior placenta previa. She has a history of three prior Cesarean deliveries. What is the most likely serious complication that can lead to obstetric hemorrhage in this woman? A. Placental abruption B. Uterine dehiscence prior to labor C. Uterine inversion D. Placenta accreta E. Uterine atony

D Placental abruption and uterine atony are both common, but, in the presence of a low-lying anterior placenta in a patient with a history of multiple Cesarean births, the diagnosis of the placenta accreta must be entertained. Placenta accreta is an abnormally firm attachment of the placenta to the uterine wall. The incidence of placenta accreta may be increasing because of the rise in the number of women with previous Cesarean sections. This is a serious obstetric complication leading to retained placenta and severe postpartum hemorrhage. Hysterectomy is frequently required due to intractable hemorrhage at delivery.

Indicate the true statements concerning oxytocin test: 1) it is a stress test; 2) it is a biophysical test assessing the placental efficiency; 3) positive result is concluded on the basis of late decelerations present after at least half of the contractions; 4) it is the test of choice in pregnant woman with placenta praevia. The correct answer is: A. all the above. B. 1,3,4. C. only 4. D. 1,2,3. E. only 1.

D. 1,2,3.

A 24-year-old G2P1 woman at 30 weeks gestation is sensitized to the D antigen. She is Rh negative and received RhoGAM during her first delivery one year ago. Which of the following statements best explains these findings? A. The patient initiated her prenatal care late during the present pregnancy B. The patient was sensitized during the previous pregnancy by receiving the RhoGAM C. Current pregnancy is too close to the first pregnancy D. The amount of fetal maternal hemorrhage was more than previously estimated E. The cause is most likely idiopathic in this case

D On rare occasion, an Rh-negative woman will subsequently be sensitized, despite prophylaxis. The protection afforded by a standard RhoGAM administration is dose-dependent. One dose will prevent Rh sensitization to an exposure of as much as 30 cc of Rh-positive red blood cells. With greater exposure, there is only partial protection and Rh sensitization may occur as a result of failure to diagnose massive transplacental hemorrhage. Alternatively, an Rh-negative woman may be sensitized in the latter part of pregnancy or soon after delivery before the post-delivery prophylaxis dose is given. Inadvertent maternal transfusion of Rh-positive blood may result in Rh sensitization to the D or another red blood cell antigen. Patients may become sensitized if they do not receive RhoGAM following an episode of antenatal bleeding or after an invasive procedure, such as amniocentesis or chorionic villus sampling. In addition, RhoGAM only confers protection against the D antigen. Therefore, despite administration of RhoGAM to Rh-negative patients, they may still become sensitized to other red blood cell antigens. Pregnancy spacing does not affect the presence of the antibody.

"A 38-year-old G1P0 woman undergoes dilation and curettage for a partial molar pregnancy. The patient and her husband are very devastated by the loss of this much-desired pregnancy. Because she feels that her ""reproductive clock"" is ticking away, the patient would like to get pregnant as soon as possible. How long should she wait before attempting pregnancy? A. After recovery from the dilation and curettage B. After the Beta-hCG normalizes C. After she has one normal menstrual cycle D. Six months after negative Beta-hCG levels E. Two years"

D Once evacuation has been accomplished, patients must be followed regularly with serial Beta-hCG levels to insure spontaneous regression. Pregnancy should be avoided during this follow-up period, and for the following six months. Effective contraception (OCP or other hormonal contraception) is strongly recommended to prevent confusion in interpreting a rising Beta-hCG as a post-molar recurrence/progression versus a new, spontaneous pregnancy. Given this patient's age and desire for a pregnancy, waiting two years decreases her fertility and increases her risks of pregnancy complications.

A 36-year-old G1 with type 1 diabetes is diagnosed with intrauterine growth restriction at 33 weeks gestation. What is the most appropriate next step in management? A. Amniocentesis B. Immediate delivery C. Weekly ultrasounds to assess fetal growth D. Antenatal testing of fetal well-being E. Observation

D Once intrauterine growth restriction is detected, the fetus needs to be evaluated periodically for evidence of well-being until delivery is deemed necessary. This will result in once or twice weekly testing, depending on the modality of assessment that is being used. Testing includes: non-stress test (NST), where the fetal heart beat is recorded over a period of at least 30 minutes while looking for accelerations with fetal movement, and the biophysical profile, which includes an ultrasound evaluation of fetal movement, fetal tone, amniotic fluid and breathing. NSTs should be performed twice weekly with at least a weekly AFI. The BPP may be performed weekly. Ultrasound for fetal growth is not useful if more frequent than every two weeks. An amniocentesis for fetal lung maturity can be considered at more advanced gestational age.

A 32-year-old G2P1 is at 42 weeks gestation. Her prenatal course was uncomplicated and she had a first trimester ultrasound confirming dates. Her cervix is 4 cm dilated and 100% effaced. She does not report contractions and states there is good fetal movement. What is the next best step in the management of this patient? A. Ultrasound to assess amniotic fluid volume B. Twice weekly non-stress test (NST) and amniotic fluid index (AFI) C. Daily biophysical profiles D. Induction E. Ultrasound to assess fetal growth

D Optimal management for the patient with a favorable cervix at greater than or equal to 41 weeks gestation is delivery. Her dilation and effacement make it likely her induction will be successful. Induction of labor in a patient with an unfavorable cervix increases the risk of Cesarean section significantly, compared to a patient who goes into spontaneous labor. It is not advisable to follow a patient who is >42 weeks with antepartum fetal testing, such as twice weekly non-stress tests with amniotic fluid index, if the gestational age is certain. Performing an ultrasound to assess fetal growth and/or amniotic fluid volume should not change the management plan which should be induction of labor at this gestational age.

A 23-year-old G1P0 comes into the office after having some light inter-menstrual spotting and cramping. She is currently sexually active and has had unprotected intercourse with two different partners over the past three months. A urine pregnancy test is positive. She does not desire to keep the pregnancy and, after an ultrasound scan in the office reveals a six-week viable intrauterine pregnancy, the patient asks about an abortion, but has no health insurance. What is the most appropriate next step in the management of this patient? a. You inform her that state Medicaid programs are not allowed to cover this service; therefore, you cannot perform the procedure b. You recommend against the procedure due to potential complications with future infertility c. You request she seeks the opinion of both of her partners before undergoing the procedure d. You support her decision for abortion after appropriate counseling e. You inform her that abortion should only be performed after 10 weeks gestation

D Patients requesting abortion should be counseled appropriately regardless of their insurance status and do not have to obtain the consent of their partner to undergo the procedure. Although there are complications associated with pregnancy termination, they are significantly fewer than complications with carrying a pregnancy. Fewest complications occur when termination is done in the first trimester. From 1990 (the year in which the number of abortions was highest) to 1995, the annual number of legally induced abortions in the United States declined by 15%. Since 1990, factors contributing to the continued decrease in the proportion of pregnancies that ended in abortion might include a decrease in the number of unintended pregnancies, changes in contraceptive practices (including an increased use of condoms among young women), reduced access to abortion services and possible changes in attitudes concerning abortion.

A 32-year-old nulliparous woman presents with amenorrhea for the last three months. She has a long history of irregular cycles, 26 to 45 days apart, for the last two years. She is otherwise in good health and is not taking any medications. She is sexually active with her husband and uses condoms for contraception. She is 5 feet 4 inches tall and weighs 140 pounds. On exam, she has a slightly enlarged, non-tender uterus. There are no adnexal masses. Which of the following is the most appropriate test to obtain in this patient? A. Thyroid stimulating hormone (TSH) B. Progesterone and estrogen C. Follicle stimulating hormone and luteinizing hormone levels (FSH and LH) D. Urine pregnancy test E. Pelvic ultrasound

D Pregnancy is the most common cause of amenorrhea. It is important to consider it early in the workup to avoid unnecessary tests, procedures and treatments that may be contraindicated during pregnancy. Although the patient has a history of irregular cycles and is using condoms for contraception, it is important to first rule out pregnancy before initiating further work-up.

A 22-year-old G2P1 woman presents for prenatal care at approximately 10 weeks gestation. Her first pregnancy was complicated by preterm premature rupture of the membranes at 28 weeks gestation. Which of the following interventions could reduce the risk of preterm premature rupture of the membranes during this pregnancy? A. Bedrest B. Placement of a cerclage C. Placement of a Tertbutaline pump D. 17 alpha-hydroxyprogesterone E. Nifedipine

D Premature rupture of the membranes occurs in approximately 10-15 % of all pregnancies. Preterm premature rupture of the membranes between 16 and 26 weeks gestation is identified in 1% of pregnancies. Preterm premature rupture of the membranes occurs in 1/3 of all preterm deliveries. The reported recurrence rate for preterm premature rupture of the membranes is approximately 32% when it occurred in the index pregnancy. Bedrest and tocolytics have not been shown to reduce the risk for PPROM, and may have detrimental effects to the mother. A cerclage may be indicated for patients with a history of an incompetent cervix. 17 alpha-hydroxyprogesterone has been shown to reduce the risk of premature labor.

A 24-year-old G1P0 woman at 34 weeks gestation is planning to breastfeed her baby. Several hormones of pregnancy are responsible in order for the breasts to produce milk. Which of the following hormones is responsible for synthesis of milk? A. Estrogen B. Oxytocin C. Cortisol D. Prolactin E. Human placental lactogen

D Progesterone, estrogen, and placental lactogen, as well as prolactin, cortisol, and insulin, appear to act in concert to stimulate the growth and development of the milk-secreting apparatus of the mammary gland. Prolactin is responsible for the synthesis of milk, but although present in large quantities during gestation, its action is inhibited by the hormones of pregnancy, particularly estrogen and progesterone. After delivery, large amounts of prolactin continue to be secreted, milk is produced after the inhibitory action of estrogen and progesterone is lifted.

A 37-year-old G4P3 woman presents in labor at term. Her medical history and prenatal course are uncomplicated. She delivers a 3500 gram infant spontaneously after oxytocin augmentation of labor. Immediately postpartum, there is excessive bleeding greater than 2000 cc. She has an IV in place. There are no lacerations and the uterus is found to be boggy. Which of the following is the most appropriate next step in the non-operative management of this patient? A. Intravenous misoprostol B. Intramuscular misoprostol C. Intravenous prostaglandin F2-alpha D. Intramuscular prostaglandin F2-alpha E. Intravenous oxytocin push

D Prostaglandin F2-alpha should be administered intramuscularly. It could also be injected directly into the uterine muscle. Prostaglandin F2-alpha should not be administered IV, as it can lead to severe bronchoconstriction. Oxytocin is administered as a short time, rapid infusion of a dilute solution (20-80 units in a liter) and not as an IV bolus/push. Misoprostol (800 to 1000 mcg) can be administered orally or rectally and is not administered IV or IM.

An 18 year-old G1P0 at 16 weeks gestation was admitted 4 days ago because of back pain, chills and fever. She has been receiving aggressive hydration and taking broad-spectrum antibiotics but continues to have spiking fever up to 102.0°F (38.9°C). Work-up reveals a right ureteral obstruction secondary to calculi. Which of the following is the most appropriate next step in the management of this patient? A. Aggressive hydration B. Change antibiotics C. Continue present antibiotics D. Pass a double-J ureteral stent E. Perform percutaneous nephrostomy

D Renal infection is the most common serious medical complication of pregnancy. Initially aggressive intravenous hydration is given to ensure adequate urinary output. Antimicrobials are begun promptly after diagnosis. The majority of patients are afebrile by 72 hours. If there is no clinical improvement by 72 hours, further evaluation is warranted including sonography to look for urinary tract obstruction (abnormal ureteral or pyelocaliceal dilatation) or calculi. Obstruction can be relieved by cystoscopic placement of a double-J ureteral stent unless long-term stenting is foreseen, then percutaneous nephrostomy is indicated. Surgical exploration is required in up to 2% of women if other conservative therapies are not successful.

An African-American couple comes to you for preconception counseling. Neither one has any significant family or genetic history. Based on their African-American descent, which of the following blood tests would you recommend? A. MCV and CBC B. Sickle cell preparation and CBC C. Peripheral blood smear and CBC D. Hemoglobin electrophoresis and CBC E. Sickle cell preparation with a hemoglobin electrophoresis, if the sickle preparation is abnormal

D Screening for carriers of both alpha and beta thalassemia is possible by evaluation of red cell indices. Although solubility tests for hemoglobin S or sickle cell preparations can be used for screening, hemoglobin electrophoresis is definitive and preferable because other hemoglobinopathies can also be detected including hemoglobin C trait and thalassemia minor. Although sickle cells can be identified on a blood smear in individuals with sickle cell disease, the cells may be absent in individuals with milder types of sickle cell disease and even in some individuals with severe sickle cell disease. Evaluation of a peripheral smear is not useful in detecting carriers for sickle cell disease.

A 34-year-old G1 woman at eight weeks gestation presents for prenatal care. She is healthy and takes no medications. Family history reveals type 2 diabetes in her parents and brothers. She is 5 feet 2 inches tall and weighs 220 pounds (BMI 40.2 kg/m2). Which of the following is the best recommendation to screen her for gestational diabetes? A. No screening required B. Screen at 24 - 28 weeks with a 50-g oral glucose challenge test C. Screen at 16 - 20 weeks with a 50-g oral glucose challenge test D. Screen now with a 50-g oral glucose challenge test E. Begin an oral hypoglycemic agent now

D Screening should be performed between 24 and 28 weeks in those women not known to have glucose intolerance earlier in pregnancy. This evaluation can be done in two steps: a 50-g oral glucose challenge test is followed by a diagnostic 100-g oral glucose tolerance test (OGTT) if initial results exceed a predetermined plasma glucose concentration. Patients at low risk are not routinely screened. For those patients of average risk screening is performed at 24 - 28 weeks while those at high risk (severe obesity and strong family history) screening should be done as soon as feasible.

A 37-year-old G1P1 has experienced symptoms of depression and difficulty concentrating the week prior to her menstrual period for the last three years, since her tubal ligation. She kept a symptom diary for three months revealing symptoms clustered around her menstrual cycle. She was diagnosed with premenstrual syndrome and began a regular exercise routine with dietary modifications, but only noticed mild relief in her symptoms. Work-up is otherwise unremarkable. Which of the following will most likely alleviate her symptoms? A. Evening primrose oil B. Ginkgo C. Progesterone cream D. Fluoxetine hydrochloride E. Levothyroxine sodium

D Selective serotonin receptor inhibitors increase the amount of active serotonin in the brain and have been found to be effective in alleviating PMS and PMDD symptoms. Patients can take the medication either every day or for 10 days during the luteal phase. Progesterone cream will not help her symptoms.

Mary is a 65-year-old G2P2 with lung metastases from cervical cancer. She was recently weaned from mechanical ventilation after being on the ventilator for four weeks. She has a tracheostomy. Mary currently has worsening pulmonary function and needs to go back on the ventilator or she will die within a few days. Mary's husband, Jim, has power of attorney for Mary's health care decisions. The attending offers Mary a choice of either no ventilation with morphine for comfort or resumption of mechanical ventilation. Mary decides she prefers to go back on the ventilator. Jim prefers that she does not go back on the ventilator because the doctor has said that Mary may never wean off of the ventilator again. Who should make the decision about whether to put Mary back on the ventilator? a. Jim b. All of them together c. Mary's doctor in consultation with Jim d. Mary e. If Jim and Mary cannot agree, consult the hospital ethics committee

D Since Mary is still competent, she can make her own decisions despite the fact that her husband has power of attorney.

A 48-year-old woman presents with complaints of a white, watery nipple discharge for four months. She has been told in the past she had fibrocystic breast changes, but otherwise has no significant medical problems or surgical history. A white nipple discharge is noted on manual expression, but the exam is otherwise normal. She was then sent for a serum prolactin level which was 45 ng/ml (normal below 40 ng/ml). What is the most appropriate next step in the management of this patient? A. Obtain a brain MRI B. Obtain a beta-hCG C. Begin Bromocriptine D. Obtain a fasting prolactin level E. Order a ductogram

D Stimulation of the breast during the physical examination may give rise to an elevated prolactin level. Accurate prolactin levels are best obtained with patients fasting. If these are still elevated, then a brain MRI would be indicated to rule out a pituitary tumor. Although pathologic factors such as hypothyroidism, hypothalamic disorders, pituitary disorders (adenomas, empty sella syndrome), chest lesions (breast implants, thoracotomy scars, and herpes zoster) and renal failure can elevate prolactin levels, a non-significant benign elevation needs to be ruled out first. A ductogram is usually indicated in patients who have bloody discharge from the nipple.

A 39-year-old G4P3 woman with an ultrasound report suggestive of a molar pregnancy is referred to your office. She is asymptomatic. She has three children and was not planning any more pregnancies. Her uterus is 16-weeks size and her Beta-hCG is >200,000 mIU/mL. What is the recommended treatment for this patient? A. Expectant management B. Induction with oxytocin C. Methotrexate D. Suction curettage E. Hysterectomy

D Suction curettage is the standard management for molar pregnancies. Hysterectomy is an option as this particular patient has completed childbearing, however, the morbidity of a hysterectomy is still considered greater than suction curettage. Induction with oxytocin would result in severe bleeding once cervical dilation and contractions developed, and expectant management would risk increased growth and progression of the mole (as well as the similar unnecessary risk of bleeding.) Methotrexate may become necessary if she develops post-molar GTD, but not as a sole means of primary treatment.

A 22-year-old patient with regular periods reports tension, depressed mood and decreased productivity towards the end of each cycle. She is otherwise healthy and maintains a high-profile job. Her past medical history is benign and she denies prior psychiatric problems. She denies smoking and drinks alcohol socially. What is the next best step in the management of this patient? A. Reassure her that her monthly symptoms are normal B. Initiate anti-depressant therapy C. Psychiatry consult D. Ascertain the timing of her symptoms each month E. Initiate psychotherapy

D Symptoms of Premenstrual Dysphoric Disorder occur in the luteal phase and are absent in the beginning of the follicular phase. It is therefore important to document the timing of symptoms each month when considering a diagnosis of Premenstrual Dysphoric Disorder. Additionally, it is important to ascertain that these symptoms are not an exacerbation of an underlying psychiatric disorder before initiating therapy.

A 25-year-old woman, gravida 2, para 1, with chronic hypertension, is at 38 weeks' gestation. Ultrasound examination shows an amniotic fluid index of 4 cm and an estimated fetal weight below the 10th percentile. A nonstress test (NST) is nonreactive with absent variability, and a subsequent contraction stress test (CST) is positive. Her Bishop score is 4. Which of the following should be the next step in managing this patient? A) Cordocentesis for fetal karyotype B) Cordocentesis for fetal blood pH C) Biophysical profile D) Immediate delivery E) Repeat contraction stress test in 1 week

D) Immediate delivery

A 32-year-old G2P2 complains of depression, weight gain and premenstrual bloating. She has suffered from these symptoms for 18 months and they have not responded to dietary changes and avoidance of alcohol and caffeine. Her only medications are multivitamins and herbs to increase her energy. She is very concerned about fatigue that often interferes with caring for her two children. A prospective symptom diary completed by the patient indicates mood symptoms, fatigue and bloating almost every day of the past two months, and regular menstrual cycles accompanied by breast tenderness. She denies feelings of wanting to hurt herself or others. Physical examination is unremarkable. Which of the following conditions is the most likely explanation for this patient's symptoms? A. Panic disorder B. Anxiety disorder C. Anemia D. Hypothyroidism E. Premenstrual dysphoric disorder

D Symptoms of hypothyroidism can mimic typical symptoms of PMS, but symptoms occur more constantly throughout the cycle. Diagnosis involves complete work-up to rule out medical illnesses, including hypothyroidism. Although fatigue can be associated with anemia, her presentation is not consistent with this diagnosis.

A 22-year-old G1 presents at 42 weeks gestation. Her cervix is long and closed. She does not report contractions and states there is good fetal movement. You discuss the benefits of induction at this time versus waiting until she goes into labor spontaneously. She agrees to proceed with an induction. Which of the following means should be used to ripen the cervix in this patient? A. Artificial rupture of membranes B. Membrane stripping C. Oxytocin infusion D. Prostaglandin E1 tablet E. RU486 (progesterone antagonist)

D The American College of Obstetricians and Gynecologists (ACOG) recommendations for the management of postterm pregnancy includes: patient records fetal kick counts, and fetal surveillance using one of the following: NST, CST, biophysical profile and delivery for nonreassuring testing. If the patient has a favorable cervix, induce at 42 weeks and, if the cervix is unfavorable, use cervical ripening agents. Membrane stripping (digital separation of chorioamnion from lower uterine segment) and artificial rupture of membranes cannot be performed in a patient with a closed cervix. Prostaglandins applied locally are the most commonly-used cervical ripening agents. RU486 is not used for cervical ripening.

A 19-year-old G3P0 with spontaneous rupture of membranes for 13 hours presented to labor and delivery. She had no prenatal care. Her vital signs are:blood pressure 120/70; pulse 72; afebrile; fundal height 36 cm; and estimated fetal weight of 2700 gm. Cervix is dilated to 1 cm, 50% effaced, -2 station. Which statement best describes the tracing seen below? A. Normal fetal heart rate with good variability and regular contractions B. Fetal tachycardia with good variability and regular contractions C. Normal fetal heart rate with poor variability and irregular contractions D. Fetal tachycardia with poor variability and regular contractions E. Normal fetal heart rate with good variability and irregular contractions https://www.apgo.org/student/uwise2/unit1intro/unit-1?quiz_id=12

D The baseline fetal heart rate is >160 with no accelerations or variability. There are regular contractions. Prolonged periods of fetal tachycardia are frequentlyfound with maternal fever or chorioamnionitis.

A 23-year-old nulliparous woman presents to the office because she has not had any menses for four months. She has a long history of irregular menstrual cycles since menarche at age 14. She is otherwise in good health and is not taking any medications. She is thin and has chronic anxiety. Her Beta-hCG is < 5 mIU/mL, and her prolactin and TSH levels are normal. What would be the next best diagnostic test to order? A. Estrogen level B. Progesterone level C. Gonadotropin releasing hormone level (GnRH) D. Follicle stimulating hormone and luteinizing hormone levels (FSH and LH) E. Dehydroepiandrosterone sulfate (DHEAS)

D The causes of hypothalamic-pituitary amenorrhea are functional (weight loss, obesity, excessive exercise), drugs (marijuana and tranquilizers), neoplasia (pituitary adenomas), psychogenic (chronic anxiety and anorexia nervosa), and certain other chronic medical conditions. In this case, the next step to make a diagnosis is to obtain FSH and LH levels, which would be expected to be in the low range. You already know that her prolactin level is normal, which is consistent with the diagnosis. Prolactin would be elevated with a prolactin-secreting pituitary adenoma.

A 23-year-old G1 undergoes a vaginal delivery and an episiotomy is performed. Upon inspection, you notice that the episiotomy has extended into the rectal sphincter and mucosa. What is the classification for this laceration? A. 1st degree B. 2nd degree C. 3rd degree D. 4th degree E. Mediolateral episiotomy

D The degree of the lacerations range from 1 through 4. They increase along with the extent of the laceration. A 4th degree laceration involves the rectal sphincter and rectal mucosa. Careful attention to identification and repair of the disrupted ends of the sphincter is important to try to minimize the risk of future fecal or flatus incontinence. A 1st degree laceration involves only the vaginal mucosa. A 2nd degree laceration involves the vaginal fascia and perineum. A 3rd degree laceration involves the rectal partial or complete transection of the rectal sphincter. A 4th degree laceration involves the external anal sphincter, the internal anal sphincter and the rectal mucosa.

"A 47-year-old G3P3 presents with a several month history of progressive abdominal bloating. She has had regular menses her entire life, but recently notes her bleeding to be heavier and occurring ""twice a month."" She is otherwise healthy and does not smoke or drink. On examination, she is 5 feet 5 inches tall and weighs 130 pounds. Her abdominal exam is notable for some mild distension, but no palpable masses. Her pelvic examination is notable for a normal appearing cervix, a deviated, but non-enlarged uterus, and a 10 cm mobile, non-tender right adnexal mass. An office endometrial biopsy reveals complex endometrial hyperplasia without atypia. What is the most likely explanation for the adnexal mass and the findings seen on the endometrial biopsy? A. Fibroid uterus B. Endometrioma C. Metastatic endometrial cancer D. Granulosa cell tumor E. Theca lutein cyst"

D The finding of an adnexal mass in a perimenopausal woman raises the suspicion of a neoplastic process. Because of the new onset of irregular bleeding and the finding of hyperplasia, the most likely explanation would be that of a granulosa cell tumor, an estrogen-secreting tumor. A theca lutein cyst is typically seen in the setting of pregnancy (molar pregnancy) and is often bilateral. A fibroid uterus may present with heavy irregular bleeding, but a pedunculated fibroid mimicking an adnexal mass is unlikely to present with such a bleeding pattern and has no correlation with hyperplasia. Severe endometriosis often presents with dysmenorrhea and is unlikely to develop in the perimenopause.

The most frequent cause of dyspareunia is: . A) Vaginismus B) Endometriosis C) Retroverted uterus D) Inadequate vaginal lubrication E) Pelvic inflammatory disease

D) Inadequate vaginal lubrication

A 23-year-old G1 woman with six weeks amenorrhea presents with lower abdominal pain and vaginal bleeding. Her temperature is 102.0°F (38.9°C) and the cervix is 1 cm dilated. Uterus is eight-week size, tender and there are no adnexal masses. Urine pregnancy test is positive. Which of the following is the most appropriate next step in the management of this patient? A. Observation B. Single-agent antibiotics C. Medical termination of pregnancy plus antibiotics D. Uterine evacuation plus antibiotics E. Laparoscopy plus antibiotics

D The management of septic abortion includes broad-spectrum antibiotics and uterine evacuation. Single agent antimicrobials do not provide adequate coverage for the array of organisms that may be involved and therefore are not indicated. A laparoscopy can be indicated if ectopic pregnancy is suspected, but it is unlikely in this case. Medical termination is not the best option since prompt evacuation of the uterus is indicated in this case.

A 30-year-old G1P1 woman presents to the emergency department with left-sided abdominal pain. Physical examination is notable for a 5 x 6 cm mobile left adnexal mass. An ultrasound is performed, which shows a left ovarian mass with cystic and solid components. Which of the following is the most likely diagnosis in this patient? A. Serous cystadenoma B. Mucinous cystadenoma C. Endometrioid tumor D. Dermoid tumor E. Brenner tumor

D The most common tumor found in women of all ages is the dermoid. The median age of occurrence is 30 years, and 80% occur during the reproductive years. Dermoids may contain differentiated tissue from all three embryonic germ layers. Dermoid tumors can contain teeth, hair, sweat and sebaceous glands, cartilage, bone, and fat.

A 28-year-old G0 underwent a routine Pap smear six weeks ago which showed low-grade squamous intraepithelial lesion (LSIL) with HPV associated changes. She currently has normal regular menses, no pelvic pain and no general complaints. Her previous medical history is negative. She has no previous abnormal Pap smears and no history of sexually transmitted infections. She has smoked a half-pack of cigarettes per day for 10 years and does not drink alcohol or use drugs. She has had three sexual partners in the past. On examination, she has a normal appearing cervix and normal bimanual exam. What is the most appropriate next step in this patient's care? A. Treat with antibiotics and repeat the Pap smear B. Loop ElectroDsurgical Excision Procedure (LEEP) C. Cold knife cone biopsy D. Colposcopy E. Repeat Pap smear today

D The most recent consensus guidelines (2006) state that management of LSIL is initial colposcopic examination (unless the woman is pregnant, postmenopausal or an adolescent). An excisional procedure, such as cold knife biopsy or LEEP, is not warranted without a tissue diagnosis of dysplasia. The Pap smear is merely a screening tool and, as such, cannot formulate a definitive diagnosis. In fact, up to 20% of patients with LSIL on Pap smear have high-grade squamous intraepithelial lesion (HSIL) on colposcopically-directed biopsy. In contrast, up to 50% of patients with LSIL on Pap smear have a negative colposcopy. Antibiotic treatment is not warranted unless there is an infection. If a Pap smear is repeated prior to six to eight weeks following the last one, reparative changes may still be happening to the cervix. This reduces the ability of the test to be a good screening tool. (The false positive and/or false negative rates can be affected). In some instances, LSIL may be followed with serial Pap smears; however, a tissue diagnosis or a colposcopy without evidence of HSIL changes must be done initially.

A 38-year-old G1P0 is admitted at 42 weeks gestation with an anencephalic infant for induction of labor. The attending physician decides not to monitor the baby's heart rate during labor because he would not intervene with a Cesarean section in the event of fetal distress or demise. The physician's action is justified by which one of the following concepts? a. Beneficence to the fetus b. Disability rights c. Maleficence to the fetus d. Non-maleficence to the patient e. Justice for the patient

D The non-maleficence principle expresses the concept that professionals have a duty to protect the patient from harm. Since an anencephalic infant will not survive, performing a Cesarean section on this patient will cause her harm. Beneficence principle expresses the concept that professionals have a duty to act for the benefit of others, and, in this case, performing a Cesarean section will not benefit the fetus.

A 34-year-old G4P3 woman at 36 weeks with a twin gestation presents in labor. She has three prior normal spontaneous vaginal deliveries at term, with the largest infant weighing 3400 grams. Twin A is breech with an estimated fetal weight of 2800 gm and twin B is vertex, with an estimated fetal weight of 3200 gm. Which of the following is an appropriate delivery option for this patient? A. Total breech extraction of twin A, vaginal delivery of twin B B. External cephalic version for twin A, vaginal delivery twin of B C. Operative vaginal delivery for twin A and vaginal delivery for twin B D. Cesarean delivery E. Vaginal delivery for twin A and Cesarean delivery for twin B

D The optimal mode of delivery for twins in which the first twin is in the breech presentation is by Cesarean section. Similar to singletons, if the first twin is breech problems can occur including head entrapment and umbilical cord prolapse. When the presenting twin is vertex and twin B is not vertex, controversy exists as to the optimal mode of delivery. A small randomized study comparing Cesarean delivery with vaginal delivery for vertex-non-vertex twins failed to show an advantage for Cesarean delivery, but did not have statistical power to address rare neonatal morbidities. Some authors have advocated external cephalic version for management of the second twin; however, observational studies have not shown any advantage of this approach compared to total breech extraction.

A 23-year-old G2P1 woman with six weeks amenorrhea presents with lower abdominal pain and vaginal bleeding. Her temperature is 102.0°F (38.9°C) and the cervix is 1 cm dilated. Uterus is eight-week size and tender. There are no adnexal masses. Urine pregnancy test is positive. What is the most likely diagnosis? A. Threatened abortion B. Missed abortion C. Normal pregnancy D. Septic abortion E. Ectopic Pregnancy

D The patient has a septic abortion. She has fever and bleeding with a dilated cervix which are findings seen with septic abortion. Threatened abortions clinically have vaginal bleeding, a positive pregnancy test and a cervical os closed or uneffaced, while missed abortions have retention of a nonviable intrauterine pregnancy for an extended period of time (i.e. dead fetus or blighted ovum). A normal pregnancy would have a closed cervix. Ectopic pregnancy would likely present with bleeding, abdominal pain, possibly have an adnexal mass, and the cervix would typically be closed.

The most reliable diagnostic finding associated with chorioamnionitis is: . A) Maternal leukocytosis B) Maternal tachycardia C) Uterine tenderness D) Maternal fever E) Maternal bacteremia

D) Maternal fever

Case where patient suddenly got shortness of breath after delivery. Medication was oxycodone + something?

a. Pulmonary function test? (due to pulmonary embolism after delivery)

A 26-year-old G0 was found to have a low-grade squamous intraepithelial lesion (LSIL) on routine Pap smear. She underwent a colposcopy with cervical biopsy. Her colposcopy was adequate and biopsy results showed CIN-I. There was no endocervical glandular involvement. Endocervical curettage showed benign cells. Which of the following is the most appropriate treatment for this patient? A. Cold knife conization B. Loop Electrosurgical Excision Procedure (LEEP) C. Cryotherapy D. Follow up Pap smear in six months E. Complete removal of the lesion

D The patient should be followed with Pap smears at six and 12 months or undergo HPV DNA testing at 12 months. Excisional or ablative procedures are not indicated for LSIL. Indications for cold knife conization (CKC) include: positive endocervical curettage, HSIL lesion too large for LEEP, patient not tolerant of examination in office, lesion extending into the endocervical canal beyond vision, or to rule out invasive cancer (classify the depth of invasion if biopsy shows invasion). It is unusual to manage low grade lesions by CKC. Indications for LEEP are similar to CKC.

A 35-year-old G3P2 woman presents for her initial prenatal care visit at 15 weeks gestation, a¬ccording to her last menstrual period. She reports that a home pregnancy test was positive about five¬¬ weeks ago. Review of her history is unremarkable and her entire family is in good health. Physical examination reveals a ten-week size uterus. Which of the following is the most appropriate next step in establishing this pregnancy's gestational age? A. Checking fetal heart tones B. Hysterosonogram C. Quantitative Beta-hCG D. Obstetrical ultrasound E. Quadruple screen

D The patient's gestational age based on her LMP and the findings on physical exam are discordant. In this case, the most reliable method of confirming gestational age is a dating ultrasound. A quantitative Beta-hCG will not reliably predict the gestational age. The uterine size on physical exam is not the most accurate way to date a pregnancy. An ultrasound performed between 14 and 20 weeks gestation should be used to date the pregnancy if there is greater than a 10 day discrepancy from the menstrual dates. First trimester ultrasound provides the most accurate assessment of gestational age and can give an accurate estimated date of confinement (EDC) to within 3-5 days.

A 33-year-old nulliparous woman presents with amenorrhea for the past 12 months. She also reports a recent onset of dyspareunia, causing her to feel anxious about having intercourse. She had menarche at age 15. Her cycles were normal until two years ago when she began skipping menses. She is otherwise in good health. She is 5 feet 4 inches tall and weighs 130 pounds. Her physical examination is completely normal. TSH and prolactin levels are normal. Urine pregnancy test is negative. What is the most likely cause of this patient's amenorrhea? A. Psychogenic B. Genital tract outflow obstruction C. Asherman's syndrome D. Premature ovarian failure E. Pituitary adenoma

D The patient's symptom of dyspareunia is likely caused by vaginal dryness, which is associated with estrogen deficiency. Hypergonadotropic amenorrhea is the result of ovarian failure or follicular resistance to gonadotropin stimulation. The history, physical exam and labs make the other possibilities less likely: psychogenic disorder (no chronic anxiety or anorexia nervosa), outflow obstruction (previously had periods), Asherman's syndrome (no history of pregnancy or intrauterine procedures), or a pituitary tumor (normal labs).

A 22-year-old G0 presents with painful menstruation that limits her activities each month. She describes the pain as spasmodic occurring on days one to three of bleeding since her cycles began. Other symptoms include nausea, nervousness, diarrhea, and headache. Her physical exam is normal with a soft, non-tender abdomen. Bimanual exam reveals a fixed uterus with uterosacral ligament nodularity. There are no adnexal masses noted. What is the most likely diagnosis in this patient? A. Premenstrual syndrome B. Premenstrual dysphoric disorder C. Primary dysmenorrhea D. Secondary dysmenorrhea E. Adenomyosis

D The physical examination in patients with primary dysmenorrhea is normal. There should not be any palpable abnormalities on abdominal, speculum, pelvic, bimanual, and rectal examinations. The restricted uterine motion found on exam suggests the possibility of endometriosis or pelvic scarring from inflammation or adhesions. These conditions must be considered in establishing the etiology of her diagnosis. Childbearing does not affect the occurrence of either diagnosis. Although the patient's symptoms, including the associated symptoms, timing of initial onset, and cyclic nature of her pain are consistent with primary dysmenorrhea, the finding on physical examination makes secondary dysmenorrhea the likely diagnosis.

A 20-year-old G1P0 woman has vaginal spotting and mild cramping for the last three days. She had her last normal menstrual period approximately seven weeks ago. She had a positive home pregnancy test. Vital signs are: blood pressure 120/72; pulse 64; respirations 18; temperature 98.6°F (37°C). On pelvic exam, she has scant old blood in the vagina, with a normal appearing cervix and no discharge. On bimanual exam, her uterus is nontender and small, and there are no adnexal masses palpable. Quantitative Beta-hCG 48 hours ago was 750 mIU/ml; today, current Beta-hCG 760 mIU/ml; progesterone 3.2 ng/ml; hematocrit 37%. Transvaginal ultrasound shows a fluid collection in the uterus with a yolk sac but no fetal pole. A 3x3 cm cyst is seen on the left ovary. There is no free fluid in the pelvis. Which of the following is the most appropriate next step in the management of this patient? A. Exploratory laparoscopy B. Treat with methotrexate C. Treat with mifepristone D. Dilation and curettage E. Repeat ultrasound in one week

D The pregnancy is abnormal based on the abnormal Beta-hCG levels and the progesterone level. In a normal pregnancy, the level should rise by at least 50% every 48 hours until the pregnancy is 42 days old (after that time, the rise in level may not follow the curve). A progesterone level of <5 ng/ml suggests an abnormal or extrauterine pregnancy. In this instance, the pregnancy is intrauterine because of the presence of a yolk sac. Dilation and curettage is an option for treatment. Other options include expectant management, misoprostol or manual vacuum aspiration. Laparoscopy and methotrexate are not indicated as this is a confirmed intrauterine pregnancy. Mifepristone is a progestin receptor antagonist and can be used as emergency contraception to prevent ovulation and blocks the action of progesterone which is needed to maintain pregnancy. In the US, Mifepristone is also used with misoprostol for pregnancy termination.

The main contraceptive action of the copper based intrauterine device is: A) Prevention of implantation of the fertilized ovum B) Cessation of ovulation C) Induced abortion D) Production of a spermicidal environment E) Elevation of serum copper level

D) Production of a spermicidal environment

The following statements are true about uterine leiomyomas EXCEPT: A) They are 5 times more common in African American women when compared to Caucasian women. B) They can be diagnosed by ultrasound. C) They have been associated with infertility. D) They can be suppressed by estrogen therapy. E) They are asymptomatic in over 50% of those who have them.

D) They can be suppressed by estrogen therapy.

A 32-year-old nulliparous woman comes to your office because she has been unable to conceive for one year. She is currently in a mutually monogamous relationship with her husband, has intercourse three times per week, and has no dyspareunia. Her menstrual cycles occur every 26-34 days. She has had seven sexual partners in the past. She was treated for multiple sexually transmitted infections including gonorrhea, chlamydia and pelvic inflammatory disease in her early twenties. She had an abnormal Pap smear about four years ago and was treated with a LEEP. What is the most likely underlying cause of infertility in this patient? A. Luteal phase defect B. Cervical stenosis C. Ovulatory dysfunction D. Tubal disease E. Endometriosis

D The rate of tubal infertility has been reported as 12% after one episode of PID, 25% after 2 episodes and 50% after three episodes. Salpingitis can develop in 15-30% of women with inadequately treated gonococcal or chlamydial infections and can result in significant long-term sequelae, such as chronic pelvic pain, hydrosalpinx, tubal scarring and ectopic pregnancy. Given this patient's history, her inability to conceive is most likely due to the long-term sequelae of a sexually transmitted infection. Although the patient had a LEEP, risk for cervical stenosis is low. She is having regular cycles; therefore, anovulation and luteal phase defect is less likely. This case emphasizes the importance of aggressive screening and treatment protocols for sexually transmitted infections, as well as counseling regarding abstinence and safer sex practices. While endometriosis can cause tubal occlusion, her clinical presentation is not consistent with endometriosis.

A 35-year-old Asian woman presents with irregular menses and hirsutism of three months duration. The patient has no family history of hirsutism. On exam, the patient was noted to have terminal hair growth on her chest and recently had laser treatment to remove similar hair on her chin. Her total testosterone is 76 ng/dl (normal) and her DHEAS is 1500µg/dl (elevated). Which of the following is the most likely diagnosis in this patient? A. Pituitary adenoma B. Ovarian tumor C. Cushing's syndrome D. Adrenal tumor E. Idiopathic

D The short duration of symptoms and the significantly elevated DHEAS support the diagnosis of an adrenal tumor as the etiology of the patient's symptoms. In addition, the patient is Asian and is less likely to have a predisposition to idiopathic hirsutism. Asians with polycystic ovarian syndrome are less likely to present with overt hirsutism than other ethnic groups.

A 26-year-old G0 woman presents with severe right lower quadrant pain associated with nausea for the last six hours, which began shortly after she finished her aerobic exercises. She has a history of suspected endometriosis, which was diagnosed two years ago, based on her severe dysmenorrhea. She has been using NSAIDs during her menses to control the pain. She is not sexually active, and is otherwise in good health. Her menstrual cycles are regular and her last menstrual period was three weeks ago. She has no history of sexually transmitted infections. Her vital signs are: blood pressure 145/70; pulse 100; temperature 99.2°F (37.3°C). She appears uncomfortable. On abdominal examination, she has moderate tenderness to palpation in the right lower quadrant. On pelvic exam, she has no lesions or discharge. A thorough bimanual exam was difficult to perform due to her discomfort. Beta-hCG <5 mIU/ml and WBC is 8,500 /microliter. A pelvic ultrasound showed a 6 cm right ovarian mass. The uterus and left ovary appeared normal. There was a moderate amount of free fluid in the pelvis. What is the most likely diagnosis in this patient? A. Appendicitis B. Exacerbation of the endometriosis C. Ovarian carcinoma D. Ovarian torsion E. Ectopic pregnancy

D The sudden onset of pain and nausea, as well as the presence of a cyst on ultrasound suggest ovarian torsion. Although appendicitis is on the differential diagnosis list, it is unlikely to have such a sudden onset of pain and a normal white count. Her endometriosis can get worse but it would be unlikely to be of such sudden onset. Although she has an adnexal mass, the Beta-hCG is negative, which rules out pregnancy.

A 36-year-old G1 began prenatal care at eight weeks gestation. At that time, the gestational age was confirmed by a transvaginal ultrasound. She is now at 36 weeks gestation. Her previous medical history reveals hypertension for eight years and class F diabetes for five years (baseline proteinuria = 1 g). She smokes one half-pack of cigarettes per day. On examination at 32 weeks gestation, her fundal height was 29 cm. At 33 weeks, biometry was consistent with 31-3/7, EFW 1827g, 25th percentile. Today, ultrasound reveals limited fetal growth over the past three weeks. Biometry is consistent with 31-5/7, EFW 1900 g, <10th percentile. What is the most likely etiology of the intrauterine growth restriction in this case? A. Genetic factors B. Congenital anomaly C. Tobacco use D. Uteroplacental insufficiency E. Perinatal infection

D There is substantial evidence from experimental animal studies that suggests that alterations in uteroplacental perfusion affect the growth and status of the fetus, as well as the placenta. This patient has significant medical diseases that are affecting her vasculature and, ultimately, limiting the substrate availability to the fetus with resultant uteroplacental insufficiency. The vascular disease is evidenced by retinopathy and proteinuria. The other choices above may all result in fetal growth restriction; however, they are not the most likely etiology in this clinical scenario.

A 33-year-old G1 at 38 weeks gestation with pregnancy complicated by type 1 diabetes was admitted for induction due to oligohydramnios. She received Cervidil (prostaglandin E2) overnight and her cervix was noted to be 3 cm dilated in the morning so oxytocin was started. After three hours on oxytocin induction, fetal heart rate was noted to be in the 160s with minimal variability and late decelerations despite resuscitation with oxygen, fluids and left lateral position. Thirty minutes after discontinuing the oxytocin, she continued to have contractions every three to four minutes with late decelerations. Her blood pressure was noted to be 138/88 and her pulse was 110. Her cervical exam was noted to be 4 cm dilated. What is the most appropriate next step in the management of this patient? A. Perform a biophysical profile B. Administer morphine C. Administer terbutaline D. Proceed with a Cesarean section E. Restart the oxytocin

D This fetus is clearly not tolerating labor. Unfortunately, there is no good way to assess fetal status at this point. A biophysical profile is not of any value in labor. The presence of late decelerations in a patient with diabetes and oligohydramnios is not reassuring and unlikely to recover. Although terbutaline may slow down the contractions, it is not recommended in a patient whose heart rate is 110. Morphine will not resolve the late decelerations.

A 13-year-old patient has had regular menses for 1 year, with debilitating pain beginning in the lower abdomen a few hours before menses and lasting 24 hours. Physical examination is completely normal. Optimal management at this time is: A) Psychiatric referral B) Diagnostic laparoscopy C) Trial of oral contraceptives D) Trial of prostaglandin synthetase inhibitors E) Reassurance with follow-up evaluation in 6 months

D) Trial of prostaglandin synthetase inhibitors

A 20-year-old G2P1 woman at 28 weeks gestation presents to labor and delivery with contractions every four minutes. On physical examination, her vital signs are: temperature 100.5°F (38.0°C); heart rate 120; respiratory rate 18, and blood pressure 110/65. Her uterine fundus is tender and the rest of the physical exam is normal. Her cervix is dilated 1 cm and is 50% effaced. Baby is in vertex presentation. Fetal heart tones are in the 150s with a category I tracing. Her white blood cell count (WBC) is 18,000/mcL. Which of the following is the most appropriate next step in the management of this patient? A. Observation B. Tocolysis C. Contraction stress test D. Labor induction E. Cesarean section

D This patient has a fever, a tender fundus, and elevated white blood cell count, which are concerning for an intra-amniotic infection. Delivery is warranted and in the case of reassuring heart tones, there are no contraindications for labor induction and a Cesarean section is not indicated at this time. Tocolytics should not be used in the case of an intra-amniotic infection. Conservative management with observation would delay diagnosis and would not be appropriate. A contraction stress test is not indicated since the patient is already contracting with reassuring fetal heart tones.

An 18-year-old G1 woman at 32 weeks gestation presents with severe abdominal pain and a small amount of bleeding. She has received routine prenatal care, smokes one pack of cigarettes per day and admits to using crack cocaine. On exam, her blood pressure is 140/80, pulse 100 and she is afebrile. Her uterus is tense and very tender. Pelvic ultrasound reveals a fundal placenta, cephalic presentation of the fetus and no other abnormalities. Cervical examination reveals blood coming through the os and is one centimeter dilated. Fetal heart tones have a baseline of 160s, with a category III tracing. Which of the following is the most likely diagnosis? A. Placenta previa B. Premature rupture of the membranes C. Preterm labor D. Placental abruption E. Chorioamnionitis

D This patient has a placental abruption. Common presenting signs of an abruption include abdominal pain, bleeding, uterine hypertonus and fetal distress. Risk factors include smoking, cocaine use, chronic hypertension, trauma, prolonged premature rupture of membranes, and history of prior abruption. Treatment would involve an emergent Cesarean section with appropriate resuscitation, including intravenous fluids and blood products as needed. A placenta previa is an abnormal location of the placenta.

A 58-year-old G3P1 presents to your office for her a health maintenance examination. She became menopausal at age 54. Her past medical history is significant for angina. She experienced a Colles' fracture 14 months ago when she tripped and fell while running after her grandson. She has not had any surgeries. She takes no medications and has no known drug allergies. She smokes 10 cigarettes a day and drinks a glass of red wine at dinner. Her father was diagnosed with colon cancer at the age of 72. Physical exam revealed a BP=120/68, P=64, BMI= 22. Her heart, lung, breast and abdominal exams were normal. Pelvic exam was consistent with vaginal atrophy and a small uterus. There was no adnexal tenderness and no masses were palpated. In addition to obtaining a bone mineral density scan, what is the next step in the management plan for this patient? A. Repeat bone mineral density in one year B. Repeat bone mineral density at age 65 C. Begin hormone replacement therapy D. Begin treatment with bisphosphonates E. Test for the presence of biochemical bone markers in the blood

D This patient has many of the major risk factors for osteoporosis including history of fracture as an adult, low body weight and being a current smoker. Patients who already have had an osteoporotic fracture may be treated on this basis alone. Prior to beginning treatment with bisphosphonates, a bone mineral density (BMD) should be documented and repeated at two-year intervals to monitor treatment. DEXA is the test of choice for measuring (BMD). A nuclear medicine bone scan may be useful to rule out a pathologic fracture from metastatic disease. General recommendations for the prevention of osteoporosis include eating a balanced diet that includes adequate intake of calcium and vitamin D, regular physical activity, avoidance of heavy alcohol consumption, and smoking cessation. Bone markers are used in research but are not yet a reliable predictor of BMD. Hormone replacement therapy is not recommended long term for disease prevention especially in patients with cardiovascular disease.

A 27-year-old G0 presents to the clinic because of concerns that she has not been able to get pregnant for the last year. She has been married for two years ago and was using birth control pills for contraception. She stopped using birth control pills when she decided to start a family one year ago. She is in good health and her only medication is a prenatal vitamin. Her periods are regular, every 28 days, with normal flow; her last period was two weeks ago. She has no history of sexually transmitted infections and no abnormal Pap smears. Her husband is also healthy with no medical problems. She is 5 feet 4 inches tall and weighs 130 pounds. Her examination, including a pelvic exam, is completely normal. Laboratory results show normal thyroid function tests and normal prolactin level. What is the most appropriate next step in the management of this patient? A. Reassurance and observation B. Perform a pelvic ultrasound C. Order a hysterosalpingogram D. Order a semen analysis E. Recommend a diagnostic laparoscopy

D This patient has primary infertility, since she has not been able to conceive for one year. She does not appear to have underlying pathology to explain why she has not conceived, and her husband's semen has not yet been examined. The male factor plays a role in about 35% of infertility cases. A pelvic ultrasound is unlikely to add any information, as the patient has normal cycles and normal exam. Although a hysterosalpingogram might be ordered in the future, the male factor needs to be ruled out first, as it is less invasive to perform. Even though this patient had been on birth control pills previously, this should not be affecting her fertility a year later. In patients who use OCPs for prolonged periods, there might be a few months delay in returning to normal fertility.

An 18 year-old G1P0 presents at 32 weeks for a routine visit. She complains of intense itching for the past 2 weeks and cannot stop scratching her arms, legs, and soles of her feet. She has tried over the counter lotions and antihistamines with no relief. She also states that her family noticed she is slightly yellow. Her vital signs are normal and there are scattered excoriations over her arms and legs. Which of the following is the best treatment in the management of this patient? A. Aggressive hydration B. Antivirals such as Acyclovir C. Antihistamines D. Ursodeoxycholic acid E. Steroids

D This patient has pruritus gravidarum, a common pregnancy-related skin condition that is a mild variant of intrahepatic cholestasis of pregnancy. There is retention of bile salt, and as serum levels increase they are deposited in the dermis. This, in turn, causes pruritus. The skin lesions are secondary to scratching and excoriation. Antihistamines and topical emollients may provide some relief and should be used initially. Ursodeoxycholic acid relieves pruritus and lowers serum enzyme levels. Another agent reported to relieve the itching is the opioid antagonist naltrexon. Hydroxychloroquine is used to treat lupus and is not indicated in this patient.

A 36-year-old married woman, gravida 3, para 3, comes to your office for annual examination and contraception. She reports that she has heavy menstrual bleeding, and would like to resume taking oral contraceptives (OCs), which she took for several years to improve her menses as a teenager. She is morbidly obese, with a history of a pulmonary embolism after her most recent pregnancy 2 years ago. Although she is certain she does not want to become pregnant in the next few years, she thinks her husband may want another child in the future. You counsel her that the best contraceptive for her at this time would be: A) progestin-only OCs B) combined OCS C) contraceptive vaginal ring D) levonorgestrel intrauterine device (IUD) E) copper-containing IUD

D) levonorgestrel intrauterine device (IUD)

A 56-year-old G3P3 woman presents to the office for her annual health maintenance exam. She is in good health and is not taking any medications. She has been postmenopausal for three years. She had an abnormal Pap smear 10 years ago, but results have been normal every year since. She is sexually active with her husband. On examination, her cervix is 1 cm above the vaginal introitus and there is moderate bladder prolapse. Her uterus is normal in size and she has no adnexal masses or tenderness. In addition to recommending a mammogram, what is the most appropriate next step in the management of this patient? A. Cystocele repair B. Pelvic ultrasound C. Total hysterectomy D Observation E. Topical estrogen

D This patient is asymptomatic from her prolapse; therefore, no intervention is necessary at this point. Cystocele repairs and hysterectomies are invasive procedures which are not indicated in this asymptomatic patient. It is not necessary to obtain a pelvic ultrasound, as her uterus is normal in size and she has no adnexal masses. Topical estrogen would not help improve the prolapse, although it might help with her vaginal dryness. She seems to be doing well with the lubricants and it is not necessary to expose her to the estrogen, especially since she still has her uterus, and estrogen treatment alone may increase her risk of endometrial cancer.

A 52-year-old G3P2 woman reports vaginal spotting and bleeding after intercourse for the past 18 months. She stopped having menses at the age of 48 and has not been on hormone replacement therapy. She also notes new onset low back pain. She has smoked two packs a day for the past thirty years. Her last gynecologic exam was 10 years ago. On physical examination, she is a thin female who appears older than her stated age. She weighs 120 pounds and is 5 feet 6 inches tall. Her pelvic examination reveals atrophy of the external genitalia and vagina, a minimal amount of dark brown blood in the vault, and a large parous cervix with a friable lesion on the anterior lip of the cervix. The uterus is normal size, non-mobile and fixed in a retroverted position. There are no palpable adnexal masses, but there is firm nodularity in the posterior cul-de-sac on rectal examination. Which of the following is the most appropriate next step in the management of this patient? A. Computerized tomography of the lower spine and pelvis B. Pap smear C. Colposcopy D. Cervical biopsy E. Pelvic ultrasound

D This patient is at high-risk for cervical cancer. Her risk factors include tobacco use and a poor screening history. The symptoms of postmenopausal and postcoital bleeding should be taken seriously, and a cervical biopsy of the suspicious cervical lesion performed. Her physical examination with fixation of the uterus and thickening of the rectovaginal septum and back pain suggests involvement of the parametria (Stage II) and possible extension to the sidewall (Stage III). A Pap smear should not be used to exclude cervical cancer, as it is a screening test and not a diagnostic test, and colposcopy would not be useful since a clinically visible lesion is already present. Although a CT scan may ultimately be needed as part of the evaluation of cervical cancer, a diagnosis must first be made by biopsy. Ultrasonography may be helpful in the diagnostic evaluation of post-menopausal bleeding, but not in the setting of an obvious cervical lesion.

A 34-year-old G1P0 woman at 39 weeks gestation presents in active labor. Her cervical examination an hour ago was 5 cm dilated, 90 percent effaced and 0 station. The baseline is 140 beats/minute. There is a deceleration after the onset of each of the last four contractions. She just had spontaneous rupture of membranes and is found to be completely dilated with the fetal head is at +3 station. What is the most likely etiology for these decelerations? A. Oligohydramnios B. Rapid change in descent C. Umbilical cord compression D. Uteroplacental insufficiency E. Head compression

D This patient is having late decelerations. Late decelerations are associated with uterine contractions. The onset, nadir, and recovery of the decelerations occur, respectively, after the beginning, peak and end of the contraction. Late decelerations are associated with uteroplacental insufficiency. A rapid change in cervical dilation and descent are not associated with late decelerations. Umbilical cord compression is associated with variable decelerations. Oligohydramnios can increase a patient's risk of having umbilical cord compression; however, it does not cause late decelerations. Head compression is associated with early decelerations.

A 22-year-old G1, who is at 38 weeks gestation with an estimated fetal weight of 2500 g, presents in active labor. She is completely dilated and effaced. The fetus is at +4 station and left occiput anterior with no molding. She has an epidural and has been pushing effectively for three hours. She is exhausted. What is the next step in management? A. Allow to continue pushing until the baby delivers B. Start Oxytocin to strengthen contractions C. Discontinue the epidural D. Forceps-assisted vaginal delivery E. Cesarean section

D This patient meets all the requirements for an operative vaginal delivery. Forceps application requires complete cervical dilation, head engagement, vertex presentation, clinical assessment of fetal size and maternal pelvis, known position of the fetal head, adequate maternal pain control and rupture of membranes. Strict adherence to the guidelines suggested by the American College of Obstetricians and Gynecologists (ACOG) for low forceps delivery does not increase the fetal or maternal risks when performed by an experienced operator.

"An 18-year-old G1P0 woman at 12 weeks gestation reports nausea, vomiting, scant vaginal bleeding and a ""racing heart."" These symptoms have been present on and off for the past four weeks. The patient has no significant past medical, surgical or family history. Vital signs are: temperature 98.6°F (37°C); heart rate 120; blood pressure 128/78. On physical examination: uterine fundus is 4 cm below the umbilicus; no fetal heart tones obtained by fetal Doppler device; cervix is 1 cm dilated with pinkish/purple ""fleshy"" tissue protruding through the os. Labs show: hemoglobin 8.2 gm/dL, quantitative Beta-hCG 1.0 Million IU/mL; thyroid-stimulating hormone (TSH) undetectable; free T4 3.2 (normal 0.7 - 2.5). An ultrasound reveals heterogeneous cystic tissue in the uterus (snowstorm pattern). Which of the following is the most appropriate next step in the management of this patient? A. Repeat quantitative Beta-hCG B. Repeat transvaginal ultrasound C. PET scan D. Chest x-ray E. CBC"

D This patient's presentation is classic for a molar pregnancy. Beta-hCG levels in normal pregnancy do not reach one million. A chest x-ray would be the most appropriate step, as the lungs are the most common site of metastatic disease in patients with gestational trophoblastic disease. Though a repeat quantitative Beta-hCG will be required on a weekly basis, an immediate post-operative value will be of little clinical utility. A PET scan is not indicated and the patient already had a CBC done.

A 29-year-old G1 woman presents at 31 weeks gestation with preterm rupture of membranes six hours ago. She notes that for the last hour she has had some occasional contractions. Her prenatal course has been uncomplicated and she takes prenatal vitamins and iron. She denies substance abuse, smoking or alcohol use. Her blood pressure is 110/70; pulse 84; temperature 98.6°F (37.0°C). What is the role of tocolysis in this patient? A. Prevent delivery B. Delay delivery until fetal lung maturity is reached C. Delay delivery for one week D. Delay delivery in order to administer steroids E. Contraindicated

D While the role of tocolysis in the setting of preterm rupture of membranes is controversial, it may be appropriate in limited settings. Tocolysis may be administered in an attempt to prolong the interval to delivery to gain time for steroids to obtain maximum benefit for the fetus. The risks of chorioamnionitis with continuing tocolytics beyond 48 hours outweighs the benefit of awaiting lung maturity. This may be reasonable in women without evidence of infection or advanced preterm labor. Admittedly, the likelihood of success in this setting is relatively poor, but the potential benefit to the fetus probably outweighs any maternal complication from tocolysis.

A 32-year-old G5P3 woman presents with left-sided abdominal pain. Her last normal menstrual period was eight weeks ago. She began having pain early this morning and it has increased to a severity of 8/10. She denies nausea or vomiting or vaginal bleeding. Her gynecological history is notable for a history of right-sided ectopic pregnancy four years ago. At that time, she had a right salpingectomy and a left tubal ligation. On physical examination: blood pressure is 90/54; pulse 108; respirations 22; and temperature 98.6°F (37.0°C). On abdominal examination, she has rebound and guarding in all quadrants, and on pelvic exam, her uterus is very tender and there is left adnexal fullness. A transvaginal ultrasound shows an empty uterus, left pelvic mass with a gestational sac and fetal pole, and a large amount of free fluid in the pelvis. Her hematocrit is 26%. What would be the next best step in the management? A. Admit for observation B. Repeat Beta-hCG level in 48 hours C. Treat with methotrexate D. Perform a laparoscopy E. Perform a dilation and curettage

D This scenario is consistent with the patient having a ruptured ectopic pregnancy. Signs of hypovolemia (tachycardia, hypotension) with peritoneal signs (rebound, guarding and severe abdominal tenderness) and a positive pregnancy test lead to the diagnosis of ruptured ectopic pregnancy. Conservative management, with observation and repeating the Beta-hCG level in 48 hours is not indicated since a diagnosis is clear and waiting can potentially be dangerous to the patient. Dilation and curettage would only be considered after laparoscopy, if needed.

A 27 year-old G1P0 at 32 weeks gestation presents complaining of cough, fever, chest pain, and dyspnea. Physical examination reveals the following: pulse 108, temperature 98.6°F (37.0°C), respiratory rate 22 per minute and right lower lobe bronchial breath sounds. Which of the following tests would be most appropriate for making a diagnosis for this patient? A. Complete blood cell count B. Mycoplasma-specific immunoglobulin G C. Urinalysis for pneumococcal antigen D. Chest x-ray E. Pulmonary function tests

D This woman presents with classic symptoms and findings for pneumonia. The typical symptoms include cough, dyspnea, sputum production, and pleuritic chest pain. Mild upper respiratory symptoms and malaise usually precede these symptoms, and mild leukocytosis is usually present. Chest radiography is essential for diagnosis, although radiographic appearance does not accurately predict the etiology of the pneumonia. Pulmonary function tests, sputum culture, serological testing, cold agglutinin identification, and tests for bacterial antigens are not recommended in uncomplicated pneumonia.

A 29-year-old G2P1 woman at 39 weeks gestation presents in early labor after spontaneous rupture of the fetal membranes. Thirty minutes after arrival, she delivers a 2650 gram male infant. A globular pale mass appears at the introitus when attempting to deliver the placenta. Her blood pressure is 90/60; pulse 104; and temperature is 98.6°F (37.0°C). What is the most likely etiology for this event in this patient? A. Multiparity B. Twin gestation C. Leiomyoma D. Uterine inversion E. Rapid labor

D Uterine inversion is an uncommon etiology of postpartum hemorrhage. Factors that lead to an over-distended uterus are risk factors for uterine inversion. Grand multiparity, multiple gestation, polyhydramnios and macrosomia are all risk factors. The most common risk factor, however, is excessive (iatrogenic) traction on the umbilical cord during the third stage of delivery. Although leiomyomas may spontaneously prolapse, it is unlikely during the peripartum period.

A 24-year-old G0 woman presents to you for preconception counseling. Her medical history is notable for type 1 diabetes mellitus, hypertension, epilepsy, and hypothyroidism. Her medications include insulin, methyldopa, valproic acid and levothyroxine. Based on her medication exposure, her infant is at greatest risk of which of the following anatomical defects? A. Duodenal atresia B. Skeletal anomalies C. Renal tubular dysgenesis D. Neural tube defects E. Omphalocele

D Valproic acid is associated with an increased risk for neural tube defects, hydrocephalus and craniofacial malformations. Insulin and methyldopa are not associated with fetal defects. Omphalocele and duodenal atresia are not increased in type 1 diabetic patients.

A 37-year-old G1P1 suffers from severe mood swings the week before her menstrual cycle. The mood swings resolve after she stops bleeding. You diagnose her with premenstrual syndrome (PMS) after obtaining further history and a normal examination. In addition to exercise, which of the following might be suggested to help decrease this patient's symptoms? A. Folic acid B. Ginkgo C. Fish oil D. Vitamin B6 E. Potassium

D Vitamin deficiency of A, E and B6 have been associated with an increase in PMS. Replacement of these vitamins might improve PMS symptoms and avoid further medical therapy.

Pelvic inflammatory disease is characterized by all of the following EXCEPT: . A) Leukocytosis B) Pelvic pain C) Fever D) Anemia E) Cervical motion tenderness

D) Anemia

A 27-year-old sexually active woman presents to your office for evaluation. She hasn't had her period for the last 3 months. Prior to that time, they were regular, every 28 days, with a light flow lasting for 4 days. In your initial evaluation of her condition, which of the following serum tests is most important? A) Prolactin B) Luteinizing hormone (LH) C) Estimated free thyroxine D) Human chorionic gonadotropin (hCG) E) Follicle stimulating hormone (FSH)

D) Human chorionic gonadotropin (hCG)

Case where patient was previously treated for Gonorrhea. Multiple sex partners. Many small erythematous papules in palms and soles. What to use to dx?

a. Rapid plasma reagin

A 55-year-old woman comes in for her first gynecologic examination since the birth of her youngest child 30 years ago. She reports being monitored by her primary doctor and is taking an oral antihypertensive with no other medical problems. She is experiencing hot flushes and irregular vaginal bleeding. On physical examination, you observe fullness in the left supraclavicular and cervical lymph nodes. Her abdomen is normal, but a pelvic examination reveals an 8-cm exophytic tumor replacing the entire cervix. She does not demonstrate any parametrial involvement. In addition to a cervical biopsy, the next diagnostic procedure should be. A) endometrial biopsy B) fractional dilation and curettage (D&C) C) cystoscopy and proctoscopy D) supraclavicular lymph node cytology or biopsy

D) supraclavicular lymph node cytology or biopsy

Question about who you would need a written consent from pregnant patient, in order to inform regarding infertility/pregnancy questions? a. The husband b. the infertility consultant c. etc.

a. The husband

Indicate the true statements concerning the double test in prenatal diagnostics: 1) it is performed between 11 and 14 week of gestation; 2) it assesses 2 biochemical markers in the blood of the pregnant woman; 3) in the case of a fetus with Down syndrome the concentration of free beta hCG subunit is decreased; 4) in the case of a fetus with Down syndrome the concentration of PAPP-A is decreased; 5) in the case of a fetus with Edwards syndrome the concentration of free beta hCG subunit is decreased. The correct answer is: A. 1,2,3,4. B. 1,3,4,5. C. 2,3,4,5. D. 1,2,4,5. E. all the above.

D. 1,2,4,5.

Indicate the true sentences regarding impending miscarriages: 1) painless vaginal bleeding is usually present; 2) gynecological examination shows cervical ripening; 3) gynecological examination shows the cervix dilated to 1-2 cm; 4) speculum examination shows the amniotic sac; 5) treatment of choice is conservative. The correct answer is: A. 1,2. B. 3,4. C. 2,5. D. 1,2,5. E. 1,3,5.

D. 1,2,5.

The indications for insemination include: 1) decreased parameters of the semen; 2) ejaculation disorders; 3) PCOS; 4) III grade endometriosis; 5) idiopathic infertility. The correct answer is: A. 1,2,3. B. 1,2,3,4. C. 1,2,4,5. D. 1,2,5. E. 3,4,5.

D. 1,2,5.

The absolute contraindications to the use of two-component hormonal contraceptives include: 1) smoking tobacco at the age over 35 years; 2) pregnancy or not excluded possibility of pregnancy; 3) migraine headaches; 4) mitral valve prolapse; 5) estrogen-dependent neoplasms. The correct answer is: A. 1,2,3,4. B. 3,4,5. C. 1,2,3. D. 1,2,5. E. all the above.

D. 1,2,5. Technically heart defects, and migraines that require ergotamine treatment are also absolute contraindications. However aortic valve prolapse and migraines not requiring treatment with ergotamines are relative contraindications.

Endometrial cancer is currently the most common female genital cancer in developed countries. The following are the characteristics of its two subtypes: 1) estrogen dependence (positive E + and P + receptor state); 2) aggressive course, frequent metastases; 3) the most common are mutations PTEN, K-ras, microsatellite instability; 4) the most common histopathological type is endometrioid endometrial cancer; 5) the most common mutations are p-53, HER2-neu, p16. The correct combination of characteristics of the first subtype of endometrial cancer is: A. 1,2,5. B. 2,4,5. C. 3,4. D. 1,3,4. E. only 1.

D. 1,3,4.

Oral hormonal anticonception is a well-documented protective factor against: 1) ovarian cancer; 2) liver cancer; 3) endometrial cancer; 4) cervical cancer; 5) renal cancer. The correct answer is: A. 2,3,4. B. 1,5. C. 1,3,4. D. 1,3. E. 2,4,5.

D. 1,3.

Indicate the clinical features of germinal neoplasms of the ovary: 1) occur mainly in young women; 2) occur mainly in post-menopausal women; 3) may be diagnosed on the basis of increased blood levels of CA-125, CEA, CA 15-3; 4) may be diagnosed on the basis of increased blood levels of LDH, AFP, hCG; 5) respond well to chemotherapy; 6) are treated only surgically; 7) are always malignant tumours. The correct answer is: A. 1,4,7. B. 3,4,6. C. 2,4,6,7. D. 1,4,5. E. 2,3,7.

D. 1,4,5.

Indicate the true statements concerning the cardiotocography (CTG) analysis: 1) short-term variability (STV) is particularly helpful in predicting the risk of metabolic acidosis in the fetus; 2) normal fetal heart rate (FHR) is 100 to 160 beats per minute; 3) lack of an acceleration in the record up to 180 minutes is regarded as normal and asa sign of deep sleep of the fetus; 4) correct long-term variability (LTV) is between 5 and 25 beats per minute; 5) sinusoidal rhythm indicates a high degree of fetal distress. The correct answer is: A. 1,3,5. B. 1,5. C. 2,3. D. 1,4,5. E. 2,3,4.

D. 1,4,5.

The combination of pharmacological therapy with a planned cesarean delivery has been shown to reduce the perinatal transmission of HIV to: A. 25%. B. 17.5%. C. 5%. D. 1.5%. E. perinatal transmission of HIV does not depend on the mode of a delivery.

D. 1.5%.

As a consequence of physiological processes in the circulatory system of a pregnant woman in the case of single pregnancy the volume of circulating blood can increase by up to: A. 20%. B. 30%. C. 40%. D. 50%. E. in a single pregnancy there is no change in the volume of circulating blood of the pregnant woman.

D. 50%.

Patient with urinary incontinence?

a. Tolterodine

A patient presents at the gynecological outpatient clinic on the 12 day after the radical hysterectomy due to endometrial hyperplasia with atypia. She complains of a constant outflow of urine from the vagina. Which of the following should be suspected? A. overactive bladder. B. exercise-induced urinary incontinence caused by the impairment of ligaments of the uterus. C. mixed urinary incontinence. D. evacuation of accumulated lymph fluid through the vaginal stump. E. vesicovaginal fistula.

E. vesicovaginal fistula.

Indicate the false statement on interpretation of CTG: A. sporadic accelerations are indicative of a healthy foetus. B. bradycardia within 100-110 bpm does not indicate that foetal health is endangered. C. cardiotocography should be implemented when the foetus is mature enough to survive outside the womb. D. decelerations are drops in the baseline heart rate lasting 20 seconds or longer with a magnitude of at least 10 bpm. E. early decelerations mirror uterine contractions.

D. decelerations are drops in the baseline heart rate lasting 20 seconds or longer with a magnitude of at least 10 bpm.

The proliferative phase of the uterine cycle is dependent on: A. progesterone produced by the corpus luteum. B. progesterone produced by granulous cells. C. estrogen produced by the corpus luteum. D. estrogen produced by granulous cells. E. estradiol produced by the dominant follicle.

D. estrogen produced by granulous cells.

Which of the following is not a germ cell tumor of the ovary? A. germinoma. B. teratoma. C. embryonal carcinoma. D. granulosa cell tumor. E. choriocarcinoma.

D. granulosa cell tumor.

Choriocarcinoma is one of the rarest but most aggressive malignant tumors in women. Its diagnosis is based on: A. magnetic resonance imaging of the abdominal cavity and pelvis. B. histopathological result from biopsy or curettage of the uterine cavity. C. histopathological result obtained during hysterectomy. D. high levels of hCG in blood serum. E. pregnancy induced hypertension before the 20th week of pregnancy.

D. high levels of hCG in blood serum.

Young girl. Maybe 17. No menses. External genitalia looked fine, but unable to visualize the vaginal canal. What do do to establish dx? a. Ultrasound for uterus/ovaries b. Measure LH/FSH

a. Ultrasound for uterus/ovaries

Case with picture that showed electro tocography. Think it showed late deceleration?

a. Uteroplacental insufficiency

Uterine leiomyomas are the most frequent benign neoplasms of the uterus. Indicate the false statement concerning their symptoms: A. they include prolonged heavy periods and intermenstrual bleedings. B. they include pain in the hypogastrium, and pressure on the bladder or the rectum. C. small leiomyomas may be asymptomatic. D. hot flushes are typical symptoms. E. leiomyomas may be the cause of infertility.

D. hot flushes are typical symptoms.

When should the next cytological examination be ordered in a 35-year-old multipara with normal results of the last three cytological examinations and without any risk factors? A. in 3 months. B. in 6 months. C. in a year. D. in 3 years. E. only if some symptoms occur, such as increased vaginal discharge, or contact bleeding.

D. in 3 years.

The range of normal values of Amniotic Fluid Index between 5 and 25 cm may be applied to the patients: A. during the whole singleton pregnancy. B. independently of the number of fetuses. C. in singleton pregnancy only after 20 weeks of gestation. D. in singleton pregnancy only after 24 weeks of gestation. E. in singleton pregnancy only after 32 weeks of gestation.

D. in singleton pregnancy only after 24 weeks of gestation. (Unsure Answer)

Which of the following is not related to the polycystic ovary syndrome? A. overweight and obesity. B. insulin resistance and increased risk of type 2 diabetes. C. increased risk of cardiovascular diseases. D. increased risk of breast cancer and rectal cancer. E. increased risk of endometrial cancer

D. increased risk of breast cancer and rectal cancer.

Indicate the true description of the influence of chorionic gonadotropin on the thyroid function: A. there is no influence. B. it lowers the TSH production which causes the decrease in thyroxine synthesis in the thyroid gland. C. it stimulates TSH production which causes the increase in thyroxine synthesis in the thyroid gland. D. it decreases TSH production and stimulates the thyroid gland to produce thyroxine. E. it increases TSH production and stimulates the thyroid gland to produce thyroxine.

D. it decreases TSH production and stimulates the thyroid gland to produce thyroxine.

The left uterine artery is a direct branch of: A. aorta. B. left common iliac artery. C. left external iliac artery. D. left internal iliac artery. E. left ovarian artery.

D. left internal iliac artery.

All of the following are features of Swyer syndrome, except from: A. XY karyotype. B. uterus present. C. primary amenorrhea. D. male phenotype. E. hypogonadism.

D. male phenotype.

A disulfiram-like reaction can be expected in a patient who was drinking alcohol while undergoing the treatment for the symptoms of a genital infection with: A. HPV virus. B. HSV virus. C. Chlamydia trachomatis. D. Trichomonas vaginalis. E. Neisseria gonorrhoeae.

D. Trichomonas vaginalis.

In the ultrasound examination of the fetus performed between the 11 and 13 week of pregnancy, the most accurate method of assessing the duration of pregnancy is the measurement of: A. biparietal diameter (BPD). B. femur length (FL). C. abdominal circumference (AC). D. crown-rump length (CRL). E. gestational sac diameter.

D. crown-rump length (CRL).

Which of the following findings on cardiotocography (CTG) is a good prognostic factor? A. decelerations. B. sinusoidal pattern. C. tachycardia. D. accelerations. E. bradycardia.

D. accelerations.

A 27-year-old patient comes to the emergency department at 29 weeks of her first pregnancy complaining about a fever up to 38.3°C for the last couple of hours, 2 episodes of vomiting and back pain. Ultrasound examination reveals a normally growing living foetus, normal placenta located in the fundus and the full bladder. On palpation the abdomen is not rigid but tender in the hypogastric area. On bimanual vaginal examination no pain during cervical motion is observed. The percussion of the lumbar area, especially on the left side, is very painful for the patient. What do you suspect? A. acute appendicitis. B. acute cystitis. C. left-sided renal calculi. D. acute pyelonephritis. E. hydronephrosis due to the pressure of the pregnant uterus on the ureter.

D. acute pyelonephritis.

The complications of fetal diabetes include: A. macrosomia. B. preterm birth, respiratory distress syndrome. C. hypoglycemia, polycythemia, hypokalemia. D. all the above. E. none of the above.

D. all the above.

The possible causes of cervical incompetence include: 1) inherited disorders; 2) traumas of the cervix; 3) state after the cervical conization; 4) multiple pregnancy. The correct answer is: A. only 1. B. 1,2. C. only 4. D. all the above. E. none of the above.

D. all the above.

Which of the following are maternal risk factors for the breech position? 1) defects of the uterus which impair fetal rotation; 2) leiomyomas of the uterus; 3) narrow pelvis; 4) excessively wide pelvis; 5) multiparity. The correct answer is: A. 1,3. B. 1,2,4. C. 1,3,5. D. all the above. E. only 5.

D. all the above.

Which of the following are absolutely contraindicated in the pharmacologicaL treatment of arterial hypertension during pregnancy? A. calcium blockers. B. beta-blockers. C. thiazide diuretics. D. angiotensin convertase inhibitors. E. clonidine.

D. angiotensin convertase inhibitors.

Chemotherapy as a separate therapeutic strategy (not as a part of a combination therapy) is the most effective in the treatment of: A. serous ovarian cancer. B. mucinous ovarian cancer. C. endometrial cancer of the uterus. | D. choriocarcinoma. E. cervical squamous cell carcinoma.

D. choriocarcinoma

Which of the following concerning hyperprolactinemia is false? A. sleep and stress are physiological conditions associated with increased secretion of prolactin. B. hyperprolactinemia is the cause of 30% of cases of secondary amenorrhea after discontinuation of oral contraceptives. C. drugs used in hyperprolactinemia treatment are cabergoline and bromocriptine. D. consequences of hyperprolactinemia include the lack of pubic hair and a decrease in the secretion of adrenal androgens. E. galactorrhoea is found in 30-90% women with hyperprolactinemia.

D. consequences of hyperprolactinemia include the lack of pubic hair and a decrease in the secretion of adrenal androgens.

A patient at 26 week of the gestation was injured in a traffic accident and subsequently was treated surgically and received 2 medications of the category B in the FDA classification. The category B denotes: A. medications with the documented risk of fetal abnormalities. B. medications with the documented risk of fetal abnormalities, but benefits from their use justify their application. C. medications that showed adverse effect on the fetus in animal reproduction studiesand there are no adequate studies in humans, but potential benefits may warrant the use of the drug in pregnant women despite a potential risk. D. medications that are relatively safe in pregnancy, with no teratogenic effects observed in animal studies, but there are no adequate studies in humans. E. medications that did not show teratogenic effects in the studies in pregnant women.

D. medications that are relatively safe in pregnancy, with no teratogenic effects observed in animal studies, but there are no adequate studies in humans.

A contraindication to breastfeeding is not: A. HIV infection. B. breast cancer. C. severe mental disorder. D. mother's use of semisynthetic penicillin. E. acute heart failure.

D. mother's use of semisynthetic penicillin.

Directly after the third stage of the delivery massive hemorrhage occurred leading to hypovolemic shock. The condition of the patient improved significantly over several days, but she is suspected of Sheehan's syndrome. Which of the following justifies this suspicion? A. her child receiving 7 in the Apgar score. B. change in respiratory rate. C. worsening of the general condition and depressive thoughts. D. no lactation. E. occurrence of diarrhea.

D. no lactation.

Which of the following is not a risk factor for urinary incontinence? A. patient's age. B. obesity. C. previous gynecological procedures. D. nulliparity. E. chronic constipation.

D. nulliparity.

An absolute contraindication to laparoscopic surgery in gynecology is not: A. patient's lack of consent. B. condition after extensive oncological surgery of the stomach and intestines. C. diffuse peritonitis. D. obesity. E. pulmonary hypertension.

D. obesity.

If hypogastric pain appears approx. 2-3 days after the administration of methotrexate in a patient treated for ectopic pregnancy, you should: A. perform urgent laparotomy with the longitudinal incision - this symptom means that there is haemorrhage because of ectopic pregnancy rupture. B. perform urgent laparoscopy - this symptom means that there is haemorrhage because of ectopic pregnancy rupture. C. perform urgent laparotomy with the transverse incision - this symptom means that there is haemorrhage because of ectopic pregnancy rupture. D. observe the patient and monitor decrease in the hCG level - this is a typical pain after the treatment with methotrexate. E. discharge the patient and inform her that the pain is most probably caused by ovulation.

D. observe the patient and monitor decrease in the hCG level - this is a typical pain after the treatment with methotrexate.

Which of the following is not a typical complication of gestational diabetes? A. polyhydramnios. B. fetal macrosomia. C. intrauterine growth retardation. D. obstetric cholestasis. E. traumatic delivery.

D. obstetric cholestasis.

Indicate the true statements concerning labor induction: 1) in the case of cholestasis of pregnancy when fatty acid concentration is 45 nmol/L the labor induction should be considered after 34 weeks of gestation; 2) in the case of gestational diabetes the labor induction should be considered after 39 weeks of gestation; 3) rupture of membranes is an absolute contraindication to labor preinduction with the use of prostaglandins; 4) in the case of pregestational diabetes the labor induction should be considered after 39 weeks of gestation; 5) in the pregnancy complicated by diabetes the labor induction is indicated when the difference between the fetal abdomen diameter and the biparietal diameter is over 4cm. The correct answer is: A. 1,2,5. B. 1,2,3. C. 1,2,3,5. D. only 2. E. 3,4.

D. only 2.

Indicate the true sentence describing endometriosis: A. retrograde menstrual flow is necessary in the pathogenesis of endometriosis. B. surgical treatment of endometriosis should not be used in women with pelvic pain. C. fertility in endometriosis is only reduced when adhesions in the peritoneal cavity are present. D. pharmacological treatment in endometriosis is limited due to the side effects of medicines. E. every woman with endometriosis is infertile.

D. pharmacological treatment in endometriosis is limited due to the side effects of medicines.

Indicate the false sentence concerning the influence of progesterone on the female body: A. progesterone has a diuretic effect. B. progesterone thickens cervical mucus. C. progesterone increases glycogen synthesis. D. progesterone increases the thickness of the myometrium and the rate of peristaltic waves of the fallopian tubes. E. progesterone increases body temperature.

D. progesterone increases the thickness of the myometrium and the rate of peristaltic waves of the fallopian tubes.

A 25-year-old woman was referred to the GP because of secondary amenorrhea. She reports galactorrhoea and periodically blurred vision. Which of the following hormones should be measured? A. adrenocorticotropic. B. glucagon. C. insulin. D. prolactin. E. testosterone.

D. prolactin.

A 25-year-old woman in 12th week of her first pregnancy made a test of antibodies against Toxoplasma gondii. The results were as follows: IgG = 300 IU/mL, IgM (-). A similar test performed 2 years earlier showed: IgG = 480 IU/mL, IgM (-). The patient should be informed that: A. there is a high risk of an infection with Toxoplasma gondii caused by the consumption of both raw meat and contaminated fruit and vegetables. B. she has to repeat the test of antibodies against T. gondii every 6-8 weeks during the whole pregnancy. C. she should undergo antiprotozoal treatment. D. she was infected with Toxoplasma gondii before the pregnancy (most probably it was a subclinical infection) and her child is not at risk of the infection. E. it is necessary to perform invasive diagnostics for the infection (amniocentesis, examination of the amniotic fluid for protozoan cysts).

D. she was infected with Toxoplasma gondii before the pregnancy (most probably it was a subclinical infection) and her child is not at risk of the infection.

The indications for elective cesarean section in twin pregnancy do not include: A. each situation of abnormal presentation of any of the foetuses. B. death of the foetus II. (according to WNL) C. TTTs syndrome (twin-to-twin syndrome). D. significant difference in foetuses weight > 20%. E. non-cephalic presentation of foetus I.

D. significant difference in foetuses weight > 20%. (>25% according to Brębowicz)

A healthy 63-year-old G0 woman comes to the office for a health maintenance exam. She has no history of abnormal Pap smears or sexually transmitted infections. She has a history of endometriosis and infertility in the past. She has been postmenopausal for 10 years and is not taking any medications. She is 5 feet 4 inches tall and weighs 130 pounds. On pelvic examination, the patient has a palpable left adnexal mass. An ultrasound shows a 5 cm complex left ovarian cyst. What is the most appropriate next step in the management of this patient? A. Observation B. Repeat ultrasound in three months C. CT scan of the abdomen and pelvis D. MRI of the pelvis E. Exploratory surgery

E A complex ovarian mass in a postmenopausal patient needs to be surgically explored. Although this could be an old endometrioma which never resolved, this cannot be assumed. If this is ovarian cancer, waiting three months can change this patient's prognosis. This complex cyst most likely will not resolve, since this is not a physiological cyst. A CT scan or MRI will not add more information and ultrasounds are typically the best imaging studies for the uterus and adnexa.

A 24-year-old G2P1 woman is diagnosed with Rh hemolytic disease at 24 weeks gestation. Measurement of which of the following in the amniotic fluid is best indicative of the severity of the disease? A. Hemoglobin B. Iron C. Anti-D antibody titer D. Glucose E. Bilirubin

E In the presence of a severely erythroblastotic fetus, the amniotic fluid is stained yellow. The yellow pigment is bilirubin, which can be quantified most accurately by spectrophotometric measurements of the optical density between 420 and 460nm, the wavelength absorbed by bilirubin. The deviation from linearity of the optical density reading at 450nm is due to the presence of heme pigment, an indicator of severe hemolysis.

A 19-year-old G1P0 woman at 41-weeks gestation with two prior prenatal visits at 35-weeks and 40-weeks, presents in active labor. Review of available maternal labs shows: blood type O+; RPR non-reactive; HBsAg negative; and HIV negative. She delivers a small female infant who cries spontaneously. On examination, you find the infant has a slightly flattened nasal bridge. Her ears are small and slightly rotated. What is the most appropriate next step in the management of this patient? A. Tell the mother the infant will be fine B. Tell the mother that her newborn has Down syndrome C. Question the patient why an amniocentesis was not performed D. Further examine the infant for wide-spaced nipples and lymphedema E. Further examine the infant for sandal gap toes and hypotonia

E A flattened nasal bridge, small size and small rotated, cup-shaped ears may be associated with Down syndrome and should prompt a survey looking specifically for other features seen with Down syndrome that include sandal gap toes, hypotonia, a protruding tongue, short broad hands, Simian creases, epicanthic folds, and oblique palpebral fissures. The initial physical findings may be a variant of normal, therefore, you should not share any concerns with the mother until you perform a detailed physical examination. Wide-spaced nipples and lymphedema are associated with Turner syndrome. It is not standard of care to offer amniocentesis to a 19-year-old, unless she has specific risk factors.

A 30-year-old woman comes to the office because she desires removal of her IUD. She had a Paragard (Copper T) intrauterine device placed for contraception four years ago and now she desires pregnancy. She is in good health and has no history of abnormal Pap smears or sexually transmitted infections. Pelvic exam shows a normal appearing cervix and no IUD string visible. Ultrasound shows the IUD in the uterine cavity. An attempt is made to remove the IUD with an IUD hook and failed. What is the most appropriate next step in the management of this patient? A. In vitro fertilization B. Laparoscopy C. Hysterosonogram D. A pelvic MRI E. Hysteroscopy

E A hysteroscopy would be easily performed either in the office or in the operating room, and the IUD could then be removed under direct visualization. This would be the best choice for this patient. In vitro fertilization requires a normal endometrial cavity so that the embryo may implant in the uterus. A retained IUD is not an indication for this procedure. Having an ultrasound showing the retained IUD in the uterine cavity, it would not be visible upon laparoscopy. If the IUD had been seen outside the uterus, laparoscopy could be offered for removal of the IUD. Hysterosonogram would not offer additional information since the pelvic ultrasound showed an intrauterine IUD. A pelvic MRI will not give any additional information what would be helpful in the management of this patient.

A 26-year-old G2P2 woman presents with urinary urgency and dysuria for the past three days. She has a history of a urinary tract infection once. She is sexually active and uses condoms for contraception. She is otherwise healthy and does not take any medications or supplements. She does not have fever, chills, flank pain or vaginal discharge. Which of the following organisms is the most likely cause of this patient's symptoms? A. Enterococcus faecalis B. Klebsiella pneumoniae C. Proteus mirabilis D. Staphylococcus saprophyticus E. Escherichia coli

E Acute cystitis in a healthy, non-pregnant woman is considered uncomplicated and is very common. Escherichia coli causes 80 to 85 percent of cases. The other major pathogens are Staphylococcus saprophyticus, Klebsiella pneumoniae, Enterococcus faecalis and Proteus mirabilis. The physician must consider antibiotic resistance when determining treatment.

A 32-year-old G1P0 woman at 10 weeks gestation presents to your office after an ultrasound evaluation has revealed a diamniotic, dichorionic twin gestation. She is very concerned about the risk for preterm delivery. Which intervention would you recommend as a possible means to reduce the risk of a preterm, low-birthweight infant? A. Bed rest B. Cervical cerclage C. Tocolytics starting at 24 weeks D. Home uterine monitoring E. Early, good weight gain

E Although prematurity has been recognized as a major cause of morbidity and mortality among twin gestations, interventions for prevention of prematurity have, in general, been unsuccessful. Studies show that an adequate weight gain in the first 20 to 24 weeks of pregnancy is especially important for women carrying multiples and may help to reduce the risk of having preterm and low-birth weight babies. These pregnancies tend to be shorter than singleton pregnancies, and studies suggest that a good early weight gain aids in development of the placenta, possibly improving its ability to pass along nutrients to the babies. Bed rest, long prescribed by obstetricians for the prevention of preterm birth, has never been shown to be efficacious, and may be associated with thromboembolic complications. An observational study of prophylactic cerclage for twin gestations failed to show any benefit. Tocolytic drugs for prevention of preterm labor in asymptomatic women with twin gestations have not been shown to be effective. Home uterine activity monitoring is another intervention that has been shown to be ineffective.

A 29-year-old G3P0 woman presents for evaluation and treatment of pregnancy loss. Her past medical history is remarkable for three early (<14 weeks gestation) pregnancy losses. Parental karyotype was normal. Which of the following is the most appropriate next step in the management of this patient? A. Place a prophylactic cerclage with her next pregnancy B. Obtain serial cervical length with her next pregnancy C. Recommend 17-hydroxyprogesterone with her next pregnancy D. Check for Factor V Leiden mutation E. Check antiphospholipid antibodies

E Antiphosphospholipid antibodies are associated with recurrent pregnancy loss. The workup for antiphospholipid syndrome includes assessment of anticardiolipin and beta-2 glycoprotein antibody status, PTT, and Russell viper venom time. There are multiple etiologies for recurrent pregnancy loss, which is defined as > two consecutive or > three spontaneous losses before 20 weeks gestation. Etiologies include anatomic causes, endocrine abnormalities such as hyper or hypothyroidism and luteal phase deficiency, parental chromosomal anomalies, immune factors such as lupus anticoagulant and idiopathic factors. Her history is not consistent with cervical insufficiency which is diagnosed typically in the second trimester by history, physical exam and other diagnostic tests, such as ultrasound. Serial cervical lengths or placement of a cerclage are not indicated in this patient. Treatment with 17-hydroxyprogesterone is indicated in patients with a history of prior preterm birth. Factor V Leiden mutation has not been associated with recurrent pregnancy loss. It can be associated with thrombotic events.

A 34-year-old G4P4 woman is diagnosed with endometritis following a Cesarean delivery three days ago. Which of the following is the most likely causative agent(s) of endometritis in this patient? A. Aerobic streptococcus B. Anaerobic streptococcus C. Aerobic staphylococcus D. Anaerobic staphylococcus E. Aerobic and anaerobic bacteria

E Bacterial isolates related to postpartum endometritis are usually polymicrobial resulting in a mix of aerobes and anaerobes in the genital tract. The most causative agents are Staphylococcus aureus and Streptococcus.

A 25-year-old G1 woman at 20 weeks gestation desires termination of the pregnancy. Her prenatal course has been unremarkable except for a chromosomal analysis positive for Trisomy 18. She desires an autopsy of the fetus. Which of the following is the most appropriate next step in the management of this patient? A. Perform a dilation and curettage B. Perform a dilation and evacuation C. Await fetal demise then start induction of labor D. Perform an induction with oxytocin E. Perform an induction with intravaginal prostaglandins

E Both medical and surgical abortions are options for this patient, depending on her personal preferences. However, if she desires an autopsy, she must undergo a medical abortion in order to have an intact fetus. Abortion is legal until viability is achieved (24 weeks gestation) unless a fetal anomaly inconsistent with extrauterine life is identified. A dilation and curettage is performed if the fetus is less than 16 weeks, while dilation and evacuation can be performed after 16 weeks by those trained in the procedure. Induction with oxytocin at this early gestational age has a high failure rate.

A 32-year-old G3P3 woman comes to the office to discuss permanent sterilization. She has a history of hypertension and asthma (on corticosteroids). She has been married for 10 years. Vital signs show: blood pressure 140/90; weight 280 pounds; height 5 feet 9 inches; and BMI 41.4kg/m2. You discuss with her risks and benefits of contraception. Which of the following would be the best form of permanent sterilization to recommend for this patient? A. Laparoscopic bilateral tubal ligation B. Mini laparotomy tubal ligation C. Exploratory laparotomy with bilateral salpingectomy D. Total abdominal hysterectomy E. Vasectomy for her husband

E Both vasectomy and tubal ligation are 99.8% effective. Vasectomies are performed as an outpatient procedure under local anesthesia, while tubal ligations are typically performed in the operating room under regional or general anesthesia; therefore carrying slightly more risk to the woman, assuming both are healthy. She is morbidly obese, so the risk of anesthesia and surgery are increased. In addition, she has chronic medical problems that put her at increased risk of having complications from surgery.

You just delivered a 32-year-old patient at term while covering for your partner. She had an uneventful pregnancy, followed by a normal spontaneous vaginal delivery. She is taking Sertraline (Zoloft), a selective serotonin uptake inhibitor (SSRI) as an antidepressant and wants to breastfeed. What is the next best step in management of this patient at this time? A. Decrease her SSRI dose by 50%, since these drugs are concentrated in the breast milk B. Consult psychiatry about changing medications and discard the expressed milk in the meantime C. Discontinue the medications so she can breastfeed D. Increase her SSRI dose, since these drugs are not concentrated in the breast milk and she is at great risk for postpartum depression E. Continue the medications, since there is negligible risk for the newborn

E Breastfeeding is beneficial to both mother and infant. Current recommendations state that SSRI medications can be safely used during lactation. Several studies show that SSRIs are secreted in breast milk, however no detectable levels of the drug were found in the infants' serum. In addition, no adverse effects were noted in the infants by either their parents or pediatricians following the infants.

A 48-year-old G3P3 woman recently had an abnormal Pap smear with high grade squamous intraepithelial lesion (HSIL). Colposcopically-directed biopsy revealed cervical intraepithelial neoplasia 3 (CIN 3). A loop electrosurgical excision procedure (LEEP) is subsequently performed. In reviewing the pathologic specimen with the pathologist, abnormal squamous cells are seen extending 2 mm beyond the basement membrane. What is the patient's diagnosis? A. CIN 1 B. CIN 2 C. CIN 3 D. Carcinoma in situ E. Microinvasive cervical cancer

E Cervical dysplasia is graded based on extent of involvement of the epithelial layer but does not extend below the basement membrane. Carcinoma in situ (CIS) represents abnormal cells involving the entire epithelium to the basement membrane. In cancer, the cells invade beyond the basement membrane. In microinvasive cancer, they invade less than 3 mm.

An 18-year-old G0 presents with a one-year history of hirsutism and acne. She had menarche at age 14 and her menses have been irregular every 26-60 days. Her sister has a similar pattern of hair growth. The patient is 5 feet 4 inches tall and weighs 180 pounds. On exam, a few terminal hairs were identified on her chin and upper lip. TSH, prolactin, total testosterone, and DHEAS levels are normal. Which of the following is the most appropriate next test to evaluate this patient's condition? A. Estradiol levels B. Serum cortisol levels C. Urinary cortisol levels D. Random blood glucose E. 17-hydroxyprogesterone

E Checking 17-hydroxyprogesterone would rule out late onset 21-hydroxylase deficiency. Normal TSH, Prolactin, total testosterone and DHEAS levels rule out pituitary or adrenal tumors. The patient could have polycystic ovarian syndrome; however, normal serum testosterone levels make it less likely.

A 26-year-old G0 women returns for a follow-up visit regarding suspected endometriosis. She has been using NSAIDs and birth control pills to help manage her pelvic pain which has been getting worse. While discussing further treatment options, she asks if there is any test or procedure you can perform to confirm her diagnosis. Which of the following would you recommend? A. Blood FSH/LH ratio and estradiol level B. Pelvic ultrasound C. CT scan of the abdomen and pelvis D. MRI of the pelvis E. Diagnostic laparoscopy

E Definitive diagnosis is based on exploratory surgery and biopsies, although endometriosis is usually initially treated based on the clinical presentation. In addition, this patient can benefit from laparoscopy, since she has failed the two most common treatments for endometriosis, NSAIDs and OCPs. There is no imaging study or blood test that can confirm the diagnosis of endometriosis.

A 33-year-old G2P1 presents at 36 weeks gestation for consultation because ultrasound revealed a fetus with biometry consistent with 30 5/7 weeks gestation. The EFW is less than the 5th percentile. Umbilical artery Doppler studies are abnormal. There is reverse end diastolic flow and the amniotic fluid volume is decreased. The AFI is 1.1 cm. Which of the following is the most appropriate next step in the management of this patient? A. Close observation with twice weekly NSTs and amniotic fluid assessments B. Close observation with twice weekly NSTs, amniotic fluid assessments and weekly umbilical artery Doppler studies C. Induction of labor D. Induction of labor at term (37 weeks gestation) E. Delivery by Cesarean section

E Delivery is indicated in a fetus with IUGR at 36 weeks gestation with oligohydramnios and abnormal umbilical artery Doppler studies. Although there is an increased incidence of fetal intolerance of labor, induction of labor is generally preferred over elective Cesarean delivery. Delivery at term is indicated in fetuses with IUGR with reassuring fetal testing including a normal amniotic fluid volume.

A 23-year-old G0 woman with last menstrual period 14 days ago presents to the office because she had unprotected intercourse the night before. She does not desire pregnancy at this time and is requesting contraception. She has no medical problems and is not taking any medications. In addition to offering her counseling and testing for sexually transmitted infections, which of the following is the most appropriate next step in the management of this patient? A. Observe for two weeks to establish if pregnancy occurred before initiating treatment B. Begin oral contraceptives now C. Begin oral contraceptives after her next normal menstrual period D. Provide emergency contraception, and return after next menstrual period E. Provide emergency contraception, then begin oral contraceptives immediately

E Emergency contraceptive pills are not an abortifacient, and they have not been shown to cause any teratogenic effect if inadvertently administered during pregnancy. They are more effective the sooner they are taken after unprotected intercourse, and it is recommended that they be started within 72 hours, and no later than 120 hours. Plan B, the levonorgestrel pills, can be taken in one or two doses and cause few side effects. Emergency contraceptive pills may be used anytime during a woman's cycle, but may impact the next cycle, which can be earlier or later with bleeding ranging from light, to normal, to heavy.

A 35-year-old G0 presents with irregular menstrual periods occurring every six to twelve weeks with occasional inter-menstrual bleeding. Currently, she has been bleeding daily for the last four weeks. She reports that her periods have always been irregular, but have become more so with heavier flow and cramping in the last year. She is sexually active with one partner. On physical exam, she is morbidly obese with no abnormalities detected on pelvic exam. Which of the following is the most appropriate next step in the management of this patient? A. Luteinizing hormone level (LH) B. Follicle stimulating hormone level (FSH) C. Testosterone level D. Pelvic CT E. Endometrial biopsy

E Endometrial biopsy should be performed to rule out endometrial hyperplasia or carcinoma given the history of irregular bleeding, coupled with the increased risk of these diagnoses in morbidly obese patients. While an ultrasound may be helpful, a pelvic CT is not useful in the workup for potential endometrial neoplasia. LH and FSH levels would not aid in the diagnostic workup and testosterone levels would not be useful, unless signs of hirsutism or virilization are present.

A 57-year-old G0 postmenopausal woman presents to her gynecologist with a complaint of vaginal bleeding for one week. The patient reports the cessation of normal menses approximately four years ago. She has had no previous episodes of irregular bleeding except when she took hormonal replacement therapy for six months. She saw her nurse practitioner five months ago and reports having a normal gynecologic evaluation and a normal Pap smear. Her past medical history is significant for hypercholesterolemia and diet-controlled diabetes mellitus. Physical exam reveals a 5 feet 3 inches tall, 275-pound woman in no acute distress. Pelvic exam demonstrated a normal vulva, urethra, vagina and cervix. Bimanual exam was normal. An endometrial biopsy was obtained and demonstrated complex atypical hyperplasia. Which of the following is this patient's greatest risk factor for developing endometrial cancer? A. Nulliparity B. Obesity C. Postmenopausal status D. Use of hormone replacement therapy E. Complex atypical hyperplasia

E Endometrial cancer is a gynecologic malignancy that has easily identifiable risk factors and typically presents with symptoms that lead to an early diagnosis. Risk factors include nulliparity, obesity, late menopause, hypertension and exposure to unopposed estrogens. Of these risk factors, obesity confers the greatest risk of developing endometrial carcinoma, especially when the patient is more than 50 pounds over ideal body weight (10-fold increase). However, in this case, the patient's greatest risk for developing an endometrial cancer is the presence of complex atypical hyperplasia (CAH) on endometrial biopsy. If left untreated, this process has approximately a 28% chance of progressing to an invasive cancer. More importantly, approximately 30% of women with a diagnosis of CAH will be found to have an invasive endometrial cancer on final pathology. Most women who develop endometrial cancer are postmenopausal, but this is less of an issue because of the finding of CAH.

A 21-year-old G1P0 woman delivered a 4000 gram infant by a low-forceps delivery after a protracted labor course that included a three-hour second stage. Her prenatal course was notable for development of anemia, poor weight gain and maternal obesity. Following the delivery, the patient was noted to have a vaginal sulcus laceration and a third-degree perineal laceration, which required extensive repair. Her hematocrit was noted to be 30% on postpartum day one. Which of the following factors places this patient at greatest risk for developing a puerperal infection? A. Third-degree perineal laceration B. Poor nutrition C. Obesity D. Anemia E. Protracted labor

E Endometritis in the postpartum period is most closely related to the mode of delivery. Endometritis can be found in less than 3% of vaginal births and this is contrasted by a 5-10 times higher incidence after Cesarean deliveries. Factors related to increased rates of infection with a vaginal birth include prolonged labor, prolonged rupture of membranes, multiple vaginal examinations, internal fetal monitoring, removal of the placenta manually and low socioeconomic status.

A 32-year-old G1 is seeing you in consultation at 35 weeks gestation. Ultrasound reveals limited fetal growth over the past three weeks. Biometry is consistent with 30-5/7, EFW 1900 g, less than 10th percentile. You counsel her about short and long-term complications for her baby. This fetus is at increased risk for all of the following adult disorders EXCEPT: A. Cardiovascular disease B. Chronic hypertension C. Chronic obstructive lung disease D. Diabetes E. Osteoporosis

E Epidemiologic studies indicate that fetal growth restriction is a significant risk factor for the subsequent development of cardiovascular disease, chronic hypertension, chronic obstructive lung disease and diabetes. Researchers suggest that the phenomenon of programming may be operable and that an adverse fetal environment during a critical period of fetal growth helps to promote these adult diseases. Osteoporosis risk factors include family history, slender body composition, prior history of osteoporosis, Asian and Caucasian ethnicity, alcohol consumption, smoking, sedentary lifestyle, excess thyroid or corticosteroids and use of anticonvulsant medications.

At one minute of life, an infant has a heart rate greater than 120 beats/minute, is crying, has acrocyanosis, gags when suctioned and is vigorously moving all four extremities. What is the APGAR score for this infant? A. 5 B. 6 C. 7 D. 8 E. 9

E Heart rate= 2, Respiratory rate= 2, Reflex = 2, Activity =2, Color =1. Therefore, the one-minute APGAR score is 9.

A 35-year-old G3P3 woman requests contraception. Her youngest child is seven years old. Her periods have been regular since she discontinued breastfeeding five years ago. Her past medical history includes depression that is controlled with antidepressants, and a history of deep venous thrombosis. She denies smoking or alcohol use. In the past, oral contraceptive pills have caused her to have severe gastrointestinal upset. What in her history makes her an ideal candidate for progestin-only pills? A. Depression B. Smoking history C. Severe nausea on combined oral contraceptives D. Lactation history E. Deep venous thrombosis

E Ideal candidates for progestin-only pills include women who have contraindications to using combined oral contraceptives (estrogen and progestin containing). Contraindications to estrogen include a history of thromboembolic disease, women who are lactating, women over age 35 who smoke or women who develop severe nausea with combined oral contraceptive pills. Progestins should be used with caution in women with a history of depression.

An 18-year-old woman comes to the office due to vaginal spotting for the last two weeks. Her menstrual periods were regular until last month, occurring every 28-32 days. Menarche was at age 13. She started oral contraceptives three months ago. On pelvic examination, the uterus is normal in size, slightly tender with a mass palpable in the right adnexal region. No adnexal tenderness is noted. Which of the following tests is the most appropriate next step in the management of this patient? A. Endometrial biopsy B. Pelvic MRI C. Pelvic sonography D. Abdominal CT Scan E. Urine pregnancy test

E It is vitally important to rule out pregnancy in the evaluation of abnormal uterine bleeding. Sonography could be considered as a next step if the pregnancy test is negative in order to evaluate the adnexal finding. Abdominal CT or MRI would not be performed in this patient unless advanced adnexal pathology was found on pelvic sonography. Endometrial biopsy would rarely be indicated in a teen with abnormal bleeding, unless morbidly obese and anovulatory.

A 39-year-old G1 presents in labor at term. The estimated fetal weight is 3200 g. She is 10 cm dilated with left sacrum anterior at +2 station. Which of the following is the most appropriate next step in the management of this patient? A. Attempt external version B. Attempt internal version C. Apply forceps D. Apply a vacuum E. Recommend a Cesarean section

E Most recent data suggests that breech infants delivered vaginally are at higher risk for neonatal complications. Therefore, it would be recommended that this patient undergo a Cesarean section, especially since this is her first pregnancy. External cephalic version is contraindicated in active labor.

A 31-year-old G0 has been diagnosed with uterine fibroids. A fluid contrast ultrasound confirmed the presence of two intramural fibroids measuring 5 x 6 cm and 2 x 3 cm that appear to be distorting the patient's uterine cavity. The patient has a two-year history of infertility. She has had a thorough infertility work up. No etiology for her infertility has been identified. Which of the following treatments is most appropriate for this patient? A. Hysteroscopy B. Uterine curettage C. Gonadotropin-releasing hormone agonist D. Uterine artery embolization E. Myomectomy

E Myomectomy is warranted in younger patients whose fertility is compromised by the presence of fibroids that cause significant distortion of the uterine cavity. A myomectomy may be indicated in infertility patients when the fibroids are of sufficient size or location to be a probable cause of infertility and when no more likely explanation exists for the failure to conceive. Hysteroscopy is a procedure that involves placing a scope through the cervical os to assess the endometrial cavity. The patient has already been diagnosed with uterine fibroids that are distorting her cavity and she has already had a fluid contrast ultrasound, so it is unnecessary to perform hysteroscopy on this patient. Treatment with GnRH agonists can be useful to shrink fibroids in anticipation of surgery, or if menopause is expected soon. This patient desires future childbearing, therefore, its use would not be an appropriate option. Uterine artery embolization can be recommended for women who have completed child-bearing because of the unclear long-term effects on fertility.

A 24-year-old Rh-negative G2P1 woman at 18 weeks gestation is positive for anti-D antibodies. In discussing the risks of Rh sensitization with her, you tell her that her fetus may be at increased risk of significant perinatal disease including fetal anemia. Which of the following non-invasive tests can detect severe fetal anemia? A. Umbilical artery systolic-diastolic ratio B. Biophysical profile C. Amniotic fluid index D. Umbilical artery blood flow E. Middle cerebral artery peak systolic velocity

E Noninvasive diagnosis of fetal anemia has been performed with Doppler ultrasonography. The use of middle cerebral artery peak systolic velocity in the management of fetuses at risk for anemia because of red cell alloimmunization has emerged as the best test for the noninvasive diagnosis of fetal anemia. All the other listed tests are for assessment of fetal well-being and non-specific to detect fetal anemia. Amniocentesis and cordocentesis have been used for many years to diagnose fetal anemia due to red cell alloimmunization. These techniques, however, are invasive and many complications are associated with their use.

A 27-year old G3P1 woman is admitted to the orthopedic service after open reduction and internal fixation of her femur status post a motor vehicle accident. Her past medical history is significant for diabetes (controlled with metformin) and a history of a deep venous thrombosis three years ago while taking an oral contraceptive. While inpatient, she has been receiving ibuprofen for pain control and oxycodone for breakthrough pain. Additionally, anticoagulation therapy was began with IV heparin, with a goal of transitioning to warfarin therapy. During her hospital stay, her preoperative labs showed a positive urine pregnancy test and an ultrasound confirmed a six week intrauterine pregnancy. Which of the following medications should be discontinued because of potential teratogenicity? A. Metformin B. Heparin C. Ibuprofen D. Oxycodone E. Warfarin

E Of the medications she is currently taking, none are contraindicated at this gestational age. Ibuprofen is safe to take until around 32 weeks gestation, when premature closure of the ductus arteriosis is a risk. While heparin is safe during pregnancy, warfarin has known teratogenic affects and should not be given. If continued anticoagulation is necessary, low molecular weight heparin is the drug of choice.

A 24-year-old G1P1 woman comes to the office requesting contraception. Her past medical history is unremarkable, except for a family history of ovarian cancer. She denies alcohol, smoking and recreational drug use. She is in a monogamous relationship. She wants to decrease her risk of gynecological cancer. Of the following, what is the best method of contraception for this patient? A. Female condoms B. Male condoms C. Copper containing intrauterine device D. Progesterone containing intrauterine device E. Combined oral contraceptives

E Oral contraceptives will decrease a woman's risk of developing ovarian and endometrial cancer. The earlier, higher dose oral contraceptive pills have been linked to a slight increase in breast cancer, but not the most recent lower dose pills. Women who use oral contraceptive pills have a slightly higher risk of developing cervical intraepithelial neoplasia, but their risk of developing PID, endometriosis, benign breast changes and ectopic pregnancy are reduced. Both hypertension and thromboembolic disorders can be a potential side effect from using oral contraceptive pills. Condoms and intrauterine devices will not lower her risk of ovarian cancer.

A 34-year-old G1P0 woman presents with vaginal spotting. An ultrasound confirms a non-viable intrauterine pregnancy. She is otherwise healthy. Her partner accompanies her and is supportive. The patient wishes to avoid any unnecessary medical interventions and asks whether she can safely let nature take its course. What is the best next step in the management of this patient? A. Immediate dilation and suction curettage B. Dilation and suction curettage in one week C. Immediate treatment with misoprostol D. Treatment with misoprostol in one week E. Expectant management

E Patients experiencing early pregnancy loss can safely consider several different treatments, including expectant management, medical treatment to assist with expulsion of the pregnancy or surgical evacuation. Provided the patient is hemodynamically stable and reliable for follow-up, expectant management is appropriate therapy. At the gestational age described, expectant management portends no increase in risk of either hemorrhage or infection compared with surgical or medical evacuation. Regardless of method chosen, the patient's blood type should be checked and rhogam administered as indicated.

A 49-year-old G1P1 comes to your office for menopause counseling. She has been experiencing severe sleep disturbances and night sweats for the past four months. She would like to begin hormone therapy, but is concerned because she has elevated cholesterol levels for which she takes medication. You explain to her that hormone therapy has the following effect on a lipid/cholesterol profile: A. Both LDL and HDL levels increase B. Both HDL and LDL levels decrease C. HDL and LDL levels are unaffected D. HDL levels increase and LDL levels are unaffected E. HDL levels increase and LDL levels decrease

E Recent data have confirmed the overall positive effects of hormone therapy on serum lipid profiles. The most important lipid effects of postmenopausal hormone treatment are the reduction in LDL cholesterol and the increase in HDL cholesterol. Estrogen increases triglycerides and increases LDL catabolism, as well as lipoprotein receptor numbers and activity, therefore causing decreased LDL levels. Hormones inhibit hepatic lipase activity, which prevents conversion of HDL2 to HDL3, thus increasing HDL levels. Hormone therapy is not currently recommended for the primary prevention of heart disease.

A 17-year-old G1P0 female at 39 weeks gestation presents with increased swelling in her face and hands over the last two days. Her blood pressure is 155/99. She has 2 plus pitting edema of the lower extremities. A 24-hour urine collection shows 440 mg of protein. What is the next best step in the management of this patient? A. Fluid restriction B. Magnesium sulfate C. Furosemide D. Hydralazine E. Delivery

E Regardless of disease severity, the only definitive therapy for preeclampsia is delivery of the fetus and placenta. This solution can occasionally be delayed in the setting of stable disease (mild or severe) when it occurs at an extremely early gestational age. Fluid management must be monitored closely in this person. Magnesium sulfate is the mainstay of therapy during labor and for 24 hours postpartum to lower the seizure threshold. Low-dose aspirin may have some benefit in decreasing the risk of preeclampsia in a subset of high-risk patients. Hydralazine is often the antihypertensive agent of choice for controlling elevated blood pressures in the acute setting.

A 22 year-old delivered her first baby five days ago after a prolonged labor and subsequent Cesarean section for arrest of cervical dilation at 7 centimeters. Fever was noted on postoperative day 2 and despite intravenous broad spectrum antibiotics, she continues to have temperature spikes above 101.3°F, 38.5°C. She is eating a normal diet and ambulating normally. On physical examination, her breasts have no erythema and nipples are intact. Her abdomen is soft, uterine fundus is firm and nontender, and her incision is healing without induration or erythema. She has normal lochia and her urinalysis is normal. Pelvic examination reveals a firm nontender uterus and no adnexal masses or tenderness. Which of the following treatments is indicated for this patient? A. Addition of antifungal therapy B. Addition of oral antibiotic therapy C. Addition of antiviral therapy D. Surgical exploration E. Heparin anticoagulation

E Septic thrombophlebitis involves thrombosis of the venous system of the pelvis. Diagnosis is often one of exclusion of other causes, but sometimes a CT scan will reveal thrombosed veins. Treatment requires addition of anticoagulation to antibiotics and resolution of fevers is generally rapid. Anticoagulation treatment is short-term. The addition of oral antibiotics has no extra benefit on a patient who is already on broad spectrum IV antibiotics. She has no evidence of fungal or viral infections, so therapy for these is not indicated. There is also no indication she needs surgery.

A 27 year-old G2P1 at 18 weeks gestation presents to the emergency room complaining of fever, nausea, vomiting, and mid-abdominal pain for the last 24 hours. For the last 12 hours, she has had no appetite. She has been healthy, but reports that her 3 year-old son has had diarrhea for 2 days. Physical examination reveals a blood pressure of 100/60, pulse 88, respiratory rate 18, and temperature 102.0°F (38.9°C). Abdominal examination reveals decreased bowel sounds and tenderness more pronounced on the right than the left. Which of the following is the next best step in the management of this patient? A. Check a complete blood count B. Abdominal and pelvic ultrasound C. Plain abdominal radiograph D. Helical computed tomography E. Graded compression ultrasound

E Suspected appendicitis is one of the most common indications for surgical abdominal exploration during pregnancy. The diagnosis of appendicitis is more difficult to make in pregnancy because anorexia, nausea, and vomiting that accompany normal pregnancy are also common symptoms of appendicitis. In addition, the enlarged uterus shifts the appendix upward and outward toward the flank, so that pain and tenderness may not be located in the right lower quadrant. Appendicitis is easily confused with preterm labor, pyelonephritis, renal colic, placental abruption, or degeneration of a uterine myoma. Peritonitis and appendiceal rupture are more common during pregnancy. The diagnosis is made based on clinical findings and graded compression ultrasonography that is sensitive and specific especially before 35 weeks gestation. This noninvasive procedure should be considered first in working up suspected acute appendicitis. Selective imaging of the appendix using helical computed tomography may be a safe and potentially reliable tool to accurately identify appendiceal changes in appendicitis, except that radiation exposure using this test is higher than graded compression ultrasonography. A plain abdominal radiograph can be used to identify air fluid levels or free air but offers little diagnostic value for appendicitis.

A 50-year-old woman, gravida 3. para 3, had her last menstrual period 12 months ago and has had no vaginal bleeding since. She is a competitive runner who works full time as a regional sales manager requiring her to do a lot of driving. She reports worsening vasomotor symptoms, which have resulted in insomnia and interruptions at work. She requests treatment to manage her symptoms so she can maintain her busy lifestyle. She does not wish to use estrogen. The nonhormonal treatment most likely to relieve her vasomotor symptoms is . A) black cohosh B) dong quai root C) clonidine D) gabapentin E) paroxetine hydrochloride

E) paroxetine hydrochloride

A 54-year-old G2P2 presents for a health maintenance examination. She has a history of breast cancer treated with mastectomy with reconstruction, chemotherapy, and is currently on tamoxifen. She has been in remission for two years and has been menopausal since the initiation of her chemotherapy. She experiences very mild hot flashes and is not sleeping well. She appears apprehensive during the examination, although her examination is completely normal except for some mild vaginal atrophy. At the conclusion of the office visit, she finally opens up and admits that she has a new boyfriend after having gone through a divorce five years earlier. She is anxious about initiating sexual activity again and wants your advice on what she should do. Which of the following is likely contributing to her anxiety? A. Breast cancer diagnosis B. Menopausal symptoms C. Body image D. Vaginal atrophy E. All of the above

E Taking a good sexual history is critical to understanding the root cause of a woman's concerns about possible sexual dysfunction. Etiologies may be physiologic, hormonal, psychologic, and often multifactorial. To distinguish among these causes, a thorough inquiry into a woman's hormonal, sexual, medical, and social history is necessary. A sexual history must include a non-judgemental and candid discussion related to her interests, desires, and practices to better understand potential causes for concern and dysfunction.

A 34-year-old G2P2 who recently underwent colposcopy with biopsy following a Pap smear which showed low-grade squamous intraepithelial lesion, comes in to discuss her results. The biopsy results showed HSIL (CIN III). Endocervical curettage showed benign endocervical cells. What is the most appropriate next step in the management of this patient? A. Radical hysterectomy B. Simple hysterectomy C. Repeat Pap smear in four months D. Human Papilloma Virus (HPV) testing E. Loop Electrosurgical Excision Procedure (LEEP)

E The LEEP procedure is usually done in the office under local intracervical anesthesia. It involves using an electrosurgical unit (similar to the Bovie in the operating room), along with a wire loop of varying sizes to remove the entire transformation zone and the dysplastic area(s) identified during colposcopy. This tissue is then sent to pathology so that the area of dysplasia can be fully evaluated. A radical hysterectomy is an option for patients with invasive cervical carcinoma stage Ia2 through IIa. This patient has only cervical dysplasia (not invasive cancer). A simple hysterectomy is not needed, as it is more involved than a LEEP. Serial Pap testing for patients with biopsy proven CIN III or HSIL is generally not recommended. Although spontaneous regression of dysplasia may occur with a high-grade lesion, the rate of regression is much lower than with LSIL. On the other hand, the progression rate of HSIL to invasive cancer is much higher (up to 12%). High risk HPV testing can be recommended as an adjunct to Pap smears such as with ASCUS Pap smears. It would not offer additional information in this patient's scenario.

A 54-year-old G2P2 post-menopausal woman presents for a health maintenance examination. She notes the new onset of a lump in her vagina, but denies any pain, abnormal bleeding or vaginal discharge. She has well-controlled diabetes mellitus and hypertension. She is recently sexually active with a new partner since the death of her husband three years ago. She smokes a half-pack per day, and has done so since age 18. On examination, she is noted to have a somewhat firm and somewhat fixed, non-tender 4 cm mass in her labia majora at the level of the bartholin's gland on the right. There is no associated erythema or discharge, and the remaining vulvar exam and pelvic exam are unremarkable. Her groin examination reveals no adenopathy. What is the most likely diagnosis: A. Lipoma B. Fibroma C. Bartholin's gland cyst D. Bartholin's gland abscess E. Bartholin's gland neoplasm

E The finding of a mass in the Bartholin's gland is highly suspicious for malignancy and requires excision/biopsy, especially in a post-menopausal women. The histology is typically adenocarcinoma. This is unlikely to be a fibroma or lipoma given the recent onset and fixed nature of the mass. A benign Bartholin's gland cyst is also unlikely given the patient's age and rather abrupt onset, and any finding of a new Bartholin's gland cyst in a post-menopausal woman should be further investigated. This is not an abscess given the absence of signs and symptoms of cellulitis or infection.

A 65-year-old G2P1 woman has been referred to you for further evaluation of postmenopausal bleeding. She initially was seen by her internist after two weeks of intermittent vaginal spotting. She reports a similar episode approximately two months ago. A recent exam and Pap smear were normal. A transvaginal ultrasound showed a homogeneous endometrial lining measuring 5.0 mm. A subsequent office endometrial sample was obtained and returned with rare atypical cells. What is the most appropriate next step in the management of this patient? A. Repeat office endometrial sample B. Follow-up ultrasound in eight to twelve weeks C. Return visit in three to six months D. Abdominopelvic CT scan E. Dilation and curettage

E The incidence of endometrial cancer in postmenopausal women with irregular bleeding ranges from 4% to 25%. More importantly, of the women ultimately diagnosed with endometrial cancer, 90% report a history of vaginal bleeding. The diagnostic test of choice is the office endometrial sampling, which has a detection rate as high as 99% in postmenopausal women. In a woman who has persistent symptoms or findings suggestive of an underlying endometrial malignancy, as does this patient with atypical cells on endometrial biopsy, further investigation with dilation and curettage is warranted.

*A 35-year-old G3P3 woman comes to the office because she desires contraception. Her past medical history is significant for Wilson's disease, chronic hypertension and anemia secondary to menorrhagia. She is currently on no medications. Her vital signs reveal a blood pressure of 144/96. Which of the following contraceptives is the best option for this patient? A. Progestin-only pill B. Low dose combination contraceptive C. Continuous oral contraceptive D. Copper containing intrauterine device E. Levonorgestrel intrauterine device

E The levonorgestrel intrauterine device has lower failure rates within the first year of use than does the copper containing intrauterine device. It causes more disruption in menstrual bleeding, especially during the first few months of use, although the overall volume of bleeding is decreased long-term and many women become amenorrheic. The levonorgestrel intrauterine device is protective against endometrial cancer due to release of progestin in the endometrial cavity. She is not a candidate for oral contraceptive pills because of her poorly controlled chronic hypertension. The progestin only pills have a much higher failure rate than the progesterone intrauterine device. She is not a candidate for the copper-containing intrauterine device because of her history of Wilson's disease.

A 17-year-old nulliparous sexually active female presents to the emergency department with pelvic pain that began 24 hours ago. She reports menarche at the age of 15 and coitarche soon thereafter. She has had four male partners, including her new boyfriend of a few weeks. Her blood pressure is 100/60, pulse 100, and temperature 102.0°F (38.9°C). On speculum examination, you note a foul-smelling mucopurulent discharge from her cervical os and she has significant tenderness with manipulation of her uterus. What is the next best step in the management of this patient? A. Outpatient treatment with oral broad spectrum antibiotics B. Outpatient treatment with intramuscular and oral broad spectrum antibiotics C. Intravenous antibiotics and dilation and curettage D. Inpatient treatment, laparoscopy with pelvic lavage E. Inpatient treatment and intravenous antibiotics

E The most likely cause of the symptoms and signs in this patient is infection with a sexually transmitted organism. The most likely organisms are both N. gonorrhoeae and chlamydia, and the patient should be treated empirically for both after appropriate blood and cervical cultures are obtained. Since the patient also has a high fever, inpatient admission is recommended for aggressive intravenous antibiotic therapy in an effort to prevent scarring of her fallopian tubes and possible future infertility.

A 23-year-old G1P0 woman at 40 weeks gestation presents to labor and delivery with contractions. At 10:00 am, her cervical exam is 2 centimeters dilated, 70% effaced and the vertex at 0 station. Clinical pelvimetry reveals an adequate pelvis and membranes are intact. The fetus is in a cephalic presentation and EFW is 3500 gms. Contractions are occurring every 3-4 minutes, based on the external monitor. Her labor slowly progresses and, at 1:00 pm, the patient has spontaneous rupture of membranes. Fetal surveillance remains reassuring. Her cervical exam is 5 centimeters dilated, 100% effaced, and 0 station. At 4:00 pm, the patient's cervical exam is unchanged. Contractions are occurring every 5-6 minutes. Which of the following is the most appropriate next step in the management of this patient? A. Perform a biophysical profile B. Have the patient ambulate C. Consent the patient for a Cesarean section secondary to failure to progress D. Continue fetal surveillance and reexamine the patient in two hours E. Begin oxytocin augmentation

E The patient has an arrest of dilatation in the active phase of labor. She is only having contractions every 5-6 minutes, so it is reasonable to start oxytocin to increase the frequency and strength of this patient's contractions. If the patient does not have cervical change once she is having more frequent contractions on oxytocin, it would be reasonable to place an IUPC (intrauterine pressure catheter) to assess the strength of the contractions. It is not yet necessary to perform a Cesarean delivery. Further observation and having the patient ambulate do not facilitate delivery. A biophysical profile is not indicated in this situation.

A 37-year-old G3P3 complains of severe premenstrual symptoms for the past two years. She finds her mood swings and irritability troubling and requests a hysterectomy, as she thinks that this procedure will alleviate her symptoms. Past medical history is only remarkable for high cholesterol and her physical examination, including pelvic examination, is normal. The patient's physician does not recommend a hysterectomy. Which of the following is the most likely explanation for the physician's recommendation not to perform a hysterectomy in this patient? A. An endometrial ablation would be preferable B. Past medical history C. Influence of thyroid hormone on symptoms D. Influence of adrenal gland on symptoms E. Influence of ovaries on symptoms

E The patient's mood swings are influenced by the hormonal shifts controlled by the hypothalamic-pituitary-ovarian axis. A hysterectomy or endometrial ablation would only resolve the menstrual bleeding component of this patient's symptoms, and have no effect on the hormonal production of the ovaries.

A 42-year-old G5P2 woman at 36 weeks gestation is diagnosed with preeclampsia. Her previous pregnancy was complicated by twins and preeclampsia at 36 weeks gestation. She also has had two spontaneous abortions at seven weeks gestation. Which of the following conditions is not associated with her increased risk for preeclampsia in this pregnancy? A. Previous history of preeclampsia B. Chronic hypertension C. Multifetal pregnancy D. Age E. Previous spontaneous abortion

E The previous history of spontaneous abortion does not put the patient at increased risk.The incidence of preeclampsia is commonly cited to be about 5 percent and is markedly influenced by parity. It is related to race and ethnicity and to genetic predisposition. Environmental factors are also likely to play a role. Other risk factors for preeclampsia include a previous history of the disease, chronic hypertension, multifetal pregnancy and molar pregnancy. In addition, patients at extremes of maternal age or with diabetes, chronic renal disease, antiphospholipid antibody syndrome, vascular or connective tissue disease or triploidy are at increased risk for developing preeclampsia

A 19-year-old G2P1 African American woman at 30 weeks gestation presents with preterm rupture of membranes six hours ago. Her prenatal course has been complicated by two episodes of bacterial vaginosis for which she was treated. She takes prenatal vitamins and iron. She denies substance abuse or alcohol use, but admits to smoking five cigarettes each day. Her prior pregnancy was delivered vaginally at 41 weeks after spontaneous rupture of membranes. Her blood pressure is 110/70; pulse 84; temperature 98.6°F (37.0°C). Pertinent sonographic findings reveal oligohydramnios and a cervical length of 30 mm. Which of the following is the most likely cause of preterm premature rupture of membranes in this patient? A. Ethnicity B. Smoking C. Previous premature rupture of membranes D. Cervical length E. Genital tract infections

E The primary risk factor for preterm rupture of membranes is genital tract infection, especially associated with bacterial vaginosis. All of the other listed options are risk factors. Smoking and prior preterm premature rupture of membranes (which she did not have previously because she delivered at 41 weeks) increases the likelihood of preterm rupture of membranes two-fold. A shortened cervical length is also a risk factor, but her cervical length is normal.

A 16-year-old nulliparous female presents to the emergency department with a two-day history of abdominal pain. She is sexually active with a new partner and is not using any form of contraception. Temperature is 101.8°F (38.8°C). On examination, she has lower abdominal tenderness and guarding. On pelvic exam, she has diffuse tenderness over the uterus and bilateral adnexal tenderness. Beta-hCG is <5. What is the most likely diagnosis in this patient? A. Ectopic pregnancy B. Appendicitis C. Acute cystitis D. Endometriosis E. Acute salpingitis

E The signs and symptoms of acute salpingitis can vary and be very subtle with mild pain and tenderness, or the patient can present in much more dramatic fashion with high fever, mucopurulent cervical discharge and severe pain. Important diagnostic criteria include lower abdominal tenderness, uterine/adnexal tenderness and mucopurulent cervicitis.

A 34 year-old G2P1 at 18 weeks gestation presents with a newly discovered lump in her left breast. Fine needle aspiration reveals adenocarcinoma. Which of the following is NOT a recommended therapy for breast cancer during pregnancy? A. Wide local excision biopsy B. Modified radical mastectomy C. Total mastectomy and node dissection D. Chemotherapy E. Radiotherapy

E There is no doubt that breast cancer is more aggressive in younger women. Whether it is more aggressive during pregnancy in young women is debatable. Slight delays (1 to 2 months) in clinical assessment, diagnostic procedures, and treatment of pregnant women with breast tumors are common. Approximately 30 percent of pregnant women with breast cancer have stage I disease, 30 percent have stage II, and 40 percent stages III or IV. Many clinical reports maintain that when breast cancer is diagnosed during pregnancy, the regional lymph nodes are more likely to contain microscopic metastases. Surgical treatment may be definitive for breast carcinoma during pregnancy and in the absence of metastatic disease a wide excisional biopsy, modified radical mastectomy, or total mastectomy with axillary node staging can be performed. Non-pregnant women receive adjunctive radiotherapy with breast-conserving surgery. However, this is not recommended during pregnancy due to sizeable abdominal scatter placing the fetus at significant risk for excessive radiation.

A 58-year-old G2P2 presents to your office complaining of two years of a vulvar rash. She has seen multiple physicians and no one has made a definitive diagnosis. The patient has experienced intermittent pruritus for one year. She has been prescribed every yeast medication known and has also used multiple over-the-counter products. She was recently given topical steroid cream, which did not alleviate her symptoms. She is a breast cancer survivor and was diagnosed and treated one year ago. She is presently on Tamoxifen. No vaginal bleeding has occurred since her menopause. On examination, her vulva is fiery red background mottled with whitish hyperkeratotic areas without a distinct lesion. No nodularity or tenderness is noted. The rest of her exam, with the exception of atrophy, is normal. What is the most likely diagnosis in this patient? A. Lichen sclerosus of the vulva B. Contact dermatitis C. Yeast vulvitis D. Psoriasis of the vulva E. Paget's disease of the vulva

E This is a typical description of Paget's disease of the vulva. Paget's is an in situ carcinoma of the vulva. The association with breast cancer is significant, but not as high as Paget's disease of the nipple. It would be unlikely for psoriasis to present this late in life. Contact dermatitis is unlikely to last for years and this woman has had therapy for yeast. Lichen sclerosus is possible and more common, but does not have the hyperkeratotic overlay and would have more likely responded to steroid use.

A 24-year-old G1P0 woman presents in active labor at 39 weeks gestation. She reports leaking fluid for the last two days. She develops a temperature of 102.0°F (38.9°C) and fetal heart rate is 180 beats/min with minimal variability. Maternal labs show: blood type O+; RPR non-reactive; HBsAg, negative; HIV negative; and GBS unknown. What will be the expected appearance of the baby at delivery? A. Vigorous, pink with normal temperature B. Vigorous, pale with low temperature C. Lethargic, pink with high temperature D. Lethargic, pale with low temperature E. Lethargic, pale with high temperature

E This patient clearly has chorioamnionitis. The fetal tachycardia may be in response to the maternal fever. Fetal tachycardia coupled with minimal variability is a warning sign that the infant can be septic. A septic infant will typically appear pale, lethargic and have a high temperature.

A 34-year-old G2P1 woman is 40 weeks gestation. She was admitted to labor and delivery in active labor 2 hours ago. Her cervix was 6 cm dilated and 100% effaced on admission. Her fetus was vertex and - 3 station. You are called to examine the patient after she experiences spontaneous rupture of membranes. The cervix is completely dilated and the fetal head is occiput anterior (OA) at +1 station. You palpate a 5 cm long section of umbilical cord in the patient's vagina. The fetal heart tracing is reassuring. The baseline is 130 beats per minute. There are multiple accelerations and no decelerations. The patient is having regular uterine contractions every 2-3 minutes. She has an epidural and is not feeling the contractions. What is the most appropriate next step in the management of this patient? A. Allow for passive descent of the fetal head with continuous fetal monitoring B. Have the patient start pushing with the contractions C. Gently attempt to replace the umbilical cord segment back up into the uterus D. Perform a forceps assisted vaginal delivery E. Elevate the fetal head with a vaginal hand and perform a Cesarean delivery

E This patient has an umbilical cord prolapse. Although fetal surveillance is reassuring, the most appropriate management is to continue to elevate the fetal head with a hand in the patient's vagina and call for assistance to perform a Cesarean delivery. It is important to elevate the fetal head in an attempt to avoid compression of the umbilical cord. Once an umbilical cord prolapse is diagnosed, expeditious arrangements should be made to perform a cesarean section. It is not appropriate to replace the umbilical cord into the uterus or allow the patient to continue to labor or perform a forceps-assisted vaginal delivery.

A 24-year-old G0 woman presents with multiple painful ulcers involving the vulva. The sores initially were fluid filled, but are now open, weeping and crusted. She reports a fever and is having difficulty voiding due to pain. She uses a vaginal ring for contraception. She has multiple sexual partners and uses condoms for vaginal intercourse. She is distraught that she may have a sexually transmitted infection. She is healthy and does not smoke or use drugs. On physical exam, she is in obvious distress. Temperature is 100.2°F (37.9°C), pulse 100. Examination of the genital tract is limited due to her discomfort. Multiple ulcers and erosions of variable size are localized to the perineum, labia minora and vestibule. Swelling is diffuse. The lesions are eroded, some with a purulent eschar. There is exquisite tenderness to touch. What further testing should be offered to this patient? A. RPR (rapid plasma regain) B. HIV C. Herpes culture D. Cervical DNA probe for gonorrhea and chlamydia E. All of the above

E This patient has classic primary herpes with painful genital ulcerations, fever and dysuria. Given the presence of one sexually transmitted infection, screening should be offered for other STIs. Resolution of the acute episode is required before a speculum can be inserted to allow endocervical sampling for gonorrhea and chlamydia. If it was a high-risk exposure, prophylactic empiric treatment could be offered to cover gonorrhea and chlamydia. The patient should be counseled that primary herpes can be acquired despite condoms and even by oral-genital inoculation. Hepatitis B vaccination should be offered to protect her against any future exposures. She should be encouraged to discuss her diagnosis with all sexual partners and to continue to reliably use latex condoms.

A 23-year-old G1 woman at six weeks gestation undergoes a medical termination of pregnancy. One day later, she presents to the emergency room with bleeding and soaking more than a pad per hour for the last five hours. Her blood pressure on arrival is 110/60; heart rate 86. On exam, her cervix is 1 cm dilated with active bleeding. Hematocrit on arrival is 29%. Which of the following is the most appropriate next step in the management of this patient? A. Admit for observation B. Repeat hematocrit in six hours C. Begin transfusion with O-negative blood D. Give an additional dose of prostaglandins E. Perform a dilation and curettage

E This patient is having heavy bleeding as a complication of medical termination of pregnancy. This is managed best by performing a dilation and curettage. It is not appropriate to wait six hours before making a decision regarding next step in management, or to just admit her for observation. Since the patient is not symptomatic from her anemia, it is not necessary to transfuse her at this time.

A 23-year-old G0 comes to the clinic because she is interested in becoming pregnant. She is in good health; however, she has not had any menses for the last two years. She had menarche at age 15, had normal periods until three years ago, when she started having periods irregularly every three months until it stopped two years ago. She has no history of pelvic infections or abnormal Pap smears. She exercises every day by running and has run four marathons in the last three years. She is 5 feet 10 inches tall and weighs 115 pounds. Her examination including a pelvic exam is normal. Laboratory results show: Results Normal Values TSH 3.5 mIU/ml 0.5-4.0 mIU/ml Free T4 0.9 ng/dl 0.8-1.8 ng/dl Prolactin 10 ng/ml <20 ng/ml FSH 6 mIU/ml 5-25 mIU/ml LH 4 mIU/ml 5-25 mIU/ml BHCG 2 mIU/ml <5 mIU/ml What is the most appropriate next step in the management of this patient? A. Check cortisol levels B. Order a brain MRI C. Obtain a pelvic ultrasound D. Check testosterone levels E. Check estrogen levels

E This patient most likely has exercise-induced hypothalamic amenorrhea, which is characterized by normal FSH and low estrogen levels. The other studies will not help determine the diagnosis. The best treatment is to encourage the patient to gain weight by decreasing exercise and increasing caloric intake. If her menses fail to resume, she may be treated with exogenous gonadotropins (LH and FSH) to help her conceive. Clomiphene citrate tends not to work as well, due to the baseline hypoestrogenic state.

A 22-year-old G0 presents with hirsutism which has been present since menarche. She states that she has laser treatments done to remove the hair on her chin every couple of months, and was wondering if there are additional treatments which might help her. She is otherwise in good health. She has normal menstrual cycles every 28 days. She is sexually active and uses birth control pills for contraception. The patient is adopted and has no information about family history. She is 5 feet 4 inches tall and weighs 125 pounds. On examination, the patient was noted to have terminal hair growth on her chest. Her TSH, Prolactin, total testosterone, DHEAS, 17-Hydorxyprogesterone levels are normal. Which of the following is the most likely underlying etiology for the hirsutism in this patient? A. Polycystic ovarian syndrome B. Side effects of the oral contraceptives C. Cushing's syndrome D. Adrenal tumor E. Idiopathic hirsutism

E This patient most likely has idiopathic hirsutism. She has no other clinical signs of polycystic ovaries, such as irregular cycles or obesity. Normal laboratory values rule out other pathogenic causes of hirsutism, such as Cushing's syndrome or adrenal tumor. Oral contraceptives are actually used for the treatment of hirsutism because they establish regular menses and lower ovarian androgen production. Additionally, they cause an increase in SHBG which allows more testosterone to be bound and unavailable at the hair follicle.

A 26-year-old G0 presents to the emergency room with eight hours of severe right lower quadrant pain associated with nausea. She has a history of suspected endometriosis, which was diagnosed two years ago based on severe dysmenorrhea. She has been using NSAIDs during her menses to control the pain. She is not sexually active. She is otherwise in good health. Her menstrual cycles are regular and her last menstrual period was three weeks ago. She has no history of sexually transmitted infections. Her vital signs are: blood pressure 145/70; pulse 100; temperature 98.6°F (37.0°C). She appears uncomfortable. On abdominal examination, she has moderate tenderness to palpation in the right lower quadrant. On pelvic examination, she has no lesions or discharge. A thorough bimanual exam was difficult to perform due to her discomfort. Beta-hCG <5 mIU/ml and hematocrit 29%. A pelvic ultrasound showed a 6 cm right ovarian mass. The uterus and left ovary appeared normal. There was a moderate amount of free fluid in the pelvis. What is the most appropriate next step in the management of this patient? A. Begin oral contraceptives B. MRI of the pelvis C. Doppler pelvic ultrasound D. CT scan of the pelvis E. Surgical exploration

E This patient most likely has ovarian torsion and needs to be surgically explored. Further imaging studies will not help beyond the information obtained on the ultrasound. A Doppler ultrasound to check the blood flow to the ovaries is controversial, as normal flow does not rule out ovarian torsion. Although oral contraceptives can help decrease the development of further cyst formation and control the pain associated with endometriosis, this patient needs immediate surgical attention due to suspected ovarian torsion.

A 23-year-old G1P1 diagnosed with postpartum depression at three months after a spontaneous vaginal delivery, has suicidal ideation and is desperate for help. Which of the following is the most appropriate next step in the treatment of this patient? A. Behavioral psychotherapy B. Anti-depressant medication C. Anxiolytic agent D. Anti-psychotic medication E. Inpatient psychiatric admissio

E This patient offered thoughts of suicidal ideation, thus inpatient management is the most appropriate choice. While behavioral psychotherapy is necessary to establish long-term strategies for coping skills, newer regimens for postpartum depression include the use of SSRI medication. SSRI medications have been shown to hasten recovery to a fully functioning state.

A 22-year-old G2P1 woman comes to your clinic today with her three-month-old daughter. She was breastfeeding without problems until about two weeks ago, when she began to experience sore nipples. The nipples are very sensitive and there is a in the breasts, which is worse when feeding. The tips of the nipples are pink and shiny with peeling at the periphery. Which of the following organism is the most likely cause of these findings? A. Group A streptrococcocus B. Group B streptrococcocus C. Staphylococcus aureus D. Staphylococcus epidermidis E. Candida

E This presentation is classic for candidiasis and should prompt an inspection of the baby's oral cavity. Candida of the nipple is associated with severe discomfort and pain. All the other above organisms are associated with classic mastitis and do not usually cause intense nipple pain.

A 27-year-old G2P1 woman at 36 weeks gestation is admitted with severe preeclampsia. Her blood pressure is 200/105. She has received two doses of IV hydralazine to lower her blood pressure. What diastolic blood pressure should you aim for in this patient? A. 50-55 mm Hg B. 60-65 mm Hg C. 70-75 mm Hg D. 80-85 mm Hg E. 90-95 mm Hg

E Treatment with an antihypertensive is indicated for blood pressures persistently greater than 160 systolic and 105 diastolic. First-line agents include hydralazine (a direct vasodilator) 5 mg IV followed by 5-10 mg doses IV at 20-minute intervals (maximum dose = 40 mg); or labetalol (combined alpha & beta-adrenergic antagonist) 10-20 mg IV followed by 20 mg, then 40 mg, then 80 mg IV every 10 minutes (maximum dose = 220 mg). The goal is not a normal blood pressure, but to reduce the diastolic blood pressure into a safe range of 90-100 mm Hg to prevent maternal stroke or abruption, without compromising uterine perfusion.

A 38-year-old G2P0 woman at 28 weeks gestation has been diagnosed with preterm labor and is currently stable on nifedipine. Her cervical exam has remained unchanged at 2 cm dilated, 75% effaced and -2 station. Her vital signs are stable and fetal heart tracing is category I. You recommend treatment with betamethasone (a steroid). Which of the following is associated with betamethasone therapy in the newborn? A. Enhancement of fetal growth B. Increased risk of infection C. Increased incidence of necrotizing enterocolitis D. Increased incidence of intracerebral hemorrhage E. Decreased incidence of intracerebral hemorrhage

E Treatment with betamethasone from 24 to 34 weeks gestation has been shown to increase pulmonary maturity and reduce the incidence and severity of RDS (respiratory distress syndrome) in the newborn. It is also associated with decreased intracerebral hemorrhage and necrotizing enterocolitis in the newborn. It has not been associated with increased infection or enhanced growth.

A 25-year-old G1 is at 17 weeks gestation. A 2 cm subserosal fibroid was noted on the anterior wall of her uterus at the time of her obstetric ultrasound at 17 weeks gestation. Which of the following treatment options for the uterine fibroid is most appropriate? A. Obtain a follow up ultrasound at six week intervals to follow growth of the fibroid B. Laparoscopy now to remove the fibroid C. Laparotomy in the late second trimester with removal of the fibroid D. Perform delivery by Cesarean section at term with removal of the fibroid after delivery of the baby and placenta E. No further treatment is necessary

E Uterine fibroids are the most common solid pelvic tumors in women. On postmortem examination, fibroids can be detected in as high as 80% of women. Most uterine fibroids are asymptomatic and do not require any treatment. Pregnant patients with fibroids usually are asymptomatic and do not have any complications related to the fibroids. Fibroids may grow or become symptomatic in pregnancy due to hemorrhagic changes associated with rapid growth, known as red or carneous degeneration. However, this is uncommon for smaller fibroids. Uncommonly, fibroids can be located below the fetus, in the lower uterine segment, or cervix, causing a soft tissue dystocia, necessitating delivery by Cesarean section. Myomectomy (removal of the fibroid) during pregnancy is contraindicated. Myomectomy at the time of Cesarean section should be avoided, if possible, secondary to the risk for increased blood loss. It is not necessary to follow the growth of fibroids during pregnancy, except for the rare cases when the fibroid is causing symptoms (primarily pain) or appear to be located in a position likely to cause dystocia.

A 42-year-old G2P2 undergoes a hysterectomy for definitive treatment of her dysmenorrhea and large uterine fibroids. The uterus is sent to pathology. Which of the following would confirm the diagnosis of fibroids? A. Blue-domed cysts greater than 3 mm B. Decidual effect in the endometrium C. Endometrial glands/stroma and hemosiderin-laden macrophages D. Invasion of endometrial glands into the myometrium E. Well-circumscribed, non-encapsulated myometrium

E Well-circumscribed, non-encapsulated myometrium confirms the diagnosis of fibroids. Leiomyosarcomas will have more than 10 mitotic figures per high power field.

A 30-year-old G1P1 woman is breastfeeding her baby and feels there is not enough milk. She is pumping in order to improve the supply of milk. You tell her that more frequent suckling would be better as it will stimulate which of the following hormones? A. Progesterone B. Insulin C. Cortisol D. Prolactin E. Oxytocin

E While prolactin is responsible for milk production, oxytocin is responsible for milk ejection. Production of oxytocin is stimulated by suckling which works better than a breast pump for stimulating the secretion of milk. Cortisol and insulin act in concert with other hormones to stimulate the growth and development of the milk-secreting apparatus.

A 45-year-old woman is referred to your office for a Pap smear that is suspicious for malignancy. The cervix appears grossly normal on speculum exam. The next most appropriate procedure is: . A) Radical hysterectomy B) Simple hysterectomy C) Cervical cone biopsy D) Cryotherapy of the cervix E) Colposcopic directed biopsy

E) Colposcopic directed biopsy

In evaluating a reproductive age woman who presents with amenorrhea, which of the following conditions will result in a positive (withdrawal) progesterone challenge test? A) Pregnancy B) Ovarian failure C) Pituitary failure D) Mullerian agenesis E) Polycystic ovary (PCOS) syndrome

E) Polycystic ovary (PCOS) syndrome

In order to stop intractable uterine bleeding from postpartum hemorrhage, ligation of the internal iliac arteries is performed. Blood flow will be greatly diminished in all of the following arteries EXCEPT: A) Obturator B) Superior gluteal C) Inferior gluteal D) Superior vesical E) Superior rectal

E) Superior rectal

A 36-year-old woman, gravida 1, para 1, presents for evaluation of cyclic heavy menstrual bleeding and increasing pelvic pressure for the past year. Her medical history is unremarkable. She had a preterm 34-week vaginal delivery 5 years ago. On examination, her vital signs are stable. A pregnancy test result is negative. Her abdomen is soft, nontender with a mass palpable just inferior to the umbilicus. Pelvic examination revealed an 18-week-sized mobile uterus. Endometrial biopsy showed secretory endometrium. Pelvic ultrasonography assessed her uterine size to be 18 cm x 10 cm x 7 cm with multiple leiomyomas, the largest of which was 8 x 6 x 4 cm. With saline infusion ultrasonographic scanning, none of the leiomyomas had substantial submucous components. The patient's hemoglobin was 8 g/dL. The patient tells you she desires treatment that will reduce both her bleeding and her pelvic pressure. She wishes to preserve her fertility. The safest and most effective long-term treatment option for this patient is: A) combination oral contraceptives B) gonadotropin-releasing hormone (GnRH) agonist injections C) uterine artery embolization D) magnetic resonance imaging (MRI)-guided focused ultrasound therapy E) myomectomy

E) myomectomy

Indicate the true statements concerning vulvar cancer: 1) most commonly it develops from the stratified squamous epithelium; 2) HPV infections play an important role in pathogenesis of some vulvar cancers; 3) supraclavicular lymph nodes are the most common site of metastasis; 4) basic treatment consists in the surgery and radiochemotherapy; 5) hormonal therapy is the most effective method of vulvar cancer treatment. The correct answer is: A. only 5. B. 1,2,3. C. 1,2,5. D. 2,3,4. E. 1,2,4.

E. 1,2,4.

A 33-year-old pregnant woman in 37th week of her second pregnancy came by the obstetric out-patient clinic because of the weakening of fetal movements. The course of the previous pregnancy was uncomplicated. The cardiotocography showed normal reactive FHR. Ultrasound examination of biophysical profile of the fetus revealed normal volume of amniotic fluid, one episode of breathing movements lasting 60 seconds, three movements of the trunk and twice hand closing and opening. The result of biophysical profile is: A. 2/10 points. B. 4/10 points. C. 6/10 points. D. 8/10 points. E. 10/10 points.

E. 10/10 points.

Which of the following are regarded as risk factors for a transverse lie? 1) oligohydramnios; 2) polyhydramnios; 3) placenta praevia; 4) multiple pregnancy; 5) single pregnancy. The correct answer is: A. 1,2,3,4. B. 1,3. C. 1,2,4,5. D. 1,3,5. E. 2,3,4.

E. 2,3,4.

The factors that increase the risk of endometrial cancer include: 1) smoking cigarettes; 2) metabolic syndrome (obesity, diabetes); 3) multiparity; 4) polycystic ovary syndrome; 5) long-term estrogen therapy not balanced with progesterone administration. The correct answer is: A. 1,3,5. B. 3,4,5. C. 2,3,4. D. 2,4. E. 2,4,5.

E. 2,4,5.

Oligohydramnios is defined as the decrease in the volume of amniotic fluid below: A. 600 mL. B. 500 mL. C. 400 mL. D. 300 mL. E. 200 mL.

E. 200 mL.

Indicate the estimated date of delivery calculated according to Naegele's rule for a woman with regular menstrual cycles of 32 days if the last menstruation started on 15 July 2018: A. 15 April 2019. B. 19 April 2019. C. 21 April 2019. D. 25 April 2019. E. 26 April 2019.

E. 26 April 2019.

The administration of specific immunoglobulin is mandatory in the case of the pregnant, who had not been affected or vaccinated, and who had contact with a patient infected with: 1) type B hepatitis virus; 2) CMV virus; 3) fifth disease virus; 4) rubella virus; 5) chickenpox virus. The correct answer is: A. 1,3,5. B. 2,3,5. C. 2,3,4. D. 1,4,5. E. 4,5.

E. 4,5.

Which of the following are used to assess the maturity of girls during puberty? 1) Bishop score; 2) Nugent score; 3) POP-Q score; 4) Tanner stages; 5) TBS (the Bethesda system) classification; 6) Lauritzen criteria. The correct answer is: A. 1,4. B. 2,5. C. 3,5. D. 4,5. E. 4,6.

E. 4,6.

A 28-year-old G2P2 woman returns today for follow up on her abnormal pap smear which reveals atypical squamous cells of undetermined significance (ASCUS). Reflex HPV testing is positive for high risk type. She has never had a prior abnormal pap smear, and has been following the recommended screening guidelines. She is asymptomatic. Her pelvic exam reveals a normal cervix with a small amount of cervical mucous. What is the next best step in the management of this patient? A. Routine screening B. Repeat Pap smear in one year C. Repeat HPV testing in one year D. Repeat co-testing with Pap and HPV in one year E. Colposcopy

E. Colposcopy

Pelvic floor disorders are the result of the weakening of the supporting and hanging apparatus as well as an excessive action of forces causing a displacement of genital organs. The factors conducive to the decline of the reproductive organ include: 1) damage to the vulvar and pelvic nerves in the course of vaginal delivery; 2) physical work in standing position and associated with carrying; 3) respiratory diseases associated with chronic cough; 4) chronic constipation; 5) post-menopausal estrogen deficiency. The correct answer is: A. 1,2,3,4. B. 1,3,4,5. C. 1,2. D. only 1. E. all the above.

E. all the above.

Which of the following are regarded as anatomical factors which can cause a miscarriage? A. endometriosis in the lesser pelvis. B. congenital defects of the uterus. C. cervical incompetence. D. uterine fibroids. E. all the above.

E. all the above.

Which of the following decreases men's fertility by lowering testosterone production? A. ethanol. B. ketoconazole. C. spironolactone. D. sulfasalazine. E. all the above.

E. all the above.

Which of the following is in favor of the diagnosis of antiphospholipid syndrome? A. vascular thrombosis. B. miscarriages. C. presence of anti-cardiolipin antibodies. D. presence of antibodies against β2-glycoprotein 1. E. all the above.

E. all the above.

The most common site for an ectopic pregnancy is: A. abdominal cavity. B. ovary. C. intramural part of the uterine tube. D. isthmus of the uterine tube. E. ampulla of the uterine tube.

E. ampulla of the uterine tube.

A heterotopic pregnancy may be a complication of an assisted reproductive program and denotes: A. ectopic pregnancy localized in the abdominal cavity. B. ectopic pregnancy localized in the scar after the cesarean section. C. ectopic pregnancy localized in the isthmus of the uterine tube. D. ectopic pregnancy localized in the cervix. E. both extra-uterine and intrauterine pregnancy occurring simultaneously.

E. both extra-uterine and intrauterine pregnancy occurring simultaneously.

Side-effects of the treatment of premenstrual syndrome with selective serotonin reuptake inhibitors include: A. hirsutism. B. loss of hair C. abnormal bleeding from the uterus. D. inhibition of ovulation. E. decreased libido.

E. decreased libido.

A 34-year-old female patient 3 months after a physiological childbirth suddenly suffered pain of the vulva. She complained that she could not sit, and while she tries to sit down she feels a resistance in the right side. In the light of reported symptoms one should: A. diagnose complications of crotch sewing and refer the patient for crotch plastic surgery. B. diagnose viral vulvitis, most probably herpetic and order acyclovir therapy. C. diagnose vulva carcinoma and order biopsy. D. diagnose Skene's gland abscess and order antibiotics. E. diagnose Bartholin's abscess, make incision and drainage.

E. diagnose Bartholin's abscess, make incision and drainage.

Finding the lambda sign on ultrasound examination in a patient at 9 week of gestation allows diagnosing: A. missed miscarriage. B. monoamniotic twin pregnancy. C. monozygotic twin pregnancy. D. singleton pregnancy. E. dichorionic twin pregnancy.

E. dichorionic twin pregnancy.

The ultrasound risk factor for shoulder dystocia is not: A. estimated fetal weight above 4500 g. B. abdominal circumference above 350 mm. C. difference in the circumference of the abdomen and fetal head over 40 mm. D. difference in the transverse abdominal and biparietal diameter over 26 mm. E. difference in the length of the clavicular bone and the humerus over 50 mm.

E. difference in the length of the clavicular bone and the humerus over 50 mm.

A neglected transverse lie is characterized with: 1) impaction of the presenting shoulder in the pelvic inlet; 2) premature rupture of the membranes; 3) total cervical effacement; 4) excessive dilation of the lower uterine wall; 5) contraction of the uterus on a highly bent fetus. The correct answer is: A. 1,2,3. B. 1,3,5. C. 2,4. D. 3,4,5. E. all the above.

E. all the above.

In patients with polycystic ovary syndrome there is an increased risk of: A. type 2 diabetes mellitus. B. hypertension. C. endometrial cancer. D. myocardial infarction. E. all the above.

E. all the above.

The increased risk of endometrial cancer has not been associated with: A. obesity. B. diabetes. C. hypertension. D. late menopause. E. endometriosis.

E. endometriosis.

This tumor can develop before puberty but the peak incidence is observed in women of reproductive age. It easily ruptures which may lead to peritoneal signs and symptoms demanding urgent surgery. In postmenopausal women it may cause abnormal bleeding from the reproductive tract. Surgical excision does not guarantee full recovery as this tumor recurrence may be observed as many as 30 years after the initial diagnosis". The Above is a characteristic of: A. ovarian cancer. B. sarcoma. C. hydatidiform mole. D. germinoma. E. granulosa cell tumor.

E. granulosa cell tumor.

The importance of GnRH analogues in gynecology has long been known. They are used in the following situations, except for: A. endometriosis treatment. B. in vitro fertilization programs. C. uterine myoma treatment. D. precocious puberty treatment. E. growth hormone deficiency treatment.

E. growth hormone deficiency treatment.

Which of the following can be usually found in a patient with menstrual disordersrelated to chronic intensive physical exercise? A. hypogonadotropic hypergonadism. B. hypergonadotropic hypergonadism. C. normal hormonal function of the ovaries. D. hypergonadotropic hypogonadism. E. hypogonadotropic hypogonadism.

E. hypogonadotropic hypogonadism.

Which of the following is not a biological effect of progesterone? A. induction of endometrial secretory changes during the ovulation cycle. B. cervical mucus change into impenetrable to sperm. C. decrease of the hypoglycemic effect of insulin. D. decrease of contractibility of the uterine muscle and fallopian tube peristalsis. E. increase in the concentration of coagulation factors II, VII, IX and X.

E. increase in the concentration of coagulation factors II, VII, IX and X.

Which of the following is not a physiological cause of increased prolactin secretion in women? A. pregnancy. B. strenuous activity. C. sleeping. D. stress. E. increased glucose level.

E. increased glucose level.

Which of the following statements concerning hydatidiform mole is false? A. complete hydatidiform mole is diploid and all the genetic material is of paternal origin. B. incomplete hydatidiform mole is usually triploid. C. principal diagnostic tool for hydatidiform mole is ultrasound imaging. D. first symptom of hydatidiform mole is usually uterine bleeding. E. most common symptoms of hydatidiform mole are excessive vomiting and the symptoms of hypothyroidism.

E. most common symptoms of hydatidiform mole are excessive vomiting and the symptoms of hypothyroidism.

Which of the following increases the risk of monozygotic twin pregnancy? A. advanced maternal age. B. use of ovulation stimulating drugs. (dizygotic) C. previous multiple pregnancy. D. long-term use of oral contraceptives. E. none of the above.

E. none of the above.

Which of the following is not among the abnormalities of the head position during labour: A. deep transverse arrest. B. brow presentation. C. occiput posterior position. D. left or right posterior position. E. occiput anterior position.

E. occiput anterior position.

Which of the following should be done in the case of a patient without risk factors for diabetes if her fasting glucose level is 110 mg/dL in the 9 week of gestation? A. oral glucose tolerance test with 75g of glucose between 24 and 28 week of gestation. B. repeated fasting glucose examination. C. recommend diabetes diet. D. assessment of the 24-hour blood glucose profile. E. oral glucose tolerance test with 75g of glucose immediately

E. oral glucose tolerance test with 75g of glucose immediately

A 35-year-old patient came to the gynecological outpatient clinic because of vaginal bleeding occurring between periods for about a year. The bleeding with dark brown blood occurs on 11 -12 day of the cycle and lasts 2-3 days. The periods occur every 30 days. The patient underwent a caesarian section two years earlier. Which of the following is the most probable cause of the bleedings? A. undiagnosed disorder of the clotting system. B. corpus luteum insufficiency. C. pelvic inflammatory disease. D. periovulatory LH surge. E. scar defect of the myometrium after the cesarean section.

E. scar defect of the myometrium after the cesarean section.

Which of the following is not the symptom related to uterine fibroids? A. menorrhagia. B. prolonged menstruation. C. hypogastric pains. D. pressure on the bladder or the rectum. E. secondary amenorrhea.

E. secondary amenorrhea.

The administration of short-acting GnRH analogues begins on the first day of the menstrual cycle and from the third day gonadotropins are added. This description regards controlled ovarian hyperstimulation with the use of: A. ICSI technique (intracytoplasmic sperm injection). B. slow freezing of reproductive material. C. protocol with GnRH antagonists. D. long protocol. E. short protocol.

E. short protocol.

A 58-year-old patient complains of vaginal dryness and postcoital bleedings. The gynecological examination reveals atrophic lesions in the vagina and normal uterus and uterine appendages. The cytological smear of the cervix is normal. The local vaginal estrogen therapy has been recommended and thus the patient should: A. receive simultaneous gestagen therapy in order to prevent the overgrowth of endometrium and the development of endometrial cancer. B. receive simultaneous gestagen therapy in order to prevent the development of breast cancer. C. receive simultaneous gestagen therapy in order to prevent the development of cervical cancer. D. receive simultaneous gestagen therapy in order to decrease the risk of a stroke. E. there is no indication for gestagen therapy in this patient.

E. there is no indication for gestagen therapy in this patient.

Which of the following blood vessels is used in the non-invasive ultrasound evaluation of foetal anaemia? A. uterine artery. B. ductus venosus. C. umbilical artery. D. middle cerebral artery. E. umbilical vein.

E. umbilical vein.

High alpha fetoprotein. Gestational age 18 weeks. What to do to establish dx? a. Amniocentesis b. Chorionic villus sampling

a. Amniocentesis

A 26 yo woman gravida 2 para 1 presents to your office at 33 weeks of gestation with nausea and vomiting, anorexia, and right-sided abdominal pain for 8 hours duration. Abdominal ultrasonography is inconclusive so a computed tomography scan is performed which reveals a right-sided appendicolith and a periappendiceal abscess. The most appropriate next step in her management is hospital admission and: a.) antibiotics and induction of labor after corticosteroids for fetal lung maturity b.) laparoscopic appendectomy c.) open appendectomy d.) cesarean delivery combined with appendectomy

a.) antibiotics and induction of labor after corticosteroids for fetal lung maturity

A healthy 20 yo woman has a 3 year history of dysmenorrhea with pain that is moderately severe during the first 3 days of her 5 day menses. Ibuprofen provides some relief. Her menstrual cycles are regular. Pelvic ultrasonography and bimanual examination results are negative. She would like to attempt pregnancy in 9 months. She requests a treatment to decrease menstrual bleeding, improve pain, and provide reliable contraception. You tell her that the best management option for her is: a.) cyclical oral contraceptives b.) continuous or long cycle OCs c.) depot leuprolide acetate (lupron) d.) depot medroxyprogesterone acetate

a.) cyclical oral contraceptives

A 55 yo woman comes in for her first gynecologic examination since the birth of her youngest child 30 years ago. She reports being monitored by her primary doctor and is taking an oral antihypertensive with no other medical problems. She is experiencing hot flushes and irregular vaginal bleeding. Physical examination you observe fullness in the left supraclavicular and cervical lymph nodes. Her abdomen is normal, but a pelvic examination reveals an 8cm exophytic tumor replacing the entire cervix. She does not demonstrate any parametrial involvement. Addition to cervical biopsy the next diagnostic procedure should be: a.) endometrial biopsy b.) fractional dilation and curettage c.) cystoscopy and proctoscopy d.) supraclavicular lymph node cytology or biopsy

a.) endometrial biopsy

A postmenopausal 60 yo woman with estrogen receptor positive breast cancer comes to see you for consultation. She has been taking tamoxifen citrate for 1 year and is anxious about the risk of developing endometrial cancer. Endometrial polyps have never been diagnosed and she has had no vaginal bleeding since menopause 10 years ago. The most appropriate next step for this patient is: a.) observation b.) progestin withdrawal test c.) transvaginal ultrasonography d.) pelvic magnetic resonance imaging e.) endometrial biopsy

a.) observation

A 31 yo woman gravida 3, para 0, comes to the clinic for evaluation of recurrent pregnancy loss. She used an intrauterine device (IUD) for contraception for 6 years because of a previous venous thrombosis episode at age 22 years. After IUD removal, she had three consecutive miscarriages between 5 weeks and 6 weeks of gestation. Chromosomal evaluation of the last products of contraception yielded a normal XY karyotype. Saline infusion sonohysterography revealed a normal uterine cavity. A fasting glucose test was normal. The best next step to diagnose her condition is: a.) parental karyotype b.) antiphospholipid antibody in luteal phase c.) endometrial biopsy in luteal phase d.) cervical cultures for chlamydia trachomatis

a.) parental karyotype

Pregnant, previously suicidal woman. Well controlled by bupropion, venlafaxine and something I can't remember. What to do? a. Discontinue all medication b. Discontinue all but bupropion c. Continue with all meds

b. Discontinue all but bupropion

Patient had lump in left breast. Mammography confirmed several small nodules in both breasts. What to do? a. MRI b. Examine again in one week

b. Examine again in one week

Question that described a young sexually active female. 4 cm mobile mass on labia majora (Bartholin cyst). How did she get it? a. Trauma b. Infection c. etc.

b. Infection

Patient was pregnant, nauseous and vomiting (but not severely). No ketones in labs? a. Hyperemesis gravidarum b. Normal pregnancy

b. Normal pregnancy

Twin pregnancy. One placenta. Separated by a thick membrane. Highest risk of? a. Twin-twin transfusion syndrome b. Preterm delivery c. Oligohydramnios

b. Preterm delivery

A 22-year old with one kid. Wanted tubal-ligation. Whats does she have the highest risk of after procedure? a. Bleeding b. Regret

b. Regret

Gestational diabetes mellitus was diagnosed in a 32 year old latin american woman gravida 1, at 28 weeks gestation 2 weeks ago. She initiated a 2,000 kcal american diabetes association diet and has been monitoring her blood sugar with a home glucometer. Her fasting blood sugar ranges from 120 to 140 and her 2 hour postprandial sugar ranges from 170 to 240. The best next step in management is: a.) increase the fiber content of her diet b.) begin a low carbohydrate diet c.) glyburide therapy d.) insulin therapy e.) initiate daily exercise regimen

b.) begin a low carbohydrate diet

A healthy 20 yo woman has a 3 year history of dysmenorrhea with pain that is moderately severe during the first 3 days of her 5 day menses. Ibuprofen provides some relief. Her menstrual cycles are regular. Pelvic ultrasonography and bi manual examination results are negative. She would like to attempt pregnancy in 9 months. She requests a treatment to decrease menstrual bleeding, improve pain, and provide reliable contraception. You tell her that the best management option for her is: a.) cyclical oral contraceptives b.) continuous or long cycle OCs c.) depot leuprolide acetate (lupron) d.) depot medroxyprogesterone acetate

b.) continuous or long cycle OCs

A 32 year old woman is preparing for infertility treatment. You counsel her that she can reduce her risk of having a fetus with neural tube defect by taking a supplement that contains the micronutrient: a.) thiamine b.) folic acid c.) pyridoxine d.) beta carotene

b.) folic acid

A 27 yo primigravid kindergarten teacher calls the office at 16 weeks of gestation because she is worried about exposure to a child with a rash. She reports that fifth disease was diagnosed in one of her pupils. She is asymptomatic. The most appropriate next step in management would be: a.) no further testing b.) maternal antibodies for parvovirus b19 c.) weekly middle cerebral artery doppler measurements d.) amniocentesis for polymerase chain reaction for parvovirus B19 e.) percutaneous umbilical blood sampling

b.) maternal antibodies for parvovirus b19

A 58 yo woman gravida 2, para 2, presents to your office for her annual well-woman examination. She became post menopausal at age 51 years and reports no bleeding or spotting since that time. She has a history of well-controlled hypertension. She exercises 5 times a week and is a nonsmoker. Her physical examination is within normal limits. Her bmi is 25. Her speculum examination reveals atrophic vaginal epithelium, normal cervix, and a small mobile uterus with nonpalpable adnexa. She informs you that endometrial cancer was recently diagnosed in her cousin; she has no other significant family history of cancer. She inquires about whether she needs screening for endometrial cancer. The best next step is: a.) expectant management b.) pelvic ultrasonography to evaluate endometrial stripe c.) endometrial biopsy d.) hysteroscopy with dilation and curettage

b.) pelvic ultrasonography to evaluate endometrial stripe

Question about the gas values of the umbilical cord right after delivery and being clamped: Arterial pH Venous pH? a. 7.05 7.15 b. 7.34 7.42 c. etc. normal is 7.18-7.38

c. etc. normal is 7.18-7.38

Patient is pregnant. Everything is normal, except for anteverted or retroverted (?) uterus. What does she have an increased risk for? a. Hemorrhage b. Premature delivery c. slightly tilted - considered normal causes incontinence or difficulty urinating during pregnancy

c. slightly tilted - considered normal causes incontinence or difficulty urinating during pregnancy

A 31 yo woman gravida 2, aborta 2, comes for consultation regarding secondary infertility. She reports a 12 month history of increasing menorrhagia and intermenstrual spotting for the past 6 months with less regular periods. Pelvic examination reveals a nontender uterus the size of an 8-week gestation. Transvaginal ultrasonography reveals a thickened irregular endometrial stripe and a 2cm submucosal leiomyoma. Lab studies reveal a hemoglobin level of 11g/dl and a normal prolactin and thyroid stimulating hormone levels. A sonohysterogram performed on cycle day 8 reveals a 1.8cm intracavitary leiomyoma. The best next step in management is: a.) clomiphene citrate (clomid, serophene) b.) intrauterine insemination c.) hysteroscopic myomectomy d.) in vitro fertilization and embryo transfer e.) ovulation induction with gonadotropins

c.) hysteroscopic myomectomy

A 30 year old nulligravid woman has been trying to conceive for the past year. She is unsure as to when she is ovulating and would like to know how she can best detect ovulation. The most reliable indicator of impending ovulation is an increase in: a.) estradiol b.) follicle stimulating hormone c.) luteinizing hormone d.) progesterone e.) basal body temperature

c.) luteinizing hormone

A 30 yo nulligravid woman has been trying to conceive for the past year. She is unsure as to when she is ovulating and would like to know how she can best detect ovulation. The most reliable indicator of impending ovulation is an increase in: a.) estradiol b.) follicle stimulating hormone c.) luteinizing hormone d.) progesterone e.) basal body temperature

c.) luteinizing hormone

A 45 yo woman is scheduled to undergo laparoscopically assisted vaginal hysterectomy for menorrhagia and uterine leiomyomas. On laparoscopic inspection of the pelvis, a 7cm complex left ovarian mass appears suspicious for cancer. You obtain pelvic washings for frozen tissue diagnosis the most appropriate next step in management is to perform a left: a.) ovarian biopsy b.) ovarian aspiration c.) ovarian cystectomy d.) salpingo-oophorectomy e.) salpingo-oophorectomy and laparoscopically assisted vaginal hysterectomy

c.) ovarian cystectomy

A 31 yo woman has a history of a prior cesarean delivery with an associated Pomeroy tubal ligation. The patient had endometritis after her delivery that was treated with antibiotics. She has recently remarried. Her husband has not fathered a pregnancy, but does have a normal semen analysis. She requests information concerning a tubal anastomosis versus in vitro fertilization to help her conceive another child. The factor that best predicts the success rate for pregnancy and delivery after a tubal anastomosis versus IVF is: a.) age of the woman requesting fertility services b.) prior history of an cesarean delivery c.) type of tubal ligation performed d.) history of endometritis e.) absence of any prior conceptions with the husband's sperm

c.) type of tubal ligation performed

A 38 yo married, nulligravid woman has been infertile for 1 year. She has a history of progressive dysmenorrhea as an adolescent that was successfully treated with the use of oral contraceptives. She discontinued taking oral contraceptives 1 year ago to attempt pregnancy and has noted recurrence of progressive dysmenorrhea especially in the past 6 months. She reports deep dyspareunia and left-sided discomfort. Pelvic examination reveals tenderness and nodularity in the posterior cul-de-sac together with left adnexal tenderness and fullness. Transvaginal ultrasonography reveals a normal uterus with a 5cm left ovarian cyst with homogenous "ground glass" appearance. The patient requests treatment for her pelvic pain and infertility. The best recommendation for this patient is: a.) leuprolide acetate (depo lupron) therapy b.) nonsteroidal anti-inflammatory agent c.) ovulation stimulation and intrauterine insemination d.) operative laparoscopy

d.) operative laparoscopy

A 30 yo nulligravid woman presents with secondary amenorrhea and a serum follicle-stimulating hormone level of 112 miu/ml found on two separate occasions that were weeks apart. Her karyotype is normal. She is contemplating using her 25 yo sister as an egg donor. Her sister has had no children and one miscarriage. While discussing the process of egg donation with her sister, you note that she has a slight stutter when she articulates and a long prominent jaw. No other abnormalities are noted. Before using the sister as a donor it would be prudent to recommend that she undergo: a.) fragile x screening b.) adrenal antibodies quantification c.) ovarian biopsy d.) transvaginal ultrasonography e.) anti-mullerian hormone measurement

e.) anti-mullerian hormone measurement

A 25 yo woman gravida 2, para 2, requests a prescription for oral contraceptives. Within the past 8 months she started having headaches that were particularly severe just before her periods. The headaches are preceded by bilateral scotoma, bilateral visual field cut, and numbness across her right cheek. She is obese with a bmi of 33. You inform her the her best hormonal contraceptive option would be: a.) continuous OC b.) levonorgestrel intrauterine device (IUD) c.) vaginal contraceptive ring d.) contraceptive patch e.) depot medroxyprogesterone acetate

e.) depot medroxyprogesterone acetate

A 43 yo woman with menses every 21-24 days has completed 3 months of therapy with clomiphene citrate (clomid, serophene) 100 mg on cycle days 5-9. She has a day 3 follicle-stimulating hormone level of 25 miu/ml and a normal hysterosalpingogram. Her male partner has a documented normal semen analysis. She asks you about her best option to become pregnant. The treatment you recommend is: a.) continue clomiphene citrate (clomid, serophene) b.) in vitro fertilization c.) gonadotropin therapy d.) oocyte donation e.) intrauterine insemination

e.) intrauterine insemination


Ensembles d'études connexes

Ch. 43 Hematologic and Immunologic Dysfunction

View Set

Economics Guided Reading 2.2-2.3

View Set

Mitosis/Meiosis Lecture Quiz Ch 6+7

View Set